NCLEX Practice Questions 1500+

Pataasin ang iyong marka sa homework at exams ngayon gamit ang Quizwiz!

The nurse is caring for a female 29 years of age who is admitted with chronic pain secondary to rheumatoid arthritis. She confides in the nurse that she would like to be able to have sex with her husband but it just hurts too much. The nurse's best response is which of the following?

Modified positions may be possible.

After instructing the male client on the performance of a testicular examination, the nurse instructs the client to perform the examination how often?

Monthly

Which of the following religious groups believe in divine healing through the "laying on of hands?"

Mormons

causes of hypoparathyroid

Most common cause is inadequate secretion of the parathyroid hormone - Could be due to the interruption of the blood supply or surgical removal of the parathyroid gland tissue during thyroidectomy, parathyroidectomy or radical neck resection - Idiopathic - spontaneously or from an unknown cause - Surgical removal may be another cause

A nurse is educating a student nurse on how STIs affect the health of their clients. Which of the following statements accurately describes an effect of an STI?

Most of the time STIs cause no symptoms, especially in women.

Which of the following statements is true for nursing care of older adults?

Most older adults are functional, benefiting from health-oriented interventions.

Change of shift report informs of newly admitted client with disoriented and combative during the night. Actions the day shift nurse should take

Move the client to a room near the nurses' station.

The nurse is examining charts to identify clients at risk for developing multiple myeloma. Which client is most at risk?

Multiple myeloma is more common in middle-aged and older clients. The median age at diagnosis is 60 years. It is twice as common in blacks as it is in whites. It occurs most often in black men.

The nurse is admitting a new client to the unit. The nurse notes that this client would need an alternate meal choice when the menu specified pork for a meal. What cultural group would require an alternative meal choice?

Muslim

Home health nurse evaluating a school age child with cystic fibrosis initiates a request for high frequency chest compression vest

My child has only a small amount of mucus after percussion therapy

*patient with severe high calcium - increased urine ca and BUN

NOT maintain seizure precautions

Which of the following involves charting information about the client and client care in chronological order?

Narrative charting. Narrative charting involves writing information about the client and client care in chronological order. In SOAP charting, everyone involved in the client's care makes entries in the same location in the chart. Focus charting follows a data, action, and response (DAR) model to reflect the steps in the nursing process. PIE charting is a method of recording the client's progress under the headings of problem, intervention, and evaluation.

Assessing a newborn following vaginal birth. Findings to report to provider

Nasal flaring

Most nurses have been taught to maintain direct eye contact when communicating with clients. Some cultural groups would not value direct eye contact with the nurse. Which cultural group would consider the direct eye contact impolite?

Native Americans

Role Ambiguity: Don't know if I'm ready to be a mom:

Negative self-esteem

A male client age 15 years is experiencing nocturnal emissions. What nursing intervention would be appropriate for this client?

No intervention is necessary as this is a normal phenomenon.

The nurse is reviewing the health history of a newly admitted patient and reads that the patient has been previously diagnosed with exostoses. How should the nurse accommodate this fact into the patient's plan of care?

No specific assessments or interventions are necessary to addressing exostoses.

The client is to have a gastrectomy. The surgeon will use a transverse incision. Prior to surgery, the nurse is checking to be sure the correct site has been marked. Identify the site that should have marked.

Note: use the marking drawn in the picture!

A mother of a 5-year-old child who was admitted to the hospital has a Protection from Abuse order for the child against his father. A copy of the order is kept on the pediatric medical surgical unit where the child is being treated. The order prohibits the father from having any contact with the child. One night, the father approaches the nurse at the nurses' station, politely but insistently demanding to see his child, and refusing to leave until he does so. What should the nurse do first?

Notify hospital security or the local authorities. The Protection from Abuse order legally prohibits the father from seeing the child. In this situation, the nurse should notify hospital security or the local authorities of this attempt to breach the order, and allow them to escort the father out of the building. The father could be jailed or fined if he violates the order. The nurse shouldn't argue or continue explaining to the father that he must leave because it could place her and the child at risk if the father becomes angry or agitated. The nursing coordinator and nurse-manager should be notified of the incident; the nurse's first priority, however, should be contacting security or the authorities.

A nursing assistant escorts a client in the early stages of labor to the bathroom. When the nurse enters the client's room, she detects a strange odor coming from the bathroom and suspects the client has been smoking marijuana. What should the nurse do next?

Notify the physician and security immediately.

The nurse is caring for a client who was given pain medication before leaving the postanesthesia care unit. Upon returning to her room, the client complains of pain and requests more pain medication. Which is the best action for the nurse to take?

Notify the physician that the client is continuing to complain of pain.

Assigning task roles for group of clients in community mental health clinic. Which should be assigned to group functioning the orienter

Noting the progress of the group toward assigned goals

A group of nurses has established a focus group and pilot study to examine the potential application of personal data assistants (PDAs) in bedside care. This study is a tangible application of:

Nursing informatics is a specialty that integrates nursing science, computer science, and information science to manage and communicate data, information, and knowledge in nursing practice. A specific application of nursing informatics is the use of PDAs in the clinical setting. The devices are less likely to be used to perform documentation or to constitute client records. Telemedicine involves the remote provision of care.

A client informs the nurse that her physician has planned a procedure that may be in conflict with the client's personal spiritual belief. The client asks the nurse for assistance. The nurse is aware that her role should include assisting the client to do which of the following?

Obtain accurate information in order to make a good decision.

A client tells a nurse that he does not think he can have the recommended heart surgery because transfusions are against his religion. What is the best response of the nurse in this situation?

Obtain all the information needed for the client to make an informed decision.

When administering a patient's eye drops, the nurse recognizes the need to prevent absorption by the nasolacrimal duct. How can the nurse best achieve this goal?

Occlude the puncta after applying the medication.

Nurses provide care to meet needs in all the human dimensions. What is one intervention nurses can implement to meet spiritual needs?

Offer a compassionate presence.

A client says, "What is that awful smell?" What sense is being used?

Olfactory

Assessing a client following vaginal delivery and notes heavy lochia and a boggy fundus. Which med should be administered

Oxytocin

A nurse is using Anandarajah and Hight's (2001) HOPE acronym to assess a client's spirituality and religious beliefs. Which of the following is a component of this acronym?

P = personal spirituality H—Sources of hope, meaning, comfort, strength, peace, love, and connection; O—Organized religion; P—Personal spirituality and practice; E—Effects on medical care and end-of-life issue.

While answering questions posed by a nurse during a health history, a young woman says, "Before my period I get headaches, am moody, and my breasts hurt." What is the client experiencing?

PMS

assessment findings for hyperpatathyroidism

PTH excess causes hypercalcemia and destruction of bones - Neurological irritation or depression - Cardiovascular: Arrythmias, hypertension, etc. - Gastrointestinal: Nausea, vomiting, constipation, decreased peristalsis

A nurse is caring for a client from Taiwan who constantly requests pain medication. What should the nurse consider when assessing the client's pain?

Pain is what the client says it is.

The clinic nurse is assessing a child who has been brought to the clinic with signs and symptoms that are suggestive of otitis externa. What assessment finding is characteristic of otitis externa?

Pain on manipulation of the auricle

The nurse is caring for a client receiving patient-controlled analgesia (PCA) for pain management. Which statement about PCA is true?

Pain relief is initiated by the client as needed.

A nurse fills the following roles in the community: health educator, personal health counselor, referral agent, trainer of volunteers, developer of support groups, integrator of faith and health, and health advocate. What is the term for this type of nurse?

Parish nurse

A hearing-impaired patient is scheduled to have an MRI. What would be important for the nurse to remember when caring for this patient?

Patient is likely unable to hear the nurse during test.

A nursing instructor has assigned a student to care for a client of Asian descent. The instructor reminds the student that personal space considerations vary among cultures. What personal space preferences are important for the student to consider when caring for this client?

People of Asian descent prefer some distance between themselves and others.

Mild anxiety:

Perception and learning is enhanced

Planning to delegate client care tasks to an assistive personnel. Which task can be delegated?

Perform gastrectomy feeding through a client's established gastrostomy tube.

Which of the following should the nurse first consider when attempting to become culturally competent?

Personal cultural beliefs and prejudices

Teaching self administration of insulin to a client with short acting and intermediate acting insulin

Pinches the skin prior to injecting the insulin

Assessing a client who has pulmonary edema. Finding to expect

Pink, frothy sputum-due to fluid leaking across the pulmonary capillaries and into the lung tissue.

The nurse is caring for a comatose, older adult with stage III pressure ulcers over two bony prominences. Which intervention should be added to the plan of care?

Place the client on a pressure redistribution bed.

Providing care for a client with a colostomy

Place the skin barrier over the stoma and hold it for 30 seconds.

A client is admitted to the facility with a productive cough, night sweats, and a fever. Which action is most important in the initial care plan?

Placing the client in respiratory isolation. Because the client's signs and symptoms suggest a respiratory infection (possibly tuberculosis), respiratory isolation is indicated. Every 8 hours isn't frequent enough to assess the temperature of a client with a fever. Monitoring fluid intake and output may be required, but the client should first be placed in isolation. The nurse should wear gloves only for contact with mucous membranes, broken skin, blood, and other body fluids and substances.

The nurse should dispose of a used needle and syringe by:

Placing uncapped, used needles and syringes immediately in the universal precaution container in the client's room.

In which of the phases of the sexual response cycle may secretions from Cowper's glands appear at the glans of the penis?

Plateau

Which of the following nonpharmacologic, independent nursing interventions may be used to promote relaxation without also causing sensory overload?

Playing music the client chooses

The nurse is aware that frequent repositioning in bed will assist in the prevention of which of the following for a client?

Pneumonia. By frequently changing positions in bed, the client can prevent the development of pneumonia, urinary stasis, and deep vein thrombosis. These movements promote blood, oxygen, and fluid circulation throughout the body systems and prevent stasis.

The nurse is caring for a bedridden, elderly adult. To prevent pressure ulcers, which intervention should the nurse include in the care plan?

Post a turning schedule at the client's bedside.

A nurse says to an older adult who is being cared for at home, "Tell me what your life was like when you were first married." What does this statement encourage the client to do?

Practice life review or reminiscence

When preparing a teaching plan for a client who is to receive a rubella vaccine during the postpartum period, the nurse should include which information?

Pregnancy should be avoided for 4 weeks after the immunization.

A nurse assessing older adults in a long-term care facility is aware that which of the following may result in sensory alterations for these clients?

Presbyopia

With aging, progressive inelasticity and thickening of the lens causes which accommodation disorder?

Presbyopia

Which nursing intervention is most appropriate for a client with multiple myeloma?

Preventing bone injury

HYPERPARATHYROIDISM/ PARATHYROID HYPERFUNCTION

Primary: Tumor, often benign, most common in people between the age of 60-70 Secondary: Occurs in those who have chronic renal failure - Constant stimulation of the parathyroid - Also due to chronic hypocalcemia, malabsorption syndromes

Caring for a client who is unconscious and requires emergency medical procedures. Unable to locate family to obtain consent

Proceed with provision of medical care

What term is used to describe the sense, usually at a subconscious level, of the movements and position of the body (especially its limbs) independent of vision?

Proprioception

Reviewing urinalysis report of client with acute glomerulonephritis expected findings

Protein

A 12-year-old has a fractured femur and is immobilized in traction as shown in the figure. What should the nurse do?

Provide opportunities for age-appropriate activities.

Which intervention should the nurse try first with a client who exhibits signs of sleep disturbance?

Provide the client with sleep aids, such as pillows, back rubs, and snacks.

Purulent:

Pus

The nurse cares for the client diagnosed with asthma. The physician orders neostigminenIM. Which of the following actions by the nurse is MOST appropriate? 1. Administer the medication. 2. Check the blood pressure and pulse. 3. Ask the pharmacy if the medication can be given orally. 4. Notify the physician.

Question: Can neostigmine be administered to a client with asthma? Strategy: "MOST appropriate" indicates that discrimination is required to answer the question. Needed Info: Neostigmine (Prostigmin) is a cholinergic (parasympathomimetic) used to treat myasthenia gravis and is an antidote for nondepolarizing neuromuscular blocking agents; side effects include nausea, vomiting, abdominal cramps, respiratory depression, bronchoconstriction, hypotension, and bradycardia. Nursing considerations include monitoring vital signs frequently, having atropine injection available, taking with milk, potentiates the action of morphine. (1) causes bronchoconstriction; notify physician (2) assessment; neostigmine causes hypotension and bradycardia; important to monitor vital signs, but priority is to notify the physician (3) medication used cautiously in clients with asthma. (4) CORRECT - cholinergics can cause bronchoconstriction in asthmatic clients; may precipitate an acute asthmatic attack

The nurse cares for the client receiving phenytoin intravenously. The nurse recognizes that the medication is administered in which of the following fluids? 1. 5% dextrose in water (D5W). 2. Lactated Ringer's solution. 3. 10% dextrose in water (D10W). 4. Normal saline.

Question: Dilantin should be mixed with which type of fluid? Strategy: Determine the outcome of each answer. Needed Info: Phenytoin (Dilantin) is an anticonvulsant; side effects include drowsiness, ataxia, nystagmus, blurred vision, gingival hypertrophy; give medication with meals to minimize GI irritation, inform client that red-brown or pink discoloration may occur; instruct about proper oral hygiene; never mix with other drugs. (1) may precipitate in any fluid containing dextrose (2) body can convert to glucose, which will precipitate med (3) may precipitate in any fluid containing dextrose (4) CORRECT - phenytoin may precipitate in any fluid containing dextrose; clear IV tubing with normal saline prior to administering to remove all traces of dextrose

The parents of an infant with myelomeningocele ask the nurse about their child's future mental ability. What is the nurse's best response?

"About one-third have an intellectual disability, but it is too early to tell about your child."

The nurse is discharging a patient home after mastoid surgery. What should the nurse include in discharge teaching?

"Don't blow your nose for 2 to 3 weeks."

Which of the following questions or statements would be most useful for the nurse to make when eliciting information about a client's sexual history?

"How would you describe the problem?

A nurse is performing discharge teaching with a client who had a total gastrectomy. Which statement indicates the need for further teaching?

"I will have to take vitamin B12 shots up to 1 year after surgery." After a total gastrectomy, a client will need to take vitamin B12 shots for life. Dietary B12 is absorbed in the stomach, and the inability to absorb it could lead to pernicious anemia.

A patient got a sliver of glass in his eye when a glass container at work fell and shattered. The glass had to be surgically removed and the patient is about to be discharged home. The patient asks the nurse for a topical anesthetic for the pain in his eye. What should the nurse respond?

"Overuse of these drops could soften your cornea and damage your eye."

Which of the following statements by a nurse would nurture spirituality by promoting love and relatedness?

"Tell me about how you get along with others."

Androgens

(adrenal sex hormones/sex steroids) -the 3rdmajor type of steroid hormones produced by the adrenal cortex - Not essential to life - Play some role in female adolescent - The adrenal gland may also secrete small amounts of estrogens (female hormones) - ACTH controls the secretion of adrenal androgens

GH Deficit in Adults

- Decreased body mass with structure - Onset noted in either childhood or adulthood - Skin thin and wrinkled - Insomnia /sleep pattern disturbances - Increased body fat/elevated cholesterol and LDH level - Lack of energy with decreased muscle strength - Depression - Ostopenia

Aldosterone Replacement

- Need to increase salt intake - Fludrocortisone acetate (florinef) -0.5-2 mg po once daily - Desoxycorticosterone acetate (DOCA) . 1-2 mg IM daily . Subcutaneous implants -last 9 to 12 months

Iatrogenic Cushing's Syndrome

- Treatment with glucocorticoids for conditions other than hormone deficit

Serum prolactin

- crucial to fast at least 3-4 hrs after awakening - Normal <20 ng/ml (non-lactating) and >200 ng/ml indicate prolactin-secreting tumor

TSH

- secreted by anterior pituitary - stimulates synthesis and secretion of the thyroid hormone/regulates the thyroid gland

Oxytocin

- secreted by posterior pituitary - induces the contraction of the smooth muscles from the reproductive organs in women - stimulates the myometrium of the uterus to contract during labor - stimulates the milk releasing reflex while breastfeeding

ADH/Vasopressin

- secreted by posterior pituitary - inhibits urine production by causing renal tubules to reabsorb water from the urine and return it to the circulatory blood

A patient is receiving 3% NaCl solution for correction of hypoatremia. During administration of the solution, the most important assessment for the nurse is to monitor is: a) Lung sounds c) Peripheral pulses d) Peripheral edema

...

med to treat acromegaly

...

The telemetry nurse is notified that the unit is receiving a new admission from the medical surgical unit. Which client currently on the telemetry unit should the nurse suggest be sent to the medical surgical unit? 1. Client with magnesium level 1.6 mg/dL (0.66 mmol/L). 2. Client scheduled for cardiac catheterization the next morning. 3. Client with digoxin level 2.4 ng/mL (3.1 nmol/L). 4. Client who reported chest discomfort during cardiac stress test.

1) CORRECT — This client is stable and can be moved to the medical surgical unit, as the magnesium level is within normal limits (1.3 to 2.3 mg/dL [0.53 to 0.95 mmol/L]). 2) This client is not stable. The client's cardiac rhythm should be monitored until the results of cardiac catheterization are known. 3) This client should remain on the telemetry unit, as the client may experience symptoms of digoxin toxicity. The normal digoxin level is 0.5 to 2.0 ng/mL (0.6 to 2.6 nmol/L). 4) Chest discomfort during a cardiac stress test indicates poor cardiovascular response to increased workload. This client is not stable and should remain on the telemetry unit.

A client is POD1 after a TURP. What amount should the nurse record as the client's total output during a shift given the following information? Intake: 4 oz OJ, 6 oz tea, IV med 50 ml, 100 ml water, 8 oz clear broth, 800 ml IV infusion, bladder irrigation 1,700 ml Output: 120 ml emesis, 40 ml drainage from JP drain, 2,550 ml

1,010 ml

Which of the following would indicate that a client has developed water intoxication secondary to treatment for diabetes insipidus? 1. Confusion and seizures 2. Sunken eyeballs and spasticity 3. Flaccidity and thirst 4. Tetany and increased blood urea nitrogen (BUN) levels.

1. Confusion and seizures Classic signs of water intoxication include confusion and seizures, both of which are caused by cerebral edema. Weight gain will also occur. Sunken eyeballs, thirst, and increased BUN levels indicate fluid volume deficit. Spasticity, flaccidity, and tetany are unrelated to water intoxication.

Following a unilateral adrenalectomy, the nurse would assess for hyperkalemia as indicated by: 1. muscle weakness. 2. tremors. 3. diaphoresis. 4. constipation.

1. muscle weakness. Muscle weakness, bradycardia, nausea, diarrhea, and paresthesia of the hands, feet, tongue, and face are findings associated with hyperkalemia, which is transient and occurs from transient hypoaldosteronism when the adenoma is removed. Tremors, diaphoresis, and constipation aren't seen in hyperkalemia.

After receiving report, a nurse should plan to assess the clients in which priority order? 1. 8 mo old receiving hydration therapy via PIV 2. 8 y/o receiving continuous chemotherapy via cental venous catheter 3. 15 y/o awaiting discharge instructions receiving prednisone therapy 4. 10 y/o 3 days postop indwelling urinary catheter

2, 1, 4, 3

The nurse understands that for the parathyroid hormone to exert its effect, what must be present? 1. Decreased phosphate level 2. Adequate vitamin D level 3. Functioning thyroid gland 4. Increased calcium level

2. Adequate vitamin D level Adequate vitamin D must be present for parathyroid hormone to exert its effect — that is to help regulate calcium metabolism. Vitamin D promotes calcium absorption from the intestines.

Normal Range for HCO3:

21-28 mEq/L

A client is seen in the clinic with a possible parathormone deficiency. Diagnosis of this condition includes the analysis of serum electrolytes. Which electrolytes would the nurse expect to be abnormal? 1. Sodium 2. Potassium 3. Calcium 4. Chloride 5. Glucose 6. Phosphorous

3. Calcium 6. Phosphorous A client with a parathormone deficiency has abnormal calcium and phosphorous values because parathormone regulates these two electrolytes. Potassium, chloride, sodium, and glucose aren't affected by a parathormone deficiency.

Normal Range for: PaCO2

35-45 mm Hg

Reviewing lab results of a toddler with hemophilia A. Which aPTT values should the nurse expect?

45 seconds

After establishing a sterile field to insert a foley, the nurse must don sterile gloves. Place the following actions in the proper sequence. 1. Interlock fingers to fit gloves onto each finger 2. Pull glove over dominant hand 3. Slip glove onto nondominant hand 4. Slip fingers of dominant hand under folded, sterile side of cuff of glove for nondominant hand 5. With nondominant hand, grasp inside of the dominant hand glove by inside of cuff

5, 2, 4, 3, 1

The nurse should recognize the greatest risk for the development of blindness in which of the following patients?

A 58-year-old Caucasian woman with macular degeneration

While changing a patient's dressing the nurse notes thick yellow-green drainage on the gauze. How should the nurse document this wounds drainage: a) Purulent b) Serous

A) Purulent

The clinic nurse sees the client today and asks about his chief complaint. The client describes to the nurse his inability to attain an erection. Which of the following would be a priority for the nurse to assess? Choose all that apply. A Medication history B Specifics about sexual problem C Sleep history D Physical activity history E History of diabetes

A, B, E

A client reports a "racing" heart, restlessness, and anxiety. BP 140/68, RR 32. The nurse should recognize which finding may explain the cardiac rhythm (sinus tachycardia)? A. Anemia B. Carotid massage C. DM D. Valsalva maneuvers

A. Anemia

A client who is recently dx with stage 4 lung cancer tells the nurse, "I want to know more about advance directives." The nurse recognizes which topics should be discussed? Select all that apply A. Living will B. Use of antibiotics C. Durable power of attorney D. Initiation of diagnostic assessments E. Provision of nutrition by natural means F. Prescription for DNR

A. Living will B. Use of antibiotics C. Durable power of attorney D. Initiation of diagnostic assessments F. Prescription for DNR

A nurse assesses a 3 mo infant and observes the anterior fontanel is closed. What action should the nurse take? A. Measure head circumference B. Review recent lab results C. Evaluate hourly urinary output D. Obtain axillary temperature

A. Measure head circumference

*78 y/o hypotyhroid, lethargy, depression - what would you check?

AMS or CV function

A client has an abrupt onset of a cluster of global changes in attention, cognition, and level of consciousness. What would be the most appropriate nursing diagnosis?

Acute Confusion

Which of the following group of terms best defines spiritual distress?

Alienation, despair

Which of the following are physical changes that occur in middle adulthood? Select all that apply. A) Body fat is redistributed. B) The skin is more elastic. C) Cardiac output begins to increase. D) Muscle mass gradually decreases. E) There is a loss of calcium from bones.

Ans: A, D, E

A nurse documents the following assessment on an older adult client's chart: "dry, thin skin." Which of the following nursing diagnoses would be appropriate for this client? A) Risk for falls B) Risk for imbalanced body temperature C) Risk for infection D) Risk for sedentary lifestyle

Ans: C

The parents of an infant are members of a faith-healing group. They refuse to give the baby antibiotics for meningitis. What does the American Academy of Pediatrics recommend for cases such as this?

Application of child abuse and neglect statutes

Providing teaching to client with peripheral arterial disease. Indications of understanding

Applies lubricating lotion to her feet to prevent cracked skin. Store the medication in the refrigerator.

In general, what is the focus of care for nurses who work with older adults?

Assisting clients to function as independently as possible

Which of the following nursing diagnosis pertains to a client's learning needs: b) Altered health maintenance related to knowledge deficit: catheter care d) Anxiety related to wife's illness

B) Altered health maintenance related to knowledge deficit: catheter care

How should the nurse position a client who is complaining of dyspnea: a) A high fowler's position with two pillows behind the head b) Orthopneic position across the over bed table

B) Orthopneic position across the over bed table

Which nursing diagnoses would the nurse use for a client prone to falls: a) Deficient knowledge b) Risk for Injury c) Risk for disuse syndrome d) Risk for suffocation

B) Risk for Injury

A client is admitted to the hospital after vomiting for 3 days. Which arterial blood gas results would the nurse expect to find in this patient: a) pH 7.30, PaCO2 50, HCO3 27 b) pH 7.47, PaCO2 43, HCO3 28 c) pH 7.43, PaCO2 50, HCO3 28

B) pH 7.47, PaCO2 43, HCO3 28

A client receives 1 unit of PRBCs. The nurse should recognize which of the following values as an expected outcome of the transfusion therapy? Pre-transfusion lab values: 2.7 million RBC, HCT 23.8%, Hgb 7.8, Ca 9.3, K 4.2 A. Hct 30% B. Hgb 8.8 C. Calcium 10.2 D. Potassium 3.5

B. Hgb 8.8

This is the nurse's third day of caring for a client who recently underwent a colectomy that resulted in a colostomy. As the nurse goes about the physical assessment, a discussion about spirituality develops. The client tells the nurse that his religion believes in a basic harmony between religion and science. You interpret this as which of the following groups?

Baha'i International Community

Formulating nursing diagnoses and client strengths is a joint function of: c) Nurse and client d) Physician and client

C) Nurse and client

When an older adult client dies from complications of a CVA, the client's partner is present at the bedside. Which of the following nursing actions should the nurse take? A. Escort partner to hallway outside room B. Ask chaplain to come be with partner C. Stay with partner at bedside D. Give partner time alone

C. Stay with partner at bedside

A nurse educates adults in preventive measures to avoid problems of middle adult years. Which of the following are the major health problems during the middle adult years?

Cardiovascular disease, cancer

A young woman has been diagnosed with human papilloma virus (HPV). As a result, she will be at increased risk for which of the following?

Cervical cancer

Caring for a multiparous client following a vaccum assisted birth. Assess for which possible complication.

Cervical laceration

Caring for a client postop after receiving moderate conscious sedation suddenly becomes restless and reports feeling lightheaded

Check the client's oxygen saturation level

Serous:

Clear

What factor is necessary to express and experience spirituality?

Connectedness with other people

A nurse walks by a client's room and observes a Shaman performing a healing ritual for the client. The nurse then remarks to a coworker that the ritual is a waste of time and disruptive to the other clients on the floor. What feelings is this nurse displaying?

Culture conflict

When assessing the patient who has a lower urinary infections (UTI), the nurse will initially ask about: c) Poor urine output d) Pain with urination

D) Pain with urination

A patient with protein calorie malnutrition who has had abdominal surgery is receiving potential nutrition (PN). Which assessment information obtained by the nurse is the best indicator that the patient is receiving adequate nutrition: a) Blood glucose is 110 m/dL b) Serum albumin level is 3.5 mg/dL d) Surgical incision is healing normally

D) Surgical incision is healing normally

A nurse observes a client's cardiac monitor change from NSR → VT. What is priority 1? A. Apply oxygen B. Administer amiodarone C. Prepare for defib D. Assess LOC

D. Assess LOC

POST OP COMPLICATIONS HYPOPHYSECTOMY

DIABETES INSIPIDUS

Black:

Debride

Admitting a client who has pneumonia. Should initiate which type of isolation precautions?

Droplet

When conducting a class on sexuality with teenagers, the nurse includes that sexuality is which of the following?

How one experiences maleness or femaleness physically, emotionally, and mentally

A client who has been using benzodiazepines for anxiety wants to add an alternative therapy. The nurse suggests biofeedback. What is the best description of biofeedback?

It is a way to concentrate on the body's response during a stressful situation.

According to the free radical theory of aging, what substance is affected by aging and causes damage?

Lipids

The nurse is conducting a class on human sexual response. The participants have understood the education when they identify that, during the excitement phase of the sexual response, the man may experience what?

Nipple erection

Teaching about lithium to client who has bipolar disorder

Notify your provider if your experience increarsed thirst

The nurse is assessing a patient with multiple sclerosis who is demonstrating involuntary, rhythmic eye movements. What term will the nurse use when documenting these eye movements?

Nystagmus

Which of the following vision deficits is a clinician justified in attributing to the normal aging process?

Presbyopia

The parents of the 6-month-old bring the infant to the pediatrician's office for a routine immunization. The nurse is to administer the immunization by intramuscular (IM) injection. The nurse recognizes that which of the following is the preferred site for an IM injection in an infant? 1. Deltoid. 2. Vastus lateralis. 3. Dorsogluteal. 4. Ventrogluteal.

Question: Where should you give an IM injection to a 6-month-old child? Strategy: Think about each site. What is the size of the muscle? Are there nerves and blood vessels in the area? Needed Info: To determine where to give injection, consider: amount and type of med, size and condition of muscle, and the ability to access site. Inject up to 0.5 mL in infant and 1 mL in child. (1) small muscle mass; radial nerve near (2) CORRECT - no blood vessels or nerves; easily accessible (3) not used until walking (about 1 year) (4) not used until walking (about 1 year).

At the advice of the physician, the client with hypertension attends classes to help quit smoking. One month later when the client visits the clinic, the nurse notes a package of cigarettes in the client's pocket. Which of the following statements, if made by the nurse, is MOST appropriate? 1. "I see that you have cigarettes in your pocket." 2. "Please give me the cigarettes." 3. "I will have to report this to the physician." 4. "You will have to enroll in another class."

Question: Which response is most therapeutic? Strategy: "MOST appropriate" indicates that discrimination is required to answer the question. Needed Info: Therapeutic communication is listening to and understanding the client while promoting clarification and insight; important for nurse to understand the client's verbal and nonverbal messages, listen for client's perception of the problem, and facilitate verbalization. (1) CORRECT - encourages client to verbalize issues and concerns; nonjudgmental (2) authoritarian (3) no need to involve physician at this point (4) need more information before determining a course of action.

The patient receiving paroxetine for obsessive-compulsive disorder tells the nurse that there is dizziness when standing up from a sitting or lying position. The nurse should recognize that this problem is PRIMARILY due to which of the following? 1. Paroxetine can cause hypoglycemia. 2. Paroxetine can affect the cerebellum. 3. Paroxetine can affect the vestibular branch of the auditory nerve. 4. Paroxetine can cause orthostatic hypotension.

Question: Why does paroxetine cause dizziness? Strategy: Think about each answer choice and its relationship to dizziness. Needed Info: Paroxetine (Paxil) is a selective serotonin reuptake inhibitor (SSRI) type of antidepressant. Effects felt in 1 - 4 weeks. Not addictive. Sudden discontinuation could lead to withdrawal symptoms. (1) inaccurate; does not change glucose metabolism (2) inaccurate (3) inaccurate (4) CORRECT - sudden drop in BP with change in position from sitting or lying to standing

Healing Touch:

Realign energy flow

Enters a client's room and sees smoke coming from a small fire in the trash can. Action to take first.

Remove the client from the room

A patient is being discharged home after mastoid surgery. What topic should the nurse address in the patient's discharge education?

Safe use of analgesics and antivertiginous agents

Parents of an infant express concern because the infant is touching his genitals. What should the nurse teach the parents?

Self-manipulation of genitals is normal behavior in an infant.

Teaching to school age child with asthma using albuterol dose inhaler instructions

Take the medication 15 min before playing sports

Which of the following assessment findings of a male client age 77 years should signal the nurse to a potentially pathologic finding, rather than a normal age-related change?

The client is oriented to person and place but is unsure of the month.

Which behavior suggests that a client has obtained relief from urticaria?

The client no longer scratches his arms.

A 5-year-old child is not gaining weight appropriately. Organic problems have been ruled out. What is the priority action by the nurse? a. Allow the child unlimited access to the sippy cup to ensure adequate hydration. b. Encourage sweets for the extra caloric content. c. Teach the mother about nutritional needs of the preschooler. d. Assess the child's usual intake pattern at home.

The correct response is D. The nurse must first assess the child's current intake to determine if there is a deficiency.

Characteristic normal urine:

Transparent

Caring for a client with a prescription for continuous passive motion machine following total knee arthroplasty

Turn off the CPM machine during mealtime.

Caring for a client reporting nausea and vomiting the past 2 days

Urine specific gravity 1.052

Caring for a newborn immediately after delivery. Which interventions should be implemented to prevent heat loss by conduction?

Use a protective cover on the scale when weighting the infant.

Providing teaching to parents of child with autism spectrum disorder

Use a reward system to modify the child's behavior

Client receiving heparin continuous IV infusion and warfarin 5 mg PO daily. Lab values are aPTT 98 seconds and INR 1.8

Withhold the heparin infusion

Client with major depressive disorder has signed informed consetn for ECT. Client states I'm not sure about this now I'm afraid it's too risky

You have the right to refuse to have the ECT even after you have agreed to it

Assessing a client whose partneer recently died. States I don't know what to do without my partner. Life is not worth living

You seem to be having a difficulty time right now

Conducting visual acuity testing using the Snellen letter chart for a school age child with eyeglasses

You should keep both eyes open during the testing.

To reduce shearing force for a bedridden client. It is most important for the nurse to: A) Put bed in high Fowler's position b) Pull the client up in at least once an hour

a) Put bed in high Fowler's position

DIABETES INSIPIDUS

a. Hyposecretion of ADH caused by stroke or trauma, or may be idiopathic b. Kidney tubules fail to reabsorb water

A client complains of severe abdominal pain. To elicit as much information as possible about the pain, the nurse should ask: a) "Do you have the pain all the time?" b) "Can you describe the pain?" c) "Where does it hurt the most?" d) "Is the pain stabbing like a knife?"

b - Asking an open-ended question such as "Can you describe the pain?" encourages the client to describe any and all aspects of the pain in his own words. The other options are likely to elicit less information because they're more specific and would limit the client's response.

A client with inflammatory bowel disease is receiving total parenteral nutrition (TPN). The basic component of the client's TPN solution is most likely to be: a) An isotonic dextrose solution. b) A hypertonic dextrose solution. c) A hypotonic dextrose solution. d) A colloidal dextrose solution.

b - The TPN solution is usually a hypertonic dextrose solution. The greater the concentration of dextrose in solution, the greater the tonicity. Hypertonic dextrose solutions are used to meet the body's calorie demands in a volume of fluid that will not overload the cardiovascular system. An isotonic dextrose solution (e.g., 5% dextrose in water) or a hypotonic dextrose solution will not provide enough calories to meet metabolic needs. Colloids are plasma expanders and blood products and are not used in TPN.

a client comes to the outpatient department complaining of vaginal discharge, dysuria, and genital irritation. Suspecting a sexually transmitted disease (STD), the physician orders diagnostic testing of the vaginal discharge. Which STD must be reported to the public health department. a. bacterial vaginitis b. gonorrhea c. genital herpes d. human papillomavirus (HPV)

b - gonorrhea must be reported to the public health department. Bacterial vaginitis, genital herpes, and HPV aren't reportable diseases

A home health nurse who sees a client with diverticulitis is evaluating teaching about dietary modifications necessary to prevent future episodes. Which statement by the client indicates effective teaching? a) "I'll increase my intake of protein during exacerbations." b) "I should increase my intake of fresh fruits and vegetables during remissions." c) "I'll snack on nuts, olives, and popcorn during flare-ups." d) "I'll incorporate foods rich in omega-3 fatty acids into my diet."

b) CORRECT ANSWER "I should increase my intake of fresh fruits and vegetables during remissions." Reason: A client with diverticulitis needs to modify fiber intake to effectively manage the disease. During episodes of diverticulitis, he should follow a low-fiber diet to help minimize bulk in the stools. A client with diverticulosis should follow a high-fiber diet. Clients with diverticular disease don't need to modify their intake of protein and omega-3 fatty acids.

When caring for a client who's being treated for hyperthyroidism, it's important to:

balance the client's periods of activity and rest.

*MD must be informed when a patient with cushings develops

bilateral lung crackles

Thioamides

block synthesis of T3 and T4

A nurse is conducting an initial assessment on a client with possible tuberculosis. Which assessment finding indicates a risk factor for tuberculosis? a) The client sees his physician for a check-up yearly. b) The client has never traveled outside of the country. c) The client had a liver transplant 2 years ago. d) The client works in a health care insurance office.

c - A history of immunocompromised status, such as that which occurs with liver transplantation, places the client at a higher risk for contracting tuberculosis. Other risk factors include inadequate health care, traveling to countries with high rates of tuberculosis (such as southeastern Asia, Africa, and Latin America), being a health care worker who performs procedures in which exposure to respiratory secretions is likely, and being institutionalized.

At which age does a child begin to accept that he or she will someday die: c) 9-12 years old d) 12-18 years old

c) 9-12 years old

Which of the following interventions would be most appropriate for the nurse to recommend to a client to decrease discomfort from hemorrhoids? a) Decrease fiber in the diet. b) Take laxatives to promote bowel movements. c) Use warm sitz baths. d) Decrease physical activity.

c) CORRECT ANSWER Use warm sitz baths. Reason: Use of warm sitz baths can help relieve the rectal discomfort of hemorrhoids. Fiber in the diet should be increased to promote regular bowel movements. Laxatives are irritating and should be avoided. Decreasing physical activity will not decrease discomfort.

anthrax first line treatment

cipro

A nurse is providing care to an older adult at home after major abdominal surgery. Which of the following nursing diagnoses would most likely be appropriate? a Adult Failure to Thrive b Anticipatory Grieving c Impaired Memory d Risk for Infection

d Risk for Infection

A nurse is caring for a client diagnosed with a cerebral aneurysm who reports a severe headache. Which action should the nurse perform? a) Sit with the client for a few minutes. b) Administer an analgesic. c) Inform the nurse manager. d) Call the physician immediately.

d) CORRECT ANSWER Call the physician immediately. Reason: The nurse should notify the physician immediately because the headache may be an indication that the aneurysm is leaking. Sitting with the client is appropriate but only after the physician has been notified of the change in the client's condition. The physician will decide whether or not administration of an analgesic is indicated. Informing the nurse manager isn't necessary.

Which of the following are considered defense mechanisms: b) denial c) Sublimation

d) denial

Sheehan's Syndrome

damage to a woman's pituitary gland caused by severe bleeding or extreme low BP during child birth

The nurse is performing a respiratory assessment on a client who has a pleural effusion. The nurse would expect that the client has:

decreased breath sounds on the affected side.

*prolactin excess in male - major complaint?

decreased libido and impotence

*nursing dx for patient with DI

disturbed sleep pattern r/t nocturia

T3

fast and short acting

*s&s hypocalcemia

loss of sensation in hands/legs - involuntary muscle spasms?

*PTU adverse effect

low WBC agranulocytosis

Assessing a client who has COPD

pH 7.31. Respiratory acidosis is an expected finding for a client who has COPD

When caring for the client who is receiving an aminoglycoside antibiotic, the nurse should monitor which laboratory value?

serum creatinine

The nurse is developing a teaching plan for a client who must undergo an above-the-knee amputation of the left leg. After a leg amputation, exercise of the remaining limb:

should begin the day after surgery.

hair, skin, nails s&s hyperthyroid

skin is flushed, soft and may feel warm and moist and to the touch due to excessive perspiration; occasionally raised and thickened over the shins, back of feet The nails margins are irregular, may grow more rapidly and separate from the nail bed

The nurse is teaching a client about using vaginal medications. The nurse should instruct the client to:

use only a water-soluble lubricant when inserting a suppository.

*post hypophysectomy for pituitary adenoma - patient will not be able to...

use toothbrush - brush teeth for several days

The nurse cares for the multipara who comes to the hospital at 29 weeks gestation with reports of backache and pelvic pressure "on and off all day." Which of the following assessments, if made by the nurse, is MOST important in determining if the patient is in premature labor? 1. The patient's history of her subjective symptoms. 2. The cervix is 50% effaced, 1 centimeter dilated. 3. The presenting part is at -1 station. 4. Regular contractions are noted on a monitor tracing.

Question: How can you tell if the patient is in true labor? Strategy: Establish priorities. When a question includes words such as "MOST important" or "FIRST," this indicates that more than one answer choice may be correct but one choice is more important than the others. Narrow the answer choices to those that are correct and then put them in order of priority. The highest priority will be the correct answer. Needed Info: Effacement: shortening and thinning of the cervix. Dilation: enlargement of opening of cervix from a few mm to an opening large enough to allow for passage of infant. Station: indicates progress of labor; relationship of presenting fetal part to imaginary line between ischial spines of pelvis in the mother. S/S premature labor: abdominal pain resembling menstrual cramps, dull backache, pelvic pressure. (1) not most important information; must differentiate regular contractions of labor from Braxton-Hicks contractions (do not occur on a regular basis, do not cause cervical dilation) (2) common occurrence especially in multiparas (women who have carried 2 or more pregnancies to viability) (3) presenting part is 1 cm above ischial spines of the pelvis; does not indicate true labor (4) CORRECT - most important; if contractions are regular (occur at least every 10 min for 1 hour), would indicate premature labor

The nurse is caring for a client with end-stage heart failure. Which statement by the client best demonstrates understanding of an advanced directive?

"A living will allows my decisions for health care to be known if I'm not able to speak for myself."

A patient who presents for an eye examination is diagnosed as having a visual acuity of 20/40. The patient asks the nurse what these numbers specifically mean. What is a correct response by the nurse?

"A person whose vision is 20/40 can see an object from 20 feet away that a person with 20/20 vision can see from 40 feet away."

A client preparing to undergo a lumbar puncture states he doesn't think he will be able to get comfortable with his knees drawn up to his abdomen and his chin touching his chest. He asks if he can lie on his left side. Which statement is the best response by the nurse?

"Although the required position may not be comfortable, it will make the procedure safer and easier to perform."

How would a nurse document the condition in which a client has a normal state of awareness?

"Aware of self and environment, responsive, well-oriented."

After explaining to a primiparous client about the causes of her neonate's cranial molding, which statement by the mother indicates the need for further instruction?

"Brain damage may occur if the molding does not resolve quickly." Caput succedaneum is common after the use of a vacuum extractor to assist the client's expulsion efforts. This edema may persist up to 7 days. Vacuum extraction is not associated with cephalohematoma. Maternal lacerations may occur, but they are more common when forceps are used. Neonatal intracranial hemorrhage is a risk with both vacuum extraction and forceps births, but it is not a common finding.

A 10-month-old child with recurrent otitis media is brought to the clinic for evaluation. To help determine the cause of the child's condition, the nurse should ask the parents:

"Do you give the baby a bottle to take to bed?" In a young child, the eustachian tube is relatively short, wide, and horizontal, promoting drainage of secretions from the nasopharynx into the middle ear. Therefore, asking if the child takes a bottle to bed is appropriate because drinking while lying down may cause fluids to pool in the pharyngeal cavity, increasing the risk of otitis media.

Which discharge instruction should a nurse give a client who's had surgery to repair a hip fracture?

"Don't flex your hip more than 90 degrees, don't cross your legs, and have someone help you put your shoes on."

Which instruction should a nurse give a client with prostatitis who is receiving co-trimoxazole double strength?

"Drink 6 to 8 glasses of fluid daily while taking this medication." The client must drink 6 to 8 glasses of fluid daily to prevent renal problems, such as crystalluria and stone formation. If the drug is effective, symptoms should improve within a few days. Sore throat and sore mouth are adverse effects; the client should report them to a physician right away. The drug causes photosensitivity, but the client should use a PABA-free sunscreen; PABA can interfere with the drug's action.

The 17-year-old client with a diagnosis of bulimia nervosa is hospitalized. The client weighs 5 lb (2.26 kg) less than her ideal weight for her height. She tells the nurse, "I do not have a problem. I am not really underweight." The nurse should respond by saying:

"Even though your weight is almost ideal for your height, purging and using laxatives are harmful to your body."

A nurse assessing the heart rate and rhythm of an 8-year-old child hears a murmur that's barely audible even in a quiet room. The child's heart rate is 80 beats/minute. The nurse should document her assessment findings as:

"Heart rate regular, grade I murmur auscultated." A grade I murmur is barely audible in a quiet room; a grade II murmur is faint but clearly audible.

What statement or question is useful for a nurse if a client asks the nurse to pray with him or her?

"How would you like us to pray?"

The nurse cares for a middle-aged client with a below-the-knee amputation. What statement indicates the need for further assessment of the client's body image?

"I hope I can handle having a prosthesis, but I am really wondering what my wife will think."

The nurse is placing a client on airborne precautions. The client asks the nurse to leave his door open. The best reply to this is:

"I must keep your door closed to prevent the spread of infection. I'll open the curtains so that you don't feel so closed in."

A patient is ready to be discharged home after a cataract extraction with intraocular lens implant and the nurse is reviewing signs and symptoms that need to be reported to the ophthalmologist immediately. Which of the patient's statements best demonstrates an adequate understanding?

"I need to call the doctor if I see flashing lights."

A client tells the nurse, "I am an atheist. I do not believe in God." What would be an appropriate response by the nurse?

"I respect what you choose to believe in."

The nurse is teaching a client with rheumatoid arthritis about how to manage the fatigue associated with this disease. Which statement by the client indicates she understands how to manage the fatigue?

"I schedule afternoon rest periods for myself in addition to sleeping 10 hours every night."

A nurse is educating a group of middle adults about health promotion. What statement by one of the participants indicates the need for additional education?

"I should eat a diet high in fats but low in fiber."

An elderly woman has been diagnosed with macular degeneration following a visit to an ophthalmologist. Which of the woman's following statements best demonstrates an accurate understanding of her new diagnosis?

"I suppose that this might be one of those things that happens when you get older."

A client has been diagnosed with hypothyroidism. Which statement by the client would demonstrate appropriate teaching by the nurse?

"I will increase fiber and fluids in my diet."

The public health nurse is addressing eye health and vision protection during an educational event. What statement by a participant best demonstrates an understanding of threats to vision?

"I'm certainly going to keep a close eye on my blood pressure from now on."

While shopping, a nurse meets a neighbor who asks about a friend receiving treatment at the nurse's clinic. What is the nurse's most appropriate response?

"I'm sorry, I can't disclose client information."

A patient has been diagnosed with glaucoma and the nurse is preparing health education regarding the patient's medication regimen. The patient states that she is eager to "beat this disease" and looks forward to the time that she will no longer require medication. How should the nurse best respond?

"In fact, glaucoma usually requires lifelong treatment with medications."

Frustrated by her worsening tinnitus, a 55-year-old female patient has sought care. Which of the following teaching points should the clinician provide to the patient?

"Initially, there are some changes in your diet that you should implement." "This might be a sign of a more serious neurologic problem that we will assess for."

The nurse meets with the client and his wife to discuss depression and the client's medication. Which comment by the wife would indicate that the nurse's teaching about disease process and medications has been effective?

"It is important for him to take his medication so that the depression will not return or get worse."

A birthing couple informs the nurse that they would like to have the placenta after the baby is born. What is the nurse's best response?

"Let me check about how to go about doing this."

During chemotherapy, a boy, age 10, loses his appetite. When teaching the parents about his food intake, the nurse should include which instruction?

"Let your child eat any food he wants."

The nurse is explaining to a client and the client's family about what to expect immediately after electroconvulsive therapy (ECT) treatments. Which of the following statements would indicate to the nurse that the teaching was effective?

"My family member will likely experience some confusion and disorientation after the treatment." Clients typically experience some confusion and disorientation after treatment, but this generally recovers quite quickly. Clients are not heavily sedated after treatment. Muscle soreness is rare. Clients do not have immediate benefits after treatment; the typical course of treatment is 6 to 10 treatments.

The nurse is assessing a client's gustatory function. What approach by the nurse will assist in assessing this sensation?

"Tell me if the taste on your tongue is sweet, sour, bitter, or salty."

A client with chronic schizophrenia is admitted to the hospital on an emergency detention. The client states to the nurse, "I didn't do anything wrong. I was just carrying out the orders God gave me to paint an X on the door of all sinners." Several hours after being admitted, the client wants to leave the hospital. In addition to explaining that the staff is concerned about the client's health and safety, which of the following should the nurse tell the client?

"The court has mandated that you undergo a 72-hour evaluation." Clients admitted on an emergency detention must remain hospitalized for the time allotted for the evaluation. In this case, the time is 72 hours. The 72 hours do not include weekends or holidays. If the treatment team completes the evaluation in less than the allotted time, they may decide to discharge the client or may institute further commitment procedures. Clients cannot sign themselves out of the hospital during this period. Family members also cannot authorize the client's release. A client on an emergency detention can be held involuntarily for 72 hours. An immediate detention is good for only 24 hours.

A male client tells the nurse that he does not understand why he feels the way he does when he is sexually excited. What would the nurse teach the client?

"The sexual response cycle includes excitement, plateau, orgasm, and resolution."

A 56-year-old patient has come to the clinic for his routine eye examination and is told he needs bifocals. The patient asks the nurse what change in his eyes has caused his need for bifocals. How should the nurse respond?

"There is a gradual thickening of the lens of the eye and it can limit the eye's ability for accommodation."

The staff at a long-term care facility have made minimal effort to secure a shared room for a couple in their late 80s, who have been married for several decades. The manager states, "I'm sure that bedroom activity is the last thing on their mind these days." How should the nurse best respond to the manager's characterization of sexuality in older adults?

"They might not be as active as in years past, but sexuality is still important for older people."

A nurse is educating a lawn-care worker on the risk of hearing loss. What might be recommended?

"Wear earplugs while using lawn equipment."

To maintain a therapeutic environment with a client and his family, the nurse can use communication techniques such as the clarification technique. An example of the clarification technique is:

"What do you mean when you say...?"

A woman tells a nurse, "My husband wants to have sex when I have my period. Is that safe?" What is an appropriate answer?

"Yes, there is no reason not to have sex then."

A woman age 70 years tells the nurse that she is still sexually active. How would the nurse respond?

"You can be sexually active as long as you want to be."

Glucocorticoids

(the "sugar" hormones) - have an influence on glucose metabolism - cortisol - essential to life - The prototype is hydrocortisone - increased hydrocortisone secretions result in high glucose levels/gluconeogenesis - Decreases protein synthesis, increase protein catabolism/breakdown

Upon assessment of a client admitted for dehydration, the nurse observes that the client appears restless and reports difficulty breathing. Upon auscultation of the client's lungs, the nurse notes bilateral basilar crackles. Which actions will the nurse take first? 1. Place the client on 2 L of oxygen by nasal cannula and auscultate the lungs. 2. Elevate the head of the bed and stop the IV infusion. 3. Decrease the IV flow rate and administer furosemide as prescribed. 4. Stop the IV infusion and notify the health care provider.

) Providing the client with oxygen via nasal cannula addresses breathing. However, there is another action the nurse will implement first. 2) CORRECT — Elevating the head of the bed will allow for a more open airway. This is the priority action. 3) Decreasing the IV flow rate and administering furosemide addresses circulation. However, there is another action the nurse will implement first. 4) Stopping the IV infusion addresses circulation. However, there is another action the nurse will implement first.

hormones that stimulate anterior pituitary

* Corticortropin-Releasing Hormone (CRH) * Thyrotropin-Releasing Hormone (TRH) * Growth-Releasing Hormone (GHRH) * Prolactin-Releasing Hormone (PRH)

treatment for pituitary hypofunction

* Surgical removal of tumor (hypophysectomy) through a transphenoidal approach is the usual treatment * Possible radiation depending on tumor size - may be used to deliver external beam radiation therapy right on the tumor (minimal effect on normal tissue) * Hormone replacement therapy

interventions for adrenal insufficiency

- Administer IV fluids and corticosteroids, monitor vital signs - If the adrenal gland does not regain function patient may need life long corticosteroid replacement and mineralocorticoids to prevent recurring of adrenal insufficiency

PRIMARY ALDOSTERONISM

- Aldosterone secreting tumor - patient exhibits profound alkalosis and hypokalemia - Hypertension is the most prominent and almost a universal sign of aldosterorism - Hypokalemic alkalosis may decrease serum calcium level resulting in tetany and paresthesia - - Glucose intolerance may occur leading to hyperglycemia - The urine volume is excessive, leading to polyuria - Serum by contrast becomes concentrated leading to polydipsia

GH Deficit in Children

- Congenital lack of GH - retarded bone age in x-rays - Failure to grow - GH and IGF levels noted to be low - Slow/delayed sexual development - no GH increase, results in pituitary dwarfism with GH challenge test

assessment findings for cushings

- Cortisol excess . Increased protein breakdown . Alteration in carbohydrate metabolism producing muscle wasting and osteoporosis . Abdominal pain, gastritis, ulcers . Immnunosuppression -slow healing of minor cuts and bruises, thin, fragile skin; easily traumatized, ecchymosis, strae . Mood swings/disturbances, psychosis . "Moon-faced" appearance - Aldosterone excess: . Weight gain, edema . Hypernatremia, hypokalemia . Hypertension - Sex steroid excess: . In female -acne, hirsutism,amenorrhea, breast atrophy, thinning of scalp hair . In male -gynecomastia, decreased libido

cortisol replacement

- Cortisone, hydrocotisone, prednisolone, prednisone - Taken BID -larger doses in a.m. and smaller doses in p.m. - Increase dose of therapy during stressful procedures or significant illnesss - May need to supplement dietary intake with added salt during GI losses of fluids during vomiting and diarrhea - Advise to carry an emergency kit with Solu-cortef and a syringe

TREATMENT AND NURSING MANAGEMENT OF THE PITUITARY GLAND HYPERFUNCTION

- Crucial to monitor and reduce elevated hormone levels with appropriate medications accordingly - Radiation therapy may be indicated in some cases and/or surgery - ongoing patient/family teaching on symptom management - Medication regimen and side effects - Electrolyte management - Post operative care if applicable/supportive care * Prolactin excess treatment - e.g. in pituitary tumors

diagnostics for cushings

- Elevated cortisol level in a.m. and p.m. - Metabolic alkalosis, decreased chloride - Elevated 17-OHCS - Elevated 17-KS

diagnostics Pheochromocytoma

- Elevated urinary catecholamines and vanillymandelic acid (VMA) - Regitine test: Measures BP before and after administration of regitine (phentolamine) - Immediate decrease of 35mmHg systolic and 25mmHg diastolic indicates positive test - CT scan and MRI - MIBG (metaiodobenzylguanidine): Radioactive dye concentrates in tumor only is vidualized on xray i.e this imaging test uses a radioactive substance (called a tracer) and a special scanner to find or confirm the presence of pheochromocytoma and neuroblastoma

acromegaly

- Enlargement of viscera - thyroid, pancreas, spleen, heart, kidneys without increase in height - Increased cardiac workload, htn, metabolic rate, acne, thick/course skin, glucose intolerance, hypoglycemia, diabetes - Respiratory issues such as sleep apnea, dyspnea, may lead to respiratory failure if untreated

post op teaching thyroidectomy

- Exercise - inform patient to do head and neck exercises 2 to 3 times a day as tolerated - Avoid iodine - Avoid premature use of hormone replacement - Risk for thyroid storm (thyroidectomy) - Teach s/s of hypothyroidism - Alternate exposure to hot and cold.

GH challenge

- Fasting GH level is drawn - Drug used to stimulate GH secretions - Arginine HCL, L-dopa, Insulin - Normal: The increase in GH will peak in 60 minutes - There's risk for hypoglycemia if insulin is used

DI diagnostics

- Fluid Deprivation Test - 8 to 12 hrs or until 3 to 5% of body weght is lost - person is weighed frequently - Plasma and urine osmolality studies performed...Inability for the specific gravity to increase is characteristic of DI... - ADH test - measures how much ADH is in the blood usually used in combination with other tests to determine what is causing the increase or decrease in ADH levels - The increase in urine osmolality and decreased urine output with this test is characteristic of DI

musculoskeletal s&s hypothyroid

- General muscular weakness and pain, including cramps, and stiffness - General joint pain, achiness, stiffness, known as "arthropathy" - Tendonitis in the arms and legs involves pain, tingling, weakness, achiness or numbness in the wrist, fingers or forearm - Tarsal tunnel syndrome - similar to carpal tunnel, with pain, tingling, burning and other discomfort in the arch of your foot - Carpel tunnel syndrome - which involves pain, tingling, weakness, achiness or numbness in the wrist, fingers or forearm

nursing care post hypophysectomy

- High risk for infection - monitor vital signs - Give antibiotics as ordered - Administer pain medications a ordered - Ineffective airway breathing - assess RR, rhythm, depth and effort - Give humidified oxygenation as ordered

causes of congenital hypothyroidism

- In most cases, the cause of congenital hypothyroidism is unknown. - Medication during pregnancy, such as radioactive iodine therapy - Maternal autoimmune disease - Too much iodine during pregnancy - Anatomic defect in thyroid gland

cardiac changes in hypothyroidism

- Include a decrease in cardiac output and cardiac contractility - Reduction in heart rate - Also significant changes including diastolic htn **For people with almost any type of heart disease, disorders of the thyroid gland can worsen old cardiac symptoms or cause new ones, and can accelerate the underlying heart disorder. **Thyroid disease can even cause cardiac problems in people with healthy hearts**

GH excess

- Increase growth in all tissues - Lead to gigantism in children - these children are extremely tall/excessive height (may be 7 to 8 feet tall) - Lead to acromegally in adults - result in soft tissue and bone deformities, prominent and course facial features such as nose, tongue, nose, lips, etc. - slow in progression

nursing care for SIADH

- Infuse hypertonic solution of Normal Saline (contain sodium) - e.g. 3% NS, 0.45% NS, etc. (give slowly) - May give diuretics such as lasix iv in cases with severe hyponatremia - Administer demeclocycline (nephrotoxic) for chronic SIADH as ordered - ADH inhibitors such as lithium - Fluid restriction - Strict monitoring of weight, intake and output - Monitor electrolytes - chemistry daily - Ongoing patient/family teaching - Supportive care

Acute Adrenal Crisis:Addisonian Crisis

- It is life threatening - Characterized by cyanosis and classic signs of circulatory shock such as pallor, tachypnea, hypotension, etc. - Patient may c/o headache, abdominal pain, nausea, diarrhea - Precipitated by stressors - Severe fluid loss - The stress of surgery -possibly hypophysectomy and adrenalectomy or dehydration resulting from preparation for tests or surgery may precipitate Addisonian Crisis

Radioactive Iodine Uptake (RAIU)

- Measures the rate of iodine uptake by the thyroid gland - Patient is given a tracer dose of iodine 123 - Simple test and provides reliable results - patients with hyperthyroidism exhibit a very high uptake of iodine 123 and those with hypothyroidism exhibit a very low uptake of iodine 123

Care for Acute Adrenal Crisis

- Medical emergency -restore BP, replace hormone therapy - Fluid replacement with D5 NS, albumin and whole blood for volume - Administer steroids -Solu-cortef 100-200 mg bolus IV, then 100 mg IVPB every 6 hours as ordered - Monitor for hypoglycemia -give 50% dextrose bolus IV, high carbohydrate diet - Minimize stressors STRICT BEDREST: - Provide periods of rest in a quiet environment - Minimize physical and psychological stressors such as cold exposure , overexertion as much as possible - Encourage to increase activity gradually as tolerated - Treat underlying cause - Protect from infection - Offer emotional support

diagnostics for PTH

- PTH intact - Serum calcium as well as phosphorus - The urinary phosphorus is elevated in the hypersecretion of the parathyroid hormone

Cushing's Syndrome

- Primary hyperfunction due adrenal nodules or hyperplasia - More common in women than in men

diagnostics for hyperpatathyroidism

- Primary parathyroidism - persistent elevation of serum calcium levels >10 and elevated parathyroid hormone - Decreased serum phosphorus - Increased urinary phosphorus and calcium - Xray showing bone changes

ADRENOCORTICAL INSUFFICIENCY/ADRENAL CORTICAL HYPOFUNCTION

- Reduced adrenal gland activity due to damage to the adrenal gland or lack of stimulation of the gland - Deficiency of cortisol, aldosterone and adrenal sex hormones/steroids (androgens)

treatment and nursing care for cushings

- Reduction of hormone levels - Pituitary surgery for ACTH secreting tumor - Adrenalectomy:Either both glands, 1 gland or resection/removal of the tumor (if 1 adrenal gland is removed; replacement therapy may be temporary necessary, if both glands are removed then replacement of corticosteroids will be lifelong) . Laparoscopic approach for single nodules . Convertional surgery:Risk for adrenal crisis, care similar for adrenal crisis. May require hormone replacement Drugs to Block Synthesis of Cortisol: -Metyrapone, mititane, aminoglultethamide -Will cause hormone deficit, replacement required

PTH

- Regulates serum calcium and phosphorus/tends to lower the blood phosphorus level and increases calcium - Activates vitamin D in the kidney - Stimulates osteoclastic activity thus increasing bone resorption - Stimulated by hypocalcemia and hyperphosphatemia - Increases renal excretion of phosphorus - Inhibited by high calcium levels (hypercalcemia) and low phosphorus levels (hypophosphatemia)

treatment and nursing care for hyperparathyoidism

- Resection of the tumor - parathyroidectomy - For asymptomatic patients with mildly elevated calcium levels, and normal renal function surgery may be delayed - Restore fluid and electrolyte balance - Replace fluids accordingly - Reduce calcium levels - Monitor lab values/potassium levels and assess arrhythmias - Administer lasix IV - Assist with ADL's and encourage weight bearing activity

treatment for aldosteronism

- Surgical removal/resection of the tumor - Encourage low sodium and high potassium diet -may need potassium supplements - Administer spiranolactone (aldactone) -potassium sparing diuretic - Monitor vital signs especially BP, daily weight, and strict I/O's

hair, skin, nails s&s hypothyroid

- Thickened dry skin, the face becomes expressionless and masklike, the tongue enlarges, the hands and feet enlarge in size - Reports of thinning hair leading to hair loss - The nails are weak and brittle

treatment for addisonian crisis

- Treatment is directed toward combating circulatory shock, restoring blood circulation - Lifelong hormone replacement if adrenal gland does not regain function (to prevent recurrence of adrenal insufficiency) - Never stop taking medications or skip doses - Identify other factors, stressors or illnesses that led to the acute episode

treatment and nursing care for hypoparathyroid

- Treatment is usually not with PTH replacement - Assess for signs of hypocalcemia - tetany, etc. - Offer lifetime calcium replacement - Give a high calcium and lowphosphorus diet - Give tums, phoslo, aluminum based anti acids with meals - they bind with phosphorus by preventing the body from absorbing the phosphorus from the food one eats, help to pass excess phosphorus out of the body in the stool, reducing the amount of phosphorus that gets into the blood (usually taken 5-10 minutes before meals)

thyroid hypofunction causes...

- Untreated permanent/physical retardation - Congenital lack of T3 and T4 - Cretinism (A congenital condition caused by a deficiency of thyroid hormone during prenatal development and characterized in childhood by dwarfed stature, mental retardation, dystrophy of the bones, childhood by dwarfed stature, mental retardation, dystrophy of the bones, and a low basal metabolism - also called congenital myxedema.

treatment and nursing care for Pheochromocytoma

- Very crucial to stabilize the BP and remove the tumor WATER: Increased cardiac output - Patient need intensive care and monitoring of BP and other vital signs - Administer IV nitroprusside titrated to reduce the BP or use of regitine IV - Change to dibenzyline po or regitine po once patient is stabilized - Administer propanolol (beta blocker) ACTIVITY AND REST: - Maintain bedrest with HOB elevated - Provide a quiet and semi-darkened room - Conserve energy as much as possible - Reassure patient and offer emotional support *Surgical Treatment: Important to stabilize the BP first - No atropine pre-operatively - Possible laparoscopic approach -not as invasive - BP very labile post-operatively - Life long hormone replacement therapy required if both adrenal glands are removed

Thyroid Storm (Thyrotoxicosis)

- a medical emergency condition and needs to be treated emergently; even before all confirmatory diagnostic tests are performed - a severe hypermetabolic state, hyperthemia associated with untreated or undertreated hyperthyroidism. - During thyroid storm person is critically ill and requires aggressive and supportive nursing care during and after the acute stage of illness - Individual's heart rate, blood pressure, and body temperature can soar to dangerously high levels - without prompt, aggressive treatment, thyroid storm is often fatal

corticosteroids

- given frequently to inhibit the inflammatory response to tissue injury (i.e. Produces anti-inflammatory effect) and to suppress allergic manifestations - Their side effects include the development of diabetes mellitus, osteoporosis, peptic ulcer, increased protein breakdown resulting in muscle wasting and poor wound healing and redistribution of fat - Diurnal secretion pattern (i.e. occurring or active during the daytime rather than at night - peaks early in the morning) - Mobilizes fat for energy production

assessment findings Pheochromocytoma

- hypertension - Episodic Paroxysmal attacks: Severe headache, palpitations, tachycardia, visual disturbances, tremors, anxiety,, chest, abdominal pain dizziness, diaphoresis, nausea, etc. - Attacks increase in frequency - High risk of ventricular fibrillation (VF), heart failure, stroke and even death

prolactin

- secreted by anterior pituitary - prepares female breast for milk production - at high levels suppresses gonadotropins

Melanin Stimulating Hormone (MSH)

- secreted by anterior pituitary - produces pigmented cells called melanin - increase skin pigmentation

Growth hormone (GH) or Somatotropin

- secreted by anterior pituitary - regulates body growth of bone and muscle - promotes protein synthesis and fat metabolism - increases blood sugar and stimulates glucose metabolism - secreted in response to hypoglycemia, exercise and protein depletion

FSH

- secreted by anterior pituitary - regulates the development and growth of the reproductive growth and processes - stimulates growth of ovarian Graafian follicle, ovulation, estrogen production in females and spermatogenesis in males

LH

- secreted by anterior pituitary - stimulates development of corpus luteum, release of oocyte, production of estrogen and progesterone in females - stimulates testosterone production/secretion and development of interstitial tissue of testes in males

Adrenocorticotropic Hormone (ACTH)/ Corticotropin

- secreted by anterior pituitary - stimulates the release of corticosteroids by the adrenal glands in situations of physiological stress

Epinephrine

- secreted by the adrenal medulla - increases HR, BP, respiratory rate, muscle strength, blood sugar, bronchodilation, and mental alertness. - Reduces the amount of blood going to the skin and increase blood flow to the major organs, the such as the brain, heart, GI system, and kidneys

Syndrome of Inappropriate Antidiuretic Hormone (SIADH)

- secretion includes excessive ADH secretion from the pituitary gland - Patient cannot excrete dilute urine, retains fluids and develops a sodium deficency (dilutional hyponatremia) - SIADH does not only occur in patients after brain surgery but brain trauma or even infection such as meningitis, tuberculosis, encephalitis, etc. - May also be d/t malignancies such as lung, pancreas and malignant tumors affecting other organs

DIAGNOSTICS OF THE PITUITARY GLAND

- through H/P exams - visual acuity and visual fields - X-rays of the epyphysis - CT scan and MRI to diagnose the presence or extent of pituitary tumors - GH studies: Serum Essay - fasting venous blood drawn early in the a.m. Normal: * Female - <5 ng/ml * Male - <10 ng/ml FSH, LH, ACTH, TSH

causes of primary hypothyroidism

- thyroid can't produce amount of hormone pituitary calls for - Iodine deficiency - Auto immune Hashimoto's thyroiditis - The most common cause of inadequate formation of the gland hypothyroidism - caused by the inflammation of the thyroid gland

causes of secondary hypothyroidism

- thyroid isn't being stimulated by pituitary to produce hormones - Thyroidectomy or irradiation of the thyroid gland - T4 synthesis defect

cardiac s&s of hypothyroidism

-Dyspnea on exertion and poor exercise tolerance - In people who also have heart disease dyspnea may be due to worsening heart failure -Bradycardia - heart rate is modulated by thyroid hormone with hypothyroidism the heart rate is typically 10 - 20 beats per minute slower than normal - Arteries are stiffer in hypothyroidism - causes the diastolic blood pressure to rise - Diastolic hypertension - one might think that, because a lack of thyroid hormone slows down the metabolism, people with hypothyroidism might suffer from hypotension - usually the opposite is true - Worsening of heart failure or the new onset of heart failure - Edema - can occur as a result of worsening heart failure - In addition, hypothyroidism itself can produce a type of edema called myxedema - Worsening of coronary artery disease (CAD) - The increase in LDL cholesterol (bad cholesterol) and in C-reactive protein seen with hypothyroidism can accelerate any underlying CAD hypothyroidism and suppressing nontoxic goiters

treatment for thyroid storm

-High doses of thioamides -Replace fluids -Reduce fever (no aspirin - it displaces the thyroid hormone from binding proteins; as a result, worsens the hypermetabolism) -Corticosteroids - inhibit peripheral conversion of T4 into T3 and have been shown to improve outcomes in patients with thyroid storm. - Propanolol ( beta blocker) - lowers the heart rate; hold for bradycardia

hormones that inhibit the anterior pituitary

-Prolactin-inhibiting hormone (PIH) -Somatostatin - inhibits growth hormone -TRH and TSH -Dopamine - inhibits TSH and prolactin -Antidiuretic hormone (ADH/Vasopressin)-regulate blood osmolality -Oxytocin - causes uterine contractions and milk ejection reflex in the breast

Pheochromocytoma

. Catecholamine secreting tumor -excess epinephrine and norepinephrine . Tumor is located in the medulla, abdominal cavity, or the sympathetic nervous system . Usually benign . Mostly seen in adults 40-60 years of age . Hypertensive child needs to be screened for pheochromocytoma

A patient complains of not having had a bowel movement since being admitted 2 days ago for multiple fractures of both lower legs. The patient is on bedrest and has skeletal traction. Which intervention would be the most appropriate nursing action: a) Administer an enema c) Ensure maximum fluid intake (3000 mL/day) d) Perform range of motion exercises to all extremeties

...

A patient substained several wounds on the legs caused by a fall. On the day after the injuries, the wounds appear and edematous. The nurse identifies the stage of healing of these wounds as long: a) Inflammatory b) Proliferate d) Remodeling

...

The RN should incorporate which instructions into the teaching plan for a client with a urinary diversion: b) Notify the physician if the stoma is deep pink and shiny c) Strands of blood appear in the urine d) Increase fluid intake

...

The nurse notes that a patient who was admitted with diabetic ketoacidosis has rapid, deep respirations. Which action should the nurse take: a) Notify the patient's health care provider b) Give the prescribed PRN lorazepam (ativan) c) Start the prescribed PRN oxygen at 2 to 4 L/min

...

When the body is subjected to invasion or trauma, the role of Europhiles is to: b) Release histamine into the circulation c) Produce specific antigens d) Phagocytize injurious agents

...

When the nurse assesses dyspnea in a client with congestive heart failure, she assesses for other manifestations of fluid volume excess including: b) Peripheral Edema c) Increased hematocrit level d) decreased urine output

...

Which potential potassium order is safe for the nurse to implement: a) Add 20 mEq of KCL to 1,000 mL of IV fluid b) 10 mEq KCL IV over 1-2 min d) 10 mEq KCL SQ

...

Which problem is most appropriate for the nurse to identify for the client with diarrhea: a) Alteration in skin integrity b) Chronic pain perception d) INeffective coping

...

A newborn is prescribed ampicillin 100 mg IV q12h. Available is ampicillin 125 mg/ml. how many ml should the nurse prepare to administer? Record answer using 1 decimal place.

0.8 ml

The nurse determines that a client's tracheostomy requires suctioning. Which action does the nurse take first? 1. Elevate the head of the client's bed to 90 degrees. 2. Quickly insert the suction catheter. 3. Preoxygenate the client. 4. Put on clean gloves.

1) A semi-Fowler, not high-Fowler, position is ideal for this client during tracheostomy suctioning. 2) The client requires preparation prior to inserting the suction catheter during this procedure. 3) CORRECT— In order to ensure the client does not experience hypoxia during tracheostomy suctioning, the nurse hyperoxygenates the client before and after each time the airway is entered for suctioning. 4) Sterile gloves are used for tracheostomy suctioning.

The nurse assesses clients for potential spousal abuse. The nurse is most concerned if a client makes which statement? 1. "It's my fault because I push my spouse's buttons." 2. "My spouse and I often disagree on many subjects." 3. "We have talked about divorce multiple times." 4. "I used to be so happy, but now I'm not."

1) CORRECT — Individuals who experience spousal abuse often accept blame, become compliant, and feel helpless. This client statement is concerning to the nurse. 2) This is not typical abuser/victim behavior, as the victim is often compliant. 3) This is not typical abuser/victim behavior, as the victim is often compliant. 4) The nurse should ask the client to elaborate. However, this is not the most alarming statement of those presented.

The nurse provides care for a client diagnosed with acquired immune deficiency syndrome (AIDS). The nurse performs discharge teaching with the client. The nurse determines teaching is effective if the client makes which statements? (Select all that apply.) 1. "I will contact the health care provider if my bed sheets become drenched with perspiration." 2. "It is safe to share toothbrushes with others." 3. "It is safe to not use condoms since we both have HIV." 4. "I will be cured if I take zidovudine as prescribed by my health care provider." 5. "I will not go to the fall festival."

1) CORRECT — The client diagnosed with AIDS is at an increased risk for infection. Wet bed sheets can indicate the development of tuberculosis. 2) A client with AIDS should not share a toothbrush or a razor under any circumstances. 3) Cross-infection with the partner's virus can increase severity of infection. 4) Zidovudine is an anti-retroviral medication that slows disease progression. This medication is not curative. 5) CORRECT — The client diagnosed with AIDS should be instructed to avoid large crowds, as this increases the risk of infection.

When assessing the incision of a client 2 days postoperatively, the nurse notes a shiny pink area with underlying bowel visible. Which action does the nurse implement? 1. Cover the area with sterile gauze soaked in normal saline. 2. Cleanse the wound with hydrogen peroxide and apply a sterile dressing. 3. Pack the opened area with sterile 3/4 inch gauze soaked in normal saline. 4. Apply antibacterial ointment and cover with clear adhesive dressing.

1) CORRECT — The data indicates that the client is experiencing an evisceration. Therefore, the appropriate action from the nurse is to immediately cover the site with a sterile dressing soaked with normal saline and contact the health care provider. 2) It is not appropriate for the nurse to use hydrogen peroxide in this situation. Normal saline is used. 3) This is an inappropriate action by the nurse. 4) This is an inappropriate action by the nurse.

The nurse supervisor is informed that three serious safety events occurred last month between 0730 and 0800. The last serious safety event occurred because the oncoming nursing shift did not know a client was receiving an IV insulin drip. Which is the priority action for the nurse supervisor to take? 1. Implement mandatory bedside reporting. 2. Discuss unsafe nursing practices with the local media. 3. Delay action until hospital risk manager has completed a full investigation. 4. Ask another nurse manager for suggestions.

1) CORRECT — The nurse supervisor needs to take action to ensure proper exchange of information during shift report, as it is essential for staff to have an opportunity for last-minute updates, to clarify information, or to receive information on care events or changes in a client's condition. Bedside report promotes staff accountability, intercepts errors, and allows nurses to better prioritize care. 2) This is not an appropriate action. The nurse supervisor's immediate priority is to proactively address safety concerns within the unit. 3) While the hospital risk manager should be involved, the nurse supervisor is responsible for client outcomes and ensuring proper exchange of information is being distributed during shift report. 4) While this may be an appropriate action by the nurse supervisor, it does not address the immediate safety issue.

The health care provider prescribes metoclopramide 2 mg/kg IV to be given to a client 30 minutes before the client receives cisplatin. The client asks the nurse why the metoclopramide is being given. Which response will the nurse give to the client? 1. "Metoclopramide prevents or reduces the side effects caused by cisplatin." 2. "Metoclopramide increases the effectiveness of the cisplatin." 3. "Cisplatin prevents or reduces the side effects of the metoclopramide." 4. "Cisplatin increases the effectiveness of metoclopramide."

1) CORRECT— Metoclopramide (Reglan) is prescribed to prevent or reduce the side effects (antiemetic) caused by cisplatin, an anti-neoplastic agent. Therefore, this is an accurate response by the nurse. 2) Metoclopramide does not increase the effectiveness of cisplatin. Therefore, this is not an accurate response by the nurse. 3) This is a false statement about the use of metoclopramide and cisplatin. Therefore, this is not an accurate response by the nurse. 4) This is a false statement about the use of metoclopramide and cisplatin. Therefore, this is not an accurate response by the nurse.

The nurse assesses a client diagnosed with Ménière disease. The client states, "I take my prescribed medications regularly, but I continue to have episodes of vertigo." Which response by the nurse is most important? 1. "Tell me about your diet." 2. "How are things going at work?" 3. "When was Ménière disease diagnosed?" 4. "What were the results of your last blood test?"

1) CORRECT— This statement allows the nurse to determine if there are dietary factors (food to medication interactions) that may be interfering with the action of the prescribed medication. 2) This response by the nurse does not allow for investigation into why the prescribed medications are not working as anticipated. 3) While it is important to document when the disease process was diagnosed, this information does not allow the nurse to investigate why the prescribed medications are not working as anticipated. 4) This question is too broad and does not allow the nurse to investigate the current situation experienced by the client. **Meniere's disease: a chronic disoreder of the inner ear involving sensorineural hearing loss, sever vertigo and tinnitus. Diet Mgmt: Low sodium (2000 mg/day), avoidance of etoh, nicotine and caffeine.

The nurse provides care for a client diagnosed with diastolic heart failure. The nurse observes the recent onset of the Atrial Fibrillation. Which is the most appropriate action for the nurse to take? 1. Administer digoxin 0.25 mg IV. 2. Instruct the client to take a deep breath and hold it. 3. Assess level of consciousness and orientation. 4. Auscultate posterior chest.

1) The nurse must assess before implementation in this situation. In addition, digoxin is not a first-line drug used to treat atrial fibrillation due to the risk of toxicity. 2) The Valsalva maneuver is not indicated in this situation, as it is used for supraventricular tachydysrhythmias. 3) CORRECT — Level of consciousness (LOC) and orientation are the best indicators regarding the effect of atrial fibrillation on cardiac output. A change in LOC and/or alertness is the earliest indication of poor cardiac output. Therefore, this is the priority action by the nurse. 4) This electrocardiogram strip indicates atrial fibrillation, which may contribute to left-sided heart failure. While it is appropriate to auscultate lung sounds, this is not the priority action in this situation.

The family member of a client diagnosed with a pneumothorax states, "I think something is wrong with that drainage device. It just got very noisy." The nurse observes that bubbling in the underwater seal is continuous compared to several hours ago. Which action does the nurse take first? 1. Clamp the chest tube at the insertion site. 2. Add sterile water to the underwater seal chamber. 3. Notify the health care provider. 4. Observe the connections of the drainage system.

1) The nurse must assess the system first and then assess the client. Clamping the chest tube at the insertion site is not an independent nursing action, as a health care provider prescription is required. 2) The water level should be at 2 cm. If a leak is present, continuous bubbling will still occur. This is not a priority action. 3) The nurse must perform an assessment prior to this action. If the system is not leaking, the nurse can call the health care provider to get additional prescriptions. 4) CORRECT — A leak in the drainage system can cause continuous bubbling. Therefore, the nurse should assess the equipment. This is the priority action.

The terminally ill client reports to the nurse that a do-not-resuscitate (DNR) prescription has been initiated. The client is concerned that family members do not accept this wish. Which is the best action made by the nurse? 1. Reassure the client that things will work themselves out. 2. Allow the next of kin to make final health care decisions. 3. Schedule a meeting with the client and family. 4. Contact the hospital social worker.

1) The nurse needs to proactively address the client's concerns, not provide reassurance that may not be appropriate. 2) The nurse needs to advocate for the client's wishes. There is no data indicating the client is unable or incapable of making this decision. 3) CORRECT — The client's family members need to acknowledge and understand the client's wishes. Therefore, a meeting with the client and family will open the lines of communication and allow time for questions/explanations. 4) The first action is to open lines of communication with the client and family. If a meeting to open the lines of communication is not effective, a social worker consult might be appropriate.

The nurse discusses the client's plan of care with the student nurse. The student nurse states, "I know the client is from another country, but the client could at least look at me when I'm talking. That is so rude." Which response by the nurse is best? 1. "I am sorry the client made you feel that way." 2. "The client doesn't look at me when I speak either." 3. "Eye contact may be a sign of arrogance in the client's country." 4. "I will ask the family if anything is bothering the client."

1) The nurse should clarify the client's cultural norms, as eye contact interpretation is not universal. 2) The nurse should clarify the client's cultural norms, as eye contact interpretation is not universal. 3) CORRECT — The nurse recognizes that eye contact interpretation is not universal. In north America, maintaining eye contact during conversation communicates respect and willingness to listen. In some cultures, however, maintaining eye contact is considered intrusive, threatening, or shows arrogance. 4) The nurse should clarify the client's cultural norms, as eye contact interpretation is not universal.

The nurse speaks with a client and the spouse who have been undergoing family counseling. The client's spouse states, "You never take any responsibility for the messes you always cause!" Which response by the nurse is best? 1. "Why do you say that?" 2. "Blaming is not effective." 3. "Let's focus only on the positives." 4. "When is the last time you two had a vacation?"

1) The use of "why" questions is often considered confrontational and not therapeutic. 2) CORRECT — Family members often blame others for failures, errors, or negative consequences of an action to keep focus away from themselves. This response by the nurse is both accurate and therapeutic. 3) The nurse needs to correct unhealthy communication patterns. Only focusing on the positives will not correct unhealthy communication patterns. 4) The nurse needs to correct unhealthy communication patterns. Asking the client and spouse when they had a vacation does not correct unhealthy communication patterns.

The nurse is supervising four unlicensed assistive personnel (UAP). The nurse will immediately intervene and provide assistance if which scope of practice violation is observed? 1. The UAP performs a routine blood glucose test on a client. 2. The UAP performs a point of care urine pregnancy test. 3. The UAP assists an older adult client with feeding. 4. The UAP restarts a client's IV fluids.

1) This action is not a scope of practice violation. The UAP can perform standard, unchanging procedures, such as a routine blood glucose test for a stable client. 2) This action is not a scope of practice violation. The UAP can perform standard, unchanging procedures (such as a urine pregnancy test) for stable clients. 3) This action is not a scope of practice violation. The UAP can perform standard, unchanging tasks (such as feeding a stable client). 4) CORRECT — This UAP action requires an intervention by the nurse. Intravenous (IV) line patency should be assessed by the nurse before restarting IV fluids, as assessment is not within the UAP's scope of practice.

The nurse evaluates client care assignments made by the student nurse. The nurse will intervene if the LPN/LVN is scheduled to care for which client? 1. Client who received methylprednisolone for lumbar radiculopathy. 2. Client who received racemic epinephrine for croup. 3. Client who received ketorolac for pleurisy. 4. Client who received tamsulosin for benign prostatic hyperplasia.

1) This is a stable client. Therefore, this assignment is within the LPN/LVN's scope of practice. 2) CORRECT — The nurse should care for this client, as the client will require frequent airway/breathing assessment. 3) This is a stable client. Therefore, this assignment is within the LPN/LVN's scope of practice. 4) This is a stable client. Therefore, this assignment is within the LPN/LVN's scope of practice.

The nurse makes client assignments on the medical surgical unit. The nurse assigns an LPN/LVN to a client diagnosed with localized herpes zoster. The LPN/LVN tells the nurse, "I have never had chickenpox." Which response by the nurse is most appropriate? 1. "Use standard precautions when providing care for the client." 2. "You will be fine, because the client is on airborne precautions." 3. "Your client assignment will be changed." 4. "Why are you concerned about providing care for the client?"

1) This is an incorrect statement and is not therapeutic. 2) This response trivializes the LPN/LVN's concern and is not therapeutic. 3) CORRECT— This is a true, therapeutic statement. 4) The use of "why" questions is confrontational and is not therapeutic.

The nurse completes documentation for a client and realizes the entry has been placed in the wrong client's medical record. Which action by the nurse is most appropriate? 1. Complete an incident report and place a copy in the client's medical record. 2. Draw a single line through each line of the incorrect entry and write a new note explaining what occurred. 3. Use correction fluid to delete the wrong entry and write in the space that the note was obliterated due to client confidentiality. 4. Copy the note into the correct client's record and indicate that it was erroneously put in the wrong client's record.

1) This is not an appropriate action by the nurse. An incident report is not placed in the client's medical record. 2) CORRECT— This is an appropriate action when correcting documentation in the client's medical record. 3) This is not an appropriate action by the nurse. Correction fluid is not used when a correction is needed to document in the medical record. 4) This is not an appropriate action by the nurse when care is documented in the wrong client's medical record.

The nurse provides care for a hospitalized older adult client who has a body mass index (BMI) of 16.1. Which is the priority action by the nurse? 1. Document the client's BMI. 2. Decrease caloric intake to 1200 calories per day. 3. Confer with a dietician. 4. Plan a return visit in 1 week.

1) While it is appropriate for the nurse to document the client's BMI in the medical record, this is not the priority action. 2) Individuals who have a BMI lower than 18.5 are at increased risk for problems associated with poor nutritional status. The client's daily caloric intake should be increased, not decreased. 3) CORRECT — The nurse should refer the client to a dietician for further evaluation, as a low BMI is associated with higher mortality rate among hospitalized clients. 4) This client requires prompt evaluation. Delaying treatment for one week negates the potential seriousness of client's current condition. **Normal BMI: 18.5 to 24.9**

The nurse provides care for an adolescent client reporting arm pain after a fall. The nurse notes bruising in multiple stages of healing. The nurse accesses the client's medical record and notes the client was treated twice last month for reported back pain after two separate falls. The client was treated two months ago for a perforated eardrum. Which action by the nurse is the priority? 1. Assess the client's anxiety level. 2. Use light touch to show support. 3. Contact social services. 4. Assess the client's pain level.

1) While the nurse should assess the client's anxiety level, a professional assessment of the client's situation takes priority over psychosocial nursing actions. 2) The use of touch may not be appropriate for this client, as it may make the client feel uneasy or threatened. 3) CORRECT — The adolescent client's history suggests that there may be abuse. The law mandates that the nurse report known or suspected child abuse by collaborating with social services and law enforcement. Therefore, this is the priority action. 4) The nurse should assess pain level. However, the professional assessment of the client takes priority over psychosocial issues, such as pain.

A nurse is completing the I&O record for a client during a shift. How many ml should the nurse document as the client's intake? Round to whole number Intake: 4 oz OJ, 6 oz water, ½ cup fruit flavored gelatin, 1 cup chicken broth, 400 mL .45% NaCl IV Output: 1,000 ml urine, 120 ml drainage from T-tube

1,060 mL

A client with hyperparathyroidism declines surgery and is to receive hormone replacement therapy with estrogen and progesterone. Which instruction would be most important to include in the client's teaching plan? 1. "Maintain a moderate exercise program." 2. "Rest as much as possible." 3. "Lose weight." 4. "Jog at least 2 miles per day."

1. "Maintain a moderate exercise program." A moderate exercise program will help strengthen bones and prevent the bone loss that occurs from excess parathyroid hormone. Walking or swimming provides the most beneficial exercise. Because of weakened bones, a rigorous exercise program such as jogging would be contraindicated. Weight loss might be beneficial but it isn't as important as developing a moderate exercise program.

Which important instruction concerning the administration of levothyroxine (Synthroid) should the nurse teach a client? 1. "Take the drug on an empty stomach." 2. "Take the drug with meals." 3. "Take the drug in the evening." 4. "Take the drug whenever convenient."

1. "Take the drug on an empty stomach." The nurse should instruct the client to take levothyroxine on an empty stomach (to promote regular absorption) in the morning (to help prevent insomnia and to mimic normal hormone release).

A client with Hashimoto's thyroiditis and a history of two myocardial infarctions and coronary artery disease is to receive levothyroxine (Synthroid). Because of the client's cardiac history, the nurse would expect that the client's initial dose for the thyroid replacement would be: 1. 25 g/day, initially. 2. 100 g/day, initially. 3. delayed until after thyroid surgery. 4. initiated before thyroid surgery.

1. 25 g/day, initially. Elderly clients and clients with cardiac disease should begin with low-dose levothyroxine increased at 2- to 4-week intervals until 100 g/day is reached. This slow titration prevents further cardiac stress. Younger clients would be started on the usual maintenance dose of 100 g/day. Clients with Hashimoto's thyroiditis don't require surgical intervention.

A female client who weighs 210 lb (95 kg) and has been diagnosed with hyperglycemia tells the nurse that her husband sleeps in another room because her snoring keeps him awake. The nurse notices that she has large hands and a hoarse voice. Which disorder would the nurse suspect as a possible cause of the client's hyperglycemia? 1. Acromegaly 2. Type 1 diabetes mellitus 3. Hypothyroidism 4. Deficient growth hormone

1. Acromegaly Acromegaly, which is caused by a pituitary tumor that releases excessive growth hormone, is associated with hyperglycemia, hypertension, diaphoresis, peripheral neuropathy, and joint pain. Enlarged hands and feet are related to lateral bone growth, which is seen in adults with this disorder. The accompanying soft tissue swelling causes hoarseness and often sleep apnea. Type 1 diabetes is usually seen in children, and newly diagnosed persons are usually very ill and thin. Hypothyroidism isn't associated with hyperglycemia, nor is growth hormone deficiency.

Before discharge, what should a client with Addison's disease be instructed to do when exposed to periods of stress? 1. Administer hydrocortisone I.M. 2. Drink 8 oz of fluids. 3. Perform capillary blood glucose monitoring four times daily. 4. Continue to take his usual dose of hydrocortisone.

1. Administer hydrocortisone I.M. Clients with Addison's disease and their family members should know how to administer I.M. hydrocortisone during periods of stress. It's important to keep well hydrated during stress, but the critical component in this situation is to know how and when to use I.M. hydrocortisone. Capillary blood glucose monitoring isn't indicated in this situation because the client doesn't have diabetes mellitus. Hydrocortisone replacement doesn't cause insulin resistance.

A client with a history of hypertension is diagnosed with primary hyperaldosteronism. This diagnosis indicates that the client's hypertension is caused by excessive hormone secretion from which gland? 1. Adrenal cortex 2. Pancreas 3. Adrenal medulla 4. Parathyroid

1. Adrenal cortex Excessive secretion of aldosterone in the adrenal cortex is responsible for the client's hypertension. This hormone acts on the renal tubule, where it promotes reabsorption of sodium and excretion of potassium and hydrogen ions. The pancreas mainly secretes hormones involved in fuel metabolism. The adrenal medulla secretes the catecholamines — epinephrine and norepinephrine. The parathyroids secrete parathyroid hormone.

The nurse is developing a teaching plan for a client diagnosed with diabetes insipidus. The nurse should include information about which hormone that's lacking in clients with diabetes insipidus? 1. Antidiuretic hormone (ADH) 2. Thyroid-stimulating hormone (TSH) 3. Follicle-stimulating hormone (FSH) 4. Luteinizing hormone (LH)

1. Antidiuretic hormone (ADH) ADH is the hormone clients with diabetes insipidus lack. The client's TSH, FSH, and LH levels won't be affected.

A client is diagnosed with syndrome of inappropriate antidiuretic hormone (SIADH). The nurse informs the client that the physician will prescribe diuretic therapy and restrict fluid and sodium intake to treat the disorder. If the client does not comply with the recommended treatment, which complication may arise? 1. Cerebral edema 2. Hypovolemic shock 3. Severe hyperkalemia 4. Tetany

1. Cerebral edema Noncompliance with treatment for SIADH may lead to water intoxication from fluid retention caused by excessive antidiuretic hormone. This, in turn, limits water excretion and increases the risk for cerebral edema. Hypovolemic shock results from, severe deficient fluid volume; in contrast, SIADH causes excess fluid volume. The major electrolyte disturbance in SIADH is dilutional hyponatremia, not hyperkalemia. Because SIADH doesn't alter renal function, potassium excretion remains normal; therefore, severe hyperkalemia doesn't occur. Tetany results from hypocalcemia, an electrolyte disturbance not associated with SIADH.

A client who suffered a brain injury after falling off a ladder has recently developed syndrome of inappropriate antidiuretic hormone (SIADH). What findings indicate that the treatment he's receiving for SIADH is effective? 1. Decrease in body weight 2. Rise in blood pressure and drop in heart rate 3. Absence of wheezes in the lungs 4. Increase in urine output 5. Decrease in urine osmolarity

1. Decrease in body weight 4. Increase in urine output 5. Decrease in urine osmolarity SIADH is an abnormality involving an abundance of diuretic hormone. The predominant feature is water retention with oliguria, edema, and weight gain. Successful treatment should result in weight reduction, increased urine output, and a decrease in the urine concentration (urine osmolarity).

A client is diagnosed with the syndrome of inappropriate antidiuretic hormone (SIADH). The nurse should anticipate which laboratory test result? 1. Decreased serum sodium level 2. Decreased serum creatinine level 3. Increased hematocrit 4. Increased blood urea nitrogen (BUN) level

1. Decreased serum sodium level In SIADH, the posterior pituitary gland produces excess antidiuretic hormone (vasopressin), which decreases water excretion by the kidneys. This, in turn, reduces the serum sodium level, causing hyponatremia. In SIADH, the serum creatinine level isn't affected by the client's fluid status and remains within normal limits. Typically, the hematocrit and BUN level decrease.

A 62-year-old client diagnosed with pyelonephritis and possible septicemia has had five urinary tract infections over the past 2 years. She's fatigued from lack of sleep; urinates frequently, even during the night; and has lost weight recently. Tests reveal the following: sodium level 152 mEq/L, osmolarity 340 mOsm/L, glucose level 125 mg/dl, and potassium level 3.8 mEq/L. Which nursing diagnosis is most appropriate for this client? 1. Deficient fluid volume related to inability to conserve water 2. Imbalanced nutrition: Less than body requirements related to hypermetabolic state 3. Deficient fluid volume related to osmotic diuresis induced by hypernatremia 4. Imbalanced nutrition: Less than body requirements related to catabolic effects of insulin deficiency

1. Deficient fluid volume related to inability to conserve water The client has signs and symptoms of diabetes insipidus, probably caused by the failure of her renal tubules to respond to antidiuretic hormone as a consequence of pyelonephritis. The hypernatremia is secondary to her water loss. Imbalanced nutrition related to hypermetabolic state or catabolic effect of insulin deficiency is an inappropriate nursing diagnosis for the client.

A client with Cushing's syndrome is admitted to the medical-surgical unit. During the admission assessment, the nurse notes that the client is agitated and irritable, has poor memory, reports loss of appetite, and appears disheveled. These findings are consistent with which problem? 1. Depression 2. Neuropathy 3. Hypoglycemia 4. Hyperthyroidism

1. Depression Agitation, irritability, poor memory, loss of appetite, and neglect of one's appearance may signal depression, which is common in clients with Cushing's syndrome. Neuropathy affects clients with diabetes mellitus — not Cushing's syndrome. Although hypoglycemia can cause irritability, it also produces increased appetite, rather than loss of appetite. Hyperthyroidism typically causes such signs as goiter, nervousness, heat intolerance, and weight loss despite increased appetite.

The nurse is performing an admission assessment on a client diagnosed with diabetes insipidus. Which findings should the nurse expect to note during the assessment? 1. Extreme polyuria 2. Excessive thirst 3. Elevated systolic blood pressure 4. Low urine specific gravity 5. Bradycardia 6. Elevated serum potassium level

1. Extreme polyuria 2. Excessive thirst 4. Low urine specific gravity Signs and symptoms of diabetes insipidus include an abrupt onset of extreme polyuria, excessive thirst, dry skin and mucous membranes, tachycardia, and hypotension. Diagnostic studies reveal low urine specific gravity and osmolarity and elevated serum sodium. Serum potassium levels are likely to be decreased, not increased.

Which outcome indicates that treatment of a client with diabetes insipidus has been effective? 1. Fluid intake is less than 2,500 ml/day. 2. Urine output measures more than 200 ml/hr. 3. Blood pressure is 90/50 mm Hg. 4. Heart rate is 126 beats/min.

1. Fluid intake is less than 2,500 ml/day. Diabetes insipidus is characterized by polyuria (up to 8 L/day), constant thirst, and an unusually high oral intake of fluids. Treatment with the appropriate drug should decrease both oral fluid intake and urine output. A urine output of 200 ml/hr indicates continuing polyuria. A blood pressure of 90/50 mm Hg and a heart rate of 126 beats/min indicate compensation for the continued fluid deficit, suggesting that treatment hasn't been effective.

The adrenal cortex is responsible for producing which substances? 1. Glucocorticoids and androgens 2. Catecholamines and epinephrine 3. Mineralocorticoids and catecholamines 4. Norepinephrine and epinephrine

1. Glucocorticoids and androgens The adrenal glands have two divisions, the cortex and medulla. The cortex produces three types of hormones: glucocorticoids, mineralocorticoids, and androgens. The medulla produces catecholamines — epinephrine and norepinephrine.

The nurse is caring for a client in acute addisonian crisis. Which laboratory data would the nurse expect to find? 1. Hyperkalemia 2. Reduced blood urea nitrogen (BUN) 3. Hypernatremia 4. Hyperglycemia

1. Hyperkalemia In adrenal insufficiency, the client has hyperkalemia due to reduced aldosterone secretion. BUN increases as the glomerular filtration rate is reduced. Hyponatremia is caused by reduced aldosterone secretion. Reduced cortisol secretion leads to impaired glyconeogenesis and a reduction of glycogen in the liver and muscle, causing hypoglycemia.

On the third day after a partial thyroidectomy, a client exhibits muscle twitching and hyperirritability of the nervous system. When questioned, the client reports numbness and tingling of the mouth and fingertips. Suspecting a life-threatening electrolyte disturbance, the nurse notifies the surgeon immediately. Which electrolyte disturbance most commonly follows thyroid surgery? 1. Hypocalcemia 2. Hyponatremia 3. Hyperkalemia 4. Hypermagnesemia

1. Hypocalcemia Hypocalcemia may follow thyroid surgery if the parathyroid glands were removed accidentally. Signs and symptoms of hypocalcemia may be delayed for up to 7 days after surgery. Thyroid surgery doesn't directly cause serum sodium, potassium, or magnesium abnormalities. Hyponatremia may occur if the client inadvertently received too much fluid; however, this can happen to any surgical client receiving I.V. fluid therapy, not just one recovering from thyroid surgery. Hyperkalemia and hypermagnesemia usually are associated with reduced renal excretion of potassium and magnesium, not thyroid surgery.

What does a positive Chvostek's sign indicate? 1. Hypocalcemia 2. Hyponatremia 3. Hypokalemia 4. Hypermagnesemia

1. Hypocalcemia Chvostek's sign is elicited by tapping the client's face lightly over the facial nerve, just below the temple. If the client's facial muscles twitch, it indicates hypocalcemia. Hyponatremia is indicated by weight loss, abdominal cramping, muscle weakness, headache, and postural hypotension. Hypokalemia causes paralytic ileus and muscle weakness. Clients with hypermagnesemia exhibit a loss of deep tendon reflexes, coma, or cardiac arrest.

Following a transsphenoidal hypophysectomy, the nurse should assess the client carefully for which condition? 1. Hypocortisolism 2. Hypoglycemia 3. Hyperglycemia 4. Hypercalcemia

1. Hypocortisolism The nurse should assess for hypocortisolism. Abrupt withdrawal of endogenous cortisol may lead to severe adrenal insufficiency. Steroids should be given during surgery to prevent hypocortisolism from occurring. Signs of hypocortisolism include vomiting, increased weakness, dehydration and hypotension. After the corticotropin-secreting tumor is removed, the client shouldn't be at risk for hyperglycemia. Calcium imbalance shouldn't occur in this situation.

The physician orders laboratory tests to confirm hyperthyroidism in a client with classic signs and symptoms of this disorder. Which test result would confirm the diagnosis? 1. No increase in the thyroid-stimulating hormone (TSH) level after 30 minutes during the TSH stimulation test 2. A decreased TSH level 3. An increase in the TSH level after 30 minutes during the TSH stimulation test 4. Below-normal levels of serum triiodothyronine (T3) and serum thyroxine (T4) as detected by radioimmunoassay

1. No increase in the thyroid-stimulating hormone (TSH) level after 30 minutes during the TSH stimulation test In the TSH test, failure of the TSH level to rise after 30 minutes confirms hyperthyroidism. A decreased TSH level indicates a pituitary deficiency of this hormone. Below-normal levels of T3 and T4, as detected by radioimmunoassay, signal hypothyroidism. A below-normal T4 level also occurs in malnutrition and liver disease and may result from administration of phenytoin and certain other drugs.

After undergoing a subtotal thyroidectomy, a client develops hypothyroidism. The physician prescribes levothyroxine (Levothroid), 25 mcg P.O. daily. For which condition is levothyroxine the preferred agent? 1. Primary hypothyroidism 2. Graves' disease 3. Thyrotoxicosis 4. Euthyroidism

1. Primary hypothyroidism Levothyroxine is the preferred agent to treat primary hypothyroidism and cretinism, although it also may be used to treat secondary hypothyroidism. It is contraindicated in Graves' disease and thyrotoxicosis because these conditions are forms of hyperthyroidism. Euthyroidism, a term used to describe normal thyroid function, wouldn't require any thyroid preparation.

A client is being treated for hypothyroidism. The nurse knows that thyroid replacement therapy has been inadequate when she notes which findings? 1. Prolonged QT interval on electrocardiogram 2. Tachycardia 3. Low body temperature 4. Nervousness 5. Bradycardia 6. Dry mouth

1. Prolonged QT interval on electrocardiogram 3. Low body temperature 5. Bradycardia In hypothyroidism, the body is in a hypometabolic state. Therefore, a prolonged QT interval with bradycardia and subnormal body temperature would indicate that replacement therapy was inadequate. Tachycardia, nervousness, and dry mouth are symptoms of an excessive level of thyroid hormone; these findings would indicate that the client has received an excessive dose of thyroid hormone.

A client with a history of chronic hyperparathyroidism admits to being noncompliant. Based on initial assessment findings, the nurse formulates the nursing diagnosis of Risk for injury. To complete the nursing diagnosis statement for this client, which "related-to" phrase should the nurse add? 1. Related to bone demineralization resulting in pathologic fractures 2. Related to exhaustion secondary to an accelerated metabolic rate 3. Related to edema and dry skin secondary to fluid infiltration into the interstitial spaces 4. Related to tetany secondary to a decreased serum calcium level

1. Related to bone demineralization resulting in pathologic fractures Poorly controlled hyperparathyroidism may cause an elevated serum calcium level. This, in turn, may diminish calcium stores in the bone, causing bone demineralization and setting the stage for pathologic fractures and a risk for injury. Hyperparathyroidism doesn't accelerate the metabolic rate. A decreased thyroid hormone level, not an increased parathyroid hormone level, may cause edema and dry skin secondary to fluid infiltration into the interstitial spaces. Hyperparathyroidism causes hypercalcemia, not hypocalcemia; therefore, it isn't associated with tetany.

A client is diagnosed with syndrome of inappropriate antidiuretic hormone secretion (SIADH). Laboratory results reveal serum sodium level 130 mEq/L and urine specific gravity 1.030. Which nursing intervention would help prevent complications associated with SIADH? 1. Restricting fluids to 800 ml/day 2. Administering vasopressin as ordered 3. Elevating the client's head of bed to 90 degrees 4. Restricting sodium intake to 1 gm/day

1. Restricting fluids to 800 ml/day Excessive release of antidiuretic hormone (ADH) disturbs fluid and electrolyte balance in SIADH. The excessive ADH causes an inability to excrete dilute urine, retention of free water, expansion of extracellular fluid volume, and hyponatremia. Symptomatic treatment begins with restricting fluids to 800 ml/day. Vasopressin is administered to clients with diabetes insipidus a condition in which circulating ADH is deficient. Elevating the head of the bed decreases vascular return and decreases atrial-filling pressure, which increases ADH secretion worsening the client's condition. The client's sodium is low and, therefore, shouldn't be restricted.

Which nursing diagnosis is most appropriate for a client with Addison's disease? 1. Risk for infection 2. Excessive fluid volume 3. Urinary retention 4. Hypothermia

1. Risk for infection Addison's disease decreases the production of all adrenal hormones, compromising the body's normal stress response and increasing the risk of infection. Other appropriate nursing diagnoses for a client with Addison's disease include Deficient fluid volume and Hyperthermia. Urinary retention isn't appropriate because Addison's disease causes polyuria.

Parathyroid hormone (PTH) has which effects on the kidney? 1. Stimulation of calcium reabsorption and phosphate excretion 2. Stimulation of phosphate reabsorption and calcium excretion 3. Increased absorption of vitamin D and excretion of vitamin E 4. Increased absorption of vitamin E and excretion of vitamin D

1. Stimulation of calcium reabsorption and phosphate excretion PTH stimulates the kidneys to reabsorb calcium and excrete phosphate and converts vitamin D to its active form, 1,25-dihydroxyvitamin D. PTH doesn't have a role in the metabolism of vitamin E.

The nurse is assessing a client after a thyroidectomy. The assessment reveals muscle twitching and tingling, along with numbness in the fingers, toes, and mouth area. The nurse should suspect which complication? 1. Tetany 2. Hemorrhage 3. Thyroid storm 4. Laryngeal nerve damage

1. Tetany Tetany may result if the parathyroid glands are excised or damaged during thyroid surgery. Hemorrhage is a potential complication after thyroid surgery but is characterized by tachycardia, hypotension, frequent swallowing, feelings of fullness at the incision site, choking, and bleeding. Thyroid storm is another term for severe hyperthyroidism — not a complication of thyroidectomy. Laryngeal nerve damage may occur postoperatively, but its signs include a hoarse voice and, possibly, acute airway obstruction.

A client has a serum calcium level of 7.2 mg/dl. During the physical examination, the nurse expects to assess: 1. Trousseau's sign. 2. Homans' sign. 3. Hegar's sign. 4. Goodell's sign.

1. Trousseau's sign. This client's serum calcium level indicates hypocalcemia, an electrolyte imbalance that causes Trousseau's sign (carpopedal spasm induced by inflating the blood pressure cuff above systolic pressure). Homans' sign (pain on dorsiflexion of the foot) indicates deep vein thrombosis. Hegar's sign (softening of the uterine isthmus) and Goodell's sign (cervical softening) are probable signs of pregnancy.

For a client with hyperthyroidism, treatment is most likely to include: 1. a thyroid hormone antagonist. 2. thyroid extract. 3. a synthetic thyroid hormone. 4. emollient lotions.

1. a thyroid hormone antagonist. Thyroid hormone antagonists, which block thyroid hormone synthesis, combat increased production of thyroid hormone. Treatment of hyperthyroidism also may include radioiodine therapy, which destroys some thyroid gland cells, and surgery to remove part of the thyroid gland; both treatments decrease thyroid hormone production. Thyroid extract, synthetic thyroid hormone, and emollient lotions are used to treat hypothyroidism.

When teaching a client with Cushing's syndrome about dietary changes, the nurse should instruct the client to increase intake of: 1. fresh fruits. 2. dairy products. 3. processed meats. 4. cereals and grains.

1. fresh fruits. Cushing's syndrome causes sodium retention, which increases urinary potassium loss. Therefore, the nurse should advise the client to increase intake of potassium-rich foods, such as fresh fruit. The client should restrict consumption of dairy products, processed meats, cereals, and grains because they contain significant amounts of sodium.

Hyperthyroidism is caused by increased levels of thyroxine in blood plasma. A client with this endocrine dysfunction would experience: 1. heat intolerance and systolic hypertension. 2. weight gain and heat intolerance. 3. diastolic hypertension and widened pulse pressure. 4. anorexia and hyperexcitability.

1. heat intolerance and systolic hypertension. An increased metabolic rate in a client with hyperthyroidism caused by excess serum thyroxine leads to systolic hypertension and heat intolerance. Weight loss — not gain — occurs due to the increased metabolic rate. Diastolic blood pressure decreases due to decreased peripheral resistance. Heat intolerance and widened pulse pressure can occur but systolic hypertension and diastolic hypertension don't. Clients with hyperthyroidism experience an increase in appetite — not anorexia.

For the first 72 hours after thyroidectomy surgery, the nurse would assess the client for Chvostek's sign and Trousseau's sign because they indicate: 1. hypocalcemia. 2. hypercalcemia. 3. hypokalemia. 4. hyperkalemia.

1. hypocalcemia. The client who has undergone a thyroidectomy is at risk for developing hypocalcemia from inadvertent removal or damage to the parathyroid gland. The client with hypocalcemia will exhibit a positive Chvostek's sign (facial muscle contraction when the facial nerve in front of the ear is tapped) and a positive Trousseau's sign (carpal spasm when a blood pressure cuff is inflated for a few minutes). These signs aren't present with hypercalcemia, hypokalemia, or hyperkalemia.

For a client in addisonian crisis, it would be very risky for a nurse to administer: 1. potassium chloride. 2. normal saline solution. 3. hydrocortisone. 4. fludrocortisone.

1. potassium chloride. Addisonian crisis results in hyperkalemia; therefore, administering potassium chloride is contraindicated. Because the client will be hyponatremic, normal saline solution is indicated. Hydrocortisone and fludrocortisone are both useful in replacing deficient adrenal cortex hormones.

In a 28-year-old female client who is being successfully treated for Cushing's syndrome, the nurse would expect a decline in: 1. serum glucose level. 2. hair loss. 3. bone mineralization. 4. menstrual flow.

1. serum glucose level. Hyperglycemia, which develops from glucocorticoid excess, is a manifestation of Cushing's syndrome. With successful treatment of the disorder, serum glucose levels decline. Hirsutism is common in Cushing's syndrome; therefore, with successful treatment, abnormal hair growth also declines. Osteoporosis occurs in Cushing's syndrome; therefore, with successful treatment, bone mineralization increases. Amenorrhea develops in Cushing's syndrome. With successful treatment, the client experiences a return of menstrual flow, not a decline in it.

When caring for a client with diabetes insipidus, the nurse expects to administer: 1. vasopressin (Pitressin Synthetic). 2. furosemide (Lasix). 3. regular insulin. 4. 10% dextrose.

1. vasopressin (Pitressin Synthetic). Because diabetes insipidus results from decreased antidiuretic hormone (vasopressin) production, the nurse should expect to administer synthetic vasopressin for hormone replacement therapy. Furosemide, a diuretic, is contraindicated because a client with diabetes insipidus experiences polyuria. Insulin and dextrose are used to treat diabetes mellitus and its complications, not diabetes insipidus.

The nurse is caring for a client with type 1 diabetes mellitus who exhibits confusion, light-headedness, and aberrant behavior. The client is still conscious. The nurse should first administer:

15 to 20 g of a fast-acting carbohydrate such as orange juice.

A 56-year-old female client is being discharged after undergoing a thyroidectomy. Which discharge instructions would be appropriate for this client? 1. "Report signs and symptoms of hypoglycemia." 2. "Take thyroid replacement medication as ordered." 3. "Watch for changes in body functioning, such as lethargy, restlessness, sensitivity to cold, and dry skin, and report these changes to the physician." 4. "Recognize the signs of dehydration." 5. "Carry injectable dexamethasone at all times."

2. "Take thyroid replacement medication as ordered." 3. "Watch for changes in body functioning, such as lethargy, restlessness, sensitivity to cold, and dry skin, and report these changes to the physician." After the removal of the thyroid gland, the client needs to take thyroid replacement medication. The client also needs to report such changes as lethargy, restlessness, cold sensitivity, and dry skin, which may indicate the need for a higher dosage of medication. The thyroid gland doesn't regulate blood glucose levels; therefore, signs and symptoms of hypoglycemia aren't relevant for this client. Dehydration is seen in diabetes insipidus. Injectable dexamethasone isn't needed for this client.

A client whose physical findings suggest a hyperpituitary condition undergoes an extensive diagnostic workup. Test results reveal a pituitary tumor, which necessitates a transsphenoidal hypophysectomy. The evening before the surgery, the nurse reviews preoperative and postoperative instructions given to the client earlier. Which postoperative instruction should the nurse emphasize? 1. "You must lie flat for 24 hours after surgery." 2. "You must avoid coughing, sneezing, and blowing your nose." 3. "You must restrict your fluid intake." 4. "You must report ringing in your ears immediately."

2. "You must avoid coughing, sneezing, and blowing your nose." After a transsphenoidal hypophysectomy, the client must refrain from coughing, sneezing, and blowing the nose for several days to avoid disturbing the surgical graft used to close the wound. The head of the bed must be elevated, not kept flat, to prevent tension or pressure on the suture line. Within 24 hours after a hypophysectomy, transient diabetes insipidus commonly occurs; this calls for increased, not restricted, fluid intake. Visual, not auditory, changes are a potential complication of hypophysectomy.

The nurse is assessing a client with Cushing's syndrome. Which observation should the nurse report to the physician immediately? 1. Pitting edema of the legs 2. An irregular apical pulse 3. Dry mucous membranes 4. Frequent urination

2. An irregular apical pulse Because Cushing's syndrome causes aldosterone overproduction, which increases urinary potassium loss, the disorder may lead to hypokalemia. Therefore, the nurse should immediately report signs and symptoms of hypokalemia, such as an irregular apical pulse, to the physician. Edema is an expected finding because aldosterone overproduction causes sodium and fluid retention. Dry mucous membranes and frequent urination signal dehydration, which isn't associated with Cushing's syndrome.

A businesswoman comes into the clinic with a progressively enlarging neck. The client mentions that she has been in a foreign country for the previous 3 months and that she didn't eat much while she was there because she didn't like the food. The client also mentions that she becomes dizzy when lifting her arms to do normal household chores or when dressing. What endocrine disorder would the nurse expect the physician to diagnose? 1. Diabetes mellitus 2. Goiter 3. Diabetes insipidus 4. Cushing's syndrome

2. Goiter A goiter can result from inadequate dietary intake of iodine associated with changes in foods or malnutrition. It's caused by insufficient thyroid gland production and depletion of glandular iodine. Signs and symptoms of this malfunction include enlargement of the thyroid gland, dizziness when raising the arms above the head, dysphagia, and respiratory distress. Signs and symptoms of diabetes mellitus include polydipsia, polyuria, and polyphagia. Signs and symptoms of diabetes insipidus include extreme polyuria (4 to 16 L/day) and symptoms of dehydration (poor tissue turgor, dry mucous membranes, constipation, dizziness, and hypotension). Cushing's syndrome causes buffalo hump, moon face, irritability, emotional lability, and pathologic fractures.

A middle-age female complains of anxiety, insomnia, weight loss, the inability to concentrate, and eyes feeling "gritty". Thyroid function tests reveal the following: thyroid-stimulating hormone (TSH) 0.02 U/ml, thyroxine 20 g/dl, and triiodothyronine 253 ng/dl. A 6-hr radioactive iodine uptake test showed a diffuse uptake of 85%. Based on these assessment findings, the nurse should suspect: 1. thyroiditis. 2. Graves' disease. 3. Hashimoto's thyroiditis. 4. multinodular goiter.

2. Graves' disease. Graves' disease, an autoimmune disease causing hyperthyroidism, is most prevalent in middle-age females. In Hashimoto's thyroiditis, the most common form of hypothyroidism, TSH levels would be high and thyroid hormone levels low. In thyroiditis, there is a low (≤2%) radioactive iodine uptake, and multinodular goiter will show an uptake in the high-normal range (3% to 10%).

A client with a history of Addison's disease and flulike symptoms accompanied by nausea and vomiting over the past week is brought to the facility. When he awoke this morning, his wife noticed that he acted confused and was extremely weak. The client's blood pressure is 90/58 mm Hg, his pulse is 116 beats/minute, and his temperature is 101° F (38.3° C). A diagnosis of acute adrenal insufficiency is made. What would the nurse expect to administer by I.V. infusion? 1. Insulin 2. Hydrocortisone 3. Potassium 4. Hypotonic saline

2. Hydrocortisone Emergency treatment for acute adrenal insufficiency (Addisonian crisis) is I.V. infusion of hydrocortisone and saline solution. The client is usually given a dose containing hydrocortisone 100 mg I.V. in normal saline every 6 hours until the client's blood pressure returns to normal. Insulin isn't indicated in this situation because adrenal insufficiency is usually associated with hypoglycemia. Potassium isn't indicated because these clients are usually hyperkalemic. The client needs normal — not hypotonic — saline solution.

Which condition would the nurse expect to find in a client diagnosed with hyperparathyroidism? 1. Hypocalcemia 2. Hypercalcemia 3. Hyperphosphatemia 4. Hypophosphaturia

2. Hypercalcemia Hypercalcemia is the hallmark of excess parathyroid hormone levels. Serum phosphate will be low (hyperphosphatemia), and there will be increased urinary phosphate (hyperphosphaturia) because phosphate excretion is increased.

When administering spironolactone (Aldactone) to a client who has had a unilateral adrenalectomy, the nurse should instruct the client about which possible adverse effect of the drug? 1. Constipation 2. Menstrual irregularities 3. Hypokalemia 4. Hypernatremia

2. Menstrual irregularities Spironolactone can cause menstrual irregularities and decreased libido. Men may also experience gynecomastia and impotence. Diarrhea, hyponatremia, and hyperkalemia are also adverse effects of spirolactone.

The nurse should expect a client with hypothyroidism to report which health concern(s)? 1. Increased appetite and weight loss 2. Puffiness of the face and hands 3. Nervousness and tremors 4. Thyroid gland swelling

2. Puffiness of the face and hands Hypothyroidism (myxedema) causes facial puffiness, extremity edema, and weight gain. Signs and symptoms of hyperthyroidism (Graves' disease) include an increased appetite, weight loss, nervousness, tremors, and thyroid gland enlargement (goiter).

A client is transferred to a rehabilitation center after being treated in the hospital for a stroke. Because the client has a history of Cushing's syndrome (hypercortisolism) and chronic obstructive pulmonary disease (COPD), the nurse formulates a nursing diagnosis of: 1. Risk for imbalanced fluid volume related to excessive sodium loss. 2. Risk for impaired skin integrity related to tissue catabolism secondary to cortisol hypersecretion. 3. Ineffective health maintenance related to frequent hypoglycemic episodes secondary to Cushing's syndrome. 4. Decreased cardiac output related to hypotension secondary to Cushing's syndrome.

2. Risk for impaired skin integrity related to tissue catabolism secondary to cortisol hypersecretion. Cushing's syndrome causes tissue catabolism, resulting in thinning skin and connective tissue loss; along with immobility related to stroke, these factors increase this client's risk for impaired skin integrity. The exaggerated glucocorticoid activity in Cushing's syndrome causes sodium and water retention which, in turn, leads to edema and hypertension. Therefore, Risk for imbalanced fluid volume and Decreased cardiac output are inappropriate nursing diagnoses. Increased glucocorticoid activity also causes persistent hyperglycemia, eliminating Ineffective health maintenance related to frequent hypoglycemic episodes as an appropriate nursing diagnosis.

Early this morning, a client had a subtotal thyroidectomy. During evening rounds, the nurse assesses the client, who now has nausea, a temperature of 105° F (40.5° C), tachycardia, and extreme restlessness. What is the most likely cause of these signs? 1. Diabetic ketoacidosis 2. Thyroid crisis 3. Hypoglycemia 4. Tetany

2. Thyroid crisis Thyroid crisis usually occurs in the first 12 hours after thyroidectomy and causes exaggerated signs of hyperthyroidism, such as high fever, tachycardia, and extreme restlessness. Diabetic ketoacidosis is more likely to produce polyuria, polydipsia, and polyphagia; hypoglycemia, to produce weakness, tremors, profuse perspiration, and hunger. Tetany typically causes uncontrollable muscle spasms, stridor, cyanosis, and possibly asphyxia.

A client is being returned to the room after a subtotal thyroidectomy. Which piece of equipment is most important for the nurse to keep at the client's bedside? 1. Indwelling urinary catheter kit 2. Tracheostomy set 3. Cardiac monitor 4. Humidifier

2. Tracheostomy set After a subtotal thyroidectomy, swelling of the surgical site (the tracheal area) may obstruct the airway. Therefore, the nurse should keep a tracheostomy set at the client's bedside in case of a respiratory emergency. Although an indwelling urinary catheter and a cardiac monitor may be used for a client after a thyroidectomy, the tracheostomy set is more important. A humidifier isn't indicated for this client.

The nurse is assessing a client with hyperthyroidism. What findings should the nurse expect? 1. Weight gain, constipation, and lethargy 2. Weight loss, nervousness, and tachycardia 3. Exophthalmos, diarrhea, and cold intolerance 4. Diaphoresis, fever, and decreased sweating

2. Weight loss, nervousness, and tachycardia Weight loss, nervousness, and tachycardia are signs of hyperthyroidism. Other signs of hyperthyroidism include exophthalmos, diaphoresis, fever, and diarrhea. Weight gain, constipation, lethargy, decreased sweating, and cold intolerance are signs of hypothyroidism.

A client visits the physician's office complaining of agitation, restlessness, and weight loss. The physical examination reveals exophthalmos, a classic sign of Graves' disease. Based on history and physical findings, the nurse suspects hyperthyroidism. Exophthalmos is characterized by: 1. dry, waxy swelling and abnormal mucin deposits in the skin. 2. protruding eyes and a fixed stare. 3. a wide, staggering gait. 4. more than 10 beats/minute difference between the apical and radial pulse rates.

2. protruding eyes and a fixed stare. Exophthalmos is characterized by protruding eyes and a fixed stare. Dry, waxy swelling and abnormal mucin deposits in the skin typify myxedema, a condition resulting from advanced hypothyroidism. A wide, staggering gait and a differential between the apical and radial pulse rates aren't specific signs of thyroid dysfunction.

The nurse is instructing a client with newly diagnosed hypoparathyroidism about the regimen used to treat this disorder. The nurse should state that the physician probably will prescribe daily supplements of calcium and: 1. folic acid. 2. vitamin D. 3. potassium. 4. iron.

2. vitamin D. Typically, clients with hypoparathyroidism are prescribed daily supplements of vitamin D along with calcium because calcium absorption from the small intestine depends on vitamin D. Hypoparathyroidism doesn't cause a deficiency of folic acid, potassium, or iron. Therefore, the client doesn't require daily supplements of these substances to maintain a normal serum calcium level.

A client with Addison's disease is scheduled for discharge after being hospitalized for an adrenal crisis. Which statements by the client would indicate that client teaching has been effective? 1. "I have to take my steroids for 10 days." 2. "I need to weigh myself daily to be sure I don't eat too many calories." 3. "I need to call my doctor to discuss my steroid needs before I have dental work." 4. "I will call the doctor if I suddenly feel profoundly weak or dizzy." 5. "If I feel like I have the flu, I'll carry on as usual because this is an expected response." 6. "I need to obtain and wear a Medic Alert bracelet."

3. "I need to call my doctor to discuss my steroid needs before I have dental work." 4. "I will call the doctor if I suddenly feel profoundly weak or dizzy." 6. "I need to obtain and wear a Medic Alert bracelet." Dental work can be a cause of physical stress; therefore, the client's physician needs to be informed about the dental work and an adjusted dosage of steroids may be necessary. Fatigue, weakness, and dizziness are symptoms of inadequate dosing of steroid therapy; the physician should be notified if these symptoms occur. A Medic Alert bracelet allows health care providers to access the client's history of Addison's disease if the client is unable to communicate this information. A client with Addison's disease doesn't produce enough steroids, so routine administration of steroids is a lifetime treatment. Daily weights should be monitored to monitor changes in fluid balance, not calorie intake. Influenza is an added physical stressor and the client may require an increased dosage of steroids. The client shouldn't "carry on as usual."

The nurse is teaching a client recovering from addisonian crisis about the need to take fludrocortisone acetate and hydrocortisone at home. Which statement by the client indicates an understanding of the instructions? 1. "I'll take my hydrocortisone in the late afternoon, before dinner." 2. "I'll take all of my hydrocortisone in the morning, right after I wake up." 3. "I'll take two-thirds of the dose when I wake up and one-third in the late afternoon." 4. "I'll take the entire dose at bedtime."

3. "I'll take two-thirds of the dose when I wake up and one-third in the late afternoon." Hydrocortisone, a glucocorticoid, should be administered according to a schedule that closely reflects the body's own secretion of this hormone; therefore, two-thirds of the dose of hydrocortisone should be taken in the morning and one-third in the late afternoon. This dosage schedule reduces adverse effects.

A client with primary diabetes insipidus is ready for discharge on desmopressin (DDAVP). Which instruction should the nurse provide? 1. "Administer desmopressin while the suspension is cold." 2. "Your condition isn't chronic, so you won't need to wear a medical identification bracelet." 3. "You may not be able to use desmopressin nasally if you have nasal discharge or blockage." 4. "You won't need to monitor your fluid intake and output after you start taking desmopressin."

3. "You may not be able to use desmopressin nasally if you have nasal discharge or blockage." Desmopressin may not be absorbed if the intranasal route is compromised. Although diabetes insipidus is treatable, the client should wear medical identification and carry medication at all times to alert medical personnel in an emergency and ensure proper treatment. The client must continue to monitor fluid intake and output and get adequate fluid replacement.

During the first 24 hours after a client is diagnosed with Addisonian crisis, which intervention should the nurse perform frequently? 1. Weigh the client. 2. Test urine for ketones. 3. Assess vital signs. 4. Administer oral hydrocortisone.

3. Assess vital signs. Because the client in Addisonian crisis is unstable, vital signs and fluid and electrolyte balance should be assessed every 30 minutes until he's stable. Daily weights are sufficient when assessing the client's condition. The client shouldn't have ketones in his urine, so there is no need to assess the urine for their presence. Oral hydrocortisone isn't administered during the first 24 hours in severe adrenal insufficiency.

Which of the following instructions should be included in the discharge teaching plan for a client after thyroidectomy for Graves' disease? 1. Keep an accurate record of intake and output. 2. Use nasal desmopressin acetate (DDAVP). 3. Be sure to get regular follow-up care. 4. Be sure to exercise to improve cardiovascular fitness.

3. Be sure to get regular follow-up care. Regular follow-up care for the client with Graves' disease is critical because most cases eventually result in hypothyroidism. Annual thyroid-stimulating hormone tests and the client's ability to recognize signs and symptoms of thyroid dysfunction will help detect thyroid abnormalities early. Intake and output is important for clients with fluid and electrolyte imbalances but not thyroid disorders. DDAVP is used to treat diabetes insipidus. While exercise to improve cardiovascular fitness is important, for this client the importance of regular follow-up is most critical.

A client is seen in the clinic with a possible parathormone deficiency. Diagnosis of this condition includes the analysis of serum electrolytes. Which electrolytes would the nurse expect to be abnormal? 1. Sodium 2. Potassium 3. Calcium 4. Chloride 5. Glucose 6. Phosphorous

3. Calcium 6. Phosphorous A client with a parathormone deficiency has abnormal calcium and phosphorous values because parathormone regulates these two electrolytes. Potassium, chloride, sodium, and glucose aren't affected by a parathormone deficiency.

Which of the following laboratory test results would suggest to the nurse that a client has a corticotropin-secreting pituitary adenoma? 1. High corticotropin and low cortisol levels 2. Low corticotropin and high cortisol levels 3. High corticotropin and high cortisol levels 4. Low corticotropin and low cortisol levels

3. High corticotropin and high cortisol levels A corticotropin-secreting pituitary tumor would cause high corticotropin and high cortisol levels. A high corticotropin level with a low cortisol level and a low corticotropin level with a low cortisol level would be associated with hypocortisolism. Low corticotropin and high cortisol levels would be seen if there was a primary defect in the adrenal glands.

A client has recently undergone surgical removal of a pituitary tumor. The physician prescribes corticotropin (Acthar), 20 units I.M. q.i.d., as a replacement therapy. What is the mechanism of action of corticotropin? 1. It decreases cyclic adenosine monophosphate (cAMP) production and affects the metabolic rate of target organs. 2. It interacts with plasma membrane receptors to inhibit enzymatic actions. 3. It interacts with plasma membrane receptors to produce enzymatic actions that affect protein, fat, and carbohydrate metabolism. 4. It regulates the threshold for water resorption in the kidneys.

3. It interacts with plasma membrane receptors to produce enzymatic actions that affect protein, fat, and carbohydrate metabolism. Corticotropin interacts with plasma membrane receptors to produce enzymatic actions that affect protein, fat, and carbohydrate metabolism. It doesn't decrease cAMP production. The posterior pituitary hormone, antidiuretic hormone, regulates the threshold for water resorption in the kidneys.

A client receiving thyroid replacement therapy develops the flu and forgets to take her thyroid replacement medicine. The nurse understands that skipping this medication will put the client at risk for developing which life-threatening complication? 1. Exophthalmos 2. Thyroid storm 3. Myxedema coma 4. Tibial myxedema

3. Myxedema coma Myxedema coma, severe hypothyroidism, is a life-threatening condition that may develop if thyroid replacement medication isn't taken. Exophthalmos, protrusion of the eyeballs, is seen with hyperthyroidism. Thyroid storm is life-threatening but is caused by severe hyperthyroidism. Tibial myxedema, peripheral mucinous edema involving the lower leg, is associated with hypothyroidism but isn't life-threatening.

A client is admitted for treatment of the syndrome of inappropriate antidiuretic hormone (SIADH). Which nursing intervention is appropriate? 1. Infusing I.V. fluids rapidly as ordered 2. Encouraging increased oral intake 3. Restricting fluids 4. Administering glucose-containing I.V. fluids as ordered

3. Restricting fluids To reduce water retention in a client with the SIADH, the nurse should restrict fluids. Administering fluids by any route would further increase the client's already heightened fluid load.

A client with hypothyroidism (myxedema) is receiving levothyroxine (Synthroid), 25 mcg P.O. daily. Which finding should the nurse recognize as an adverse reaction to the drug? 1. Dysuria 2. Leg cramps 3. Tachycardia 4. Blurred vision

3. Tachycardia Levothyroxine, a synthetic thyroid hormone, is given to a client with hypothyroidism to simulate the effects of thyroxine. Adverse reactions to this agent include tachycardia. The other options aren't associated with levothyroxine.

A client with hyperthyroidism is about to receive radioactive iodine as an outpatient. What safety measures should the nurse teach the client to protect his family while he undergoes treatment? 1. Good hand washing 2. How to isolate himself in one room of the house 3. Use of disposable eating utensils 4. Not worrying about precautions

3. Use of disposable eating utensils The client with hyperthyroidism can receive radioactive iodine as an outpatient with some precautions, such as using disposable eating utensils, and avoiding kissing, sexual intercourse, and holding babies. Good hand washing is always necessary to prevent the spread of infection; however, it provides no protection against radioactive iodine therapy. Isolation isn't necessary, but radiation precautions are.

Which of the following would the nurse expect to assess in an elderly client with Hashimoto's thyroiditis? 1. Weight loss, increased appetite, and hyperdefecation 2. Weight loss, increased urination, and increased thirst 3. Weight gain, decreased appetite, and constipation 4. Weight gain, increased urination, and purplish-red striae

3. Weight gain, decreased appetite, and constipation Hashimoto's thyroiditis, an autoimmune disorder, is the most common cause of hypothyroidism. It's seen most frequently in women over age 40. Weight gain, decreased appetite, constipation, lethargy, dry cool skin, brittle nails, coarse hair, muscle cramps, weakness, and sleep apnea are symptoms of Hashimoto's thyroiditis. Weight loss, increased appetite, and hyperdefecation are characteristic of hyperthyroidism. Weight loss, increased urination, and increased thirst are characteristic of uncontrolled diabetes mellitus. Weight gain, increased urination, and purplish-red striae are characteristic of hypercortisolism.

A 35-year-old female client who complains of weight gain, facial hair, absent menstruation, frequent bruising, and acne is diagnosed with Cushing's syndrome. Cushing's syndrome is most likely caused by: 1. an ectopic corticotropin-secreting tumor. 2. adrenal carcinoma. 3. a corticotropin-secreting pituitary adenoma. 4. an inborn error of metabolism.

3. a corticotropin-secreting pituitary adenoma. A corticotropin-secreting pituitary adenoma is the most common cause of Cushing's syndrome in women ages 20 to 40. Ectopic corticotropin-secreting tumors are more common in older men and are often associated with weight loss. Adrenal carcinoma isn't usually accompanied by hirsutism. A female with an inborn error of metabolism wouldn't be menstruating.

When caring for a client who's being treated for hyperthyroidism, it's important to: 1. provide extra blankets and clothing to keep the client warm. 2. monitor the client for signs of restlessness, sweating, and excessive weight loss during thyroid replacement therapy. 3. balance the client's periods of activity and rest. 4. encourage the client to be active to prevent constipation.

3. balance the client's periods of activity and rest. A client with hyperthyroidism needs to be encouraged to balance periods of activity and rest. Many clients with hyperthyroidism are hyperactive and complain of feeling very warm. Consequently, it's important to keep the environment cool and to teach the client how to manage his physical reactions to heat. Clients with hypothyroidism — not hyperthyroidism — complain of being cold and need warm clothing and blankets to maintain a comfortable temperature. They also receive thyroid replacement therapy, often feel lethargic and sluggish, and are prone to constipation. The nurse should encourage clients with hypothyroidism to be more active to prevent constipation.

The nurse is assessing a client with possible Cushing's syndrome. In a client with Cushing's syndrome, the nurse would expect to find: 1. hypotension. 2. thick, coarse skin. 3. deposits of adipose tissue in the trunk and dorsocervical area. 4. weight gain in arms and legs.

3. deposits of adipose tissue in the trunk and dorsocervical area. Because of changes in fat distribution, adipose tissue accumulates in the trunk, face (moonface), and dorsocervical areas (buffalo hump). Hypertension is caused by fluid retention. Skin becomes thin and bruises easily because of a loss of collagen. Muscle wasting causes muscle atrophy and thin extremities.

When instructing the client diagnosed with hyperparathyroidism about diet, the nurse should stress the importance of: 1. restricting fluids. 2. restricting sodium. 3. forcing fluids. 4. restricting potassium.

3. forcing fluids. The client should be encouraged to force fluids to prevent renal calculi formation. Sodium should be encouraged to replace losses in urine. Restricting potassium isn't necessary in hyperparathyroidism.

An incoherent client with a history of hypothyroidism is brought to the emergency department by the rescue squad. Physical and laboratory findings reveal hypothermia, hypoventilation, respiratory acidosis, bradycardia, hypotension, and nonpitting edema of the face and pretibial area. Knowing that these findings suggest severe hypothyroidism, the nurse prepares to take emergency action to prevent the potential complication of: 1. thyroid storm. 2. cretinism. 3. myxedema coma. 4. Hashimoto's thyroiditis.

3. myxedema coma. Severe hypothyroidism may result in myxedema coma, in which a drastic drop in the metabolic rate causes decreased vital signs, hypoventilation (possibly leading to respiratory acidosis), and nonpitting edema. Thyroid storm is an acute complication of hyperthyroidism. Cretinism is a form of hypothyroidism that occurs in infants. Hashimoto's thyroiditis is a common chronic inflammatory disease of the thyroid gland in which autoimmune factors play a prominent role.

A client is admitted to an acute care facility with a tentative diagnosis of hypoparathyroidism. The nurse should monitor the client closely for the related problem of: 1. severe hypotension. 2. excessive thirst. 3. profound neuromuscular irritability. 4. acute gastritis.

3. profound neuromuscular irritability. Hypoparathyroidism may slow bone resorption, reduce the serum calcium level, and cause profound neuromuscular irritability (as evidenced by tetany). Hypoparathyroidism doesn't alter blood pressure or affect the thirst mechanism, which usually is triggered by fluid volume deficit. Gastritis doesn't cause or result from hypoparathyroidism.

A client has received 2.5L of IV fluid. How many ml will be recorded? = 2500 mL A client received 3 tsp and 1 tbsp of oral fluids. How many ml should be recorded?

30 ml

During preoperative teaching for a client who will undergo subtotal thyroidectomy, the nurse should include which statement? 1. "The head of your bed must remain flat for 24 hours after surgery." 2. "You should avoid deep breathing and coughing after surgery." 3. "You won't be able to swallow for the first day or two." 4. "You must avoid hyperextending your neck after surgery."

4. "You must avoid hyperextending your neck after surgery." To prevent undue pressure on the surgical incision after subtotal thyroidectomy, the nurse should advise the client to avoid hyperextending the neck. The client may elevate the head of the bed as desired and should perform deep breathing and coughing to help prevent pneumonia. Subtotal thyroidectomy doesn't affect swallowing.

Which of the following is the most common cause of hyperaldosteronism? 1. Excessive sodium intake 2. A pituitary adenoma 3. Deficient potassium intake 4. An adrenal adenoma

4. An adrenal adenoma An autonomous aldosterone-producing adenoma is the most common cause of hyperaldosteronism. Hyperplasia is the second most frequent cause. Aldosterone secretion is independent of sodium and potassium intake as well as of pituitary stimulation.

A client with severe head trauma sustained in a car accident is admitted to the intensive care unit. Thirty-six hours later, the client's urine output suddenly rises above 200 ml/hour, leading the nurse to suspect diabetes insipidus. Which laboratory findings support the nurse's suspicion of diabetes insipidus? 1. Above-normal urine and serum osmolality levels 2. Below-normal urine and serum osmolality levels 3. Above-normal urine osmolality level, below-normal serum osmolality level 4. Below-normal urine osmolality level, above-normal serum osmolality level

4. Below-normal urine osmolality level, above-normal serum osmolality level In diabetes insipidus, excessive polyuria causes dilute urine, resulting in a below-normal urine osmolality level. At the same time, polyuria depletes the body of water, causing dehydration that leads to an above-normal serum osmolality level. For the same reasons, diabetes insipidus doesn't cause above-normal urine osmolality or below-normal serum osmolality levels.

A 68-year-old client has been complaining of sleeping more, increased urination, anorexia, weakness, irritability, depression, and bone pain that interferes with her going outdoors. Based on these assessment findings, the nurse would suspect which disorder? 1. Diabetes mellitus 2. Diabetes insipidus 3. Hypoparathyroidism 4. Hyperparathyroidism

4. Hyperparathyroidism Hyperparathyroidism is most common in older women and is characterized by bone pain and weakness from excess parathyroid hormone (PTH). Clients also exhibit hypercalciuria-causing polyuria. While clients with diabetes mellitus and diabetes insipidus also have polyuria, they don't have bone pain and increased sleeping. Hypoparathyroidism is characterized by urinary frequency rather than polyuria.

Which nursing diagnosis takes highest priority for a client with hyperthyroidism? 1. Risk for imbalanced nutrition: More than body requirements related to thyroid hormone excess 2. Risk for impaired skin integrity related to edema, skin fragility, and poor wound healing 3. Disturbed body image related to weight gain and edema 4. Imbalanced nutrition: Less than body requirements related to thyroid hormone excess

4. Imbalanced nutrition: Less than body requirements related to thyroid hormone excess In the client with hyperthyroidism, excessive thyroid hormone production leads to hypermetabolism and increased nutrient metabolism. These conditions may result in a negative nitrogen balance, increased protein synthesis and breakdown, decreased glucose tolerance, and fat mobilization and depletion. This puts the client at risk for marked nutrient and calorie deficiency, making Imbalanced nutrition: Less than body requirements the most important nursing diagnosis. Options 2 and 3 may be appropriate for a client with hypothyroidism, which slows the metabolic rate.

Which of the following is the most critical intervention needed for a client with myxedema coma? 1. Administering an oral dose of levothyroxine (Synthroid) 2. Warming the client with a warming blanket 3. Measuring and recording accurate intake and output 4. Maintaining a patent airway

4. Maintaining a patent airway Because respirations are depressed in myxedema coma, maintaining a patent airway is the most critical nursing intervention. Ventilatory support is usually needed. Although myxedema coma is associated with severe hypothermia, a warming blanket shouldn't be used because it may cause vasodilation and shock. Gradual warming with blankets would be appropriate. Thyroid replacement will be administered I.V. and although intake and output are very important, these aren't critical interventions at this time.

For a client with Graves' disease, which nursing intervention promotes comfort? 1. Restricting intake of oral fluids 2. Placing extra blankets on the client's bed 3. Limiting intake of high-carbohydrate foods 4. Maintaining room temperature in the low-normal range

4. Maintaining room temperature in the low-normal range Graves' disease causes signs and symptoms of hypermetabolism, such as heat intolerance, diaphoresis, excessive thirst and appetite, and weight loss. To reduce heat intolerance and diaphoresis, the nurse should keep the client's room temperature in the low-normal range. To replace fluids lost via diaphoresis, the nurse should encourage, not restrict, intake of oral fluids. Placing extra blankets on the bed of a client with heat intolerance would cause discomfort. To provide needed energy and calories, the nurse should encourage the client to eat high-carbohydrate foods.

The nurse is caring for a client who had a thyroidectomy and is at risk for hypocalcemia. What should the nurse do? 1. Monitor laboratory values daily for an elevated thyroid-stimulating hormone. 2. Observe for swelling of the neck, tracheal deviation, and severe pain. 3. Evaluate the quality of the client's voice postoperatively, noting any drastic changes. 4. Observe for muscle twitching and numbness or tingling of the lips, fingers, and toes.

4. Observe for muscle twitching and numbness or tingling of the lips, fingers, and toes. Muscle twitching and numbness or tingling of the lips, fingers, and toes are signs of hyperirritability of the nervous system due to hypocalcemia. The other options describe complications for which the nurse should also be observing; however, tetany and neurologic alterations are primary indications of hypocalcemia.

The nursing care for the client in addisonian crisis should include which intervention? 1. Encouraging independence with activities of daily living (ADLs) 2. Allowing ambulation as tolerated 3. Offering extra blankets and raising the heat in the room to keep the client warm 4. Placing the client in a private room

4. Placing the client in a private room The client in addisonian crisis has a reduced ability to cope with stress due to an inability to produce corticosteroids. Compared to a multibed room, a private room is easier to keep quiet, dimly lit, and temperature controlled. Also, visitors can be limited to reduce noise, promote rest, and decrease the risk of infection. The client should be kept on bed rest, receiving total assistance with ADLs to avoid stress as much as possible. Because extremes of temperature should be avoided, measures to raise the body temperature, such as extra blankets and turning up the heat, should be avoided.

A client is admitted to the health care facility for evaluation for Addison's disease. Which laboratory test result best supports a diagnosis of Addison's disease? 1. Blood urea nitrogen (BUN) level of 12 mg/dl 2. Blood glucose level of 90 mg/dl 3. Serum sodium level of 134 mEq/L 4. Serum potassium level of 5.8 mEq/L

4. Serum potassium level of 5.8 mEq/L Addison's disease decreases the production of aldosterone, cortisol, and androgen, causing urinary sodium and fluid losses, an increased serum potassium level, and hypoglycemia. Therefore, an elevated serum potassium level of 5.8 mEq/L best supports a diagnosis of Addison's disease. A BUN level of 12 mg/dl and a blood glucose level of 90 mg/dl are within normal limits. In a client with Addison's disease, the serum sodium level would be much lower than 134 mEq/L, a nearly normal level.

The nurse explains to a client with thyroid disease that the thyroid gland normally produces: 1. iodine and thyroid-stimulating hormone (TSH). 2. thyrotropin-releasing hormone (TRH) and TSH. 3. TSH, T3, and calcitonin. 4. T3, T4, and calcitonin.

4. T3, T4, and calcitonin. The thyroid gland normally produces thyroid hormone (T3 and T4) and calcitonin. TSH is produced by the pituitary gland to regulate the thyroid gland. TRH is produced by the hypothalamus gland to regulate the pituitary gland.

When assessing a client with pheochromocytoma, a tumor of the adrenal medulla that secretes excessive catecholamine, the nurse is most likely to detect: 1. a blood pressure of 130/70 mm Hg. 2. a blood glucose level of 130 mg/dl. 3. bradycardia. 4. a blood pressure of 176/88 mm Hg.

4. a blood pressure of 176/88 mm Hg. Pheochromocytoma, a tumor of the adrenal medulla that secretes excessive catecholamine, causes hypertension, tachycardia, hyperglycemia, hypermetabolism, and weight loss. It isn't associated with the other options.

When assessing a client with pheochromocytoma, a tumor of the adrenal medulla that secretes excessive catecholamine, the nurse is most likely to detect: 1. a blood pressure of 130/70 mm Hg. 2. a blood glucose level of 130 mg/dl. 3. bradycardia. 4. a blood pressure of 176/88 mm Hg.

4. a blood pressure of 176/88 mm Hg. Pheochromocytoma, a tumor of the adrenal medulla that secretes excessive catecholamine, causes hypertension, tachycardia, hyperglycemia, hypermetabolism, and weight loss. It isn't associated with the other options.

The client is being evaluated for hypothyroidism. During assessment, the nurse should stay alert for: 1. exophthalmos and conjunctival redness 2. flushed, warm, moist skin 3. systolic murmur at the left sternal border 4. decreased body temperature and cold intolerance

4. decreased body temperature and cold intolerance Hypothyroidism markedly decreases the metabolic rate, causing a reduced body temperature and cold intolerance. Other signs and symptoms include dyspnea, hypoventilation, bradycardia, hypotension, anorexia, constipation, decreased intellectual function, and depression. The other options are typical findings in a client with hyperthyroidism.

Before undergoing a subtotal thyroidectomy, a client receives potassium iodide (Lugol's solution) and propylthiouracil (PTU). The nurse would expect the client's symptoms to subside: 1. in a few days. 2. in 3 to 4 months. 3. immediately. 4. in 1 to 2 weeks.

4. in 1 to 2 weeks. Potassium iodide reduces the vascularity of the thyroid gland and is used to prepare the gland for surgery. Potassium iodide reaches its maximum effect in 1 to 2 weeks. PTU blocks the conversion of thyroxine to triiodothyronine, the more biologically active thyroid hormone. PTU effects are also seen in 1 to 2 weeks. To relieve symptoms of hyperthyroidism in the interim, clients are usually given a beta-adrenergic blocker such as propranolol.

The nurse teaches a client with newly diagnosed hypothyroidism about the need for thyroid hormone replacement therapy to restore normal thyroid function. Which thyroid preparation is the agent of choice for thyroid hormone replacement therapy? 1. methimazole (Tapazole) 2. thyroid USP desiccated (Thyroid USP Enseals) 3. liothyronine (Cytomel) 4. levothyroxine (Synthroid)

4. levothyroxine (Synthroid) Levothyroxine is the agent of choice for thyroid hormone replacement therapy because its standard hormone content gives it predictable results. Methimazole is an antithyroid medication used to treat hyperthyroidism. Thyroid USP desiccated and liothyronine are no longer used for thyroid hormone replacement therapy because they may cause fluctuating plasma drug levels, increasing the risk of adverse effects.

A client is scheduled for a transsphenoidal hypophysectomy to remove a pituitary tumor. Preoperatively, the nurse should assess for potential complications by: 1. testing for ketones in the urine. 2. testing urine specific gravity. 3. checking temperature every 4 hours. 4. performing capillary glucose testing every 4 hours.

4. performing capillary glucose testing every 4 hours. The nurse should perform capillary glucose testing every 4 hours because excess cortisol may cause insulin resistance, placing the client at risk for hyperglycemia. Urine ketone testing isn't indicated because the client does secrete insulin and, therefore, isn't at risk for ketosis. Urine specific gravity isn't indicated because although fluid balance can be compromised, it usually isn't dangerously imbalanced. Temperature regulation may be affected by excess cortisol and isn't an accurate indicator of infection.

During a follow-up visit to the physician, a client with hyperparathyroidism asks the nurse to explain the physiology of the parathyroid glands. The nurse states that these glands produce parathyroid hormone (PTH). PTH maintains the balance between calcium and: 1. sodium. 2. potassium. 3. magnesium. 4. phosphorus.

4. phosphorus. PTH increases the serum calcium level and decreases the serum phosphate level. PTH doesn't affect sodium, potassium, or magnesium regulation.

A client with Addison's disease comes to the clinic for a follow-up visit. When assessing this client, the nurse should stay alert for signs and symptoms of: 1. calcium and phosphorus abnormalities. 2. chloride and magnesium abnormalities. 3. sodium and chloride abnormalities. 4. sodium and potassium abnormalities.

4. sodium and potassium abnormalities. In Addison's disease, a form of adrenocortical hypofunction, aldosterone secretion is reduced. Aldosterone promotes sodium conservation and potassium excretion. Therefore, aldosterone deficiency increases sodium excretion, predisposing the client to hyponatremia, and inhibits potassium excretion, predisposing the client to hyperkalemia. Because aldosterone doesn't regulate calcium, phosphorus, chloride, or magnesium, an aldosterone deficiency doesn't affect levels of these electrolytes directly.

The nurse is caring for a client with diabetes insipidus. The nurse should anticipate the administration of: 1. insulin. 2. furosemide (Lasix). 3. potassium chloride. 4. vasopressin (Pitressin).

4. vasopressin (Pitressin). Vasopressin is given subcutaneously in the acute management of diabetes insipidus. Insulin is used to manage diabetes mellitus. Furosemide causes diuresis. Potassium chloride is given for hypokalemia.

The nurse is administering eye drops to a patient with glaucoma. After instilling the patient's first medication, how long should the nurse wait before instilling the patient's second medication into the same eye?

5 minutes

A client has a prescription for 5,000 unit IV bolus for heparin followed by an IV drip at 1,000 units/hr. A premixed bag of 25,000 units in 250 ml of D5W is available. The nurse should set the infusion pump to deliver how many ml for the bolus dose?

50 ml

Normal Range for pH:

7.35- 7.45

Nurse on pediatric unit received shift report on 4 children. Which should the nurse assess first?

A 10-year old child who is awaiting surgery for an appendectomy and experienced sudden relief of pain. Can indicate peritonitis from a ruptured appendix.

Outpatient mental health clinic clients which of the following clients is effectively using sublimation as a defense mechanism

A client who channels her energy into a new hobby following the loss of her job

Nurse must recommend clients for discharge in order to make room for several critically injured clients from a local disaster

A client who has cellulitis and is receiving oral antibiotics every 8 hours.

Postpartum nurse is caring for 4 clients which one should be on seizure precautions.

A client who is as 33 wks of gestation and has severe gestational hypertension

Just received change of shift report on 4 clients. Which should the nurse assess first

A client who is postop with abdominal distention and no bowel sounds

Received change of shift report on 4 clients. Which one should the nurse intervene to prevent food and medication interaction?

A client who is receiving a MAOI and is requesting a cheeseburger for dinner. -Contains tyramine can cause hypertension

Charge is providing educational session about infection control for a group of staff on isolation precautions

A client who requires airborne precautions should be placed in a negative pressure airflow room

Mental health clinic caring for clients. Which of the following clients is using dissociation as a defense mechanism?

A client who was abused as a child describes the abuse as if it happened to someone else

The nurse is caring for a Mexican American who is Catholic. The nurse wishes to learn more about the culture by consulting a key informant. Which of the following religious practitioners would be most knowledgeable about the beliefs held by individuals of Mexican ethnicity?

A curandera

Working in ED triaging 4 clients. Which of the following clients should the nurse recommend for treatment first?

A middle aged adult client who has unstable vital signs

hematologic s&s hnypothyroid

A moderate anemia is usual and may be caused by lack of thyroxine - The blood film may show mild macrocytosis and acanthocytosis - There is also a reduced oxygen need and thus reduced erythropoietin secretion - Autoimmune thyroid disease, especially myxedema or Hashimoto's disease, is associated with pernicious anemia

A patient is being discharged home from the ambulatory surgical center after cataract surgery. In reviewing the discharge instructions with the patient, the nurse instructs the patient to immediately call the office if the patient experiences what?

A new floater in vision

Teaching a group of newly licensed nurses about the need to complete an incident report. Which of the following is an example of reportable incident and an indication for completing a report

A nurse administered a medication via the wrong route.

While reviewing the chart for an assigned client before beginning care, a student notes that the client does not belong to a specific religion. Based on this information, what should the student interpret about the client?

A person may be deeply spiritual but not profess a religion.

While assessing a neonate at 4 hours after birth, the nurse observes an indentation with a small tuft of hair at the base of the neonate's spine. The nurse should document this finding as what finding?

A small tuft of hair and an indentation at the base of the neonate's spine is termed spina bifida occulta. This condition usually occurs between the L5 and S1 vertebrae with failure of the vertebrae to completely fuse. There are usually no sensory or motor deficits with this condition. Spina bifida cystica includes meningocele, myelomeningocele, and lipomeningocele. Meningocele is characterized by a saclike protrusion filled with spinal fluid and meninges. Usually, this condition is associated with sensory and motor deficits. Myelomeningocele is characterized by a saclike protrusion filled with spinal fluid, meninges, nerve roots, and spinal cord. With myelomeningocele, there are usually sensory and motor deficits.

A community health nurse is testing the theory of locus of control (LOC). Which of the following client's demonstrates the internal control concept of this theory: a) A client who takes an active role in all health decisions b) A client who allows the primary care provider to make all the decisions c) A client who does not make any decisions without his/her souse's input d) A client who relies on information from the local hospital for his.her health needs

A) A client who takes an active role in all health decisions

On one of the first days working alone, the novice nurse must provide teaching on tracheotomy care to the client as well as the client's spouse. This nurse is not familiar with the teaching aspect. The best action for the nurse is to: a) ASk the nurse mentor to assist with the teaching after reviewing the procedure b) Read the policy and procedure manual before the teaching session c) Do the best the nurse can by remembering what was taught in nursing school d) ASk for a different assignment until the nurse feels comfortable with this one

A) ASk the nurse mentor to assist with the teaching after reviewing the procedure

Which nursing action will be included when the nurse is doing a wet-to-dry dressing change for a patient's Stage III sacrel pressure ulcer: a) Administer the ordered PRN oral opoid 30 min before the dressing change b) Soak the old dressing with sterile saline a few minutes before removing them

A) Administer the ordered PRN oral opiod 30 min before the dressing change

The nurse is reviewing laboratory data for a patient who is receiving total parental nutrition. Which lab value should be immediately brought to the physicians attention: a) BUN of 60 c) Serum glucose 328 d) Potassium of 3.5

A) BUN of 60

Outcome statement is: a) Client will ambulate without a walker by 6 weeks b) Client will ambulate freely in house c) Client will not fall

A) Client will ambulate without walker by 6 weeks

The nursing action most appropriate for a client who has an infection and develops a fever of 99.8' F is to: a) Continue to monitor the patient's temp b) Administer an antipyretic

A) Continue to monitor the patient's temp

A student nurse who claims to be very uncreative and dose not understand why it is necessary to assess and develop new ideas in the clinical area. The best response by the nurse educator is: a) Creativity allows unique solutions to unique problems b) Not all your answers are going to be from your textbook c) Creativity makes nursing fun d) You'll get bored if you don't learn to be creative

A) Creativity allows unique solutions to unique problems

A client has an open wound that is yellow and black. Using the RYB color code, which nursing intervention needs to occur first? a) Debride the area with wet-to-dry dressing b) Apply topical antibiotic ointment

A) Debride the area with wet-to-dry dressing http://www.slideshare.net/lezzoj/wound-dressing

A client recovering from abdominal surgery refuses analgesia, saying that he is "fine, as long as he dosen't move." Which nursing diagnosis should be a priority: A) Deficient Knowledge (pain control measures) b) Ineffective Health Maintenance

A) Deficient Knowledge (pain control measures)

A nursing activity that is carried out during the evaluation phase of the nursing process is: a) Determining if interventions have been effective in meeting patient's outcome b) Documenting the nursing care plan in the progress notes in the medical record c) Deciding whether the patient's health problems have been completely resolved d) Asking the patient to evaluate whether the nursing care provided was satifactory

A) Determining if interventions have been effective in meeting patient's outcomes

A patient complains of pain during circumfusion of the shoulder when the nurse moves the arm behind the patient which question should the nurse ask: a) Do you have difficulty in putting on a jacket b) Are you able to feed yourself without difficulty

A) Do you have difficulty in putting on a jacket

During a well-child visit, a mother tells the nurse that her 4- year old daughter typically goes to bed at 10:30 pm and awakens each morning at 7 am. She does not take a napin the afternoon. Which is the best response by the nurse: a) encourage the mother to consider putting her daughter to bed between 8 and 9 pm d) Reassure her that her daughter's sleep pattern is normal and that she has outgrown her need for an afternoon nap

A) Encourage the mother to consider putting her daughter to bed between 8 and 9 pm

The mother of a 1 month old infant is concerned because the infant has had vomiting and diarrhea for 2 days. What instructions should the nurse give this infants mother: a) Have the infant be seen by a physician b) Give the infant at least 2 ounces of juice every 2 hours

A) Have the infant be seen by a physician

The nurse has completed discharge teaching for a client who will be going home on oxygen therapy. Which statement made by the client, would indicate that this client needs further instruction: a) I will replace my cotton blankets with polyester ones b) My son will not be able to smoke when I am around

A) I will replace my cotton blankets with polyester ones

The shift change while the nursing staff was waiting for the adult children of a deceased client to arrive. The oncoming nurse has never met the family. Which of the following initial greetings is most appropriate: a) I'm very sorry for your loss b) I'll take you in to view the body

A) I'm very sorry for your loss

The patient has been admitted with complaints of shortness of breath for 2 week duration and has received the nursing diagnosis impaired gas exchange. Which admission laboratory result would support the choice of this diagnosis: a) Increased hematocrit b) Decreased BUN

A) Increased Hematocrit

The nursing process is a dynamic process. This means that it: a) Is ever changing to the client's needs b) Conveys the force or power of the health team

A) Is ever changing in response to the client's needs

The nurse anticipates that osteoposis may result from prolonged immobilization because of: a) Lack of weight bearing, which decreases osteoblastic activity b) Decreased dietary calcium intake

A) Lack of weight bearing, which decreases osteoblastic activity

A patient has the following arterial blood gas (ABG) results: ph 7.32, PAO2 88 mmHg, PaCO@ and HCO3 16 mEqL. The nurse interprets these results as: a) Metabolic acidosis b) Metabolic alkalosis c) Respiratory acidosis d) Respiratory alkalosis

A) Metabolic Acidosis

At SAM, a nurse checks the amount of solution left in a potential nutrition infusion bag for an assigned client. It is a 3000 mL bag with 1000 mL remaining. The solution is running at a rate of 100 mL/hr. The bag was hung the previous day at noon. The nurse plans to change the infusion bag and tubing today at: a) Noon b) 2 pm

A) Noon

A patient with an open abdominal wound has a complete blood cell (CBC) count and differential, which indicate an increase in white blood cells (WBC)s and a shift to the left. The nurse anticipates that the next action will be to: a) Obtain wound cultures b) start antibiotic c) Reddress the wound with wet-to-dry dressing d) Continue to monitor the wound for purulent drainage

A) Obtain wound cultures

An 85 year old client has impaired hearing. When creating the care plan which intervention should have the highest priority: a) Obtaining an amplified telephone b) Teaching the importance of changing his position

A) Obtaining an amplified telephone

Which of these patients in the clinic will the nurse plan to teach about risks associated with obesity: a) Patient who has a BMI of 18 kg/m2 b) Patient with a waist circumference 34 inches (86 cm) d) patient whose waist measures 30 in. (75 cm) and hips measure 34 in. *85 cm)

A) Patient who has a BMI of 18 kg/m2

A patient in the hospital has a history of functional urinary incontinence. Which nursing action will be included in the plan of care: a) Place a bedside commode near the patient's bed b) Demonstrate the use of the Crede maneuver to the patient

A) Place a bedside commode near the patient's bed

The edges of a patient's appendectomy incision are approximated, and no drainage is noted. The nurse documents on the client's wound record that the incision appears to be healing by: a) Primary intention b) Secondary intention

A) Primary Intention

Which of these nursing actions included in the plan of care for a patient who is receiving intermittent tube feedings through a percutaneous endoscopic gastrostomy )PEG) tube may be delegated to an LPN/LVN: a) Providing skin care to the area around the tube site b) Assessing the patient's nutritional status at least weekly

A) Providing skin care to the area around the tube site

After completing a scheduled every 2-hour turn by turning the patient to the left side, the nurse notices a reddened are over the coccyx. The area blanches when the nurse compresses it with thumb pressure. One hour later, the nurse reassesses the area and finds the redness has disappeared. How should the nurse document this area: a) Reactive hyperemia c) Stage II pressure ulcer d) Stage III pressure ulcer

A) Reactive Hyperemia

The nurse is caring for an 80 year old female nursing home resident who has been admitted to the hospital with pneumonia and is becoming progressively more confused. Her vital signs are: Temp 101' F, Pulse 112, Resp. 28 and BP 100/70. ABG results include pH 7.50, PaCO@ 25 mmHg, and bicarbonate level 18 mEq/L. The nurse interprets these findings to indicate: a) Respiratory acidosis secondary to hypoexmia b) Respiratory acidosis secondary to anxiety

A) Respiratory acidosis secondary to hypoeximia

The nurse is developing a plan of care for a client with a newly created ileostomy. The priority nursing diagnosis for this client is: a) Risk for deficient fluid volume related to excessive fluid loss from ostomy b) Disturbed body image related to presence of ostomy

A) Risk for Deficient Fluid Volume related to Excessive Fluid loss from Ostomy

A patient who has just been started on continuous tube feedings of a full strength commercial formula at 100 mL/hr using a closed system method has six diarrhea stools the first day. What action should the nurse plan to take: a) Slow the infusion rate of the tube feeding b) Check the gastric residual volumes more frequently c) Change the internal feeding system and formula every 8 hrs d) Discontinue administration of water through the feeding tube

A) Slow the infusion rate of the tube feeding

When asked to sign the permission form for surgical removal of a large but noncancerous lesion on her face, the client begins to cry. Which of the following is the most appropriate response: a) Tell me what it means to you to have surgery b) you must be very glad to be having this lesion removed

A) Tell me what it means to you to have surgery

The nurse obtains all of the following assessment data about a patient with deficient fluid volume caused by a massive burn injury. Which of the following assessment data will be of greatest concern: a) The BP is 90/40 mm/Hg c) Oral fluid intake is 100 mL for the last 8 hours d) There is prolonged skin tenting over the sternum

A) The BP is 90/40 mm/Hg

A nurse is practicing the concept of holism to the client. Which of the following is the best example of this: a) The nurse considers how the loss of a client's job will affect the regulation of the client's diabetes b) The nurse makes sure to do a complete teaching regarding pharmacological interventions c) The nurse is careful to follow physician treatments on schedule d) The nurse is able to prioritize the needs of the client assigned according to Maslow's hierarchy

A) The nurse consider's how the loss of a client's job will affect the regulation of the client's diabetes

Which outcome is appropriate for the client problem "ineffective gas exchange" for the client recently diagnosed with COPD: a) The patient demonstrates the correct way to pursed lip breathe b) The client lists three signs/symptoms to report to the Health Care provider

A) The patient demonstrates the correct way to pursed lip breathe

The nurse case manager is concerned about A particular client being discharged from the hospital. Which of the following factors, if present for this client, would alert the nurse to possible problems with treatment adhearance: a) The prescribed therapy is costly and of unknown duration b) The therapy will require no lifestyle changes of the client c) The client has not had difficulty understanding the regimen d) The client's culture is supportive of Western medicine

A) The prescribed therapy is costly and of unknown duration

A client who has just been diagnosed with pancreatic cancer is quite upset and verbal. The nurse has the following diagnoses: anxiety related to unfamiliarity of disease process, manifested by restlessness tachycardia. The etiology of this diagnoses is which of the following: a) Unfamiliarity of disease process b) Anxiety c) Restlessness d) Tachycardia

A) Unfamiliarity of disease process

A client has a serum sodium concentration of 160 mEq/L and exhibits generalized weakness and confusion. The nurse should plan to initiate: a) Fluid restrictions c) Monitoring of urine specific gravity d) Seizure precautions

A) fluid restrictions

The nurse is caring for a client today who asks to have the coffee removed from his tray. The client informs the nurse that his religion precludes the intake of caffeine. The nurse is aware that religions often guide daily living habits in which of the following ways? Choose all that apply. A) Diet B) Medical treatments C) Clothing D) Education E) Automobile makes

A, B, C

The nurse is caring for client 82 years of age who is struggling to adapt to hearing loss as he ages. The nurse performs which of the following interventions to assist the client in adapting to this sensory deficit? Choose all that apply. Make sure he wears his hearing aid. Speak in a lower tone of voice. Speak so he can observe your lip movement. Keep his environment clear of clutter. Orient to person, place, and time frequently

A, B, C

Then nurse is caring for a female client today. As the nurse is giving the client her morning medications, she begins a conversation about her belief in a higher power. The nurse knows that this can be interpreted as which of the following? Choose all that apply. A) Faith B) Spirituality C) Religion D) God E) Atheism

A, B, C, D

According to Havighurst, which of the following are developmental tasks of middle adulthood? Select all that apply. A) Accept and adjust to physical changes. B) Maintain a satisfactory occupation. C) Assist children to become responsible adults. D) Maintain social contacts and relationships. E) Relate to one's spouse or partner as a person.

A, B, C, E

Which of the following are conditions that must be met for a person to receive the necessary data to experience the world? Select all that apply. A stimulus must be present. A receptor or sense organ must receive the stimulus and convert it to a nerve impulse. The nerve impulse must be conducted along a nervous pathway from the receptor or sense organ to the brain. The stimulus must be recognized by the cardiovascular system and sent to the brain. The person must physically and mentally recognize the stimulus and accept or reject it in the brain. A particular area in the brain must receive and translate the impulse into a sensation.

A, B, C, F

A client states, "I want my children to have my favorite photographs, because I will not need them anymore since my wife died." How should the nurse respond? A. "Are you thinking about death and dying?" B. "Do you think this would make your partner happy?" C. "Your children will be happy you are thinking of them." D. "It seems like this decision makes you very sad."

A. "Are you thinking about death and dying?"

A home health nurse is performing an admission assessment on a client who had a knee arthroplasty 1 week ago. Which of the following client statements should concern the nurse most? A. "I am so glad to be off those blood thinners." B. "I will keep a pillow under my knee when I am in bed." C. "I am planning to use a wheelchair to help me get around." D. "I plan to take Motrin instead of prescribed Lortab for pain control."

A. "I am so glad to be off those blood thinners."

An asymptomatic client who was exposed to Ebola 1 week ago telephones the ED. Which of the following statements would be accurate for the triage nurse to tell the caller? A. "I will need to notify the health department." B. "Household contacts should be given preventative antibiotics." C. "Since you are symptom free, you were not infected." D. "A throat culture is an effective way to diagnose the infection."

A. "I will need to notify the health department."

A nurse provides education regarding contraceptive methods to a group of women. Which client is a candidate for combination oral contraceptive therapy? A. 45 y/o woman with rheumatoid arthritis B. 30 y/o woman with hx of breast cancer C. 18 y/o woman with elevated liver enzymes D. 27 y/o woman with hx of DVT

A. 45 y/o woman with rheumatoid arthritis

A client who has long term liver cirrhosis is admitted with portal HTN. Which intervention should the nurse implement to prevent complications? A. Administer stool softeners daily B. Place client in high fowler's C. Infuse .9NS at 125 ml/hr D. Offer client low protein, high calorie snacks

A. Administer stool softeners daily

A nurse is assisting with a class of a group of adolescents on smoking cessation. Which strategy should the nurse include? A. Ask several former adolescent smokers to discuss why/how they quit smoking B. Invite parents to discuss their concerns about having a child who smokes C. Discuss consequences of tobacco addiction using pictures to illustrate effects D. Introduce smokeless tobacco to participants as alternative to cigarettes

A. Ask several former adolescent smokers to discuss why/how they quit smoking

A client who had a gastric resection 1 month ago occasionally reports dizziness, sweating, and fluttering in the chest after meals. What should the nurse advise the client to do? A. Avoid consuming milk, sweets, sugars B. Reduce amount of protein and fat C. Eat small, frequent meals D. Drink liquids 1h before or after meals E. Sit up for at least 1h after each meal

A. Avoid consuming milk, sweets, sugars D. Drink liquids 1h before or after meals

A nurse plans teaching for parents of a toddler about healthy food choices. Which foods should be recommended? Select that apply A. Baked deboned chicken B. Chocolate chip cookie C. Slices of peeled apple D. Yogurt E. Popcorn F. Peanuts

A. Baked deboned chicken C. Slices of peeled apple D. Yogurt

A nurse reviews the dietary journal of a client recently prescribed phenelzine. Which foods should be of concern? A. Beef jerky B. Whole milk C. Grape juice D. Cream cheese

A. Beef jerky

A community health nurse provides teaching about the Zika virus to a group of women who are pregnant. Which information should be included? Select all that apply A. Breastfeeding is encouraged for infants B. Travel to infected areas should be avoided C. Vaccination is recommended for prevention D. Infection can occur through sexual intercourse E. Apply mosquito repellent when going outdoors F. Flu like symptoms

A. Breastfeeding is encouraged for infants B. Travel to infected areas should be avoided D. Infection can occur through sexual intercourse E. Apply mosquito repellent when going outdoors F. Flu like symptoms

A nurse reviews a client's lab results and should report which to the provider? Select all that apply A. Calcium 8 B. Sodium 130 C. Potassium 4.5 D. Magnesium 1.9 E. Phosphorus 3.5

A. Calcium 8 B. Sodium 130

A client is receiving TPN and lipids. The nurse should recognize which of the following measures should be implemented? Select all that apply A. Change TPN infusion tubing q24h B. Discontinue infusion of lipids after 12h C. Clean IV injection port before/after each time it is used D. Increase rate if infusion falls behind schedule E. Monitor BSL before meals and at bedtime

A. Change TPN infusion tubing q24h B. Discontinue infusion of lipids after 12h C. Clean IV injection port before/after each time it is used

A school nurse observes several children playing on a playground. Which child should concern the nurse most? A. Child squatting after a game of ball B. 2 children arguing with each other C. Child breathing heavily after running D. Child climbing on swing set supports

A. Child squatting after a game of ball

A charge nurse prepares end of shift report for a group of clients. Which lab results require follow up? Select all that apply A. Client scheduled for sx, Hct 32 B. Client scheduled for angioplasty, creatinine 1.6 C. Client with type 1 DM, Hba1c 5.9 D. Client with DVT receiving warfarin, INR 2.5 E. Client with PE receiving heparin aPTT 40 sec

A. Client scheduled for sx, Hct 32 B. Client scheduled for angioplasty, creatinine 1.6 E. Client with PE receiving heparin aPTT 40 sec

A client plans to leave the facility AMA. Which actions should the nurse implement? Select all that apply A. Contact the provider B. Notify security department C. Ask client to sign informed consent D. Request d/c prescription by provider E. Inform client of complications that may occur w/o tx

A. Contact the provider E. Inform client of complications that may occur w/o tx

A nurse plans care for a client dx with a CVA. Which members of the interprofessional team should participate in planning care? Select all that apply A. Dietician B. Hospice nurse C. Speech therapist D. Physical therapist E. Rapid response team

A. Dietician C. Speech therapist D. Physical therapist

An adolescent was admitted 12h ago following a MVA. Multiple skeletal fractures were sustained. The client is in balanced-suspension traction. Which of the following assessment findings requires immediate intervention by the nurse? A. Disorientation B. Shallow respirations C. Chest pain with positioning D. Bloody drainage at pin site

A. Disorientation

A nurse provides care for a client who has recently returned from Liberia (Ebola). Which symptoms should be reported immediately? Select all that apply A. Fever B. Dysuria C. Epistaxis D. Diarrhea E. Vomiting

A. Fever C. Epistaxis D. Diarrhea E. Vomiting

A client who has schizophrenia is currently experiencing an exacerbation of symptoms. Which actions should the nurse implement? Select all that apply A. Focus on reality based topics B. Explain why the president will never attend lunch C. Ask client to describe auditory hallucination D. Monitor food and fluid intake E. Provide supportive and structured environment

A. Focus on reality based topics C. Ask client to describe auditory hallucination D. Monitor food and fluid intake E. Provide supportive and structured environment

A client remains in the ICU 48h post intubation. The nurse recognizes which interventions are needed to assist in prevention of VAP? Select all that apply A. Frequent hand hygiene B. Oral decontamination C. Wearing a face mask D. Client positioned supine E. Clean oral suction device F. Turn q2h

A. Frequent hand hygiene B. Oral decontamination E. Clean oral suction device F. Turn q2h

A 55 y/o woman reports frequency, urgency, and burning upon urination. Which factor most likely contributed to this problem? A. Frequent intercourse B. Increase intake of cranberry juice C. Started jogging exercise program D. Initiation of estrogen replacement therapy

A. Frequent intercourse

A home health nurse evaluates the items a school age child packed for lunch and a snack at school. Which items should the nurse suggest be replaced with a healthier option? Select all that apply A. Fruit loops B. Grapes C. Animal crackers D. Fig bar E. Skim milk F. Cupcakes

A. Fruit loops F. Cupcakes

A nurse is organizing care for 4 clients. Which of the following tasks should the nurse instruct the AP to perform? A. Measure emesis from a client who was recently admitted with dehydration B. Bathe/shampoo hair for a client who was just admitted after a MVA C. Provide assistance for a client who is requesting a bedpan after a lumbar puncture D. Decrease oxygen on NC for client being discharged with COPD

A. Measure emesis from a client who was recently admitted with dehydration

A nurse prepares to administer meds through an NG tube. Which actions should the nurse take? Select all that apply A. Measure gastric residual B. Flush with at least 15 ml sterile water C. Aspirate gastric contents and check for pH D. Push meds through using a syringe E. Mix crushed tablets and administer using a 60 ml syringe

A. Measure gastric residual B. Flush with at least 15 ml sterile water C. Aspirate gastric contents and check for pH D. Push meds through using a syringe

A client who is semi-comatose after a CVA, has a NG tube (decompression or feeding) and was started on TPN (electrolytes, vitamins, minerals, glucose, proteins, sometimes lipids via central line to SVC) today. Which of the following should the nurse implement to prevent fluid volume deficit (dehydration)? A. Monitor blood glucose q4-6h B. Determine total fluid intake q8h C. Give boluses of water through NG tube D. Increase oral fluid intake to 3L/day

A. Monitor blood glucose q4-6h

A child who has hemophilia is being discharged home. The nurse should teach the parents to sue which measures if a child sustains an injury? Select all that apply A. Place ice over injured tissue B. Provide passive ROM C. Apply pressure directly if bleeding D. Soak affected area in warm water E. Keep injured extremity above heart F. Administer replacement clotting factors

A. Place ice over injured tissue C. Apply pressure directly if bleeding E. Keep injured extremity above heart F. Administer replacement clotting factors

An older adult client reports recurring calf pain after walking 1-2 blocks that disappears with rest (intermittent claudication). The client has weak pedal pulses (artery), and skin on the left lower leg is shiny and cool to touch. Which of the following nursing interventions is appropriate at this time? A. Position left leg dependently B. Elevate left leg above heart C. Immobilize left leg to prevent further injury D. Assess dorsiflexion and extension of left foot

A. Position left leg dependently

A client is receiving magnesium sulfate 1 g/hr. the nurse is unable to elicit a patellar DTR and RR 10. Which of the following is the priority nursing action? A. Prepare to administer calcium gluconate B. Arrange for an emergency c-section C. Review previous lab results D. Verify infusion rate of medication

A. Prepare to administer calcium gluconate

A client who has Parkinson's disease is prescribed selegiline (MAOI). The nurse should provide dietary teaching that includes avoiding which foods? Select all that apply A. Red wine B. Soy sauce C. Watermelon D. Aged cheese E. Cured sausage

A. Red wine B. Soy sauce D. Aged cheese E. Cured sausage

A client was recently placed in seclusion after exhibiting violent behavior. What is the first action of the nurse? A. Review medical hx for potential contraindications to seclusion B. Obtain verbal prescription now and request medical evaluation within 12h C. Maintain seclusion if client continues to exhibit signs of delirium D. Administer propofol 80 mg IV, repeat as needed

A. Review medical hx for potential contraindications to seclusion

A client has a sealed radiation implant. Which actions should the nurse implement? Select all that apply A. Save linens in client's room B. Assign client to a private room C. Limit each visitor to 30 min/day D. Instruct friends to stand 3 ft from client E. Place "Caution: radioactive material" sign on door

A. Save linens in client's room B. Assign client to a private room C. Limit each visitor to 30 min/day E. Place "Caution: radioactive material" sign on door

A nurse provides discharge teaching to a client prescribed clozapine. Which of the following instructions should the nurse include? A. Schedule weekly lab tests B. Decrease fiber in diet C. Monitor BP for HTN D. Avoid consuming aged cheeses and wine

A. Schedule weekly lab tests

A nurse assess a client who has hyperthyroidism. Which finding is expected? A. Tachycardia B. Thickened tongue C. Narrow pulse pressure D. Hypoactive DTRs

A. Tachycardia

A nurse suspects the client has developed tension pneumothorax during a right pleural thoracentesis. Which findings will be present? Select all that apply A. Trachea deviated to left B. Temp 38.8C (102F) C. Chest pain increases on right side D. HR increases from 98 to 130 E. RR decrease from 30 to 18

A. Trachea deviated to left C. Chest pain increases on right side D. HR increases from 98 to 130

A client is newly prescribed isosorbide mononitrate. Upon review of the client's admission hx, which of the following findings should concern the nurse most? A. Use of vardenafil B. Report of frequent HA C. Hx of MI D. Administration of metoprolol

A. Use of vardenafil

A client who has just bed diagnosed with rheumatoid arthritis is required to receive 3 mo of methotrexate sodium (anti-neoplastic/cancer drug) therapy. The nurse recognizes that which of the following are associated with the therapy? Select all that apply A. WBC count 1,200 mm B. Weight gain 2.27 kg (5 lb) C. Oral temp 37.2 C (99 F) D. Urine specific gravity 1.003 E. Platelets 5,000 mm

A. WBC count 1,200 mm E. Platelets 5,000 mm

A nurse cares for a client who is prescribed alteplase. The concurrent use of which medication should be of concern? A. Warfarin B. Metoprolol C. Furosemide D. Levothyroxine

A. Warfarin

A client has a pulmonary embolism. Initial tx includes 40% O2 via venturi mask, IV heparin, and bed rest. Which finding should indicate to the nurse therapy is effective? A. aPTT 70 seconds B. PaO2 75 C. CT scan of chest positive for infiltrate D. Calf edema and erythema resolved

A. aPTT 70 seconds

A nurse assesses a client who has chronic emphysema and HTN. Which ABG should be most consistent with this client's medical history? A. pH 7.37, PaCO2 62, HCO3 36, PaO2 65 B. pH 7.24, PaCO2 83, HCO3 24, PaO2 58 C. pH 7.27, PaCO2 44, HCO3 16 PaO2 91 D. pH 7.36, PaCO2 30, HCO3 19, PaO2 85

A. pH 7.37, PaCO2 62, HCO3 36, PaO2 65 Respiratory acidosis Compensated

Which nursing diagnosis should the nurse expect to see in a care plan for a client in sickle cell crisis?

Acute pain related to sickle cell crisis

During the neurological assessment, the nurse asks the client if she has any problems with her sense of smell. The client relates that she lives near a factory that emits an obnoxious odor, but that she no longer notices the smell. The nurse tells Joan that this phenomenon is normal and is called which of the following?

Adaptation

When developing a care plan for an older adult (age 65 and older), the nurse should consider which challenges faced by clients in this age-group?

Adjusting to retirement, deaths of family members, and decreased physical strength

Creating plan of care for child with acute lymphooid leukemia and an absolute netrophil count of 400/mm3

Administer granulocyte colony-stimulating factor to the child

Newly admitted child. Which of the following actions should the nurse include in the plan

Administer high dose antibiotic therapy

Some religious beliefs may conflict with prevalent health care practices. For example, what type of treatment is prohibited by the doctrine of Jehovah's Witnesses?

Administering blood transfusions

causes of adrenocortical insufficiency

Adrenal cortex destruction possibly due to: -Tumors - Infection such as tuberculosis, histoplasmosis, etc. - Addison's disease -an auto immune disease (the most common cause) and is more common in women than in men - Could also be due to the inability to synthesize hormones (Congenital adrenal hyperplasia) -inherited genetic defect that limits production of one of the many enzymes the adrenal glands use to make cortisol

Preparing an educational session about advocacy to a group of nurses

Advocacy is a leadership role that helps others to self actualize

A teenager states, "Old people are different. They don't need the same things that young people do." What is this statement an example of?

Ageism

Rural community health nurse developing a plan to improve health care delivery for migrant farmworkers.

Agency for Healthcare Research and quality

post op thyroidectomy

Airway clearance - laryngeal nerve damage, laryngospasm - requires an airway inserted - tracheotomy set is kept at bedside) - dyspnea - can also occur as a result of edema in the glottis or hematoma formation )surgical evacuation of the hematoma is required) - Bleeding - may be due to subcutaneous hemorrhage or a hematoma formation - (observe the sides and back of patient's neck plus the anterior dressing for bleeding) - monitor vital signs, c/o sensation of fullness or pressure at the incision site -Protect the incision - keep dressing intact - Risk of hypocalcemic tetany (The most common complication after total or near-total thyroidectomy secondary to hypoparathyroidism, which occurs in about a third of cases - When symptoms develop they can range from mild paresthesias to painful tetany and even life-threatening complications, such as laryngeal spasm or arrhythmia. - Symptomatic hypocalcemia is also the primary reason for a prolonged hospitalization after thyroidectomy. - A successful thyroid operation is dependent in part on preventing or effectively treating hypocalcemia-related symptoms: parathyroid trauma/removal - treat with calcium gluconate

Caring for older adult who is experiencing chronic anorexia and receiving tube feedings. Lab values that indicate additional nutrients are needed

Albumin 2.8 g/dL

Caring for a client in stage 4 of labor who is receiving oxytocin via continuous IV. Priority assessment

Amount of vaginal bleeding

What data indicates to the nurse that placental detachment is occurring?

An abrupt lengthening of the cord

Which of the following best describes a stimulus?

An act or agent that initiates a response by the nervous system

Each of the major religions has several characteristics in common. What is one of those characteristics?

An ethical code defines right and wrong.

An 86-year-old client with dementia is being discharged after treatment for a hip fracture. In reviewing the notes, the nurse identifies that the sole care-giver at home is an adult daughter with a moderate intellectual disability. Which is the most important action the nurse should ensure is in place before discharging the client home?

An immediate home visit is arranged with the visiting nurse service and the social worker.

An older adult client who has had a colostomy for over 10 years states, "I won't need any teaching about colostomies. I understand how to change the bag and care for my colostomy, but I'm not sure how to best clean my stoma." What does this statement indicate?

An incongruent relationship

Caring for a group of clients which one should the nurse see first

An older adult client who is anxious and attempting to pull out his IV line

The nurse is providing care for a patient who has benefited from a cochlear implant. The nurse should understand that this patient's health history likely includes which of the following? Select all that apply. A) The patient was diagnosed with sensorineural hearing loss. B) The patient's hearing did not improve appreciably with the use of hearing aids. C) The patient has deficits in peripheral nervous function. D) The patient's hearing deficit is likely accompanied by a cognitive deficit. E) The patient is unable to lip-read.

Ans: A, B

A boy age 13 years visits the school nurse's office and asks to speak privately with her. He looks very upset and embarrassed and struggles to make eye contact with the nurse. After some stuttered stops and starts, he finally asks about masturbating and if the things he has heard about it are true. The nurse provides correct education and information for the young man about masturbation by telling him which of the following? Choose all that apply. A) It is a technique of self-stimulation B) People do it regardless of age or gender C) Masturbation can lead to blindness D) Masturbation is not dirty or wrong E) Masturbation can decrease intelligence

Ans: A, B, D

One of the adverse events that Medicare will no longer reimburse the hospital for is an in-hospital fall. Fall prevention is a major part of nursing and risk management. In order to reduce the risk of falling, the nurse must: A) ensure that the patient wears his prescription glasses when up. B) post signs to alert staff to the patient at high risk for falls. C) always assist every patient with ambulation. D) assess the patient's fatigue level. E) monitor gait and balance.

Ans: A, B, D, E

The nursing instructor is talking with the junior nursing class about male reproductive issues. The instructor tells the students that the causes of erectile dysfunction include which of the following? Select all that apply. A) Alcoholism B) Spinal cord trauma C) Tadalafil D) Phosphodiesterase-5 inhibitors E) Diabetes

Ans: A, B, E

A public health nurse is teaching a health promotion workshop that focuses on vision and eye health. What should this nurse cite as the most common causes of blindness and visual impairment among adults over the age of 40? Select all that apply. A) Diabetic retinopathy B) Trauma C) Macular degeneration D) Cytomegalovirus E) Glaucoma

Ans: A, C, E

A patient is scheduled for enucleation and the nurse is providing anticipatory guidance about postoperative care. What aspects of care should the nurse describe to the patient? Select all that apply. A) Application of topical antibiotic ointment B) Maintenance of a supine position for the first 48 hours postoperative C) Fluid restriction to prevent orbital edema D) Administration of loop diuretics to prevent orbital edema E) Use of an ocular pressure dressing

Ans: A, E

A nurse who provides care in a long-term care facility recognizes the need to promote health rather than solely treating illness. Which of the following measures should the nurse encourage among the older adult resident population of the facility? Select all that apply. A) Encourage frequent naps in order to ensure adequate sleep and rest. B) Encourage residents to take dietary supplements when safe. C) Conduct activities at a slower pace and allow residents time to respond. D) Encourage residents to engage in the present rather than perform reminiscence. E) Promote self-care and only assist residents when it is necessary.

Ans: B, C, E

An older adult with a recent history of mixed hearing loss has been diagnosed with a cholesteatoma. What should this patient be taught about this diagnosis? Select all that apply A) Cholesteatomas are benign and self-limiting, and hearing loss will resolve spontaneously. B) Cholesteatomas are usually the result of metastasis from a distant tumor site. C) Cholesteatomas are often the result of chronic otitis media. D) Cholesteatomas, if left untreated, result in intractable neuropathic pain. E) Cholesteatomas usually must be removed surgically.

Ans: C, E

A woman complains of pain with intercourse. What client medications should the nurse check for that contribute to dyspareunia?

Antihistamines

Which measure would be most effective for the client to use at home when managing the discomfort of rhinoplasty 2 days after surgery?

Apply ice compresses.

What should a nurse do when administering pilocarpine?

Apply pressure on the inner canthus to prevent systemic absorption.

A male nurse is preparing to take the vital signs of a female patient. Which ethnic group would consider this improper?

Arab Muslim

A patient has informed the home health nurse that she has recently noticed distortions when she looks at the Amsler grid that she has mounted on her refrigerator. What is the nurse's most appropriate action?

Arrange for the patient to be assessed for macular degeneration.

An older adult patient has been diagnosed with macular degeneration and the nurse is assessing him for changes in visual acuity since his last clinic visit. When assessing the patient for recent changes in visual acuity, the patient states that he sees the lines on an Amsler grid as being distorted. What is the nurse's most appropriate response?

Arrange for the patient to visit his ophthalmologist.

Caring for 4 clients. Which task should be delegated to assistive personnel

Arrange the lunch tray for a client who has a hip fracture.

Patient refused a newly open fentanyl patch. Which of the following actions should the nurse take?

Ask another nurse to witness the disposal of the new patch.

When coaching a client to improve their health, which strategy is the most effective for the nurse to use to help clients take an active role in their health care?

Ask clients for their views of their health and health care. One of the best strategies to help empower clients to manage their health is to ask them their view of situations and to respond to what they say. This technique acknowledges that clients' opinions have value and relevance to the interview. It also promotes an active role for clients in the process. Use of a questionnaire or written instructions is a means of obtaining information but promotes a passive client role. Asking whether clients have questions encourages participation, but alone it does not acknowledge their views.

A nurse is making a home care visit to a client with a hearing deficit. What can she do to facilitate communication with the client?

Ask for permission to turn off the television set during the visit.

How would a nurse assess a client for pupillary accommodation?

Ask the client to focus on an object as it is brought closer to the nose.

As a component of a head to toe assessment, the nurse is preparing to assess convergence of the client's eyes. How should the nurse conduct this assessment?

Ask the client to follow her finger as she slowly moves it towards the client's nose.

Caring for a client who has gastrointestinal bleeding and an NG tube in place. Action that should be taken while performing gastric lavage.

Ask the client to lie on the left side

The nurse is providing discharge education to an adult patient who will begin a regimen of ocular medications for the treatment of glaucoma. How can the nurse best determine if the patient is able to self-administer these medications safely and effectively?

Ask the patient to demonstrate the instillation of her medications.

During discharge teaching the nurse realizes that the patient is not able to read medication bottles accurately and has not been taking her medications consistently at home. How should the nurse intervene most appropriately in this situation?

Ask the social worker to investigate community support agencies.

Assessing correct placement of a client's NG feeding tube prior to administering a bolus feeding.

Aspirate contents form the tube and verify the pH level

A patient has lost most of her vision as a result of macular degeneration. When attempting to meet this patient's psychosocial needs, what nursing action is most appropriate?

Assess and promote the patient's coping skills during interactions with the patient.

A client is admitted to the emergency department following an overdose of barbiturates. What should the nurse do first?

Assess ventilation and assist ventilation as needed.

A patient has just arrived to the floor after an enucleation procedure following a workplace accident in which his left eye was irreparably damaged. Which of the following should the nurse prioritize during the patient's immediate postoperative recovery?

Assessing and addressing the patient's emotional needs

A patient with mastoiditis is admitted to the post-surgical unit after undergoing a radical mastoidectomy. The nurse should identify what priority of postoperative care?

Assessing for mouth droop and decreased lateral eye gaze

An Asian American male client is operated on for gallstones. On the postoperative night, the nurse finds that the client is not sleeping and is tossing and turning. When asked about analgesics, the client expresses that he does not have pain. What nursing action is most appropriate?

Assessing for non-verbal expressions of pain

The mother of an 11-month-old boy who was born at 24 weeks' gestation is concerned about his size and motor skills. What information should the nurse provide?

Assessment of a premature infant's growth and development should be based on the adjusted age rather than the chronologic age. An 11-month-old who was born at 24 weeks' gestation (4 months early) should exhibit the growth and development of a 7-month-old infant.

The nurse and a colleague are performing the Epley maneuver with a patient who has a diagnosis of benign paroxysmal positional vertigo. The nurses should begin this maneuver by performing what action?

Assisting the patient into a sitting position

After mastoid surgery, an 81-year-old patient has been identified as needing assistance in her home. What would be a primary focus of this patient's home care?

Assisting the patient with ambulation as needed to avoid falling

The nurse is providing health education to a patient newly diagnosed with glaucoma. The nurse teaches the patient that this disease has a familial tendency. The nurse should encourage the patient's immediate family members to undergo clinical examinations how often?

At least once every 2 years

While admitting a clilent who is having elective surgery tomorrow, the nurse asks if he has a preferred religion or faith. The client indicates that he does not believe in a higher power and therefore has no preferred religion. The nurse knows that which of the following terms describes the client's feelings about religion?

Atheist

The nurse is admitting a patient to the unit who is scheduled to have an ossiculoplasty. What postoperative assessment will best determine whether the procedure has been successful?

Audiometry

Caring for a client receiving hemodalysis with AV fistula

Auscultate the affected extremity for a bruit

Assessing a newborn heart rate follow actions the nurse should take

Auscultate the apical pulse and count beats for at least 1 min.

Reviewing client's right with the nurses on the unit. Informed consent promotes which of the following ethical principles

Autonomy

A client underwent cataract removal with an intraocular lens implant. The nurse is giving the client discharge instructions. These instructions should include which of the following?

Avoid straining during bowel movements.

A nurse is teaching preventative measures for otitis externa to a group of older adults. What action should the nurse encourage?

Avoiding the use of cotton swabs

A multigavid client in labor at 38 weeks' gestation has been diagnosed with Rh sensitization and probably fetal hydrops and anemia. When the nurse observes the fetal heart rate pattern on the monitor, which of the following patterns is most likely? a. early deceleration pattern b. sinusoidal pattern c. variable deceleration pattern d. late deceleration pattern

B - The fetal heart rate of a multipara diagnosed with Rh sensitization and probably fetal hydrops and anemia will most likely demonstrate a sinusoidal pattern that resembles a sine wave. It has been hypothesized that this pattern reflects an absence of autonomic nervous control over the fetal heart rate resulting from severe hypoxia. This client will most likely requires a cesarean delivery to improve the fetal outcome. Early decelerations are associated with cord compression; and late deceleration are associated with poor placental perfusion

As a young adult single mother of a second-grade child has to make a decision regarding the teacher for her child will have in third grade and asks the nurse for advice: All other variables being equal which choice is best: a) A woman with 35 year old of teaching experience b) A man who is 40 years old

B) A man who is 40 years old

During a routine physical, an 11 year old tells the nurse that many students in school are "doing it". How should the nurse respond to this statement: a) Tell the client to talk with parents about sexual matters b) ASk what "doing it" means to the client

B) ASk what "doing it" means to the client

A client is hospitialized with numerous acute health problems. According to Maslow's Basic needs model, which nursing diagnosis would take the highest priority: a) Risk for injury related to unsteady gait b) Altered nutrition, less than body requirements related to inability to absorb nutrients c) Self-care deficit related to weakness and debilitation d) Powerlessness related to chronic disease state

B) Altered nutrition, less than body requirements related to inability to absorb nutrients

The most appropriate manner in which to state an intervention directed towards assisting a client with ambulation is: a) Assist patient with ambulation b) Ambulate with client, using gait belt, two times daily for 15 minutes

B) Ambulate with client, using gait belt, two times daily for 15 minutes

Which statement best reflects the nurse's assessment of the fifth vital sign: a) Do you have any complaints b) Are you experiencing any discomfort right now

B) Are you experiencing any discomfort right now

The nurse primarily uses the nursing process in the care of patient's: a) To explain nursing interventions to other health care professionals b) As a problem solving tool to identify and treat patient's health care problems c) As a scientific based process of diagnosing the patient's health care problems d) To establish nursing theory that incorporates the biopsychosocial nature of humans

B) As a problem solving tool to identify and treat patient's health care needs

A client tells the nurse that she does not want to get into the tub for a morning bath. The client has not been bathed for several days. What should the nurse do? a) Assign UAP the task of giving the client's bath b) ASk the client the usual way bathing occurs at home c) Skipping the patient's bath and documenting "refused" is not following at client-centered approach d) Tell the client that a bath is needed and ignore the client's comment

B) Ask the client the usual way bathing occurs at home

The daughter of an 80 year old man is aphastoc after suffering a cerebrovascular accident (stroke) express concern that their father is "always" exposing and playing with himself and his catheter. While they are in the room. Upon assessment the nurse finds the patient pulling on and rubbing his penis. What is the nurse's priority action: b) Assess the client's penis for irritations from the catheter c) ASk the client to keep his linens at waist level when he has visions

B) Assess the client's penis for irritations from the catheter

An older adult lives in a facility that provides, housing, group meals, personal care and support, social activities, and minimal health care services. What type of facility does this describe? A) Nursing home B) Assisted living C) Accessory apartment D) Home modification

B) Assisted living

One of the client's assigned to the nurse's care is to receive a medication that the nurse is not familiar with and is not not listed in the drug reference manual. The best action of the nurse is to: a) Follow the physician's order as written and give the medication b) Call the pharmacy and do further investigating before administering the medication c) Ask the client about this medication d) Call the physician and ask what the medication is and what it is used for

B) Call the pharmacy and do further investigating before administering the medication

After the nurse implements diet instructions for a patient with heart disease the patient can explain the information but fails to make recommended dietary changes. The nurse's evaluations that: a) Learning did not occur because the patient's behavior did not change b) Choosing not to follow the diet is the behaviors that resulted from learning c) The nursing responsibility for helping the patient make dietary changes has been fulfilled d) The teaching methods were ineffective in helping the patient learn the dietary information

B) Choosing not to follow the diet is the behaviors that resulted from learning

When the nurse is evaluating the fluid balance for a patient admitted for hypervolemia associated with multiple draining wounds, the most accurate assessment to include is: a) Skin turgor b) Daily weight c) Presence of edema

B) Daily Weight

Which are the following are normal physiological changes that occur during non- REM sleep: b) Decrease in pulse d) drop in basal metabolic rate

B) Decrease in pulse

A 43 year old is diagnosed with type 2 diabetes mellitus after being admitted to the hospital with an infected foot wound. When applying principles of adult learning, which teaching strategy by the nurse is most likely to be effective: a) Discuss the importance of blood glucose control in maintenance of long term health b) Demonstrate the correct method for cleaning and redressing the wound to the patient c) Assure the patient that the nurse is an expert on management of diabetes complications

B) Demonstrate the correct method for cleaning and redressing the wound to the patient

When preparing to teach an 82 year old Hispanic patient who lives with an adult daughter ways to improve nutrition, which action should the nurse take first: a) Ask the daughter about the patient's food preference b) Determine who shops for groceries and prepare meals

B) Determine who shops for groceries and prepare meals

Upon entering the room, the client is found crying along with the client's spouse. The nurse decides to sit with both of them, offering presence and listening to their fears instead of the planned education. This is an example of which of the following: B) Determining the nurse's needs for assistance c) Supervision delegated care d) Reassuring the client

B) Determining the nurse's needs for assistance

Two days after surgery for an Ileal conduit, the patient will not look at the stoma or participate in care. The patient insists that no one but the ostomy nurse specialists care for the stoma. The nurse identifies a nursing diagnosis of: a) Anxiety related to effects of procedure on lifestyle b) Disturbed body image related to change in body function

B) Disturbed body image related to change in body function

Nurses must use critical thinking in their day-to-day-practice, especially in circumstances surrounding client care and wise use of resources. In which of the following situations would critical thinking be most beneficial: a) Administering IV push medications to critically ill patients b) Educating a home health patient about treatment options c) teaching a new parent car seat safety d) Assisting an orthopedic client with the proper use of crutches

B) Educating a home health patient about treatment options

Which nursing intervention would be the most beneficial in preparing the patient psychologically for ileostomy surgery: a) Include the patient's family in preoperative teaching sessions b) Encourage the patient to express his or her concerns and to ask questions regarding the management of the ileostomy

B) Encourage the patient to express his or her concerns and to ask questions regarding the management of the ileostomy

An older patient receiving intravenous fluids at 175 mL/HR is demonstrating crackles, shortness of breath, and distended neck veins. The nurse recognizes these findings as being which complication of intravenous fluid therapy: b) Fluid volume excess c) Pulmonary embolism

B) Fluid volume excess

Which statement indicates the client needs a sensory aid in the home: a) I tripped over that throw rug again b) I can't hear the doorbell

B) I can't hear the doorbell

The nurse is caring for a client who uses cathartics frequently. Which statement made by the client indicates an understanding of the discharge teaching: a) In the future I will eat a banana every time I take the medication b) I don't have to have a bowel movement every day

B) I don't have to have a bowel movement every day

A patient who is suspected of experiencing respiratory distress from a left-sided pneumothorax should be positioned: a) On the right side b) In semi-fowler's position

B) In the Semi-Fowler's Position

The nurse teaching a 32 year old man with renal failure about the path physiologic mechanism of acid-base balance recognize that the instructions have been understood when the client says: a) I lose too much acid through my kidneys b) My breathing increases to correct imbalances

B) My breathing increases to correct imbalances

What is wrong with the following outcome? Client will be able to climb one flight of stairs without shortness of breath: a) Nothing is wrong b) No target time is given

B) No target time is given

A patient returns to the clinic with recurrent dysuria after being treated with trimethoprium and sulfamethoxazole (Bactrim) for 3 days. Which action will the nurse plan to take: a) remind the patient about the need to drink 1000 mL of fluids daily b) Obtain a midstream urine specimen for culture and sensitivity testing

B) Obtain a midstream urine specimen for culture and sensitivity testing

When the client has arrived at the nursing unit from surgery, the nurse is most likely to give priority to which of the following assessments? a) pain tolerance b) Pain intensity

B) Pain Intensity

A client has been having pain without any clear pathology for cause. The most appropriately written nursing diagnoses for this client would be which of the following: a) Pain due to unknown factors b) Pain related to unknown etiology c) Pain caused by psychosomatic condition d) Pain manifested by client's report

B) Pain related to unknown etiology

A client has joined a fitness club and is working with the nurse to design a program for weight reduction and increased muscle tone. The client has tried exercise in the past with success, but has not been participating in a program for some time. In order to assess the potential for success with this client, the nurse should evaluate which of the behavior- specific conditions: a) Interpersonal influences b) Perceived benefits of action c) Situational influences

B) Perceived benefits of action

What is primary function of a family? a) Provide everything each member wants b) Provide an environment that supports growth of individuals

B) Provide an environment that supports growth of individuals

The newly admitted client has contractures of both lower extremities. What nursing intervention should be included in the client's plan of care: a) Weight-bearing activities to stimulate joint relaxation b) Range of motion exercises to prevent worsening of contractures c) Exercises to strengthen flexor muscles

B) Range of motion exercises to prevent worsening of contractures

Because of significant concerns about financial problems a middle-aged client complains of difficulty sleeping. Which outcome would be the most appropriate for the nursing care plan? By day 5, the client will: b) Report falling asleep within 20 to 30 minutes c) Have a plan to pay all bills

B) Report falling asleep within 20 to 30 minutes

A client is attending classes on building positive relationships with significant others as well as learning skills to be open minded and respectful to those whose opinions are different. This client is focusing on which component of wellness: a) Physical b) Social c) Emotional d) Environment

B) Social

Which action can the nurse delegate to nursing assistive personnel (NAP) who help with treatment of a patient admitted with tuberculosis and placed on airborne precautions: a) Teach the patient about how to use tissues to dispose of respiratory secretions b) Stock the patients room with all necessary personal protective equipment c) Interview the patient to obtain the names of family members and close contacts d) tell the patient's family members the reason for the use of airborne precautions

B) Stock the patient's room with all the necessary personal protective equipment

The nurse is organizing a wellness project to educate teenagers about keeping their bodies healthy. Which information about diet and exercise should be included: a) Diet is the most important predictor of health b) The most important factors for maintaining health are diet and activity

B) The most important factors for maintaining health are diet and activity

A patient with poor circulation to the feet requires teaching about foot care. Which learning goal should the nurse include in the teaching plan? a) The nurse will demonstrate the proper technique for trimming toenails b) The patient will list three ways to protect the feet from injury by discharge d) The patient will understand the rationale for proper foot care after instructions

B) The patient will list three ways to protect the feet from injury by discharge

The nurse is preparing written handouts to be used as part of the standardized teaching plan for patient's who have been recently diagnosed with diabetes. Which of the following statements would be appropriate to include in the handouts: a) Polyphagia, polydipsia, and polyuria are common symptoms of Diabetes mellitus b) The use of the right foods can help in keeping blood glucose at a near-normal level c) Some diabetes control blood glucose with oral medications or nutritional interventions d) Diabetes mellitus is characterized by chronic hyperglycemia and the associated symptoms

B) The use of the right foods can help in keeping blood glucose at a near-normal level

Which factor reduces the risk of electrical hazards: a) two-pronged electrical plugs b) Three-prolonged electrical plugs

B) Three-prolonged electrical plugs

The nurse notes that the tube fed client has shallow breathing and dusky color. The feeding is running at the prescribed rate. What is the nurses priority action: a) Place the client in high fowler's position b) Turn off tube feeding d) Assess the patient's bowel sounds

B) Turn off the tube feeding

The nursing diagnosis Risk for Impaired Skin Integrity related to sensory-perception disturbance would best fit a client who: a) Cut a foot by stepping on broken glass b) Uses a wheelchair due to paraplegia

B) Uses a wheelchair due to paraplegia

An oxygen delivery system is prescribed for a client with chronic obstructive pulmonary disease (COPD) to deliver a precise oxygen concentration. Which of the following types of oxygen delivery systems would the nurse anticipate to be prescribed: a) Face tent b) Venture Mask

B) Venture Mask

When reviewing both the client's problem list against the various identified nursing diagnoses, both of which include client and family input, the nurse is utilizing of the following processes to minimize diagnostic error: a) Understanding what is normal vs. what is not normal b) Verifying c) Consulting resources d) Basing diagnoses on patterns

B) Verifying

A patient with frequent urinary tract infections ask the nurse how she can prevent the reoccurence. The nurse should teach the client to: a) Douche after intercourse b) Void every three hours

B) Void every three hours

The nurse is caring for the patient with clostridum difficile. Which intervention should the nurse implement to prevent nosocomial spread to other clients: a) Wash hands with betadine for 2 min after giving care b) Wear nonsterile gloves when handling GI excretions

B) Wear nonsterile gloves when handling GI excretions

To assess a patient's readiness to learn before planning, teaching activities, which question should the nurse ask: a) What kind of work and leisure activities do you do b) What information do you think you need right now c) Do you have any religious beliefs that are inconsistent with the treatment

B) What information do you think you need right now

During an initial interview the client makes this statement, I'm really not that sick or in pain right now. The nurses best response is: a) It's ok to be worried surgery is a big step b) What kind of questions do you have about your surgery c) I think these are things you should be asking your doctor d) have you had surgery before

B) What kind of questions do you have about your surgery

The nurse's client today informs her that he receives regular visits from a parish nurse, who helps him with his diet-controlled diabetes. The nurse is aware that parish nurses do which of the following? Choose all that apply. A) Provide medicine at no charge B) Refer clients to appropriate resources C) Perform physical assessments D) Promote personal responsibility for health E) Provide health education

B, D, E

The U.S. Religious Landscape Survey of 35,000 Americans by the Pew Research Center's Forum on Religion &amp; Public Life (2008) finds that most Americans are religious and they have a nondogmatic approach to faith. Which of the following statements accurately describes religion in America? Select all that apply. A) A majority of Americans who are affiliated with a religion believe their religion is the only way to salvation. B) More than half of Americans rank the importance of religion very highly in their lives, attend religious services regularly, and pray daily. C) A plurality of adults who are affiliated with a religion want their religion to adjust to new circumstances or adopt modern beliefs and practices. D) Significant minorities across nearly all religious traditions see a conflict between being a devout person and living in a modern society. E) While more than 50% of Americans believe in the existence of God or a universal spirit, there is considerable variation in the nature and certainty of this belief. F) Sixty percent of adults believe that God is a person with whom people can have a relationship; but one out of four—including about half of Jews and Hindus—see God as an impersonal force.

B, D, F

A client is schedule for an endoscopy the following morning. Effective teaching? A. "The oral laxative may be mixed with lemon or lime soda." B. "I will not eat or drink anything after midnight." C. "My stools may be chalky white for a couple of days." D. "After the procedure, I will take my daily medicine."

B. "I will not eat or drink anything after midnight."

A nurse provides care for a child admitted with pain crisis r/t sickle cell anemia. What is priority 1? A. Start oxygen B. Administer IV fluids C. Apply warm compresses D. Send type and cross

B. Administer IV fluids

A client develops welling of eyes, face, tongue, and lips after administering PCN IV. Which action should the nurse perform first? A. Give diphenhydramine 25 mg IV B. Administer epinephrine 0.2 ml IM C. Raise HOB 45 degrees D. Prepare to administer 1L fluid bolus

B. Administer epinephrine 0.2 ml IM

A client reports a 3 day hx of fever, fatigue, and HA. Which action should the nurse implement after a dx of Lyme disease is confirmed? A. Give acetaminophen B. Administer oral doxycycline C. Place on contact precautions D. Decrease stimuli in environment

B. Administer oral doxycycline

A client is admitted to the ICU following an open reduction and internal fixation of the right femur. The client is suddenly agitated and reports chest pain and SOB. Petechiae are present over the chest. Which action should the nurse perform immediately? A. Obtain pulse ox reading B. Administer oxygen via facemask C. Give methylprednisolone IV D. Request therapist to provide an albuterol aerosol

B. Administer oxygen via facemask

A nurse is organizing care for a group of clients. Which of the following should the nurse assign to a UAP? A. Record a client's VS during blood transfusion B. Assist a client who is requesting a bedpan 1 day after hysterectomy C. Offer a pamphlet regarding advanced directives to a newly admitted client D. Ask a client if pain was relieved after administration of acetaminophen

B. Assist a client who is requesting a bedpan 1 day after hysterectomy

A client is alert and oriented but anxious and short of breath. After vagal maneuvers and medication administration, the cardiac rhythm (ventricular tachycardia) has not changed. The nurse should prepare to assist with which procedure? A. Defibrillation B. Cardioversion C. Pacemaker insertion D. Echocardiogram

B. Cardioversion

A nurse receives report and recognizes which client is priory? A. Murphy's sign in a client who has abd pain B. Chvostek's sign in a client with hypoparathyroidism C. Na 146 in a dehydrated client D. Mg 1.1 in an alcoholic client

B. Chvostek's sign in a client with hypoparathyroidism

A nurse is observing a client attempt 3 point crutch walking. Which of the following should be of concern to the nurse? A. Client keeps arm flexed B. Client moves leg with opposite arm C. Client backs up to a chair before sitting D. Client asks for a height adjustment for crutches

B. Client moves leg with opposite arm

Which findings should the nurse recognize as expected physiological changes in the older adult client? Select all that apply A. Increased need for sleep B. Decreased hearing acuity C. Increased night blindness D. Decreased ROM E. Increased sensitivity to cold

B. Decreased hearing acuity C. Increased night blindness D. Decreased ROM E. Increased sensitivity to cold

A nurse admits a client who is terminally ill for palliative tx. Which actions are required to implement the advance directive (living will and POA/surrogate/proxy)? Select all that apply A. Provide written info to family B. Document advance directive status C. Ensure living will reflects current decisions D. Notify members of health care team regarding status E. Change allow natural death (AND) status based on provider decision

B. Document advance directive status C. Ensure living will reflects current decisions D. Notify members of health care team regarding status

While d/c a sheath placed in the left femoral artery, a client bleeds profusely. After stabilizing the client, the nurse should clean up any remaining blood using: A. Industrial detergent with a biohazard category 1 rating B. EPA registered sodium hypochlorite product C. Chlorine bleach solution of 5:5 ratio measurement D. Equal parts solution of ammonia and chloride

B. EPA registered sodium hypochlorite product

A nurse participates in a health screening for a group of young adults. Which findings are risk factors for cardiac disease? Select all that apply A. BMI 24 B. Heart disease in close relative C. BP 165/88 D. Walking for 30 min, 5x/week E. Using smokeless tobacco products F. Females who take estrogen for birth control

B. Heart disease in close relative C. BP 165/88 E. Using smokeless tobacco products

A client has quadriplegia secondary to a spinal cord injury (above T6). Which of the following interventions should the nurse incorporate to prevent autonomic dysreflexia? Select all that reply A. Instruct client to wear a medic alert bracelet B. Keep indwelling catheter free of kinks C. Monitor BP for hypertensive changes D. Encourage high fiber diet and administer stool softeners E. Perform DRE if no daily bowel movement

B. Keep indwelling catheter free of kinks D. Encourage high fiber diet and administer stool softeners

A child is entering college and the parents ask about the need for a meningococcal conjugate vaccine. Which factor should guide the nurse's response? A. URI are more common on college campuses B. Living in a dorm increases risk of exposure to the disease C. Adults who contract meningitis frequently have complications D. Receiving the vaccine provides guaranteed protection

B. Living in a dorm increases risk of exposure to the disease

A school aged child who reports a stomach ache has bruises on his upper arms. When questioned, the child states, "My mom hurt me." What should the nurse do first? A. Document findings B. Notify child protective services C. Care for child's immediate needs D. Identify supportive resources for mother

B. Notify child protective services

A nurse provides care for a client who recently had a tracheostomy placed. Which equipment should be placed at the bedside? Select all that apply A. Nasal cannula B. O2 C. Bag valve mask D. Suction equipment E. 2 tracheostomy tubes

B. O2 C. Bag valve mask D. Suction equipment E. 2 tracheostomy tubes

A client is prescribed levothyroxine. Which of the following symptoms should concern the nurse most? A. Weight loss B. Palpitations C. Heat intolerance D. Increased appetite

B. Palpitations

A client has not voided 8h following removal of an indwelling bladder catheter. Which of the following should be the nurse's initial action? A. Increase fluids B. Perform bladder scan C. Place indwelling catheter D. Provide assistance to bathroom

B. Perform bladder scan

A nurse is caring for a client who is scheduled for electroconvulsive therapy (ECT: brain cells stimulated to fire neurotransmitter- series of ECT). Which of the following medications should the nurse withhold prior to therapy? A. Atropine sulfate B. Phenytoin C. Methohexital D. Succinylcholine

B. Phenytoin

A nurse is caring for a client who is 24h post-acute MI and reports, "I can't breathe now tha ti am lying down after lunch." Which of the following should the nurse's initial action be? A. Administer IV furosemide B. Place client tin high fowlers C. Begin oxygen 4-6L NC D. Auscultate anterior and posterior lungs bilaterally

B. Place client tin high fowlers

An adolescent who has sickle cell anemia is admitted for cellulitis of the leg. The nurse should prioritize which of the following interventions? A. Applying cold compresses to affected area B. Planning activities that encourage energy conservation C. Administering oral and parenteral rehydration therapies D. Educating client and family about prevention of crisis

B. Planning activities that encourage energy conservation

A nurse receives a request from 4 clients at the same time. which of the following clients should the nurse address first? A client who: A. Needs to void 1h after removal of an indwelling urinary catheter B. Report restlessness and SOB postop fractured femur C. Asks for a stool softener 2 days after an MI D. Demands to take prescribed insulin

B. Report restlessness and SOB postop fractured femur

A nurse admits a client who sustained a C3 spinal cord injury. Which of the following findings should the nurse recognize as priority of care? A. HR 52/min B. Respirations 10/min C. Temperature 97.0F (36.0C) D. BP 88/54 mmHg

B. Respirations 10/min

A client receives a transfusion of PRBCs and tells the nurse, "My IV site is painful and looks like it is swollen." Which of the following actions should the nurse take? A. Double check the blood type with the unit of blood with another nurse B. Start a new IV at another site and resume transfusion at the new site C. d/c transfusion and send remaining blood and tubing to lab D. Continue to monitor site for s/s infection or infiltration

B. Start a new IV at another site and resume transfusion at the new site

A client who has a PIV is scheduled for a platelet count to be drawn. After cleansing the site, which action should the nurse take? A. Clamp catheter and change lumen cap B. Withdraw 10 ml of blood and discard it C. Access site with a non-coring needle D. Flush catheter with 10 ml of sterile water

B. Withdraw 10 ml of blood and discard it

All of the following are factors to consider when caring for clients with limited income. Which one is the most important?

Basic human needs may go unmet

Which of the following nurse's actions carries the greatest potential to prevent hearing loss due to ototoxicity?

Be aware of patients' medication regimens and collaborate with other professionals accordingly.

A nurse is caring for a client who is unconscious. Which of the following is a recommended guideline for communication with this client?

Be careful what is said in front of the client as he or she might hear you.

For a diabetic client with a foot ulcer, the physician orders bed rest, a wet-to-dry dressing change every shift, and blood glucose monitoring before meals and at bedtime. Why are the wet-to-dry dressings used for this client?

Because they debride the wound and promote healing by secondary intention.

A 78-year-old female patient has been scheduled for outpatient cataract surgery. Which of the following signs and symptoms most likely prompted the patient to initially seek care?

Bilateral blurred vision and visual distortion

Which of the following health care practices may be influenced by a young woman's religion?

Birth-control measures

Developing discharge plan for school age child with thrombocytopenia. What should the child avoid

Blowing the nose it increases the risk for bleeding or hemorrhaging.

Assessing a client who has depressive disorder taking amitriptyline. Identify adverse effects.

Blurred vision

The nurse is caring for an elderly female with osteoporosis. When teaching the client, the nurse should include information about which major complication?

Bone fracture

Assessing a client with history of seeking counseling for relationship problems. Shows the nurse multiple superficial self-inflicted lacerations on her forearms. This behavior should be identified as a characteristic of which personality disorder

Borderline-emotionally unstable, have troubled interpersonal relationships and often engage in harmful behaviors such as cutting, substance use and suicidal thoughts.

The client suffered a myocardial infarction (MI) and has shared with the nurse that he is reluctant to resume sexual activity. He is worried about having another MI. The nurse discusses various methods of sexual expression and points out that the most important body area for sexual arousal and stimulation is which of the following?

Brain

A client with Tourette syndrome is seen in an outpatient clinic. The client has multiple tics occurring several times per day. The nurse notices that the client has a difficult time completing tasks such as activities of daily living (ADLs). In which of the following ways can the nurse best help this client?

Break down tasks into small achievable steps

Dietary teaching for a new prescription of phenelzine

Broccoli, Yogurt, Cream Cheese

Assess 1 wk old for palmer grasp reflex

By touching the palms of the infant's hands near the base of the digits causing the flexion of the fingers

A 40-year-old executive who was unexpectedly laid off from work 2 days earlier complains of fatigue and an inability to cope. He admits drinking excessively over the previous 48 hours. This behavior is an example of: a. alcoholism b. manic episode c. situational crisis d. depression

C - A situational crisis results from a specific event in the life of a person who is overhwlemed by the situation and reacts emotional. Fatigue, insomnia, and inability to make decisions are common signs and symtpoms. The situations crisis may precipitate behavior that causes a criss (alcohol or drug abuse). There isn't enough information to label this client an alcoholic. A manic episodes is characterized by euphoria and labile effect. Symptoms of depression are usually present for 2 or more weeks.

The nurse uses the PLISSIT format in helping client's who have sexual dysfunction. Which action by the nurse best reflects the "P" section of this format: a) ASk the physician for permission to discuss sexual topics with the client c) Acknowledge the clients spoken and unspoken sexual concerns when providing care

C) Acknowledge the clients spoken and unspoken sexual concerns when providing care

The client being admitted from the ED is diagnosed with a fecal impaction. Which nursing intervention should be implemented: c) Administer an oil retention enema d) Prepare for an UGI X-ray

C) Administer an oil retention enema

The nurse is caring for a patient diagnosed with pneumonia who is having shortness of breath and difficulty breathing. Which intervention should the nurse implement first: a) Take the client's vital signs b) Check the client's pulse oximetry c) Administer oxygen via nasal cannula

C) Administer oxygen via nasal cannula

A patient is receiving tube feedings through a percutaneous endoscopic gastrostomy (PEG). Which action will the nurse include in the plan of care: a) Keep the patient positioned on the left side b) Obtain a daily x-ray to verify tube placement c) Check the gastric residual volume every 4 to 6 hours

C) Check the gastric residual volume every 4 to 6 hours

The client experienced female circumcision as a puberty ritual while living in Africa as a child. What condition should the nurse monitor the client as an adult: c) Chronic urinary tract infection d) Tendency for postpartum hemorrhage

C) Chronic Urinary Tract Infection

A client with acute pancreatitus has an abnormally low serum calcium level. During a bath the nurse cleans the client's face with a cloth, and the lips, nose, and side of the face. When documenting this information the nurse would state that the patient's facial twitching indicates the presence of: c) Chrostek's sign d) Bell's palsey

C) Chrostek's Sign

An 80 year old client is transferred to a long term care facility. On the second night, he becomes confused and agitated. What is the most appropriate nursing diagnosis? c) Disturbed Sensory Perception d) Disturbed Thought Process

C) Disturbed Sensory Perception

The client is admitted to a comprehensive rehabilitation center for continuing care,following a motor vehicle crash. While the admitting nurse will develop the initial care who will be involved with the ongoing planning of this client's care: a) The admitting nurse continues to assume that responsibility b) All nurses who work with the client c) Everybody involved in the client's care d) The client and the client's support system

C) Everybody involved in the client's care

The nurse is caring for an 80 year old patient with the medical diagnosis of heart failure. The patient has edema, orthopnea, and confusion. Which nursing diagnosis is most appropriate for this client: c) Excess fluid volume related to retension of fluids as evidence by edema and orthopnea d) Excess fluid volume related to cognitive heart failure as evidence by edema and confusion

C) Excess fluid volume related to retension of fluids as evidence by edema and orthopnea

A 76 year old patient has an open surgical wound on the abdomen that contains a creamy exudate and small areas of deep granulation tissue. The nurse documents the wound as a: a) Red wound b) Yellow wound c) Full thickness wound

C) Full thickness wound

Which question should the nurse ask when assessing a patient who has a history of benign prostatic hyperplasia (BPH): c) Has there been a decrease in the force of your urinary stream d) Have you been experiencing any difficulty in achieving an erection

C) Has there been a decrease in the force of your urinary stream

While assisting the client with a bath, the nurse encourages full range of motion in all the client's joints. Which activity would best support range of motion in the hand and arm: b) Move the wash basin farther toward the foot of the bed so the client must reach c) Have the client brush their hair and teeth d) Move each of the patient's hand and arm joints through passive range of motion

C) Have the client brush their hair and teeth

The nurse obtains this information when assessing a 74 year old patient in the outpatient clinic. Which finding os of the highest priority when the nurse is planning care for the patient: c) History of recent loss of balance and fall d) Complaint of left hip aching when jogging

C) History of recent loss of balance and fall

Wanting to know more about the client's pain experience, the nurse continues to explore different questioning techniques. Which of the following is the best example of an open-ended question for this situation: a) Is your pain worse at night b) What brought you to the clinic c) How has the pain impacted your life d) You're feeling down about having pain, aren't you

C) How has the pain impacted your life

Which statement made by a post menopausal client, would the nurse evaluate as indicating the need for further assessment: a) For some reason, I have more sexual desire than ever c) I am so glad that I don't need to worry about sex anymore d) Sex certainly takes longer that it used to, but im getting used to that

C) I am so glad that I don't need to worry about sex anymore

When learning how to implement the nursing process into a plan of care for a client, the student nurse realizes the part of the purpose of the nursing process is to: a) Deliver care to a client in an organized way b) Implement a plan that is close to the medical model c) Identify client needs and deliver care to meet those needs d) Make sure that standardized care is available to clients

C) Identify client needs and deliver care to meet those needs

The 45 year old client reports that she has no interest in sex and that she and her husband have not had intercourse in 16 years. How does the nurse interpret this assessment data: c) If both partners share the same lack of desire there is often not a problem d) This situation is so unnatural that some dysfunction is present

C) If both partners share the same lack of desire there is often not a problem

The client has a documented Stage III pressure ulcer on the right hip. What NANDA nursing diagnosis problem statement is most appropriate for use with this client: c) Impaired tissue integrity d) Risk for Injury

C) Impaired Tissue Integrity

A patient with a stroke is paralyzed on the left side of the body and has developed a pressure ulcer on the left hip. The best nursing diagnoses for this patient is: a) Impaired physical mobility related to left-sided paralysis b) Risk for impaired tissue integrity related to left-sided weakness c) Impaired skin integrity related to altered circulation and pressure d) Ineffective tissue perfusion related to inability to move independently

C) Impaired skin integrity related to altered circulation and pressure

A nurse is providing a back rub to a client just after administering a pain medication, with the hope that these two actions will help decrease the client's pain. Which phase of the nursing process is this nurse implementing: a) Assessment b) Diagnosis c) Implementation d) Evaluation

C) Implementation

Upon assessment the nurse notes that the client is dyspneic; has bibasilar crackles, and tires easily upon exertion. Which nursing diagnosis is best supported by these assessment details: b) Anxiety c) Ineffective airway clearance d) Impaired gas exchange

C) Ineffective Airway Clearance

A client just had a baby following a long labor and difficult delivery. Which of the following nursing diagnoses is formulated correctly: a) Constipation, due to tissue trauma, manifested by no bowel movements for two days b) Risk for infection, because of new incision, related to episiotomy c) Ineffective breast feeding, related to lack of motivation, secondary to exhaustion d) Altered urinary elimination, secondary to childbirth

C) Ineffective breast feeding, related to lack of motivation, secondary to exhaustion

An example of correctly written nursing diagnoses statement is: a) Altered tissue perfussion related to heart failure b) Risk for impaired tissue integrity related to sacrel redness c) Ineffective coping related to response to biopsy test results d) Altered urinary elimination related to urinary tract infection

C) Ineffective coping related to response to biopsy test results

The client who is bed-bound complains of abdominal pain. Bowel sounds are present. What action should be taken by the nurse: b) Palpate for bladder fullness c) Inspect the sacrel area for edema d) Use the PRN order to medicate the client with an antacid

C) Inspect the sacrel area for edema

Which nursing intervention should be applied to a client with a nursing diagnosis of Risk for Skin Integrity impairment related to immobility: a) Encourage client to eat at least 40% of meals b) Restrict fluid intake c) Keep lines dry and wrinkle free

C) Keep linens dry and wrinkle free

A client is exhibiting signs and symptoms of acute confusion/delirium. Which strategy should the nurse implement to promote a therapeutic environment: c) keep the room organized and clean d) Use restraints for client safety

C) Keep the room organized and clean

Which information noted by the nurse when caring for a patient with a bladder infection is most important to report to the health care provider: c) Left-sided flank pain d) Temp 100.1" F

C) Left-Sided Flank Pain

A client who describes his pain as 6 on a scale of 1 to 10 is classified as having which of the following: c) Moderate to severe pain d) Very severe pain

C) Moderate to severe pain

The nurse is collecting information from a client's family. The client is confused and not able to contribute to the conversation. The spouse's states, "This is not normal behavior". The nurse documents this is which of the following: a) inference b) Subjective data c) Objective data d) Secondary subjective

C) Objective data

When assessing the musculoskeletal system the nurse's initial action will usually be to: b) Have the patient move the extremities against resistance c) Observe the patient's body build and muscle configuration

C) Observe the patient's body build and muscle configuration

All of the following nursing actions are included in the plan of care for the patient who is malnourished. Which action is appropriate for the nurse to delegate to nursing assistive personnel (NAP): c) Offer the patient the prescribed nutritional supplement between meals d) Assess the patient's strength while ambulating the patient in the room

C) Offer the patient the prescribed nutritional supplement between meals

Which behavior is characteristic of someone who is coping well with stress: c) Sets aside 30 min a day to exercise d) has no hobbies

C) Sets aside 30 min a day to exercise

How should the nurse use the JCAHO 2006 National Patient Safety Goals to improve communication among caregivers: a) Review a list of look-a-like sound-a-like drugs used in the organization c) Studying a list of abbreviations that are not to be used throughout the organization d) Use the client's room number as an identifier

C) Studying a list of abbreviations that are not to be used throughout the organization

The aspect of an older adult's history indicating a risk, for developing hyperatremia is that the client: c) Takes an over the counter antacid d) Has had frequent urinary tract infections

C) Takes an over the counter antacid

In planning preoperative teaching for a patient undergoing a Roux-en y gastric bypass as treatment for morbid obesity the nurse places the highest priority on: b) Discussing the necessary postoperative modifications in lifestyle c) Teaching the patient proper coughing and deep breathing techniques

C) Teaching the patient proper coughing and deep breathing techniques

The nurse assesses an open area over a patient's greater trochanted that is approximately 10 cm in diameter. The tissue around the area is edematous and feels boggy. The edges of the wound cup in toward the center. Which additional findings would indicate to the nurse that this is a Stage IV pressure ulcer: b) The crater extends into the subcutaneous tissue c) The joint capsule of the hip is visable

C) The joint capsule of the hip is visable

Which information obtained during the nurse assessment of the patient's nutritional- metabolic pattern may indicate the risk for musculoskeletal problems: c) The patient is 5 ft. 2 inches and weighs 180 lbs. d) The patient prefers whole milk to nonfat milk

C) The patient is 5 ft. 2 inches and weighs 180 lbs.

The nurse is writing the plan of care for a client who is confined to bed. Which intervention should be included to help reduce the effects of shearing forces on the client's skin: b) Coat the patient's back and buttocks with baby powder after bathing c) Use a turn sheet lifted by two staff member to move the client in bed

C) Use a turn sheet lifted by two staff member to move the client in the bed

The client has been close to death for some time and the family asks how the nurse will know when the client has actually died,. Which of the following would be the most accurate response from the nurse: c) When there is no apical pulse d) When the extremities are cool and dark in color

C) When there is no apical pulse

Prior to finalizing a family orientated nursing care plan and implementing interventions, it is essential for the nurse to perform which of the following: a) Meet with all family members simultaneously c) establish a trusting relationship with the family as a group

C) establish a trusting relationship with the family as a group

Dementia is a disorder that progresses over several years, with increasing confusion, forgetting family, and disorientation in familiar surroundings. A common problem with dementia patients is sundowning syndrome, which is described as ... A) a behavior change at sunset as the client becomes more fatigued, listless, and disoriented. B) occasional onset of marked confusion, wandering and feeling lost during the afternoon, before sunset. C) habitual agitation, restlessness, and confusion that occurs after dark. D) increasing sleeplessness at night because the patient cat-naps during the day.

C) habitual agitation, restlessness, and confusion that occurs after dark.

A client who has cirrhosis and elevated serum ammonia levels is hospitalized with dehydration. The client asks for a larger portion of meat with each meal. How should the nurse respond? A. "I can ask the dietician to send you larger meat portions." B. "It would be better to increase your intake of fluids." C. "Protein intake needs to remain the same, but you can have more side dishes." D. "Your calories need to be limited. Would you like sugar free gelatin?"

C. "Protein intake needs to remain the same, but you can have more side dishes."

A client who has bipolar disorder reports a desire to purchase a car for a friend. How should the nurse respond? A. "Will your friend be here soon?" B. "That is very kind and generous of you." C. "This really requires further discussion." D. "Can you remain financially stable if you assist your friend?"

C. "This really requires further discussion."

A nurse provides care for a neonate delivered at 28 weeks gestation. After post-delivery assessment and prior to transferring the neonate to NICU, what is priority 1? A. Repeat VS B. Assess involution progression C. Allow mother to see and touch neonate (move quickly) D. Place neonate skin to skin with mom

C. Allow mother to see and touch neonate (move quickly)

A nurse enters the client's room who is the foot of the bed, lying on the floor. Which of the following should be the initial nursing action? A. Examine client for injuries B. Obtain pulse and BP C. Assess VS and LOC D. Determine intensity of pain with ROM

C. Assess VS and LOC

4 days after a ventral hernia repair, a client who is obese (stress on sutures) and has a hx of COPD vomits and reports severe abdominal pain. The oxygen saturation is 90%. Which of the following actions should the nurse implement first? A. Give ondansetron hcl IV B. Encourage pursed lip breathing C. Assess surgical incision site D. Apply low dose oxygen via NC

C. Assess surgical incision site

10 days after chemotherapy, a client's WBC is 1,000. Which discharge instructions should the nurse provide? Select all that apply Increase intake of raw fruits and vegetables A. Avoid using public transport B. Sanitize personal toothbrush daily C. Avoid cleaning cages of household pets D. Wear mask when around other people

C. Avoid cleaning cages of household pets D. Wear mask when around other people

A nurse provides teaching to a client who is 40 weeks gestation and scheduled to receive dinoprostone. Which of the following information should be discussed? A. An indwelling catheter will be inserted B. Terbutaline is given after contractions begin C. Bed rest is required for 15-30 minutes after application D. Oxytocin will be administered within 4h of first dose

C. Bed rest is required for 15-30 minutes after application

A nurse arrives at a work site explosion. Which of the following clients should be triaged first? A client who has: A. Open fracture reporting pain level 10 B. Fixed pupils and agonal respirations C. Burns to face with respiratory stridor D. Type 2 DM who is disoriented

C. Burns to face with respiratory stridor

A 2 y/o is admitted to the hospital for elective sx. Which info should be highest priority for the nurse to document on the plan of care? A. Child's ability to separate from parents B. Parent's understanding of child's hospitalization C. Child's ritual and routines at home D. Parent's religious practices and beliefs

C. Child's ritual and routines at home

A client who has stage 1 Alzheimer's disease exhibits signs of irritability and is reluctant to perform hygiene care. Which action should the nurse include when counseling the family? A. Be matter-of-fact and insist on tasks to be completed B. Suggest family hire personal assistant to provide care C. Establish and maintain routine for daily hygiene activities D. Ask provider for additional medication to use PRN

C. Establish and maintain routine for daily hygiene activities

A nurse prepares to administer meds to a client who has asthma. Which of the following effects should the nurse recognize as an adverse response to bronchodilator therapy? A. Limited routes of administration B. Hyperkalemia C. Increased myocardial oxygen use D. Hypoglycemia

C. Increased myocardial oxygen use

A nurse completes a physical assessment on a client. Which finding is priority 1? A. Beau's lines on left index fingers B. Several papules on lower abdomen C. Increased tactile fremitus of thorax D. Cerumen accumulation in right ear canal

C. Increased tactile fremitus of thorax

A client's blood type is AB+ and the nurse observes the blood infusing is labeled type B-. Which action should the nurse implement? A. Stop transfusion immediately B. Prepare to administer antipyretics C. Monitor client for any adverse reactions D. Check client for adventitious breath sounds

C. Monitor client for any adverse reactions

A nurse should recognize which symptom as an adverse effect associated with long-term use of haloperidol? A. Rapid weight loss of 4.5 kg (10 lb) over 2 weeks B. WBC decrease from 10,000 → 6,000 C. Repetitive involuntary motions of face or mouth D. Inability to recall events after taking meds

C. Repetitive involuntary motions of face or mouth

A client receives daily peritoneal dialysis. Today the client infused 2,000 ml of dialysate solution and 3h later 1,000 ml returned. Which instruction should the nurse provide? A. This is a normal occurrence B. Advance the catheter into the abdomen C. Turn from side to side to increase the return D. Milk the catheter using thumb and index finger

C. Turn from side to side to increase the return

A nurse provides info at a health care clinic screening about HTN. Which factors place the client at risk for HTN? Select all that apply A. Participates in aerobic exercise daily B. Reports BP 110/70 to 118/74 C. Uses smokeless tobacco products frequently D. Maintain triglyceride level 150-190 E. Relies on frozen meal choices for dinner

C. Uses smokeless tobacco products frequently D. Maintain triglyceride level 150-190 E. Relies on frozen meal choices for dinner

The following pattern is observed on the fetal monitor for a client who is receiving oxytocin: multiple contractions with short resting period, duration of contractions 100-115 seconds and FHR baseline 100. Which of the following actions should the nurse perform first? A. Notify the provider B. Administer oxygen by face mask C. d/c oxytocin infusion D. Prepare to administer terbutaline

C. d/c oxytocin infusion

an older adult client has been vomiting for the past 4 days. The client is febrile, tachycardic, and has postural hypotension. Which ABG results should the nurse recognize as consistent with this client's history? A. pH 7.24, PaCO2 83, HCO3 42 B. pH 7.49, PaCO2 29, HCO3 22 C. pH 7.48, PaCO2 46, HCO3 32 D. pH 7.27, PaCO2 32, HCO3 16

C. pH 7.48, PaCO2 46, HCO3 32 Metabolic alkalosis Partially compensated

*Women with HRT - should not have if...

CV issues

*monitor cushings for...

CV issues

A client with a personality disorder is upset and calls the nurse a "stupid cow." Which of the following is the most effective initial response by the nurse to this client's behavior?

Calmly discuss the inappropriateness of displacing anger to others.

The nurse is admitting a client from China to the medical-surgical unit with a diagnosis of cancer. While doing the client's assessments, the client speaks of her naturalistic beliefs related to health care and the importance of the yin/yang theory. Based on her cancer diagnoses, the idea that cancer is considered a cold illness in the culture, and her yin/yang beliefs, which meal will the patient most likely order for lunch?

Chicken noodle soup with crackers, fruit crisp, and hot tea

A father, mother, grandmother, and three school-aged children have immigrated to the United States from Thailand. Which member(s) of the family are likely to learn to speak English more rapidly?

Children

A nurse is caring for a hospitalized child. What would the nurse consider to meet the spiritual needs of the child?

Children have definite perceptions of God.

Which meal would be appropriate for the child with osteomyelitis to choose?

Children with osteomyelitis need a diet that is high in protein and calories. Milk, eggs, cheese, meat, fish, and beans are the best sources of these nutrients.

The nurse on the medical-surgical unit is reviewing discharge instructions with a patient who has a history of glaucoma. The nurse should anticipate the use of what medications?

Cholinergics

Serous Sanguineous:

Clear and blood tinged

Following a motorcycle accident, a 17-year-old man is brought to the ED. What physical assessment findings related to the ear should be reported by the nurse immediately?

Clear, watery fluid is draining from the patient's ear.

A client has just expelled a hydatidiform mole. She's visibly upset over the loss and wants to know when she can try to become pregnant again. How should the nurse respond?

Clients who develop a hydatidiform mole must be instructed to wait at least 1 year before attempting another pregnancy, despite testing that shows they have returned to normal. A hydatidiform mole is a precursor to cancer, so the client must be monitored carefully for 1 year by an experienced health care provider. Discussing this situation at a later time or checking with the physician to give the client something to relax does nothing to address the client's immediate concerns. Advising the client to wait until all tests are normal is a vague response and provides the client with little information.

The nurse is caring for a client with bipolar disorder who was recently admitted to an inpatient unit and is experiencing a manic episode. What is a priority nursing intervention for this client?

Closely monitor the client's eating and sleeping habits.

A nurse engages in professional rituals as a means to standardize practice and ensure efficiency. In doing so, the nurse integrates understanding of which of the following as a characteristic?

Common and observable expressions of culture

A client who's 7 months pregnant reports severe leg cramps at night. Which nursing action would be most effective in helping the client cope with these cramps?

Common during late pregnancy, leg cramps cause shortening of the gastrocnemius muscle in the calf. Dorsiflexing or standing on the affected leg extends that muscle and relieves the cramp. Although moderate exercise promotes circulation, walking 2 hours daily during the third trimester is excessive. Excessive calcium intake may cause hypercalcemia, promoting leg cramps; the physician must evaluate the client's need for calcium supplements. If the client eats a well-balanced diet, calcium supplements and additional servings of high-calcium foods may be unnecessary.

Caring for a newborn with herpes simplex virus which precautions should be initiated

Contact

School age child with scald burns on both hands and wrists suspected abuse

Contact child protective services.

*The nurse provides medication instruction to a client who is prescribed 50 mcg/hour dose of transdermal fentanyl every 3 days. Which statement made by the client indicates understanding of the instructions? 1. "I should avoid placing a heating pad over the medication patch." 2. "If I develop a fever, less medication will be absorbed through my skin." 3. "The medication patch should be folded in half and put in the trash." 4. "I will leave the old patch on for a couple of hours after putting on the new one."

Correct: 1 Rationale: ANY heat source, including hot baths & electric blankets, will increase the absorption of the medication thru the skin. Statement indicated correct understanding. 2: A fever increases med absorption thru the skin. 3: Med patch should be folded in half with the adhesive side on the inside and flushed down the toilet. 4: Med will continue to be absorbed from both patches, increasing the risk of adverse effects.

The nurse provides care for a client who underwent a vagotomy with antrectomy to treat a duodenal ulcer. Postoperatively, the client develops dumping syndrome. Which client statement indicates to the nurse that further teaching is necessary? 1. "I should eat bread with each meal." 2. "I should eat smaller meals more frequently." 3. "I should lie down after eating." 4. "I should avoid drinking fluids with my meals."

Correct: 1 Rationale: Carbohydrates increase the risk of dumping syndrome.

The nurse preceptor observes the novice nurse obtain blood through a peripherally inserted central catheter (PICC). Which observation requires an intervention by the nurse preceptor? 1. The nurse discards 1 mL of blood prior to obtaining the blood sample. 2. The nurse uses a 10 mL syringe to flush through the port of the catheter. 3. The nurse applies clean gloves prior to beginning the procedure. 4. The nurse uses the push-pause technique to flush the catheter.

Correct: 1 Rationale: Novice nurse should discard 3-5 mL of blood to prevent contamination of a blood sample with IV fluids/meds. 2: 10 mL syringe is recommended to reduce pressure on the lumen of the PICC line during the flush. 3: Clean gloves are used when drawing blood from PICC line 4: The push-pause technique reduces the risk of clot formation and damage to the PICC line.

The nurse in the emergency department prepares to administer morphine sulfate to a client. Which action does the nurse take first? 1. Verify the client's name and date of birth. 2. Document the amount used on the medication record. 3. Determine if the client has a responsible driver. 4. Ensure the client's call light and belongings are within reach.

Correct: 1 Rationale: Nurse must verify clients' identity before administering meds, at least with 2 identifiers

The nurse admits a client to the postpartum unit and provides instruction about the postpartum process. The nurse determines that teaching is effective if the client makes which statement? 1. "I will call for assistance the first time I want to get out of bed." 2. "I can expect to pass clots the size of golf balls for the first 24 hours." 3. "I will use lanolin on my nipples when I breast feed my baby." 4. "I will allow my baby to suck no more than 5 minutes on each breast."

Correct: 1 Rationale: Only true statement regarding postpartum care.

*The health care provider prescribes an increase in the parenteral nutrition (PN) infusion rate from 50 mL/hour to 100 mL/hour. The PN is infusing through a peripherally inserted central catheter (PICC) device. Which is the priority action for the nurse? 1. Assess hourly urine. 2. Evaluate total serum protein level. 3. Assess vital signs (VS) every 4 hours. 4. Evaluate aspartate aminotransferase (AST) test.

Correct: 1 Rationale: PN is hyperosmolar and will pull fluid into the intravascular space, thereby causing osmotic diuresis. Fluid volume will affect the ABCs. Therefore, monitoring UO is the priority nursing action. 2: PN is high in protein, necessitating the need to monitor the total serum protein level. This will not impact ABC. 3: Changes in fluid volume may impact VS. However, this is too broad. Urine output is a better indicator of intravascular volume. 4: Not appropriate

*The nurse provides care for a client who is diagnosed with depression and anxiety. The client states, "I feel overwhelmed because I'm the only caregiver for my two children." Which response by the nurse is best? 1. "Do you participate in any religious or spiritual activities?" 2. "What can we do to help take your mind off things?" 3. "You do not plan to have any more children, do you?" 4. "Why do you not work outside the home?"

Correct: 1 Rationale: Spirituality and religious beliefs have the potential to exert influence on how people understand the meaning and purpose in their lives. The beliefs can also impact the use of critical judgment and the ability to problem solve. 2: Distraction is not always the best technique. Nurse should assess the client's coping mechanisms 3: a judgmental question 4: "why" questions is confrontational and not therapeutic

The nurse provides care to a client who has a chest tube and pleural drainage system placed for the treatment of a right-sided pneumothorax. The suction control chamber is set at 20 cm and tubing is attached to the wall suction. Which finding will the nurse expect to observe after the insertion of the chest tube? 1. Bubbling in the water-seal chamber. 2. Serosanguinous drainage in the collection chamber. 3. Fluctuation in the suction control chamber during coughing. 4. One cm sterile water in the water-seal chamber.

Correct: 1 Rationale: The water seal chamber bubbles d/t the pneumothorax 2: serosanguinous drainage is not anticipated, shouldn't have any drainage or very scant drainage 3: fluctuation is expected in the water seal chamber when the client forcefully coughs; not expected after the initial insertion 4: nurse expects 2 cm sterile water in the water seal chamber to prevent reentry of air into the pleural space

A client returns to the recovery area after a colonoscopy procedure. Intravenous midazolam was administered during the procedure. The procedure was completed at 1115. The recovery room nurse reviews the sedation chart below. Based on this information, which is the most appropriate action for the nurse to take? Time (preprocedure) O2Sat: 96% BP: 132/84 LOC: Alert/Oriented Pulse/Pain: 84, 0/10 RR: 18 Time (1115) O2Sat: 92% BP: 124/78 LOC: Sleepy/Arousable Pulse/Pain: 76, 0/10 RR: 14 Time (1130) O2Sat: 94% BP: 130/80 LOC: Sleepy/arousable Pulse/Pain: 80, 0/10 RR: 16 Time (1145) O2Sat: 93% BP: 140/86 LOC: Arouses to command Pulse/Pain: 72, 1/10 RR: 15 Time (1215) O2Sat: 92% BP: 160/88 LOC: Arouses to command Pulse/Pain: 66, 1/10, nausea RR: 16 1. Recheck blood pressure in 15 minutes. 2. Administer ondansetron 4 mg IV. 3. Obtain a 12-lead electrocardiogram (ECG). 4. Assist client to get dressed.

Correct: 1 Rationale: VS should be within 20% of pre-procedure values. While midazolam more commonly causes hypotension, the elevated BP is greater than a 20% change in baseline values, indicating that the client is not stable. 2: Expected, nausea may occur p the procedure 3:ECG is done to determine heart rate changes and dysrythmias. 4: Client is not stable

*A client diagnosed with rheumatoid arthritis (RA) is prescribed 50 mg etanercept subcutaneous weekly. The client reports joint swelling, symmetrical joint pain, and deformities of both hands. Which finding does the nurse report to the health care provider? 1. White cell count 14,000/mm 3 (14 x 10 9/L). 2. C-reactive protein 1.2 mg/dL. 3. Serum hemoglobin 9 mg/dL (90 g/L). 4. Sedimentation rate 22 mm/hr.

Correct: 1 Rationale: WBC of 14,000 may indicate active infection (normal: 4,500 to 10,500), which is a contraindication to etanercept. 2,3,4: Expected findings with moderate to severe RA clients. **Etanercept: DMARDs, MOA: binds to TNF (a mediator of inflammatory response) = decreased inflammation and slowed preogression of RA/spondylitis/psoriasis.**

The client receives a blood transfusion and experiences a hemolytic reaction. The nurse anticipates which assessment findings for this client? (Select all that apply.) 1. Hypotension. 2. Low back pain. 3. Wet breath sounds. 4. Fever. 5. Urticaria. 6. Severe shortness of breath

Correct: 1,2,4 Rationale: CLIENT c hemolytic transfusion reaction will experience a drop in BP, low back pain and an elevated temp. 3,4: wet breath sounds and SOB/dyspnea is expected for a client c circulatory overload 5: urticaria/hives is expected c an allergic reaction

*The nurse provides care for a client who is prescribed assist-control mechanical ventilation with positive end-expiratory pressure (PEEP) of 5 cm H 2O. Which actions will the nurse include in the client's plan of care? (Select all that apply.) 1. Strict handwashing before suctioning. 2. Brushing teeth every 12 hours. 3. Elevating the head of the bed 20 degrees. 4. Administering pantoprazole 40 mg intravenous daily. 5. Changing client position every 2 hours.

Correct: 1,4,5 Rationale: -Hand hygiene will reduce risk of VAP -Pantoprazole, a proton pump inhibitor, will decrease the risk of aspiration of gastric contents. -Repositioning and turning every 2 hours reduces the risk of atelectasis, PNA and skin breakdown 2: Oral care and teeth brushing should be at least every 8 hours. 3: HOB should be at least 30 degrees

*The nurse provides care for an older adult client who is diagnosed with a fractured ulna. The client reports falling frequently. Which client statements require that the nurse collect more information? (Select all that apply.) 1. "I keep my bedroom pitch black at night." 2. "My adult child secured all electrical cords against the baseboards." 3. "The bottoms of my shoes have rubber soles." 4. "My sister gave me her cane before she died." 5. "I have my vision checked every 3 years." 6. "I prefer for my pants to fit loosely around my waist."

Correct: 1,4,5,6 Rationale: These statements requires follow-up by the nurse *Reducing fall risk: -have a night light -secure all electrical cords -rubber soles on bottom of shoes -cane should be in proper height with client -visual examinations every 1-2 years -no loose fitting pants on waist

*A pediatric client is diagnosed with pneumonia and prescribed ampicillin 50 mg/kg oral suspension every 6 hours. The child weighs 18 lb (8.181818 kg). The ampicillin is available in 125 mg/5 mL. How many mL will the nurse administer for each dose? (Record your answer rounding at the end of your calculations to the nearest whole number.)

Correct: 16 mL Solution: Client Dose: 50 x 8.181818 = 409.0909 mg x mL = (5mL/125mg) x (50mg/1kg) x (1kg/2.2lbs) x 18lbs. x mL = 4500/275 x mL = 16.3636364 ~ 16

*The nurse provides care for pregnant and postpartum clients. Which client does the nurse see first? 1. Client at 6 weeks' gestation, reporting that the LPN/LVN could not obtain fetal heart tones with a Doptone. 2. Client at 5 days postpartum, reporting bright red, bloody discharge. 3. Client at 22 weeks' gestation, reporting feeling fetal movement four times in the last hour. 4. Client at 2 days postpartum, reporting urinary incontinence.

Correct: 2 Rationale: Lochia rubra (endometrial sloughing that is bloody with a fleshy odor) should last 1-3 days. Therefore, client is unstable. Nurse should assess the client's lochia amount and color in addition to monitoring VS. 1: Client is stable. Fetal heart tones cannot be heard with Doptone until 8-12 weeks gestation. 3: Reassuring sign of fetal well being; fewer than 3 fetal mov'ts in a 1 hour period would indicate a potential issue 4: Stable; urinary incontinence is common during the postpartum period. Nurse should teach the client to perform Kegel exercises to tighten pubococcygeal muscles and avoid diuretics.

*The LPN/LVN reporting to the nurse says, "You may want to see the client recently diagnosed with pancreatic cancer. I am not sure how well things are going." The nurse enters the room and finds the client sitting quietly, looking out the window. As the nurse approaches the client, the client does not look at the nurse. Which is the most appropriate response by the nurse? 1. "Sleep problems are common during times of stress. Have you had difficulty sleeping?" 2. "Tell me what you know about your diagnosis and the treatment you will receive." 3. "How would you describe your overall health status up to this time of your life?" 4. "How have you handled any health problems you experienced in the past?"

Correct: 2 Rationale: MOST imp't to determine client's perception of the health problem. Open-ended statement. Strategy: need to address the problem and better to ask open-ended questions. It is more imp't to deal with the here and now.

*A client who is diagnosed with end-stage kidney disease is prescribed hemodialysis treatments three times a week. After two weeks of treatment, the client states, "I have a headache when the dialysis finishes. Is this normal?" Which is the most appropriate response by the nurse? 1. "I have seen this a lot in clients. Don't worry too much about it." 2. "Headaches may occur at the beginning of treatment and should improve over time." 3. "Have you experienced any headaches similar to these in the past?" 4. "Why are you so worried about this? It is a common side effect."

Correct: 2 Rationale: Nurse must provide correct info in a therapeutic way. Headache, nausea and fatigue may occur after hemodialysis d/t disequilibrium syndrome. This is caused by rapid removal of electrolytes and solutes from blood. A reduction of blood flow during dialysis decreases the risk of disequilibrium syndrome. 1: This response is about the nurse, not the client; this also negates client's concern 3: It is more imp't for the nurse to address the here and now versus if the client has experienced similar headaches in the past 4: "why" questions are not therapeutic

At a rehabilitation center for clients with spinal cord injuries (SCIs), the nurse conducts an orientation session for a group of unlicensed assistive personnel (UAP). Which statement is most important for the nurse to include? 1. "The clients may appear angry at times." 2. "Obtain the client's permission before touching the client." 3. "Most clients arrive believing they will walk out of here." 4. "Personnel in this environment often need counseling."

Correct: 2 Rationale: This statement provides the UAP c info needed to provide care for a client c SCI. Therefore, this isa priority when delegating tasks to the UAP who provides client care. 1,3,4: MAY be true but does not provide info regarding care for SCI patients.

*The nurse performs triage in the emergency department (ED). An unemancipated adolescent minor requests to be treated. The registration clerk states the adolescent requires guardian consent for treatment. Which action should the nurse take next? 1. Triage the client after guardian consent has been obtained. 2. Ask the unemancipated minor about the medical reason for seeking treatment. 3. Request that the health care provider perform a medical screening exam. 4. Notify the nursing supervisor.

Correct: 2 Rationale: Unemancipated minors an consent to medical tx if they have a specific medical condition (i.e. pregnancy, pregnancy-related conditions, minor tx for custodial child, STI info & tx, substance abuse tx and mental health tx). 1: depending on why the minor is seeking treatment, guardian consent may not be necessary and could breach HIPAA guidelines 3: Every person who presents to the ED and requests tx should receive medical screening exam from HCP. 4: it is not appropriate

*The nurse reviews the medical record of a client who is confused. The client has soft wrist and ankle restraints in place. The nurse determines care is effective if which actions are documented? (Select all that apply.) 1. Restraints secured tightly to the skin. 2. Client placed in room next to the nursing station. 3. Restraints attached to side rails on the client's bed. 4. Informed consent for the restraints obtained from the client's spouse. 5. Client alert and oriented x 3. 6. Client placed in the prone position.

Correct: 2,4 Rationale: An appropriate action that promotes client's safety, consent is obtained by proxy since the client is confused 1: tight application interferes c circulation and potentially can can neurovascular injury. Nurse should be able to insert 2 fingers under the restraint 3: restraints should be attached to the bed frame; client could be injured if restraint is secured to the side rail and it is lowered 5: restraints should be d/c as soon as client becomes alert and oriented 6: prone position while in restraints increases the client's risk of suffocation

The nurse meets with the parent of an adolescent male who presents for an annual health maintenance visit. The parent voices concern that the child has recently become clumsy and uncoordinated. Which response by the nurse is correct? 1. "Your son might have attention deficit hyperactivity disorder." 2. "I'll talk with the health care provider about assessing for subtle motor dysfunction." 3. "Your son's clumsiness is expected at this age." 4. "This may be an early sign of depression."

Correct: 3 Rationale: Adolescent males experience a rapid rate of physical growth, which can cause clumsiness and a lack of coordination. This statement is accurate and addresses the parent's concern. 1,4: This is a false statement about clumsiness and lack of coordination in adolescent males, as these manifestations are not associated with attention deficit hyperactivity disorder (ADHD) nor depression. Therefore, this response by the the nurse is not correct. 2: Inappropriate for the nurse tos uggest to HCP the need to assess for subtle motor dysfunction.

The nurse instructs a student nurse about the correct way to set up a sterile field. The nurse determines that teaching is effective if which action is observed? 1. The student nurse places the supplies at the edge of the sterile field. 2. The student nurse wears a gown and gloves at all times. 3. The student nurse sets up the sterile field above waist level. 4. The student nurse opens supplies with sterile gloves.

Correct: 3 Rationale: Appropriate action and indicates accurate understanding of the sterile field

During a urinary bladder catheter insertion, with a size 16 French catheter on an older adult male, the nurse feels increased resistance. Which is the most appropriate action for the nurse to take? 1. Withdraw the catheter and apply more lubricant. 2. Instruct the client to take a deep breath and bear down. 3. Stop the insertion and instruct the client to take deep breaths. 4. Withdraw the catheter and notify the health care provider.

Correct: 3 Rationale: Instructing the client to take deep breaths will relax the urethral muscles and facilitate passage thru the prostate gland. 4: the nurse determines if there is something that can resolve the issue prior to contacting the HCP

The nurse in the emergency department (ED) assesses a client diagnosed with tonic-clonic epilepsy. The client's spouse states that the client has been taking phenytoin as prescribed, but has not been feeling well lately. Which client observation most concerns the nurse? 1. Reddish-brown urine, and the client reports constipation. 2. Acne, hirsutism, and gingival hyperplasia. 3. Ataxia, slurred speech, and nystagmus. 4. The left arm is in a sling and the client walks with a limp.

Correct: 3 Rationale: Slurred speech and ataxia both present an airway concern. 1,2,4: Important manifestations but none impact the ABCs

The client approaches the triage desk in the emergency department (ED) and reports exposure to chemicals after a truck overturned. The client has powder and unknown liquid substances on the clothing. The client is diaphoretic and reports difficulty breathing. Which action does the nurse take first? 1. Escort the client to the decontamination room. 2. Notify the health care provider. 3. Put on appropriate protective gear. 4. Deliver high flow oxygen via a mask.

Correct: 3 Rationale: The nurse's first priority is to protect self and put on the appropriate protective gear. 4: Action is appropriate after the nurse puts on protective gear, as the goal is to prevent the spread of contamination.

A client is admitted to the emergency department (ED). The family reports the client had a sudden onset of left-sided facial droop and slurred speech at home. The nurse observes left-sided muscle weakness. Which is the most important question for the nurse to ask? 1. "What over-the-counter medications does your parent take?" 2. "What was your parent doing when the symptoms began?" 3. "When did you notice the onset of your parent's symptoms?" 4. "Does your parent have a history of high blood pressure?"

Correct: 3 Rationale: Time is of the essence when providing care to a client who experiences ischemic stroke, as thrombolytic therapy is only effective for 4.5 to 6 hours from onset of sx. This is the priority assessment question as thrombolytic therapy can restore circulation for this client. 2:A hemorrhagic stroke may be precipitated by strenous activity. This question is important to differentiate whether the client is experiencing a hemorrhagic or ischemic stroke but not the priority. 4: HTN or high blood pressure is a common risk factor for all types of stroke. Although this is an appropriate assessment question, it does not address the here and now.

*The home care nurse instructs a client diagnosed with multiple sclerosis (MS). The client states, "I have poor concentration and difficulty pronouncing words." The nurse notes that the client's speech is slow and slurred. Which client statement indicates to the nurse that further teaching is necessary? 1. "I will sit up straight when I talk and will feel confident." 2. "I will turn off the TV when speaking and look at the person with whom I am talking." 3. "During a conversation, I will carefully build up to my most important points." 4. "If words fail me, I will draw a picture."

Correct: 3 Rationale: Verbal communication often causes fatigue for MS clients. Therefore, client is taught to make important points first prior to the onset of fatigue. 1,2,3: INDICATES appropriate understanding from the client.

*The client diagnosed with chronic lymphocytic leukemia (CLL) is scheduled for a bone marrow aspiration and biopsy. The client says, "I am frightened. I have never had this test before, and I don't know what to expect." Which statements will the nurse include when responding to the client's concerns? (Select all that apply.) 1. "We will move you to the operating room where the test is always performed." 2. "The bone in the front of your chest will be used for the biopsy specimen." 3. "A tight pressure dressing will be placed over the test site after the procedure." 4. "You will not feel any discomfort as the local anesthetic is injected." 5. "There is a risk of bleeding, so we will monitor the test site frequently."

Correct: 3,4 Rationale: A bone marrow biopsy can cause bleeding and a pressure dressing is applied to reduce the risk of bleeding. Therefore, both are accurate and appropriate for the nurse t o include in teaching. 1: BMA/biopsy may be done in a client room or treatment room. OR is not required. 2: Sternum may be used for BMA but not enough marrow available for biopsy. 4: Client will feel some stinging and discomfort during bone marrow biopsy. This is false reassurance.

A client diagnosed with malnutrition is prescribed continuous enteral feedings through a newly placed gastrostomy tube. Which actions will the nurse include in the client's plan of care? (Select all that apply.) 1. Cover the insertion site with an adhesive bandage. 2. Add 8 hours of feeding to the bag at a time. 3. Rotate the gastrostomy tube 360 degrees once daily. 4. Auscultate for whoosh of air through the gastrostomy tube. 5. Check for slight in-and-out movement of the gastrostomy tube.

Correct: 3,5 Rationale: Gtube should be rotated 360 degrees daily (to reduce risk of skin irritation and breakdown) and a slight in-&-out mov't indicates that the GTube is not embedded in the stomach wall. 1: Gtube insertion site should be covered c a sterile bandage to reduce infection until the stoma is healed. AN adhesive bandage is not used, as this may cause the tube to become dislodged along with increasing risk of infection. 2: Only 4 hours of feeding should be added to the bag to reduce risk of bacterial contamination. 4: Insertion of air is not recommended for GTube placement assessment.

*The nurse reviews the medical record of a client recently diagnosed with Guillain-Barré syndrome. The client has flaccid paralysis of both legs, a history of coronary artery bypass surgery 3 weeks ago, and a 20-year history of hypertension and hypercholesterolemia. The client was also recently diagnosed with type 2 diabetes mellitus (DM). The nurse prepares to apply anti-embolism stockings to both legs. Which priority action does the nurse implement? 1. Assess for bilateral pretibial edema. 2. Palpate both calves for pain. 3. Ask the client the reason for application of anti-embolism stockings. 4. Palpate bilateral pedal pulse strength.

Correct: 4 Rationale: Best indication of PAD and circulation in the extremities is to monitor the client's pedal pulses. In addition, decreased circulation is a contraindication for an anti-embolism stockings. 1: Some edema is expected to an immobile client. Purpose of TED socks may be to reduce edema 2: Venous thromboembolism is a contraindication for anti-embolism stockings. However, client may not have calf pain c VTE in the deeper veins. Pain is also considered psychosocial.

The nurse provides care to a client who is diagnosed with a stroke and is admitted to a rehabilitation center. The client has left-sided pronator drift and decreased dorsiflexion strength of the left extremity. The nurse notes the client bumps into the left wall when ambulating with a walker. The client leans to the left when sitting in a chair or wheelchair. Which is the most appropriate action for the nurse to take? 1. Place the client's favorite watch on the left wrist. 2. Provide a written list for the client to follow during morning care. 3. Instruct the client to choose a dress for the day. 4. Position the client so the right side faces the door of the room.

Correct: 4 Rationale: Client has R side stroke c L side unilateral neglect syndrome. Therefore, the client cannot see out of the left side of both eyes. Safety is a priority when providing care. To enhance safety, the nurse positions the client for best vision so that the client is not scared or upset by approaching people. 1,3: Psychosocial 2: Client recovering from stroke may have short attention span or visual difficulties, making reading with comprehension a difficult task. Nurse should provide verbal instructions c short sentences.

A client is brought to the emergency department (ED) by friends reporting a dry mouth, frequent urination, extreme thirst, and no fluid intake for the last 8 hours. The friends report the client may not have taken insulin during the last couple of days. The nurse reviews prescriptions from the health care provider. Which prescription does the nurse implement first? 1. Administer 20 mEq potassium chloride orally. 2. Begin regular insulin at 0.1 units/kg/hour. 3. Obtain a 12-lead electrocardiogram. 4. Begin infusion of 0.9 % NaCl at 1 L per hour.

Correct: 4 Rationale: During DKA, osmotic diuresis occurs and the client is at significant risk for fluid volume deficit. Since this deficit impacts the ABCs (specifically circulation), this is the priority prescription for the nurse to implement. 1: Mild to moderate hyperkalemia is often seen during the initial phase of DKA. Once an insulin drip is initiated, causing potassium to move into the cells, a KCl prescription may be appropriate. 2: does not address ABC's Restoring volume is the priority for this client 3: This does not address the client's actual problem

*The community health nurse conducts a program for suicide prevention at a high school. The nurse discusses high-risk groups for suicide. The nurse determines that further teaching is necessary if students from the group make which statement? 1. "Adolescents are at risk to commit suicide." 2. "Depressed people are at risk to commit suicide." 3. "History of previous suicide attempts put people at risk." 4. "People grieving a loss for 9 months are at risk."

Correct: 4 Rationale: Grief is a normal human response that occurs in response to loss. The entire grieving process may take up to 3 years. Therefore, this statement indicates the need for further instruction. 1) Males over the age of 50 years and adolescents ages 15 to 19 years are at risk for suicide. This statement indicates correct understanding of the information presented. 2) Indications of depression include low self-esteem, feelings of helplessness/hopelessness, and a sense of doom or failure. Individuals who are depressed are at an increased risk to commit suicide. This statement indicates correct understanding of the information presented. 3) A suicide attempt is the result of the client turning aggression and rage toward self. Anyone with a history of a previous suicide attempt is at risk for another attempt. This statement indicates correct understanding of the information presented.

*The nurse receives a phone call from a client's adult child who states, "I just got here to see my elderly parent, and I think heat stroke has occurred. I think the air conditioning is not working and the house is very hot." The adult child reports that the parent is confused, very thirsty, nauseated, and in pain. Which is the most appropriate statement for the nurse to make? 1. "If perspiration is present, heat stroke has not occurred." 2. "Give your parent cool fluids to drink immediately." 3. "What medications does your parent take daily?" 4. "Remove any excess clothing immediately."

Correct: 4 Rationale: Removing the parent's clothing will begin the cooling process, thereby enhancing circulation. Other measures to reduce temp can be implemented once this step occurs. 1: Provides education to family but does not address the immediate concern 2: Client is at risk for aspiration d/t altered mental status, should be NPO. 3: This is an assessment question but info is not immediately needed.

*The nurse provides care for a young adult client requiring an emergent appendectomy. The health care provider explains to the client the risks and benefits of the procedure. However, the client refuses to sign the informed consent. The client states, "No one is removing any organs from my body because it is against my religious beliefs. I'm leaving!" The client's mother insists the client receive the operation. Which response does the nurse make to the client? 1. "I am going to apply soft wrist and ankle restraints." 2. "Let us contact the hospital chaplain to mitigate the situation." 3. "Intravenous diazepam will help calm your nerves before the procedure." 4. "It is your decision to refuse medical treatment."

Correct: 4 Rationale: The competent client has the right to make personal choice without interference 1: false imprisonment 2: contacting the hospital chaplain violates the client's rights 3: use of diazepam in this situation would be considered a chemical restraint. Psychotropic drugs cannot be used to control behavior

*The nurse works on the medical surgical unit. The nurse-to-client ratio is 1:10. Which action does the nurse take first? 1. Document the situation in writing. 2. Refuse the client assignment. 3. Delegate tasks to the LPN/LVN. 4. Notify the nursing supervisor.

Correct: 4 Rationale: Thisi s the priority action, as the nurse-to-client ratio is proportionately high. This action alerts the nursing supervisor of the situation so nurses can be "floated" from other departments, if available. 1: Notifying the supervisor is the priority. Nurse should provide documentation to the nursing admin, but the documentation does not relieve the nurse of responsibility if clients suffer harm because of inattention. It does show that the nurse attempted to act appropriately. 2: Refusing the client assignment could be regarded as abandonment 3: Nurse maintains responsibility for client outcomes. Problem is nurse-to-client ratio.

*The nurse provides care for the client immediately after arrival in the emergency department (ED). Emergency personnel report that the client was involved in a head-on collision with immediate loss of consciousness. Which is the first action taken by the nurse? 1. Determine Glasgow Coma Scale (GCS) score. 2. Assess bilateral blood pressure. 3. Check bilateral pupillary response to light. 4. Determine oxygen saturation levels.

Correct: 4 Rationale: When prioritizing care for a client, nurse uses the ABC's (airway, breathing, circulation). Oxygen saturation levels allow the nurse to monitor the client's airway (priority). 1: GCS is used to assess ABC and neuro status for clients c head trauma. It is appropriate but too broad and will take longer. 2: Assessing BP is monitoring for circulation. However, airway is priority and increases in arterial CO2 will increase ICP. 3: Nurse assesses neuro status (eg. PERRLA) after ABC.

The nurse reviews the medical record of a client diagnosed with acute kidney injury. It is most important for the nurse to review which lab value? 1. Fasting blood glucose. 2. Serum uric acid. 3. Serum protein. 4. Urine specific gravity.

Correct: 4 Rationale: When providing care for a client diagnosed with acute kidney injury, it is important for the nurse to monitor circulation by reviewing the client's urine specific gravity, which is a good indicator of fluid volume. 1,2,3: Not necessary to review for acute kidney injury

The nurse provides care for the client diagnosed with a hypertensive emergency. The client is prescribed sodium nitroprusside 0.3 mcg/kg/min. The client weighs 176 lb (80 kg). The concentration of the sodium nitroprusside is 50 mg/250 mL. What rate will the nurse set for the per hour amount on the micro infusion pump? (Record your answer rounding at the end of the calculation using one decimal place.)

Correct: 7.2 mL/hr Solution: x mL/hr = (250mL/50mg) x (1mg/1000mcg) x (24mcg/1min) x (60min/1hr) =360000/50000 =7.2

Assessing a client taking propranolol. Indication of adverse reaction

Coughing at night.

A client has undergone a left hemicolectomy for bowel cancer. Which activities prevent the occurrence of postoperative pneumonia in this client?

Coughing, breathing deeply, frequent repositioning, and using an incentive spirometer

Which aging theory describes a chemical reaction that produces damage to the DNA and cell death?

Cross-linkage theory

Despite the presence of a large number of elderly residents of Asian heritage, a long-term care facility has not integrated the Asian concepts of hot and cold into meal planning. Which of the following should the nurses at the facility recognize this as an example of?

Cultural blindness

What is the term that describes the inability of a person to recognize his or her own values, beliefs, and practices as well as those of others, because of strong ethnocentric tendencies?

Cultural blindness

A nurse is providing care for a Cambodian client. The nurse says, "You have to get up and walk whether you want to or not." What is this statement an example of?

Cultural imposition

A nurse is conducting a health assessment for an African American client. What should the nurse consider in terms of cultural sensitivity?

Cultural risk factors for alterations in health and normal racial variations

A Mexican immigrant who migrated to the United States and lives in a Spanish-speaking community with other relatives is taken to the ER following a fall at work. He is admitted to the hospital for observation. The nurse is aware tht this client is at risk for:

Cultural shock

Which of the following statements is true of factors that influence communication?

Culture and lifestyle influence the communication process.

A client needs to be transferred to the oncology unit for further care. Which of the following information is necessary to include in the transfer report?

Current client assessment. The nurse should include the current assessment of the client in the transfer report because it enables the receiving nurse to prepare for the client before arrival and to clarify any information from written transfer summaries they may have obtained.

While assessing the fundus of a multiparous client on the first postpartum day, the nurse performs hand washing and dons clean gloves. Which of the following should the nurse do next? a. place the non-dominant hand above the symphysis pubis and the dominant hand at the umbilicus b. ask the client to assume a side-lying position with the knees flexed c. perform massage vigorously at the level of the umbilicus if the fundus feels boggy d. place the client on a bedpan in case the uterine palpation stimulates the client to void

D - The nurse should place the non-dominant hand above the symphysis pubis and the dominant hand at the umbilicus to palpate the fundus. This prevents the uterine inversion and trauma, which can be very painful to the client. The nurse should ask the client to assume a supine, not side-lying, position with the knees flexed. The fundus can be palpated in this position and the perineal pads can be evaluated for lochia amounts. The fundus should be massaged gently if the fundus feels boggy. Vigorous massaging may fatigue the uterus and cause it to become firm and then boggy again. The nurse should ask the client to void before fundal evaluation. A full bladder can cause discomfort to the client, the uterus to be deviated to one side, and postpartum hemorrhage.

Which client is at greatest risk for experiencing sensory overload: c) A 16 year old listening to loud music d) An 80 year old client admitted for emergency surgery

D) An 80 year old client admitted for emergency surgery

A 52 year old man is scheduled for an annual physical exam. The nurse will plan to teach the patient about: c) Normal decreases in testosterone level d) Annual prostate specific antigen testing (PSA)

D) Annual prostate specific antigen testing (PSA)

Immediate surgery is planned for a patient with acute abdominal pain. The question used by the nurse that will elicit the most complete information about the patient's coping-stress tolerance pattern is: b) What do you think caused this abdominal pain c) How do you feel about yourself and your hospitalization d) Are there other major problems that are a concern right now

D) Are there other major problems that are a concern right now

The nurse assess a surgical patient in the morning of the first postoperative day and notes redness and warmth around the incision. Which action by the nurse is most appropriate: a) Obtain wound cultures b) document the assessment d) Assess the wound every 2 hours

D) Assess the wound every 2 hours

A 78 year old who has been admitted to the hospital with dehydration is confused and incontinent of urine. Which nursing action will be best to include in the plan of care: c) Insert an indwelling catheter until the symptoms have resolved d) Assist the patient to the bathroom every 2 hours during the day

D) Assist the patient to the bathroom every 2 hours during the day

In discussing diet modifications the nurse encourages a client with cellulitus and severe inflammation to include: c) Pretzels d) Citrus fruit

D) Citrus Fruit

During the assessing component of the nursing process, the primary reason for interviewing the client is to: c) Provide emotional therapy d) Collect data

D) Collect Data

The patient's teaching plan includes this goal, "The patient will select 2 gram sodium diet from the hospital menu for the next three days". Which evaluation method will be best for the nurse to use. When determining whether teaching was effective: a) Check the sodium content of the patient's menu choices over the next three days c) Have the patient list favorite foods that are high in sodium and foods that could be substituted for these favorites d) Compare the patient's sodium intake over the next three days with the sodium intake before the teaching was implemented

D) Compare the patient's sodium intake over the next three days with the sodium intake before the teaching was implemented

A client reports to the nurse that she has been taking barbiturate sleeping pills every night for several months and now wishes to stop taking them. Which statement is the most appropriate advice for the nurse to provide the client? c) Discontinue taking the pills d) Continue taking pills and discuss tapering the dose with the primary care provider

D) Continue taking the pills and discuss tapering the dose with the primary care provider

A 72 year old who has benign prostatic hyperplasia is admitted to the hospital with chills, fever, and vomiting. Which finding by the nurse will be most helpful in determining whether the patient has an upper tract infection (UTI): c) Foul smelling urine d) Costovertebral tenderness

D) Costovertebral Tenderness

The nurse observes nursing assistive personnel (NAP) taking the following actions when caring for a patient with a retention catheter. Which action requires that the nurse intervene: c) Using an alcohol based hand cleaner before performing catheter care d) Disconnecting the catheter from the drainage tube to obtain a specimen

D) Disconnecting the catheter from the drainage tube to obtain a specimen

A client has a history of sleep apnea. Which is the most appropriate question for the nurse to ask? c) Have you had chest pain with or without activity d) Do you have difficulty with daytime sleepiness

D) Do you have difficulty with daytime sleeping

When assessing a 64 year old woman, the nurse notes that the patient has lost 1 inch in height since the previous visit 2 years ago. The nurse will plan to teach the patient about: c) Magnetic reasonable imaging (MRI) d) Dual energy x-ray absorption (OEXA)

D) Dual Energy X-ray Absorption (OXEA)

When providing care using evidence-based practice, the nurses uses: a) Clinical judgement based on experience c) Evidence-based guidelines in addition to clinical expertise d) Evaluation of data showing that the patient outcomes are met

D) Evaluation of data showing that the patient outcomes are met

A patient who has been admitted to the hospital for surgery tells the nurse, 'I do not feel right about leaving my children with my neighbor", which action should the nurse take next: a) Reassure the patient that these feelings are common for parents b) Have the patient call the children to ensure that they are doing well c) Call the neighbor to determine whether adequate childcare is being provided d) Gather more data about the patient's feeling about the child-care arrangements

D) Gather more data about the patient's feeling about the child-care arrangements

A patient is taking a potassium-wasting diurectic for treatment of hypertension. The nurse will teach the patient to report symptoms of adverse effects such as: a) personality change b) Frequent loose stools c) Facial muscle spasms d) Generalized weakness

D) Generalized weakness

A patient who has a wound infection after major surgery has only been taking in about 50% to 75% of the ordered meals and states, "Nothing on the menu really appeals to me." Which action by the nurse will be most effective in improving the patient's oral intake: a) Make a referral to the dietician d) Have family members bring in favorite foods from home

D) Have family members bring in favorite foods from home

A nursing student is learning the application of the nursing process to client care. When questioned by the student about the reason for implementing a nursing diagnosis, the nurse's professor responds: "The nursing diagnosis statement: a) Describes client problems that nurses are licensed to treat c) Includes the disease the client has during the treatment of care d) Helps standardize care for all clients

D) Helps standardized care for all clients

The nurse has admitted a patient with a new diagnoses of pneumonia and explained to the patient that together they will plan the patient's care and set goals for discharge. The patient says, "How is that different from what the doctor does?" Which response by the nurse is most appropriate: c) Nurses perform many of the procedures done by physicians, but nurses are here in the hospital for a longer time than doctors d) In addition to caring for you while you are sick, the nurses will assist you to develop an individualized plan to maintain your health

D) In addition to caring for you while you are sick, the nurses will assist you to develop an individualized plan to maintain your health

During an admission nursing assessment, a client with diabetes describes his leg pain as a "dull, burning sensation." The nurse recognizes this description to be characteristic of which type of pain: c) Visceral d) Neuropathic

D) Neuropathic

When the nurse is planning for the physical examination of an alert 86 year old patient. Adaptions to the examination technique should include: a) Speaking slowly when directing the patient b) Avoiding the use of touch as much as possible c) Using slightly more pressure for palpation of the liver d) Organizing the sequence to minimize position changes

D) Organizing the sequence to minimize position changes

The patient has been experiencing difficulty and straining when expelling feces. Which intervention should the nurse discuss with the client: c) Encourage the client to use a cathartic laxative on a daily basis d) Place the client on a high fiber diet

D) Place the client on a high fiber diet

The nurse is caring for a client diagnosed with early osteoporosis. Which intervention is most applicable for this client: a) Institute an exercise plan that includes weight-bearing activities b) Protect the client's bones with strict bed rest d) Provide the client with assisted range of motion exercising twice daily

D) Provide the client with assisted range of motion exercising twice daily

While providing hygiene care to a confused older adult client diagnosed with Alzheimer's disease, the nures is called to the nursing station. To ensure patient safety the nurse must do what? A) Ask a family member to stay with him. B) Cover him with a blanket for warmth. C) Reattach the restraints. D) Put side rails up before leaving the client.

D) Put side rails up before leaving the client.

A patient who has required prolonged mechanical ventilation has the following arterial blood gas results: pH 7.48, PaCO@ 32 mmttg, and HCO 25 mEq/L. The nurse interprets these results as: a) Metabolic acidosis b) Metabolic alkalosis c) Respiratory acidosis d) Respiratory alkalosis

D) Respiratory Alkalosis

A patient who is having difficulty breathing is admitted to the hospital. The best approach for the nurse to use to obtain a complete health history is to: a) Obtain subjective data about the patient's family membrane b) Omit subjective data collection and obtain the physical examination c) Use the health care provider's medical history to obtain subjective data d) Schedule several short sessions with the patient to gather subjective data

D) Schedule several short sessions with the patient to gather subjective data

The nurse is doing bowel and bladder retraining for the client with oaraplegia. Which of the following is NOT a factor for the nurse to consider: c) Fluid intake d) Sexual Function

D) Sexual Function

A nurse in instructing a hospitalized client with a diagnosis of emphysemia about measures that will enhance the effectiveness of breathing during dyspneic periods. Which of the following position will the nurse instruct the client to assume: c) Sitting in a recliner chair d) Sitting on the side of the bed and leaning on an over bed table

D) Sitting on the side of the bed and leaning on an over bed table

A patient returns to the surgical nursing unit following a vertical banded gastroplasty with a nasogastric tube to low, intermittent suction and a patient controlled analgesia (PCA) machine for pain control. Which nursing action should be included in the postoperative plan of care: b) Offer sips of sweetened liquids at frequent intervals c) remind the patient that PCA use may slow the return of bowel functions d) Support the surgical incision during patient coughing and turning in bed

D) Suport the surgical incision during patient coughing and turning in bed

When admitting a patient who has just Arrived on the medical unit with severe abdominal pain, what should the nurse do first: A) Complete only basic demographics data before addressing the patient's abdominal pain b) Medicate the patient for the abdominal pain before attending to the health history and examination c) Inform the patient that the abdominal pain will be treated as soon as the health history is completed d) Take the initial vital signs and then deal with the abdominal pain before completing the health history

D) Take the initial vital signs and then deal with the abdominal pain before completing the health history

While the nurse is assessing a 62 year old man, the patient says he does not respond to sexual stimulation the way he did when he was younger. The nurse's best response to the patient's comment is: c) Erectile dysfunction is a common problem with older man d) Tell me more about how your sexual response has changed

D) Tell me more about how your sexual response has changed

Which would be an expected outcome for a client with the following nursing diagnoses self-care deficit related to congnitive impairment: a) The client will be able to name the staff that works on the day shift b) The client will eliminate safety hazards in her environment c) The nurse will stress the importance of adequate fluid intake d) The client with supervision will brush her teeth

D) The client with supervision will brush her teeth

The nurse has formulated a diagnosis of Activity Intolerance related to Decreased Airway Capacity for chronic asthma. In looking at the client's coping skills, the nurse realizes that the patient has a vast knowledge about the disease and what exacerbates symptoms in particular situations. The nurse will utilize this information because: a) Strengths can be an aid to mobilizing health and the healing process c) It will be easier for the nurse to educate the client about other interventions d) The nurse wont have to spend time going over the pathology of the client's disease

D) The nurse wont have to spend time going over the pathology of the client's disease

The nurse has just received change-of-shift report about the following four patients which patient will the nurse assess first: a) The patient who has multiple black wounds on the feet and ankles b) The newly admitted patient with a stage IV pressure ulcer on the coccyx c) The patient who needs to be medicated with multiple analgesics before a scheduled dressing change d) The patient who has been receiving immunosuppressants medications and has a temp of 102' F

D) The patient who has been receiving immunosuppressants medications and has a temp of 102' F

The nurse is performing an admission assessment on a 20 year old college student who is being admitted for electrolyte disorders of unknown etiology. Which assessment is most important to report to the health care provider: c) The patient has history of weight fluctuations d) The patient's serum potassium level is 2.9 mEq/L

D) The patient's serum potassium level is 2.9 mEq/L

Nurses often utilize systems theory to assess family units. Which example illustrates a family unit that does NOT meet the criteria of a well-functioning system? c) Each member's personal boundaries are well defined d) The primary activities of each member focus on personal purposes

D) The primary activities of each member focus pn personal purposes

While admitting a patient to the medical unit, the nurse learns that the patient does not read well. This information will guide the nurse in determining: a) The degree of patient motivation and readiness to learn b) What information the patient will be able to understand c) That the family must be included in the teaching process d) Which instructional strategies should be used in teaching

D) Which instructional strategies should be used in teaching

The nurse is developing a weight loss plan for a 21 year old patient who is morbidly obese. Which statement by the nurse is most likely to help the patient in loosing weight on the planned 1000 calorie diet: c) Most of the weight that you lose during the first weeks of dieting is water weight rather than fat d) You are likely to start to notice changes in how you feel with just a few weeks of diet and exercise

D) You are likely to start to notice changes in how you feel with just a few weeks of diet and exercise

A client who has metastatic ovarian cancer states, "I do not want any more treatment." How should the nurse respond? Select all that apply A. "I understand, I would feel the same way." B. "Why do you want to stop the treatment?" C. "Do you want to discuss a different type of medication?" D. "Tell me more about your decision to discontinue chemotherapy." E. "Would you like to speak with a hospice representative?"

D. "Tell me more about your decision to discontinue chemotherapy." E. "Would you like to speak with a hospice representative?"

A LPN reports the following data to the supervising RN regarding data collected for a client who has CHF: pulse ox 85%, RR 48/min and labored. What is the priority action at the time? A. LPN will administer IV furosemide B. Respiratory therapist will be notified C. Client will be prepared for CXR D. Care of client will be reassigned to an RN

D. Care of client will be reassigned to an RN

A woman who has PROM is admitted for observation. Which finding should concern the nurse? A. FHR 160 B. Maternal temp 37.2C (99F) C. Irregular uterine contractions D. Cloudy amniotic fluid

D. Cloudy amniotic fluid

12h after an open reduction and internal fixation of the left femur, a client reports increasingly severe pain, numbness, and tingling over the left lower leg. Which action should the nurse take? A. Elevate effected leg on 2 pillows B. Assess pain intensity and medicate as needed C. Loosen external fixator to relieve pressure D. Compare color, temperature, and sensation of toes bilaterally

D. Compare color, temperature, and sensation of toes bilaterally

Which of the following actions should the nurse take after drawing blood from a client who has an implanted venous port? A. Check for blood return B. Access port with noncoring needle C. Apply 20% benzocaine to injection site D. Flush port with 5 ml of 100 units heparin/mL

D. Flush port with 5 ml of 100 units heparin/mL

A nurse is caring for a client who has a chest tube. The nurse notes the chest tube has become disconnected from the chest drainage system. Which of the following actions should the nurse take? A. Reposition client to high fowler's B. Increase suction to chest drainage system C. Place client on low flow oxygen via NC D. Immerse end of chest tube in bottle of sterile water

D. Immerse end of chest tube in bottle of sterile water

A client has type 1 DM. After reviewing the medical record, which interevntions should the nurse implement first? Click on tabs -Lab: glucose 580, K 5.7, Na 150, ketones in urine, urine specific gravity 1.030 (dehydration) -VS: temp 38.3C (101F), 136 HR, 50 RR, -Assessment: GCS 13, kussumal respirations, lungs clear bilaterally, sinus tach, abdomen flat and soft, A. Reassess urine for ketones B. Begin continuous IV insulin infusion C. Administer 20 mEq K+ IV over 2h D. Infuse 1L of 0.9% sodium chloride over 1h

D. Infuse 1L of 0.9% sodium chloride over 1h

A nurse is coordinating client care. Which of the following should the nurse delegate to the PN (priority- safety)? The client who: A. Has new prescription for PCA B. Requires intermittent suctioning of newly placed tracheostomy C. Has tension pneumothorax and requires a chest tube D. Is scheduled for transfer to a physical rehabilitation facility

D. Is scheduled for transfer to a physical rehabilitation facility

A nurse observes an infant who has Tetralogy having a hypercyanotic episode. What is priority one? A. Draw ABGs B. Initiate IV therapy C. Prepare for intubation D. Place in knee chest position

D. Place in knee chest position

A nurse prepares an older client for a schedule colonoscopy. Which of the following should be the nurse's initial action? A. Chill bowel cleansing solution B. Monitor frequency of elimination C. Provide oral intake of clear liquids D. Place portable commode at bedside

D. Place portable commode at bedside

A client presents to the ED and reports G3P2 (2 births after 20 weeks). Which of the following should be the nurse's initial action after observing a present part? A. Provide emotional support to client B. Notify L&D staff members C. Time frequency and duration of contractions D. Prepare for delivery of infant in ED

D. Prepare for delivery of infant in ED

A nurse from the adult med-surg is assigned to the pediatric unit. Which of the following would be an appropriate assignment? A. Toddler admitted with epiglottitis B. School age child scheduled for excision of Wilms tumor C. Infant recovering from repair of a cleft lip and palate D. Preschooler who had surgical fixation of a fractured humerus

D. Preschooler who had surgical fixation of a fractured humerus

Following mass shooting, a nurse in the ED provides care for a client who has been triaged with a green tag. What is priority 1? A. Clean wounds B. Administer lorazepam C. Check serum electrolytes D. Provide reassurance of safety

D. Provide reassurance of safety

A client who has recently undergone sx for a tracheostomy is now at home. The nurse recognizes a need for immediate intervention when the caregiver does which of the following? A. Places an air humidifier at bedside B. Suctions intermittently for 15 seconds C. Cuts a 4x4 to put around tracheostomy tube D. Removes ties before cleaning tracheostomy

D. Removes ties before cleaning tracheostomy

A nurse initiates emergency protocol on the medical unit during a fire. Which client should be evacuated first? A client who is: A. Receiving mechanical ventilation B. Recovering from a below the knee amputation C. Prescribed continuous oxygen therapy D. Scheduled for cholecystectomy the following day

D. Scheduled for cholecystectomy the following day

1h after a client has a cardiac cath and stent placement using an approach via left femoral artery, the nurse should be most concerned about which finding? A. Left pedal pulse 1+ and slightly cooler than right, right pedal pulse 2+ B. Client rates discomfort of 3/10 in left groin area C. Cardiac monitor shows 1-2 PVCs/min D. VS: HR 120, BP 90/60, RR 22, temp 99F

D. VS: HR 120, BP 90/60, RR 22, temp 99F

An infant is admitted for management of dehydration secondary to rotavirus infection. Which laboratory finding should the nurse anticipate? A. pH 7.24, PaCO2 83, HCO3 24 B. pH 7.49, PaCO2 31, HCO3 18 C. pH 7.52, PaCO2 18, HCO3 26 D. pH 7.31, PaCO2 30, HCO3 16

D. pH 7.31, PaCO2 30, HCO3 16 Metabolic acidosis Partially compensated

The nurse is caring for an infant diagnosed with nonorganic failure to thrive. Which action should be included in the plan of care for the infant?

Daily weights are an appropriate intervention for an infant with failure to thrive. It would be inappropriate for the nurse to encourage the mother to continue to try to feed the infant when crying because the infant may develop further aversion to eating. It is also inappropriate to assume that abuse has taken place; there is no information in the stem to suggest this. The parents would benefit from a community support group; however, the nurse cannot require the parents to attend a community support group prior to discharge.

What type of cognitive responses might a nurse assess in a client with sensory deprivation?

Decreased attention span, difficulty problem solving

Caring for a client with a prescription of chlorpromazine indications of effectiveness

Decreased hallucinations

A client's blood glucose level is 45 mg/dl. The nurse should be alert for which signs and symptoms?

Decreased level of consciousness (LOC), anxiety, confusion, headache, and cool, moist skin.

What is one reason for the "middle-aged spread" often seen in middle adults?

Decreased physical activity

s&s SIADH

Decreased urine output, thirst, confusion, lethargy, agitation, seizures, coma, etc.

Manger preparing an educational session for staff about how to provide cost effective care

Delegate non-nursing tasks to ancillary staff.

A nurse documents the following on a client chart: "client manifests difficulties with spatial orientation, memory language, and changes in personality." What state of arousal/awareness does this describe?

Delirium

ADH water deprivation test

Designed to see how concentrated the urine and blood are in the absence of water for several hrs and what the cause is - Need to be NPO after midnight on a.m. of exam - no coffee, tea or smoking - Weight, BP, urine output, specific gravity and plasma osmolality are obtained for baseline data and then monitored every hr there after - ADH (vasopressin) is then given in NS iv over 2 hrs (exam can take up to 8 hrs) - Test distinguishes ADH deficit from renal dysfunction) - Increase in urine osmolality and decreased plasma osmolality after ADH is given confirms ADH Deficit - No change after ADH is given indicates Renal Disorder/Dysfunction

Updating the plan of care for a client who is 48 hr post op following laryngectomy and unable to speak. Plan to take first

Determine the client's reading skills

The nurse's assessment of a patient with significant visual losses reveals that the patient cannot count fingers. How should the nurse proceed with assessment of the patient's visual acuity?

Determine whether the patient is able to see the nurse's hand motion.

Assessing a client who has had a stroke. For which shoudl the nurse initiate referral for occupational therapy

Difficulty performing ADLs

While obtaining a health history, the nurse learns that the client is allergic to bee stings. When obtaining this client's medication history, the nurse should determine if the client keeps which medication on hand?

Diphenhydramine hydrochloride (Benadryl)

A patient's ocular tumor has necessitated enucleation and the patient will be fitted with a prosthesis. The nurse should address what nursing diagnosis when planning the patient's discharge education?

Disturbed body image

A patient is scheduled to have an electronystagmography as part of a diagnostic workup for Ménière's disease. What question is it most important for the nurse to ask the patient in preparation for this test?

Do you currently take any tranquilizers or stimulants on a regular basis?

Caring for a client with a new prescription of Clonidine. Adverse side effects

Dry mouth

Cushing's Disease

Due/secondary to ACTH excess/adrenocortical activity - Ectopic ACTH secretion by the lung or pancreatic tumors

Which of the following characteristics differentiates open-angle glaucoma from angle-closure glaucoma?

Dysfunction of the aqueous humor drainage system

What term is used to describe painful intercourse?

Dyspareunia

A client with iron deficiency anemia has been admitted to the medical-surgical unit. Which assessment findings are characteristic of this type of anemia?

Dyspnea, tachycardia, and pallor

Assessing client for compartment syndrome expected findings

Edema

A client comes to the emergency department complaining of chest pain. An electrocardiogram (ECG) reveals myocardial ischemia and an anterior-wall myocardial infarction (MI). Which ECG characteristic does the nurse expect to see?

Elevated ST segment

A client who has recently had surgery for prostate cancer expresses to the nurse feelings of anger toward God, his church, and the clergy. Which intervention is appropriate for this client?

Encouraging the client to discuss concerns with the clergy

The nurse has taken shift report on her patients and has been told that one patient has an ocular condition that has primarily affected the rods in his eyes. Considering this information, what should the nurse do while caring for the patient?

Ensure adequate lighting in the patient's room.

A home health care nurse has observed that a client 80 years of age, who has multiple chronic health problems, takes a total of 19 medications on either a scheduled or PRN (as needed) basis. How should the nurse address this client's risk of harm from polypharmacy?

Ensure that the client's care is coordinated and encourage the primary care provider to review her medication regimen.

Providing discharge teaching about disease management for a client who has a new diagnosis of DM. Priority activities

Ensure the client understand the medication regimen. Because this position limits the flow of th instilled solution out of the stomach and prevents aspiration.

The nurse is caring for a client infected with methicillin-resistant Staphylococcus aureus (MRSA). What's the major infection control measure to reduce MRSA and other nosocomial pathogens in a health care setting?

Ensuring that personnel wash their hands before and after contact with every client

After completing a shift, a nurse realizes that documentation on a client was not completed before leaving the unit. Which of the following actions by the nurse is most appropriate?

Enter the information tomorrow stating it is a late entry.

A nurse is caring for an elderly woman from a far eastern culture. How does the nurse demonstrate awareness of culturally competent care?

Establishing effective communication.

A client who has difficulty sleeping expresses to the nurse that watching television may help him relax and get sleep. The nurse disregards the client's concern and suggests drinking warm milk before going to bed. Which cultural characteristic is the nurse demonstrating?

Ethnocentrism

A client is to be discharged from an acute care facility following treatment for right leg thrombophlebitis. The nurse notes that the client's leg is pain-free, without redness or edema. The nurse's actions reflect which step of the nursing process?

Evaluation

A nurse is caring for a client who is visually impaired. Which of the following is a recommended guideline for communication with this client?

Explain reason for touching client before doing so.

A patient has had a sudden loss of vision after head trauma. How should the nurse best describe the placement of items on the dinner tray?

Explain the location of items using clock cues.

A patient with chronic open-angle glaucoma is being taught to self-administer pilocarpine. After the patient administers the pilocarpine, the patient states that her vision is blurred. Which nursing action is most appropriate?

Explaining that this is an expected adverse effect

Client who has acute blood loss following trauma refuses potentially life saving blood transfusion

Explore the client's reasons fro refusing the treatment

The nurse in the ED is caring for a 4 year-old brought in by his parents who state that the child will not stop crying and pulling at his ear. Based on information collected by the nurse, which of the following statements applies to a diagnosis of external otitis?

External otitis is characterized by aural tenderness.

A nurse assesses a client's eyes by testing the cardinal fields of vision for coordination and alignment. What eye characteristic is being assessed by this process?

Extraocular movements

Preparing to transfer a client to a rehab facility who had a stroke Family is concerned about the level of care the client will receive?

Facilitate an interdisciplinary conference at the new facility for the family

Admission assessment on a client who had a recent positive pregnancy test. First day of her last menstrual period was May 8 . Nagel's rule EDB

February 15

A client rings a call bell to request pain medication. Upon performing the pain assessment, the nurse informs the client that she will return with the pain medication. The nurse's promise to return with the pain medication is an example of which principle of bioethics?

Fidelity.

Preparing to teach about dietary management to a client with Crohn's disease with a enteroenteric fistula. Which of the following nutrients should the nurse instruct the client to decrease in his diet

Firber

The nurse is admitting a 55-year-old male patient diagnosed with a retinal detachment in his left eye. While assessing this patient, what characteristic symptom would the nurse expect to find?

Flashing lights in the visual field

The American Cancer Society recommends routine screening to detect colorectal cancer. Which screening test for colorectal cancer should the nurse recommend?

Flexible sigmoidoscopy beginning at age 50

Assessing a client who has bipolar. Which of the following alterations in speech is the client using

Flight of Ideas

A nurse at an urgent care clinic is assessing a client who reports impaired vision in one eye . Which indicates the client has a detached retina

Floating dark spots

A patient has just returned to the surgical floor after undergoing a retinal detachment repair. The postoperative orders specify that the patient should be kept in a prone position until otherwise ordered. What should the nurse do?

Follow the order because this position will help keep the retinal repair intact.

The nurse is working in a support group for clients with acquired immunodeficiency syndrome (AIDS). Which point about preventing transmission of the human immunodeficiency virus (HIV) is most important for the nurse to stress?

Following safer-sex practices

Then nurse is caring for a hospice client who tells the nurse that she is worried about how she has treated a younger sister. She asks the nurse how to make things right. The nurse recognizes this as which of the following spiritual needs?

Forgiveness

Manager in long term care having difficulty with staffing weekend shift is planning to implement changes to the scheduling procedure.

Form a committee of staff members to investigate current staffing issues.

A client age 71 years has recently integrated large amounts of blueberry and pomegranate juice into her diet, touting their antioxidant properties that mitigate the effects of separated high-energy electrons. The client's actions reflect which of the following theories of aging?

Free radical theory

Policy and procedure dictate that hand washing is a requirement when caring for clients. Which statement about hand washing is true?

Frequent hand washing reduces transmission of pathogens from one client to another.

Cytomegalovirus (CMV) is the most common cause of retinal inflammation in patients with AIDS. What drug, surgically implanted, is used for the acute stage of CMV retinitis?

Ganciclovir

A patient presents at the ED after receiving a chemical burn to the eye. What would be the nurse's initial intervention for this patient?

Generously flush the affected eye with normal saline or water.

On otoscopy, a red blemish behind the tympanic membrane is suggestive of what diagnosis?

Glomus tympanicum

A 6-month-old infant is brought to the ED by his parents for inconsolable crying and pulling at his right ear. When assessing this infant, the advanced practice nurse is aware that the tympanic membrane should be what color in a healthy ear?

Gray

A 6-year-old child is brought to the pediatric clinic for the assessment of redness and discharge from the eye and is diagnosed with viral conjunctivitis. What is the most important information to discuss with the parents and child?

Handwashing can prevent the spread of the disease to others.

Community health nurse providing teaching on home safety for older adults

Have grab bars installed around your bathtub and toilet.

Caring for a client with active TB about disease transmission

Have the client wear a surgical mask while being transported outside the room.

Assessing an adolescnet who has been taking ibuprofen for 6 months to treat juvenile idiopathic arthritis. Assess for adverse effects

Have you had any stomach pain or bloody stools

The nurse caring for a Native American client plans care understanding that one belief of Native American healing practices is which of the following?

Healing takes time.

According to Erikson, the middle adult is in a period of generativity versus stagnation. What happens if developmental tasks are not achieved?

Health needs become a major concern

A group of high school students is attending a concert, which will be at a volume of 80 to 90 dB. What is a health consequence of this sound level?

Hearing loss may occur with a decibel level in this range.

A nurse is performing a physical examination of a primigravid client who's 8 weeks pregnant. At this time, the nurse expects to assess:

Hegar's sign. When performing a vaginal or rectovaginal examination, the nurse may assess Hegar's sign (softening of the uterine isthmus) between the 6th and 8th weeks of pregnancy.

Assessing a client who received 2 units of packed RBCs 48 hr ago. Findings that indicate the therapy is effective?

Hemoglobin 14.9 g/dL

A nurse consulting with a nutrition specialist knows it's important to consider a special diet for a client with chronic obstructive pulmonary disease (COPD). Which diet is appropriate for this client?

High-protein

A client requests the nurse not touch his lips when administering his oral medications. Based on the nurse's understanding of the major religions, the nurse identifies this request as reflecting which of the following?

Hinduism

The nurse is assigned to a 52-year-old male patient. He is talkative and usually friendly when the nurse enters his room. Today, however, he is standing at the mirror and says: I lost my job because the company downsized, there isn't anything I can do. As his caregiver, the nurse recognizes this expression of concern is related to which of the following?

His career goals and retirement plans are compromised

The nurse is reviewing a client's prenatal history. Which of the following is a significant factor in anticipating complications in labor and birth?

History of postpartum hemorrhage (PPH)

thyroid nodules - hot-warm-cold

Hot nodules - hyperfunctioning of thyroid tissue - cancer in 5-9% - Warm nodules - moderate functioning of thyroid - cancer in 5-9% - Cold nodules - hypofunctioning of thyroid tissue - cancer in 15-20%

A nurse is monitoring a client receiving tranylcypromine sulfate. Which serious adverse reaction can occur with high dosages of this monoamine oxidase (MAO) inhibitor? You Selected:

Hypertensive crisis. The most serious adverse reaction associated with high doses of MAO inhibitors is hypertensive crisis, which can lead to death. Although not a crisis, orthostatic hypotension is also common and may lead to syncope with high doses. Muscle spasticity (not flaccidity) is associated with MAO inhibitor therapy. Hypoglycemia isn't an adverse reaction of MAO inhibitors.

assessment findings for hypoparathyroidism

Hypocalcemia and tetany - Neurological symptoms such as stridor, tingling sensation, and spasms, seizures, Trusseau's sign (positive when carpopedal spasm is induced by occluding the blood flow to the arm for 30 minutes with a BP cuff - Autoimmune - occurs when the body tissues are attacked by its own immune system - Positive Chvostek's sign (when a sharp tapping over the facial nerve in front of the parotid gland and anterior to the ear causes spasm or twitching of the mouth - Cardiovascular: arrhythmias, prolonged QT interval, cardiac arrest - Calcifications: cataracts, soft tissues,malformations of the teeth, including weakened tooth enamel and misshapen roots of the teeth -Hypocalcemia, hyperphosphatemia

Caring for a client who is in resuscitation phase of burn injury

Hyponatremia

anterior pituitary hypofunction

Hyposecretion of one or more of the pituitary hormones caused by tumors, trauma, encephalitis, autoimmunity, or stroke Hormones most often affected are growth hormone (GH) and gonadotropic hormones (luteinizing hormone, follicle-stimulating hormone), but thyroid-stimulating hormone (TSH), adrenocorticotropic hormone (ACTH), or antidiuretic hormone (ADH) may be involved.

reproductive s&s hypothyroid

Hypothyroidism influences ovarian function by decreasing levels of sex-hormone-binding-globulin (SHBG) and increasing the secretion of prolactin - Prolactin, the hormone which stimulates milk production for breastfeeding, also affects ovulation and menstrual cycles. - Prolactin inhibits Follicle-Stimulating Hormone (FSH) and gonadotropin releasing hormone (GnRH) - With high levels of prolactin, which can be caused by hypothyroidism, ovulation is not triggered and a woman cannot get pregnant - In men, low FSH and GnRH caused by elevated levels of prolactin can prevent the maturation of sperm

Providing teaching about advance directives to a middle adult client. Which response indicates understanding?

I can designate my partner as my health care surrogate

Talking with a client with stage IV breast cancer. Which statement is a constructive use of a defense mechanism

I told my doctor that I would like to start a support group for other women who are sick in my community

Providing info to a client immediately before Romberg test

I will be checking you once with your eyes open and once with them closed

Discharge instructions about newborn care to a client 2 days postpartum

I will cover my baby's body when I wash her hair. I will use the bulb syringe first in her mouth and then in her nose.

Providing education to the parent of school age child with asthma. Indicates understanding?

I will make sure my child receives a yearly influenza vaccination

*statement made by patient with DI requires further teaching

I will wean myself out ADH inhalant

Discussing common prenatal discomfort 20 weeks gestational

I will wear a supportive bra overnight

Charge nurse notices one of the nurses on the shift frequently violates unit policies by taking an extended amount of time on break. Statement that addresses the conflict

I would like to talk to you about the nit policies regarding break time.

A nurse is explaining the use of an IUD to a female client interested in obtaining contraception. Which of the following statements regarding the IUD is correct?

IUDs seem to affect the way the sperm or egg moves.

Conducting a mental status examination on newly admitted client. Priority assesment

Ideals of self harm

An obese client is admitted to the hospital for abusing amphetamines in an attempt to lose weight. Which nursing intervention is appropriate for this client?

Identify alternative ways for the client to lose weight.

A nurse has developed a plan of care to meet the needs of a client with the nursing diagnosis of Spiritual Distress. What would be an expected outcome?

Identify factors in life that challenge spiritual beliefs.

A client in the intensive care unit will experience less sensory overload in which of the following situations?

If a clock displays date, time, AM/PM

A client develops a facial rash and urticaria after receiving penicillin. Which laboratory value does the nurse expect to be elevated?

IgE. Immunoglobulin E (IgE) is involved with an allergic reaction. IgA combines with antigens and activates the complement system. IgB coats the surface of B lymphocytes. IgG is the principal immunoglobulin formed in response to most infectious agents.

When performing an assessment, the nurse collects the following data: impaired coordination, decreased muscle strength, limited range of motion, and the client's reluctance to move. This data indicates which nursing diagnosis?

Impaired mobility

caring for a client who has schizophrenia who recently started risperidone

Implement fall precautions for the client

Which of the following statements is true of the older adult population?

Incontinence is not a part of aging.

If you determine that the infant in the question above is experiencing normal spitting up associated with his developmental age, develop a brief teaching plan to review with the mother.

Increase the frequency and decrease the volume of feedings; make sure the infant burps at least two or three times per feeding; keep the infant upright for 20 to 30 minutes after feeding; avoid placing the infant in an infant seat after feeding. If vomiting increases or becomes projectile or if the infant is failing to gain weight, notify the primary care provider.

Assessing a client with decreased visual acuity due to cataracts. Identify physiological changes

Increased opacity of the lens

Caring for a client with COPD and becomes extremely short of breath. Interventions by the nurse requires completion of an incident report

Increasing oxygen via nasal cannula to 6 L/min

According to Erikson's theory of development, chronic illness can interfere with which stage of development in an 11-year-old child?

Industry versus inferiority

Child with sickle cell anemia and is having a vaso-occlusive crisis

Infuse IV fluids

Goitrogens

Inhibit T3 andT4 synthesis - soybeans, cabbage, strawberries, etc

Myxedema coma

Initially patient may show signs of depression, lethargy and diminished cognitive status -Respiratory drive depression resulting in alveolar hypoventilation, progressive carbon dioxide retention and coma

Caring for a client with a magnesium level of 2.5 mEq/l

Initiate continuous cardiac monitoring

A multigravid client at 34 weeks' gestation who is leaking amniotic fluid has just been hospitalized with a diagnosis of preterm premature rupture of membranes and preterm labor. The client's contractions are 20 minutes apart, lasting 20 to 30 seconds. Her cervix is dilated to 2 cm. The nurse reviews prescriptions (see chart). Which prescription should the nurse initiate first?

Initiate fetal and contraction monitoring. The nurse should initiate fetal and contraction monitoring for this client upon arrival to the unit. This gives the nurse data regarding changes in fetal and maternal contraction status before completing the other prescriptions. Next, the betamethasone would be given to begin the maturation process for the fetal lungs. The nurse should then start an intravenous infusion to provide a line for immediate intravenous access, if needed, and provide hydration for the client. The nurse should obtain the urine specimen prior to administering any antibiotic therapy, if prescribed.

Caring for a child with a fever and fluid filled vesicles on trunk and extremities. Identify priority

Initiate transmission based precautions

Preparing to mix haloperidol lactate 5 mg/mL and diphenhydramine 25 mg/1.5 mL to administer IM to an agitated client

Inject air into both vials prior to withdrawing the medications.

Adrenal Medulla

Inner layer of the adrenal gland, functions as part of the autonomic nervous system -often released into the bloodstream in response to stress or fright and prepare the body for "fight-or-flight"/produces "fight or flight" catecholamines -catecholamines are adrenaline (epinephrine), noradrenaline (norepinephrine), and dopamine.

Caring for a client who has a fecal impaction actions to be taken when digitally evacuating the stool

Insert a lubricated gloved finger and advance along the rectal wall.

A child has been experiencing recurrent episodes of acute otitis media (AOM). The nurse should anticipate that what intervention is likely to be ordered?

Insertion of a ventilation tube

Oncology unit nurse is administering doxorubicin to breast cancer patient

Inspect the client's mucosa for petechiae every 8 hrs. This med causes thrombocytopenia and increases the risk of bleeding

Performing an abdominal assessment on a client Sequence of actions

Inspect, auscultate, purcuss then palpate.

When performing an abdominal assessment, the nurse should follow which examination sequence?

Inspection, auscultation, percussion, and palpation

A nurse is teaching a patient with glaucoma how to administer eye drops to achieve maximum absorption. The nurse should teach the patient to perform what action?

Instill the medication in the conjunctival sac.

Preparing to assist with a thoracentesis for a client who has pleurisy

Instruct the client to avoid deep breathing during the procedure

caring for a client with DVT

Instruct the client to elevate the affected extremity when sitting

Preparing client for paracentesis. Action to take

Instruct the client to void.

The nurse is caring for a patient who has undergone a mastoidectomy. In an effort to prevent postoperative infection, what intervention should the nurse implement?

Instruct the patient to protect the ear from water for several weeks.

Which of the following hospital units is more likely to cause severe sensory alterations?

Intensive care

A client who has been diagnosed with renal calculi reports that the pain is intermittent and less colicky. Which nursing action is most important at this time?

Intermittent pain that is less colicky indicates that the calculi may be moving along the urinary tract. Fluids should be encouraged to promote movement, and the urine should be strained to detect passage of the stone. Hematuria is to be expected from the irritation of the stone. Analgesics should be administered when the client needs them, not routinely. Moist heat to the flank area is helpful when renal colic occurs, but it is less necessary as pain is lessened.

A nurse is assessing a client on the first day after major abdominal surgery. Which of the following internal stimuli would be increased and affect client responses?

Intravenous lines, pain

A woman is using Depo-Provera as a method of birth control. What common side effect should the nurse explain to the client?

Irregular bleeding

Continuous bladder irrigation following transurethral resection of prostate. Bladder spasms and decreased urinary output

Irrigate the catheter with 0.9% sodium chloride irrigation

Teaching a client who newly diagnosed with DM. Instructions on manifestations of hypoglycemia

Irritability

infant with hydrocephalus 6 hr post op following ventriculoperitoneal shunt placement . finding to report to provider

Irritability when being held. signifies increased ICP

Several residents of a long-term care facility have developed signs and symptoms of viral conjunctivitis. What is the most appropriate action of the nurse who oversees care in the facility?

Isolate affected residents from residents who have not developed conjunctivitis.

The nurse conducting a class on human sexuality includes which of the following about gender identity?

It may be the same as or different from biologic gender.

Caring for a client who is taking valproic acid for seizure control. Adverse effect to monitor and report

Jaundice-clients are at risk for liver damage.

The nurse has answered the telephone at the nurses' station, and the individual on the line states that there is bomb in the healthcare facility. What is the nurse's best response?

Keep the individual on the line in order to gather more information about the details of the threat. If a bomb threat is received, the nurse should keep the caller on the line and talking as long as possible in order to gather information about the location of the bomb and a description of the bomb and the caller. The threat must be reported promptly, but the nurse should not hang up in order to do this.

A client has sustained a right tibial fracture and has just had a cast applied. Which instruction should the nurse provide regarding cast care?

Keep your right leg elevated above heart level.

Which of the following vision disorders is most likely to have an infectious etiology?

Keratitis

A nurse assesses a mother who rocks and cuddles her newborn son. What sense is the mother stimulating?

Kinesthetic

During a client assessment, the nurse has the client close his eyes. She then places her finger on his right thigh. She asks the client where he is being touched and he answers "my right thigh." This is an example of which sense?

Kinesthetic

Personal space and distance is a cultural perspective that can impact nurse-client interactions. What is the best way for the nurse to interact physically with a client who has a different cultural perspective on space and distance?

Know the client's cultural personal space preferences.

A patient presents to the ED complaining of a sudden onset of incapacitating vertigo, with nausea and vomiting and tinnitus. The patient mentions to the nurse that she suddenly cannot hear very well. What would the nurse suspect the patient's diagnosis will be?

Labyrinthitis

The nurse is obtaining a health history from a patient of Puerto Rican descent. Which of the following is most likely to be a health problem that has a cultural connection for this patient?

Lactose enzyme deficiency

A nurse observes the following pattern on an EFM for an intrapartum client. Which action should the nurse take? A. Continue to monitor B. Perform vaginal exam and reposition C. Reposition client and apply oxygen D. Prepare for immediate cesarean birth

Late decelerations C. Reposition client and apply oxygen

The nurse observes the unlicensed assistive personnel (UAP) obtain a capillary glucose sample. Which is the best location for obtaining a blood glucose sample?

Lateral aspect of finger; end of finger is not recommended d/t less blood flow and more nerve fibers.

The nurse is planning the care of a patient who is adapting to the use of a hearing aid for the first time. What is the most significant challenge experienced by a patient with hearing loss who is adapting to using a hearing aid for the first time?

Learning to cope with amplification of background noise

The nurse is providing home care for a client who traditionally drinks herbal tea to treat an illness. How should the nurse respond to a request for the herbal tea?

Let me check with the doctor to make sure it is okay to drink the tea with your medicines.

How can religious, life-affirming influences be compared with basic human needs?

Life-affirming influences encourage self-actualization.

The nurse is caring for a client taking an anticoagulant. Which instruction regarding anticoagulant therapy should the nurse give the client?

Limit foods high in vitamin K.

A client with peptic ulcer disease is taking ranitidine. What is the expected outcome of this drug?

Limit gastric acid secretion. Histamine-2 (H2) receptor antagonists, such as ranitidine, reduce gastric acid secretion. Antisecretories, or proton-pump inhibitors, such as omeprazole, help ulcers heal quickly in 4 to 8 weeks. Cytoprotective drugs, such as sucralfate, protect the ulcer surface against acid, bile, and pepsin. Antacids reduce acid concentration and help reduce symptoms.

The nurse delivers a client's 10 a.m. medications. The client is away from his room for a diagnostic study. Which action is most appropriate for the nurse to take?

Lock the medications in the medicine preparation area until the client returns.

In which of the following health care settings is a client more likely to be at risk for sensory deprivation?

Long-term care

Which of the following visual deficits would be categorized as homonymous hemianopia?

Loss of the same side of the visual field in each eye

What spiritual need is believed to underlie all religious traditions and is common to all people?

Love and relatedness

A nurse is providing dietary instructions to a client with a history of pancreatitis. Which of the following instructions would be most appropriate?

Maintain a high-carbohydrate, low-fat diet. A client with a history of pancreatitis should avoid foods and beverages that stimulate the pancreas, such as fatty foods, caffeine, and gas-forming foods; should avoid eating large meals; and should eat plenty of carbohydrates, which are easily metabolized. Therefore, the only correct instruction is to maintain a high-carbohydrate, low-fat diet. An increased sodium or fluid intake is not necessary because chronic pancreatitis is not associated with hyponatremia or fluid loss.

An Anglo American client reports to the primary health care facility with symptoms of fever, cough, and running nose. While interviewing the client, which of the following points should the nurse keep in mind?

Maintain eye contact while talking.

Preparing a sterile field for sterile dressing change.

Maintain sterile objects within the line of vision

The advanced practice nurse is attempting to examine the patient's ear with an otoscope. Because of impacted cerumen, the tympanic membrane cannot be visualized. The nurse irrigates the patient's ear with a solution of hydrogen peroxide and water to remove the impacted cerumen. What nursing intervention is most important to minimize nausea and vertigo during the procedure?

Maintain the irrigation fluid at a warm temperature.

hypothalamus

Maintains homeostasis by regulating the internal environment such as the heart rate, body temperature, water balance and the pituitary gland secretions

Caring for a client with a new diagnosis of terminal cancer. Client states he would like to go home to be with family and loved ones.

Make a referral for social services

Preparing to teach a group of newly licensed nurses about effective time management. Include as priority

Making a list of activities to complete

A postpartum client's husband calls the nurse and says, "My wife feels funny." The nurse enters the room and notes blood gushing from the client's vagina, pallor, and a rapid, thready pulse. What should be the nurse's first intervention?

Massage the fundus. Postpartum hemorrhage results in excessive vaginal bleeding and signs of shock, such as pallor and a rapid, thready pulse. Placental separation causes a sudden gush or trickle of blood from the vagina, rise of the fundus in the abdomen, increased umbilical cord length at the introitus, and a globe-shaped uterus. Uterine involution causes a firmly contracted uterus, which cannot occur until the placenta is delivered. Cervical lacerations produce a steady flow of bright red blood in a client with a firmly contracted uterus. The priority measure to correct postpartum hemorrhage is to massage the fundus. Packing the uterus with sterile gauze is contraindicated. The physician will have to be called but the nurse must first intervene.

Client who has fluid volume overload. Which tasks can nurse delegate to UAP

Measure the client's daily weight

Thyrotropin-releasing hormone (TRH) Test

Measures TSH before and after giving TSR Excessive increase in TSH - primary hypothyroidism

What is the most significant difficulty regarding sexuality faced by people taking medications for hypertension?

Medications change sexual functioning.

A nurse is developing a plan of care for an older adult with chronic heart disease. Which of the following factors must be considered?

Medications, hospitalizations, and medical supplies increase economic difficulties.

The nurse caring for a Native American client should inquire if the client utilizes which of the following?

Medicine man or woman

A nurse is caring for a client undergoing opiate withdrawal, which causes severe physical discomfort and can be life-threatening. To minimize these effects, opiate users are commonly detoxified with:

Methadone is used to detoxify opiate users because it binds with opioid receptors at many sites in the central nervous system but doesn't have the same deleterious effects as such opiates as cocaine, heroin, and morphine. Barbiturates, amphetamines, and benzodiazepines are highly addictive and using these drugs would make detoxification treatment necessary.

The nurse on the medical unit is performing a physical assessment on a newly admitted patient. To locate the point of maximum impulse (PMI) of the patient's heart, the nurse's hand (fingertips) should be placed over which of the following locations? 1. The fifth intercostal space directly over the sternum. 2. The second intercostal space to the right of the sternum. 3. The second intercostal space to the left of the sternum. 4. The fifth intercostal space at the midclavicular line.

Question: How do you locate the PMI? Strategy: Picture the anatomy of the heart and its position in the body. Needed Info: PMI: forward thrust of L ventricle during systole produces normal pulsation on chest wall; indicates size and position of heart; should be felt in 1 intercostal space; if larger, indicates ventricular enlargement. (1) position of R ventricle (2) best for aortic valve sounds (3) best for pulmonic valve sounds (4) CORRECT

The nurse cares for the 4-year-old diagnosed with a fractured pelvis due to an auto accident. The nurse prepares the child for the application of a hip spica cast. The nurse recognizes that it is MOST important to include which of the following in the child's plan of care? 1. Obtain a doll with a hip spica cast in place. 2. Tell the child that the cast will feel cold when it is put on the skin. 3. Reassure the child that the cast application is painless. 4. Introduce the child to another child who has a hip spica cast.

Question: How do you prepare a 4-year-old for the procedure? Strategy: "MOST important" indicates that discrimination is required to answer the question. Needed Info: Preschool children (age 36 months - 6 years) fear injury, mutilation, and punishment; allow child to play with models of equipment; encourage expression of feelings; spica cast immobilizes the hip and knee. (1) CORRECT - preschoolers need to see and play with dolls and equipment; explain procedure in simple terms and explain how it will affect the child (2) may feel a warm or burning sensation under cast while it dries, due to chemical reaction between the plaster and the water (3) will be placed on special cast table that holds the child's body; turning to apply the cast may be painful (4) more important to allow child to play with doll with a hip spica cast; viewing the cast may be frightening.

The nurse plans care for the adult with pneumonia. The patient is to be suctioned PRN. Which of the following techniques, if used by the nurse, MOST accurately describes proper suctioning? 1. Apply suction with rotation, for no more than 20 seconds, as the catheter is inserted. 2. Apply suction, for no more than 10 seconds, as the catheter is both inserted and withdrawn. 3. Apply suction, for no more than 10 seconds, as the catheter is withdrawn. 4. Apply suction each time the patient inhales.

Question: How do you suction an adult? Strategy: Think about the outcome of each answer choice. Needed Info: Pneumonia: infection of the lungs due to viruses/bacteria, aspiration of food/fluids or inhalation of toxic chemicals. S/S: fever, chills, hemoptysis, dyspnea, fatigue. Treatment: antibiotics. Nursing responsibilities: turn, cough, deep breath, Fowler's position; suction to remove secretions and provide open airway; use 12 - 14 French catheter; use suction pressure less than 120 mm Hg and gently rotate the catheter 360 degrees. Complications: infection, trauma, hypoxemia, dysrhythmias. (1) not done for that length of time or during insertion, which would cause trauma to the mucous membrane (2) no suction when inserted (3) CORRECT - short time of suctioning, when pulling out catheter; too long can cause hypoxia, dysrhythmias; hyperoxygenate before, during, and after (4) suctioning process not correlated with breathing pattern.

The nurse cares for the client who is to receive warfarin sodium. The nurse recalls that which of the following is the mechanism of action of this medication? 1. It inhibits prothrombin synthesis. 2. It prevents conversion of fibrinogen to fibrin. 3. It inactivates thrombin. 4. It inhibits platelet aggregation.

Question: How does Coumadin work? Strategy: Think about each answer choice. Needed Info: warfarin sodium (Coumadin): long acting anticoagulant that inhibits Vitamin K-dependent clotting factors. Side effects: excessive dosage may cause hemorrhage, rash, fever. Prothrombin time (PT) used to control dosage. Therapeutic range is 1.5 - 2 times normal level. Antidote vitamin K (phytonadione: Mephyton). May eat consistent amounts of green leafy vegetables containing vitamin K. (1) CORRECT (2) action of heparin (3) action of heparin (4) action of aspirin and dipyridamole (Persantine).

The nurse cares for the client receiving lansoprazole. The nurse recognizes that lansoprazole has which of the following effects on the gastrointestinal system? 1. It increases bowel motility. 2. It reduces bowel motility. 3. It neutralizes gastric acid secretion. 4. It decreases gastric acid secretion.

Question: How does lansoprazole work? Strategy: Think about the actions of lansoprazole. Needed Info: Lansoprazole (Prevacid) is a protein pump inhibitor used to treat and prevent stomach and intestinal ulcers. It reduces gastric acid production. Prevacid 24HR should be taken only once every 24 hours for 2 weeks. May take 4 days to work. (1) no effect on bowel motility; laxatives (bisacodyl: Dulcolax) increase motility, these are contraindicated for pt with abdominal pain (2) no effect on bowel motility (3) effect of antacids such as aluminum hydroxide with magnesium hydroxide (Maalox) or aluminum hydroxide with magnesium hydroxide and simethicone (Mylanta) (4) CORRECT

The nurse cares for the patient with Parkinson's disease who is receiving levodopa. The nurse recalls that levodopa works by which of the following actions? 1. It blocks central cholinergic receptors. 2. It restores dopamine levels in extrapyramidal centers. 3. It releases dopamine and other catecholamines from neuronal storage sites. 4. It activates dopaminergic receptors in the basal ganglia.

Question: How does levodopa work? Strategy: Think about each answer choice and how it relates to Parkinson's disease. Needed Info: Parkinson's disease: caused by impairment of dopamine-producing cells in the brain. Levodopa is converted to dopamine in the body to supply the extrapyramidal centers in the brain. Side effects: hemolytic anemia, aggressive behavior, dystonic movements, depression, hallucinations, dizziness, orthostatic hypotension. (1) action of benztropine mesylate (Cogentin) used with levodopa; side effects: urinary retention, dry mouth, constipation; takes 2 - 3 days before effects are seen (2) CORRECT - don't take with vit B6 or fortified cereals: will block effects (3) action of amantadine (Symmetrel) used with levodopa; side effects: irritability, insomnia, dizziness; take after meals (4) action of bromocriptine (Parlodel) used with levodopa; side effects: dizziness, HA, orthostatic hypotension, abdominal cramps, pleural effusion; take with meals.

The nurse cares for the patient on the telemetry unit. The patient's orders include nifedipine 10 mg PO TID. The patient asks the nurse how the medication works. Which is the BEST response by the nurse? 1. "It constricts the coronary arteries." 2. "It increases myocardial contractility." 3. "It decreases myocardial oxygen demand." 4. "It promotes coronary artery spasms."

Question: How does nifedipine work? Strategy: Determine the outcome of each answer choice. Is it desirable? Needed Info: nifedipine (Procardia): antianginal medication that is a calcium channel blocker (inhibits calcium ion flow across cardiac and smooth muscle). Side effects: light-headedness, HA, hypotension, hypokalemia. Nursing responsibilities: monitor BP and potassium levels. (1) dilates coronary arteries (2) decreases myocardial muscle contractility; digoxin increases myocardial contractility (3) CORRECT - antianginal (4) inaccurate.

The client is admitted to the hospital for surgery on a ruptured anterior cruciate ligament in the right knee. Following surgery, the physician prescribes morphine sulfate to be administered using a patient-controlled analgesia (PCA) pump. Which explanation by the nurse BEST describes this method of pain medication administration? 1. "You will contact your nurse when you feel pain, and the nurse will bring pain medication to add to your intravenous pump." 2. "You will receive a large dose of pain medication continually from an intravenous pump." 3. "You will be able to self-administer a preset dose of pain medication as needed by pressing a button connected to the intravenous pump." 4. "You will be able to self-administer an unlimited amount of pain medication as needed by pressing a button connected to the intravenous pump.

Question: How is a PCA pump used? Strategy: "BEST" indicates that there may be more than one correct response. Each part of the answer choice must be correct. Needed Info: PCA amount allows patients to control administration of IV analgesics; preloaded pump system administers preset amount of medication when button is pushed by patient; predetermined lock-out time interval; amount of medication is displayed on front of machine; reduces pulmonary complications, and patient is more alert. (1) contact with a nurse is not required (2) client does not get a large amount of medication continuously, which might lead to an unintentional drug overdose (3) CORRECT - client is able to pace the rate of medication being dispensed by taking responsibility for when it is administered; provider continues to take responsibility for the amount administered at any one time (4) client gets only a pre-set amount of medication at any one time.

The client is admitted for evaluation of a convulsive disorder. An electroencephalogram (EEG) is scheduled. The client asks the nurse how an EEG is performed. Which of the following explanations by the nurse is MOST accurate? 1. "Several small electrical shocks are given that feel like pinpricks." 2. "Electrodes are attached to the head and the electrical activity of the brain is evaluated." 3. "A radiopaque substance is injected into an artery and x-rays are taken." 4. "A radioactive material is injected intravenously followed by a brain scan."

Question: How is an EEG performed? Strategy: Picture the test being performed. Needed Info: EEG: measurement of the electrical activity of the brain to evaluate seizure disorders. Nursing responsibilities: keep awake night before test; shampoo client's hair; stimulants (coffee, tea, cigarettes, cola), antidepressants, tranquilizers, anticonvulsants held for 24 - 48 hours before test to avoid alteration (particularly lowering) of the seizure threshold. (1) not accurate; no shocks given; painless procedure (2) CORRECT -readings taken awake, asleep, while hyperventilating, viewing flickering lights (3) cerebral angiography; nursing responsibilities: check for allergies to contrast medium and iodine; prep: NPO 6 - 8 hours, will feel heat sensation when dye injected; post-test: bed rest 6 - 24 hours, pressure dressing over insertion site 6 - 12 hours, force fluids for 24 hours to excrete contrast medium (4) brain positron emission tomography (PET) scan: preparation: empty bladder; isotope injected, takes 2 hours to be absorbed in brain; post-test: force fluids, urine does not need special care.

The nurse cares for the client receiving pain medication via a patient controlled analgesia pump (PCA). The syringe contains hydromorphone 6 mg in 30 mL. The client is prescribed hydromorphone 0.2 mg/hour IV per the PCA pump. How many milliliters per hour does the client receive?

Question: How much hydromorphone is needed an hour on this PCA pump? Strategy: Utilize the correct equation to figure out the answer in mL per hour. Needed Info: Set the answer up as a ratio proportion. The answer is being calculated from the concentration of hydromorphone (Dilaudid) available in order to figure out the hourly rate to be set on the pump. CORRECT ANSWER: 1 mL / hour. (0.2 mg / 6 mg) x 30 mL = 1 mL/hour

The nurse teaches the group of men about testicular cancer and testicular self-examination. The nurse instructs the men to perform testicular self-examination at which frequency? 1. Weekly. 2. Monthly. 3. Yearly. 4. Biannually.

Question: How often should men do a testicular exam? Strategy: Think about each answer choice. Needed Info: Testicular cancer most common in men 15 - 34 years old; best to do after a shower when the body is warm and relaxed; hold scrotum in the palm of hand; roll each testicle between thumb and fingers. Symptoms of cancer: painless enlargement or heaviness in testicle. (1) too often (2) CORRECT (3) too infrequent (4) too infrequent.

The father of the day-old infant tells the nurse that he will be driving his wife and infant home from the hospital. It is MOST important for the nurse to make which of the following recommendations for how the infant should be transported? 1. In a front-facing infant car seat in the back seat. 2. In a rear-facing infant car seat in the back seat. 3. In an infant seat on the wife's lap in the passenger seat. 4. In the wife's arms in the back seat.

Question: How should a newborn be transported in a car? Strategy: Think about the outcome of each answer choice. Needed Info: The American Academy of Pediatrics recommends that children under 2 years of age use a rear-facing infant car seat. Children should ride in the rear of a vehicle until they are 13 years old. (1) bone structure inadequate to handle motor vehicle accidents (2) CORRECT - until a minimum age of 2 years, can be longer if the child is small for age (3) unsafe (4) unsafe.

The nurse cares for the postoperative patient who is to receive psyllium. When administering psyllium, the nurse uses which of the following techniques? 1. Mix with 6 ounces of orange juice; let stand for 1 minute, then administer it. 2. Mix with 8 ounces of water; administer it immediately followed by another 8 ounces of water. 3. Sprinkle on the patient's food; add 4 ounces of water and mix until well blended. 4. Pour into 8 ounces of milk; let it stand for 1 minute, then administer it.

Question: How should you mix psyllium? Strategy: Think about the outcome of each answer choice. Needed Info: psyllium (Metamucil): bulk-forming laxative used to treat constipation; on contact with water it forms a bland, gelatinous bulk that promotes peristalsis; can be mixed with water, milk, or fruit juice. (1) do not let stand; use 8 oz fluid (2) CORRECT (3) should not be chewed (4) do not let stand.

The patient with peripheral vascular disease is returned to the room following a right below-the-knee amputation (BKA). During the first 24 hours postoperatively, how does the nurse position the patient's residual limb? 1. Elevates the stump by raising the foot of the bed on blocks. 2. Dangles the stump over the side of the bed. 3. Abducts the stump by placing pillows between the legs. 4. Places the stump in correct anatomical alignment.

Question: How should you position a patient after a BKA? Strategy: Determine the outcome of each answer choice. Needed Info: Common complication after amputation is hip flexion contracture, resulting from elevation of stump on pillows. (1) CORRECT - increases venous return, prevents edema, promotes comfort (2) increases edema (3) legs should be adducted, not abducted (4) after 24 hours.

The pregnant woman is given an epidural anesthetic in preparation for cesarean section. Following administration of the epidural, the patient's blood pressure falls from 120/84 to 94/50. The nurse recognizes that it is ESSENTIAL to assist the patient into which of the following positions? 1. Supine. 2. Sitting. 3. Side-lying. 4. Trendelenburg.

Question: How should you position the patient? Strategy: "Essential" indicates that this is a priority question. Think about the outcome of each answer choice. Needed Info: Spinal anesthetic: local anesthetic injected into the lumbar intervertebral space beyond the dura mater into the subarachnoid space, which blocks pain sensations and movement. Epidural: local anesthetic injected into the lumbar intervertebral space outside the dura mater, which blocks pain sensations only, not movement. Complication of regional anesthetics: sympathetic nerve fibers blocked, hypotension due to loss of vasoconstrictor ability. Prehydrate before regional anesthetic to ensure adequate blood volume. (1) position impedes blood return to maternal heart due to pressure of fetus on vena cava; worsens hypotension (2) causes greater pooling of blood in legs, contributing to hypotension (3) CORRECT - optimizes blood return from lower extremities; displaces heavy uterus from inferior vena cava (4) weight of uterus against diaphragm impedes respirations.

The nurse in the well child clinic receives a call from a parent stating the parent's child attended a birthday party the day before with a child who had a facial rash and was diagnosed with erythema infectiosum (fifth disease). The parent is concerned that the parent's child may develop the disease. Which of the following responses by the nurse is BEST? 1. "Your child will not develop the disease." 2. "Look for a rash in 4 to 14 days." 3. "Bring your child into the clinic this afternoon." 4. "Does your child have a facial rash now?

Question: Is a child with a rash due to fifth disease contagious? Strategy: Think about the outcome of each answer. Needed Info: Erythema infectiosum (fifth disease) is a virus caused by human parvovirus B19; symptoms include erythema on face, lacy red rash on trunk and limbs. May have cold-like symptoms prior to onset of rash; treatment includes antipyretics, analgesics, and anti-inflammatory drugs. (1) CORRECT - fifth disease is a virus that is found in respiratory secretions; not contagious after the rash develops (2) incubation period is 4 to 14 days but may be as long as 20 days; an infected person is contagious prior to development of rash, whereas the child at the party already had a rash; (3) no reason for child to come to clinic (4) no reasons to assess for a rash.

The nurse performs discharge teaching for the patient with chronic renal failure. The nurse recognizes that teaching has been successful if the patient makes which statement? 1. "I will weigh myself every morning, before I eat breakfast." 2. "I will restrict my sodium and protein intake." 3. "I will take my antivomiting pills right after I eat." 4. "I will avoid between-meal snacks."

Question: What are appropriate self-care activities for a patient with chronic renal failure? Strategy: Recall the pathophysiology of chronic renal failure. Needed Info: Chronic renal failure is the slow progressive loss of renal function; gain of 2 pounds or more in 24 hours indicates fluid retention; antiemetics can be taken 30 to 60 minutes before meals; to increase nutritional balance, sodium and protein are not restricted, and between-meal snacks are encouraged. (1) CORRECT - most accurate measurement (2) not useful approach (3) need to be taken before the meal (4) less likely to prompt nausea.

The nurse cares for children in the pediatric clinic. After assessing a 3-year-old, the nurse instructs the child's parent about safety precautions. The nurse determines teaching is effective if the parent states which of the following? Select all that apply. 1. "My child wears a helmet while riding in the bike seat on the back of my bike." 2. "Our cleaning products at home are in a closed cabinet below the kitchen sink." 3. "Our medications are kept in childproof containers in a locked cabinet." 4. "My child sits in a forward-facing seat with a harness while riding in the car." 5. "Since we call vitamins 'candy,' our child eagerly takes them each day." 6. "The number to poison control is posted on our refrigerator."

Question: What are correct child safety precautions? Strategy: Consider each answer as a child safety precaution. More than one answer will be correct. Needed Info: Unintentional injury is the leading cause of death among children aged 1 - 4 years, accounting for about one-third of their deaths. (1) CORRECT - bike helmets prevent concussions if the child falls off the bike (2) need for locked cabinet, not just closed door (3) CORRECT - the locked cabinet is key to preventing accidental poisoning; childproof containers are not sufficient by themselves (4) CORRECT - the American Academy of Pediatrics in June 2011 recommended using a backward-facing seat for children under 2 years of age; use a forward-facing seat with a harness for children over 2 years (5) does not give child idea of possible danger of taking more. (6) CORRECT - posting an emergency contact in a visible spot allows faster response when time is critical.

The nurse prepares to change the central line dressing on the child. Arrange the following steps of the procedure in the correct order from first to last. All options must be used. 1. Open sterile towel to create a field. 2. Apply nonsterile gloves. 3. Apply an occlusive dressing. 4. Remove old dressing and wash hands. 5. Cleanse the area with acetone and alcohol swabs in a circular motion. 6. Cleanse the area with 1% povidone-iodine swabs. 1. 1, 2, 4, 5, 6, 3 2. 1, 4, 2, 6, 5, 3 3. 2, 4, 1, 5, 6, 3 4. 2, 4, 1, 6, 5, 3

Question: What are the steps of a central line dressing change for a child? Strategy: Be careful! Arrange the steps of the dressing change in the correct order in which to perform the task. Needed Info: The central line dressing needs to be changed regularly, at least every 48 hours using sterile technique. The site must be cleansed properly, and signs of infection need to be assessed. CORRECT ANSWER: 2, 4, 1, 5, 6, 3 (2) The first step is to apply nonsterile gloves. (4) The second step is to remove old dressing and wash hands. (1) The third step is to open sterile towel to create a field. (6) The fourth step is to cleanse the area with 1% povidone-iodine swabs. (5) The fifth step is to cleanse the area with acetone and alcohol swabs in a circular motion. (3) The sixth step is to apply an occlusive dressing.

The nurse cares for the patient who has syndrome of inappropriate antidiuretic hormone (SIADH). Which of the following findings does the nurse recognize as consistent with this diagnosis? 1. Increased urine output; decreased serum sodium. 2. Decreased urine output; increased serum sodium. 3. Increased urine output; increased serum sodium. 4. Decreased urine output; decreased serum sodium.

Question: What are the symptoms of SIADH? Strategy: Think about how the disease affects the body. Needed Info: SIADH: seen with head injury, encephalitis, lung cancer, myxedema. Excessive amounts of antidiuretic hormone (ADH) from posterior pituitary results in water imbalance; water intoxication occurs due to fluid retention; opposite of diabetes insipidus. S/S: decreased LOC (cerebral edema, increased intracranial pressure), seizures, coma, sodium less than 120 mEq/L. Treatment: fluid restriction (500 - 600 mL/24 hr), sodium replacement, diuretics. Nursing responsibilities: daily weight, neuro checks, I + O, check electrolytes, position flat in bed. (1) decreased urine output (2) in SIADH, the serum sodium is washed out, resulting in a decreased level (3) diabetes insipidus due to lack of ADH; S/S: polyuria, polydipsia; treatment: vasopressin (Pitressin) (4) CORRECT - alderosterone is suppressed, causing hyponatremia with normal or increased plasma volume

The nurse administers morphine sulfate as ordered to the patient reporting severe pain. The nurse recognizes that which of the following signs indicates morphine toxicity? 1. The patient has blurred vision. 2. The patient's pupils are pinpoint. 3. The patient's pupils are unequal. 4. The patient's pupils are dilated.

Question: What are the symptoms of morphine toxicity? Strategy: Think about what causes each symptom. Does it relate to morphine? Needed Info: Morphine: narcotic analgesic used for severe pain. Side effects: sedation, hypotension, N + V, urinary retention, and physical dependence. Nursing responsibilities: check pupils and respirations. (1) inaccurate (2) CORRECT - indicators of morphine toxicity include pinpoint pupils and/or respirations less than 13 per min (3) not seen (4) not seen.

The nurse observes the behavior of the patient seen in the emergency room. Which of the following indicates to the nurse that the patient is experiencing a panic level of anxiety? 1. Reduced sensory input, distorted perception, behavioral disorganization. 2. Behavioral disorganization, inability to negotiate simple life demands, increased ability to concentrate. 3. Heightened sensory awareness, impaired cognitive function, distorted perception. 4. Increased pulse, increased muscle tension, rate of speech and volume are adequate for communication.

Question: What behaviors indicate a panic level of anxiety? Strategy: All symptoms must be correct in order for the answer to be correct. Needed Info: Anxiety is feeling of dread or fear in the absence of external threat, or disproportionate to the nature of the threat. In panic level anxiety, the patient is unable to see, hear, or function. Assess level of anxiety, decrease environmental stimuli, use unhurried approach, and stay with the patient. (1) CORRECT - reduced sensory input, distorted perception, behavioral disorganization are indicators of panic (2) will have decreased ability to concentrate with panic (3) sensory input is decreased with panic (4) pulse, muscle tension, rate of speech and volume all increase with panic.

The nurse performs screening on a group of older adult males. The nurse recognizes that which of the following is the MOST frequent cause of their urinary problems? 1. Degeneration of the renal arteries. 2. Degeneration of prostatic tissue. 3. Hyperplasia of the renal arteries. 4. Hyperplasia of the prostate gland.

Question: What causes urinary problems in elderly males? Strategy: Think about the answers. Needed Info: Symptoms of benign prostatic hypertrophy (BPH) include dysuria, frequency, urgency, decreased urinary stream, hesitancy, and nocturia; later symptoms may include cystitis, hydronephrosis, or urinary calculi. (1) renal blood vessels become thickened and more rigid, rather than degenerated (2) prostate enlarges, rather than degenerates, due to aging (3) blood vessels become thickened and more rigid, but they do not cause the manifestations of BPH (4) CORRECT - prostate enlargement (hyperplasia) causes urethral obstruction

The nurse cares for the emaciated patient admitted with Crohn's disease (regional enteritis). The nurse expects the patient to be placed on which of the following diets? 1. High-calorie, high-protein, high-residue. 2. Low-calorie, low-protein, low-residue. 3. High-calorie, high-protein, low-residue. 4. Low-calorie, low-protein, high-residue.

Question: What diet is used for Crohn's disease? Strategy: Think about Crohn's disease. Needed Info: Crohn's disease (regional enteritis): inflammatory bowel disease involving segments of the terminal ileum and proximal colon. The entire wall of the colon is affected. Restricts absorption of nutrients. S/S: right lower quadrant abdominal pain, diarrhea, weight loss, low-grade fever. Remissions and exacerbations seen. Treatment: meds: antidiarrheals (loperamide), antispasmodics, anticholinergics, sulfonamides (sulfasalazine), steroids. (1) high residue; may cause diarrhea (2) nutritional deficiencies (3) CORRECT - nonirritating, high in nutrients and minerals (4) nutritional deficiencies and high residue; may cause diarrhea.

The nurse conducts a physical assessment of the newly admitted client on the medical unit. When auscultating breath sounds over the trachea, the nurse normally expects to hear sounds that can be BEST characterized as which of the following? 1. Soft and low pitched. 2. Coarse and rumbling. 3. Fine and crackling. 4. Loud and high pitched.

Question: What do breath sounds sound like over the trachea? Strategy: "Best categorized" indicates that there may be more than one correct response. Think about what causes each breath sound. Needed Info: Use diaphragm of stethoscope, have patient take slow, deep breaths through the mouth. If crackles or wheezes are heard, ask patient to cough to see if sound changes. (1) inaccurate; vesicular: peripheral parts of lungs (2) inaccurate; rhonchi/wheezes: air over mucus; abnormal (3) inaccurate; rales: air over fluid; abnormal (4) CORRECT - hollow, harsh sounding; air passing through a tube

The woman is brought to the emergency room complaining of severe left lower quadrant pain. She tells the nurse that she performed a home pregnancy test and believes she is 8 weeks pregnant. On admission the patient's vital signs are pulse 90, blood pressure 110/70, respirations 20. A half-hour later her vital signs are pulse 120, blood pressure 86/50, respirations 26. The nurse recognizes that the change in the patient's vital signs indicates which of the following? 1. The patient's pain may have increased. 2. The patient may be bleeding internally. 3. The patient may be frightened. 4. The patient may have an infection.

Question: What do the changes in vital signs mean? Strategy: Think about the significance of each assessment. Needed Info: Ectopic pregnancy: fetus implanted outside of uterus, usually the fallopian tube. (1) usually BP increases with pain increase (2) CORRECT - increased P, decreased BP = decreased intravascular volume; shock (3) usually BP + P increases with fear (4) usually won't change BP unless in septic shock.

The nurse cares for the newly admitted patient who reports abdominal cramping and generalized weakness. When the nurse sends a stool sample to the lab for a guaiac test, what positive finding could be expected? 1. White blood cells. 2. Red blood cells. 3. Ova and parasites. 4. Mucus.

Question: What does a guaiac test indicate? Strategy: Think about each answer choice. Needed Info: Guaiac fecal occult blood test (G-FOBT): occult means hidden, often done as a screening test for colon cancer. Positive indicates need for further studies. (1) not accurate (2) CORRECT - none usually found (3) not accurate (4) not accurate.

The patient visits the physician for HIV testing. The physician notifies the patient that the results are positive. The patient asks the nurse what this means. The nurse's response should be based on recognition of which of the following? 1. The patient has AIDS. 2. The patient will develop AIDS within the year. 3. The patient has been exposed to the HIV virus. 4. The patient has been infected with the HIV virus.

Question: What does it mean if a person is HIV positive? Strategy: Think about each answer choice: Is it true about AIDS? Needed Info: AIDS (acquired immunodeficiency syndrome): caused by human immunodeficiency virus (HIV). Alters the functioning of immune system. Transmission: contact with blood and body fluids (semen). Test that detects presence of antibodies: enzyme-linked immunosorbent assay (ELISA). Test that confirms presence of virus: Western blot. (1) not true; AIDS: defined as presence of complications (such as opportunistic infections--Pneumocystis pneumonia) from HIV virus (2) not completely certain; AIDS Related Complex (ARC) has symptoms (fever, drenching night sweats, weight loss, fatigue, lymphadenopathy) without opportunistic infections (3) no test determines exposure; only detects infection. (4) CORRECT - HIV virus is considered infected and infectious

The nurse cares for the woman who has just delivered her first child, a boy weighing 6 lb 2 oz. The Apgar scores at one and five minutes are 8 and 9. The nurse recognizes that these scores indicate which of the following? 1. An isolette should be ready in the nursery for close observation of this infant. 2. The newborn is making an optimal transition to extrauterine life. 3. The parents will need emotional support to deal with a less than perfect infant. 4. Apgar scores correlate well with future emotional and intellectual development.

Question: What does this Apgar mean? Strategy: Think about each answer choice. Is it true about Apgar score? Needed Info: Apgar scores, checked at 1 and 5 min, are used to assess a newborn's initial adaptation to extrauterine life. There are five categories, each of which gets a score of 0 - 2: heart rate, resp effort, muscle tone, reflex irritability, color. (1) not needed; radiant warmer (2) CORRECT - good Apgar (3) Apgar score is fine (4) no relationship.

The parent of the adolescent being admitted to the psychiatric unit reports that the adolescent has become increasingly withdrawn at home. During the admission interview with the nurse, the patient says, "When I look in the mirror, I cannot see myself." The nurse recognizes that the patient is experiencing which of the following? 1. Displacement. 2. Dissociation. 3. Denial. 4. Depersonalization.

Question: What does this describe? Strategy: Think about each answer choice. Needed Info: Adolescence provides a time for development of a healthy self-concept and discovering one's role in life. If the road to these discoveries is blocked, the person experiences depersonalization. (1) unconscious placing of emotions onto others (boss yelling at employee, person yelling at spouse) (2) splitting off anxiety producing experiences (multiple personalities) (3) refusal to acknowledge reality (will not accept bad news). (4) CORRECT - feelings of unreality concerning self or environment

The nurse teaches the client how to increase dietary potassium. The client knows bananas are high in potassium but she does not like their taste. What foods should the nurse recommend the client include in the diet? 1. Potatoes, spinach, raisins. 2. Rhubarb, tofu, celery. 3. Carrots, broccoli, yogurt. 4. Onions, corn, oatmeal.

Question: What foods are high in potassium? Strategy: Think about each food. Needed Info: Potassium functions in water balance in cells, protein synthesis, and heart contractility; primary sources include grains, meats, vegetables, and fruits. (1) CORRECT - potassium content: potatoes 610 mg, spinach 838 mg, raisins 1,089 mg, for a total of 2,537 mg (2) potassium content: rhubarb 548 mg, tofu 9 mg, celery 114 mg, for a total of 671 mg (3) potassium content: carrots 221 mg, broccoli 254 mg, yogurt 251 mg, for a total of 726 mg (4) potassium content: onions 318 mg, corn 192 mg, oatmeal 132 mg, for a total of 642 mg.

The 72-year-old parent is brought to the clinic as a patient by the adult child who reports that the patient is not eating well but is otherwise healthy. The patient is not taking any medications. The nurse determines that which of the following meals is the BEST choice for this patient's nutritional needs? 1. Grilled cheese sandwich, cookie, tea. 2. Broiled chicken, broccoli, skim milk. 3. Raisin toast, tapioca pudding, apple juice. 4. Liver and onions, decaffeinated coffee, jell-o.

Question: What foods are in a nutritionally well balanced diet? Strategy: Think about the type of diet that is needed for an elderly person. Needed Info: Patients with anorexia (poor appetite) need nutrient-dense, high-calorie diet. Nutritionally well-balanced diet contains foods from 6 basic food groups: bread, cereal, rice, and pastas (6 - 11 servings); vegetable group (3 - 5 servings); fruit (2 - 4 servings); milk, yogurt, cheese group (2 - 3 servings); meat, poultry, fish, dry beans, eggs, nuts group (2 - 3 servings); fats, oils, and sweets (use sparingly). (1) some protein, some carbohydrate, some fluid (2) CORRECT - source of protein, vegetable, milk (3) some carbohydrate, some fruit (4) source of iron and fluids.

The nurse cares for the client experiencing an episode of acute pain. Which of the following physiologic changes does the nurse expect to see in this client during this episode? 1. Decreased blood pressure. 2. Decreased heart rate. 3. Decreased skin temperature. 4. Decreased respirations.

Question: What happens to the vital signs when a client is in pain? Strategy: Think about the cause of each vital sign change. Is it consistent with pain? Needed Info: Pain causes increased blood pressure and heart rate, which leads to increased blood flow to the brain and muscles; rapid irregular respirations lead to increased oxygen supply to brain and muscles; increased perspiration removes excessive body heat; increased pupillary diameter leads to increased eye accommodation to light. (1) blood pressure increases to enhance alertness to threats (2) heart rate increases (3) CORRECT - skin cools due to diaphoresis (4) respirations increase.

The nurse cares for the patient 1 hour after a percutaneous liver biopsy. The nurse is MOST concerned if which of the following is observed? 1. The patient frequently coughs after deep breathing. 2. The patient lies on the right side with a pillow under the costal margin. 3. The LPN/LVN obtains the blood pressure and pulse every 15 minutes. 4. The patient reports mild pain radiating to the right shoulder.

Question: What indicates a complication of a liver biopsy? Strategy: "Nurse is MOST concerned" indicates a complication. Needed Info: Sampling of tissue by needle aspiration; preparation for procedure includes administer IM vitamin K, NPO morning of exam, instruct patient to hold breath; post-procedure nursing care includes position on right side for 1 - 2 hours, maintain bed rest for 24 hours, obtain frequent vital signs to monitor for hemorrhage. (1) CORRECT - avoid coughing or straining to prevent hemorrhage (2) prevents hemorrhage or escape of bile (3) vital signs monitored every 10 - 15 minutes during first hour and every 30 minutes for next 1 - 2 hours; assess for hemorrhage (4) referred pain often happens, associated with current liver problems.

The nurse in the same-day surgery department cares for the client after a sigmoidoscopy. Which of the following symptoms, if exhibited by the client an hour after the procedure, MOST concern the nurse? 1. The client reports fullness and pressure in abdomen. 2. The client reports grogginess and thirst. 3. The client reports lightheadedness and dizziness. 4. The client reports mild pain and cramping in abdomen.

Question: What indicates a complication of a sigmoidoscopy? Strategy: Think about what causes each symptom. Needed Info: Direct visualization of the sigmoid colon, rectum, and anal canal; laxative night before exam and enema or suppository morning of procedure; NPO at midnight. Post-procedure: allow client to rest; observe for hemorrhage, perforation; encourage fluids. (1) expected after this procedure (2) expected after this procedure; has been NPO since midnight before the procedure; midazolam (Versed) is used to aid in relaxation during procedure; atropine is used during procedure to decrease peristaltic activity (3) CORRECT - could signify hypovolemic shock due to bowel perforation (4) expected after this procedure.

The nurse cares for the patient after a traditional cholecystectomy. The patient has a nasogastric tube connected to suction, an IV of D5W infusing into the right arm, and a T-tube and Penrose drain in place. The nurse is MOST concerned by which of the following findings? 1. The systolic blood pressure is 10 mm Hg lower than it was preoperatively. 2. There is 250 cc of bloody drainage from the T-tube during the first 24 hours. 3. There is 30 cc of serosanguineous drainage in the Penrose drain during the first 24 hours. 4. The patient experiences a 2 degree temperature decrease in the evening after surgery.

Question: What indicates a complication? Strategy: Think about each answer. Needed Info: T-tube ensures drainage of bile from common bile duct until edema in area decreases; protect skin around incision from bile drainage irritation; observe for jaundice. (1) decrease could be due to position change or fluids lost during surgery (2) CORRECT - would expect drainage of 400 mL/day with gradual decrease in amount; will be bloody initially and change to greenish-brown; T-tube getting dislodged is most frequent cause of ineffective drainage (3) drainage is expected, prevents accumulation of fluid in the incision (4) expected outcome of taking temperature by a different method, as patient is now a mouth-breather.

The nurse educator presents an in-service for staff on family dynamics. The nurse educator identifies which behavior as being associated with a functional family process related to communication? 1. Acknowledgment of personal needs and role responsibilities. 2. Congruence between verbal and nonverbal messages. 3. Ability to meet emotional needs of family members. 4. Appropriate responsibility for other family members' needs.

Question: What indicates a functional family process? Strategy: Think about each answer. Needed Info: Family function is how individual members relate to each other; functional communication is characterized by clear direct messages and by requesting and receiving feedback; dysfunctional communication is characterized by double-bind communication, contradictions, inconsistencies, obscure speech, and misunderstandings. (1) indicates a functional family, but this is not necessarily related to communication (2) CORRECT - indicates a functional family; double-bind communication, by contrast, is an example of conflicting messages (3) one function of a family system is assisting members to meet their physical, emotional, and safety needs; dysfunctional families are unable to meet these needs, but this is not necessarily related to communication (4) indicates a functional family, but this is not necessarily related to communication.

The nurse evaluates the progress of the patient hospitalized with depression. The nurse considers which of the following statements by the patient as an indication of improvement? 1. "I slept well last night." 2. "I can't seem to stop eating." 3. "I feel tired." 4. "I am feeling sad."

Question: What indicates an improvement in a patient who has been depressed? Strategy: Think about each answer choice. How does it relate to depression? Needed Info: S/S: unkempt appearance, lack of energy, change in sleep pattern, weight loss, decreased concentration, slowed motor activity. (1) CORRECT - depression indicated by excessive sleeping or difficulty falling asleep, staying asleep, or awakening too early (2) sign of depression (3) sign of depression (4) sign of depression.

The nurse plans care for the patient admitted reporting fever, vomiting, and diarrhea. The nurse writes the following nursing diagnosis on the patient's care plan: "Fluid volume deficit." The nurse recognizes that which of the following changes in laboratory values BEST demonstrates improvement in the patient's condition? 1. Decreased specific gravity of urine, decreased hematocrit. 2. Increased specific gravity of urine, increased hematocrit. 3. Decreased specific gravity of urine, increased hematocrit. 4. Increased specific gravity of urine, decreased hematocrit.

Question: What indicates an improvement in fluid volume deficit? Strategy: Think about physiology. Needed Info: Urine specific gravity depends on hydration; normal: 1.010 - 1.030; will increase if patient is dehydrated. Hematocrit measures % volume of RBCs in whole blood; normal: men 42 - 50%, women 40 - 48%; increases in severe dehydration (volume). (1) CORRECT - specific gravity and hematocrit increase with dehydration (2) ongoing fluid volume deficit (3) does not best indicate improvement; specific gravity is decreased but hematocrit still increased (4) does not best indicate improvement; hematocrit is decreased but specific gravity still increased.

The nurse cares for the woman at 37 weeks gestation. The nurse is MOST concerned by which finding? 1. The patient reports right quadrant pain. 2. The patient's BP is 150/95. 3. The patient has 1+ proteinuria. 4. The patient has 3+ pitting edema of the ankles.

Question: What indicates impending eclampsia? Strategy: "MOST concerned" indicates a complication. Question unstated. Read answer choices for clues. Needed Info: Eclampsia is seizures in a pregnant woman not related to a pre-existing brain condition. Providers intervene before that advanced stage, when symptoms of impending eclampsia occur. Preeclampsia causes hypertension, proteinuria, and edema; symptoms of severe preeclampsia include BP of 150 - 160/100 - 110 mm Hg; 4+ proteinuria; headache, epigastric pain; treatment for severe preeclampsia includes bed rest, vital signs and fetal heart tones, monitor I + O, seizure precautions, and administer magnesium sulfate. (1) CORRECT - indicates impaired liver function, sign of impending eclampsia (2) elevated BP less than 160/110 considered moderate preeclampsia (3) indicates mild preeclampsia (4) dependent edema, indicative of mild preeclampsia.

The patient is admitted to the hospital for a myelogram using a water-soluble dye. What information is MOST important for the nurse to obtain about the patient's medication history? 1. Is the patient currently taking any antihypertensives? 2. Is the patient currently taking any nonsteroidal anti-inflammatory medications? 3. Is the patient currently taking any antibiotics? 4. Is the patient currently taking any antidepressants or antipsychotics?

Question: What information do you need from the patient's medication history? Strategy: Think about each medication given. How does it relate to a myelogram? Needed Info: Meds that lower the seizure threshold such as phenothiazines (chlorpromazine), MAO inhibitors (isocarboxazid, phenelzine), tricyclic antidepressants (imipramine, amitriptyline), CNS stimulants, psychoactive drugs (methylphenidate) should be held for 48 hours before and 24 hours after test. The reason to stop such meds is that their presence could increase the risk of seizures. (1) will not affect test (2) will not affect test (3) will not affect test. (4) CORRECT - need to hold any meds that lower seizure threshold

The 80-year-old patient is admitted to the hospital with a diagnosis of carcinoma of the colon. A hemicolectomy is scheduled. On admission, the patient appears disheveled and is restless and confused. It is MOST important that the nurse obtain the answer to which of the following questions? 1. Which prescription and/or over-the-counter medications is the patient taking? 2. What is the medical history of the patient's family? 3. What was the patient's previous occupation? 4. Has the patient smoked cigarettes in the past?

Question: What information is MOST important to get about the patient since he is confused? What is a frequent cause of confusion in the elderly? Strategy: Picture the patient as described. Needed Info: Polypharmacy, the taking of multiple medications, is common in elderly adults, accounting for the dispensing of one-third of prescribed medications. Decreases in kidney and liver functioning can allow a buildup of toxic chemicals. (1) CORRECT - confusion can be caused by drug toxicity (2) not most important (3) not most important (4) not most important.

The nurse cares for the patient during the acute phase of a cerebrovascular accident (stroke). The nurse gives the HIGHEST priority to which of the following? 1. Maintaining musculoskeletal function. 2. Maintaining nutritional status. 3. Maintaining respiratory function. 4. Maintaining skin integrity.

Question: What is MOST important for a patient right after a CVA? Strategy: Remember your ABCs: airway, breathing, circulation. Needed Info: Manifestations of stroke vary with the involved cerebral vessel and the area of the brain affected; women more likely to report nontraditional manifestations; manifestations always sudden in onset, focal, and usually one-sided. (1) ROM, positioning (2) soft diet due to possible dysphagia, or tube feedings (3) CORRECT - ABCs first (4) bed rest 48 - 72 hours during acute phase.

The patient is admitted to the hospital with a ruptured ectopic pregnancy. A laparotomy is scheduled. The nurse recognizes it is MOST important to include which goal on the patient's preoperative care plan? 1. Fluid replacement. 2. Pain relief. 3. Emotional support. 4. Respiratory therapy.

Question: What is MOST important for a patient with a ruptured ectopic pregnancy? Strategy: Remember Maslow's hierarchy of needs. Meet physical needs before addressing psychosocial needs. Needed Info: Dehydration can become a serious danger in a preoperative patient, especially if bleeding occurs. Fluid replacement prevents that complication. Pain relief can be well handled in the postoperative period. Emotional support, while important, is a lower priority than ensuring that the patient's life is not in danger. (1) CORRECT - physical need; IV fluids, blood transfusion (2) not most important (3) not most important (4) not a problem.

The nurse prioritizes the needs of the patient who has been raped. Which of the following is MOST important initially? 1. Emotional needs. 2. Physical needs. 3. Hygiene needs. 4. Legal responsibilities.

Question: What is MOST important for a person who has been raped? Strategy: Establish priorities. Remember Maslow's hierarchy of needs. Meet the physical needs before you address safety, love, belonging, esteem, and self-actualization needs. Needed Info: Major effort in working with the person raped concentrates on first collecting physical evidence, such as proof of injuries and possible specimens for DNA testing. Emotionally charged crime. (1) not most important initially (2) CORRECT - Maslow's hierarchy: physical is first (3) not most important initially (4) not most important initially.

Arterial blood gases (ABGs) are ordered for the patient following a myocardial infarction. After obtaining the ABGs, which of the following measures is MOST important for the nurse to implement? 1. Obtain ice for the specimen. 2. Apply a sterile dressing to the site. 3. Apply direct pressure to the site. 4. Observe the site for hematoma formation.

Question: What is MOST important for you to do after obtaining an ABG? Strategy: Picture yourself doing the procedure. Set priorities. Care for the patient first and the equipment or samples second. Needed Info: ABGs: measurement of partial pressure of oxygen, carbon dioxide, and pH of blood; assessment of acid-base status of body; use a heparinized syringe; needle inserted 45 - 60 degrees to skin surface and advanced into radial artery; apply pressure after needle removed; put specimen on ice. (1) should be done, but not most important; care for the patient first (2) bandage applied (3) CORRECT - prevents bleeding, hematoma; maintain for at least 5 min, 15 min if on anticoagulant (4) not highest priority; check for discoloration, numbness, tingling, temperature.

The nurse cares for the client after a ureterolithotomy. The nurse notes that the client has a left ureteral catheter in place. The nurse includes which of the following in the client's plan of care? 1. Clamp the catheter for short periods of time. 2. Irrigate the catheter every 2 hours. 3. Gently advance the catheter if no drainage is observed. 4. Instruct the client that urine from the catheter should be clear.

Question: What is a correct action for a client with a ureteral catheter? Strategy: Determine the outcome of each answer. Needed Info: Ureterolithotomy is surgical removal of calculus from the ureter; do not irrigate ureteral catheter, check incisional drain, check surgical dressing; encourage oral fluids. (1) due to small size of catheter, do not clamp (2) do not irrigate, since that would change the direction of the fluid flow; measure intake and output (3) do not advance catheter, may cause trauma (4) CORRECT - immediately after surgery, a small amount of blood-tinged urine is normal but then it becomes clear; increase fluid to promote flow of urine

The nurse reviews patient assignments made by the student nurse. The nurse determines that assignments are appropriate if a nursing assistant is assigned to which patient? 1. The patient scheduled for an MRI. 2. The unconscious patient who requires mouth care. 3. The patient admitted for uncontrolled seizures. 4. The patient with diabetes who requires foot care.

Question: What is a correct patient assignment for a nursing assistant? Strategy: Think about each answer. Needed Info: Delegate standard, unchanging procedures to a nursing assistant. (1) requires assessment and teaching (2) CORRECT - mouth care for an unconscious patient can be delegated to the nurse assistant; the nurse must first assess for a gag reflex (3) nurse must care for patient; assess for patent airway, adequate respirations, and circulatory status (4) nursing assistant can perform foot care for a nondiabetic patient.

The nurse performs teaching for the client being discharged with a new ileostomy. The nurse includes which of the following statements in the discharge teaching? 1. "Change the appliance every day." 2. "The ileostomy does not require irrigation." 3. "Decrease your fluid intake." 4. "Apply cream around the stoma."

Question: What is a correct statement about caring for an ileostomy? Strategy: Determine the outcome of each answer. Needed Info: Ileostomy is an opening into the ileum from the abdominal wall for evacuation of feces; drainage bag with pectin-based skin barrier must be worn at all times. (1) appliance should only be changed if there is a leak (2) CORRECT - no need to irrigate; stool remains loose and cannot be controlled with irrigation (3) maintain high fluid intake due to loss of fluids through the ileostomy (4) prevents the appliance from adhering to skin; increases the incidence of leaking.

The nurse has completed discharge instructions for the primigravida at 29 weeks gestation who is hospitalized for treatment of deep vein thrombosis (DVT). Which of the following statements, if made by the patient to the nurse, indicates that teaching has been successful? 1. "I will give myself heparin every day." 2. "I should check my leg once a week." 3. "I will massage my leg nightly." 4. "I can take Pepto-Bismol for diarrhea."

Question: What is a correct statement about deep vein thrombosis and pregnancy? Strategy: "Teaching has been successful" indicates a correct response. Needed Info: Pregnancy, immobility, obesity, and surgery are risk factors for deep vein thrombosis. Heparin is anticoagulant that blocks conversion of prothrombin to thrombin; side effects include hematuria and bleeding gums; monitor partial thromboplastin time (PTT). (1) CORRECT - heparin does not cross the placenta; considered safe during pregnancy (2) not an aspect of self-care (3) contraindicated with DVT; clot can be released (4) bismuth subsalicylate (Pepto-Bismol) increases the anticoagulant effectof heparin;, do not use together.

The patient is admitted to the unit for treatment of acute glomerulonephritis. The nurse teaches the patient about the disease and the treatment required. The nurse determines that teaching is successful if the patient makes which of the following statements? 1. "Who would have thought that a sore throat two weeks ago would cause this!" 2. "I am in the hospital because my grandmother receives dialysis three times per week." 3. "I'm glad that I don't have to restrict my activities." 4. "My roommate is going to bring me a double cheeseburger with bacon."

Question: What is a correct statement about glomerulonephritis? Strategy: Think about what the words mean. Needed Info: Acute glomerulonephritis is usually caused by beta hemolytic streptococcal infection; symptoms include fever, chills, hematuria, weakness, pallor, weight gain, lung rales, and fluid overload. (1) CORRECT - inflammatory reaction of the kidney to infection that occurs on skin or in the throat (2) no relationship; damage to glomerulus caused by an immunological reaction (3) bed rest ordered during acute phase to guard against hematuria and proteinuria; promotes diuresis (4) usually anorexic; protein and sodium limited until BUN, creatinine and BP normal.

The nurse performs discharge teaching for the client receiving trifluoperazine. The nurse determines that teaching is successful if the client verbalizes which of the following? 1. "I should take two pills at night if I have difficulty sleeping." 2. "I cannot breastfeed my baby while I am taking this medication." 3. "I may experience frequent tearing in my eyes." 4. "I will have to increase my calorie intake daily."

Question: What is a correct statement about trifluoperazine? Strategy: Think about each statement. Needed Info: Trifluoperazine (Stelazine) is an antipsychotic phenothiazine; side effects include pseudoparkinsonism, dystonia, akathisia, tardive dyskinesia; instruct client to avoid alcohol, report urine retention or constipation, use sunblock, and chew sugarless gum or suck on hard candy to relieve dry mouth. (1) cannot increase or decrease dosage without physician approval (2) CORRECT - breastfeeding is contraindicated because trifluoperazine is excreted in breast milk (3) side effect is dry eyes--use artificial tears (4) may have weight gain; should decrease calorie intake and exercise frequently.

The nurse teaches the patient who is lactose intolerant about alternative ways to obtain adequate amounts of calcium in the diet. Which of the following items, if selected by the patient, indicates understanding of which foods to eat? 1. Eggs and green leafy vegetables. 2. Instant breakfast mixes and skim milk. 3. Cottage cheese and yogurt. 4. Custard and mashed potatoes.

Question: What is a good source of calcium for a lactose intolerant patient? Strategy: Eliminate foods that contain milk. Needed Info: Lactose intolerance: inability of intestine to absorb milk due to deficiency in enzyme that breaks down milk sugar (lactase). Calcium: milk, seafood, orange juice, cereals, and dark green leafy vegetables such as spinach, kale, and mustard greens. Yogurt and hard cheeses may be tolerated because of how they are processed. (1) CORRECT - good sources of Ca (2) contain lactose (3) contain lactose; yogurt may be tolerated; cottage cheese is a soft cheese and would not be tolerated ( 4) made with milk, contains lactose.

The nurse counsels the client in the outpatient psychiatric clinic for treatment of a fear of water. The nurse recognizes that a phobia can BEST be described as which of the following? 1. A form of sublimation that is adaptive to the client. 2. A persistent fear that is excessive and unrealistic. 3. A persistent uncontrolled thought precipitated by anxiety. 4. A manipulative behavior used to achieve secondary gain.

Question: What is a phobia? Strategy: Think about each answer. Needed Info: Client feels apprehension, anxiety, helplessness when confronted with the feared object; nursing considerations include: avoid confrontation and humiliation, do not focus on trying to stop the client from being afraid, use systematic desensitization, try relaxation techniques. (1) sublimation is a defense mechanism; diversion of unacceptable, instinctual drives into socially acceptable outlets (2) CORRECT - phobia is a lasting and unreasonable fear caused by a specific object or situation that poses little or no danger (3) this is description of an obsession (4) this is when one person attempts to influence another person in order to meet one's own needs or desires.

The nurse conducts the admission interview of the client scheduled for surgical repair of an inguinal hernia. Which of the following client statements MOST concerns the nurse? 1. "I am allergic to bananas." 2. "I am allergic to shellfish." 3. "I am allergic to peanuts." 4. "I am allergic to milk."

Question: What is a potential complication to the surgery? Strategy: "MOST concerned" indicates something is wrong. Needed Info: Indication of latex allergy includes urticaria, rash, wheezing, rhinitis, conjunctivitis, bronchospasm, and anaphylactic shock; instruct client to avoid latex products. (1) CORRECT - latex products are used extensively during surgery; certain food allergies may indicate an allergy to latex; foods include apricots, cherries, grapes, kiwis, passion fruit, mangoes, bananas, avocados, chestnuts, tomatoes, and peaches (2) a concern if client going to be given a dye (3) can cause anaphylactic reaction; latex allergy is concern prior to surgery (4) can cause diarrhea; latex allergy is concern prior to surgery.

The nurse on the surgical unit is assigned to care for two patients in traction. The nurse recognizes that when caring for the patient in traction it is MOST important to take which of the following actions? 1. Allow the weight to hang freely at all times. 2. Encourage the patient to limit body movements. 3. Immediately remove the weights if the patient complains of discomfort. 4. Give pain medication regularly.

Question: What is a priority when caring for a client in traction? Strategy: Determine the outcome of each answer. Needed Info: Traction reduces fractures, alleviates pain and muscle spasms, prevents or correct deformities, and promotes healing; maintain straight alignment of ropes and pulleys, ensure that weights hang freely, frequently inspect skin for areas of breakdown, maintain position for countertraction. (1) CORRECT - necessary for proper pull of traction (2) twisting and turning may be prohibited, but movement of unaffected extremities is encouraged (3) weights should not be removed, changes pull of traction (4) pain should be investigated rather than routinely treated; pain may indicate circulatory impairment.

The nurse cares for clients being treated for narcotic abuse. The nurse recognizes that which of the following data obtained during a client history presents the HIGHEST risk for the client developing a disease process? 1. The use of multiple drugs. 2. Intravenous administration of narcotics. 3. Unsuccessful efforts to decrease drug use. 4. Legal difficulties encountered as a result of drug use.

Question: What is a risk factor for IV drug users? Strategy: Think about each answer. Needed Info: Symptoms of narcotic abuse include marked respiratory depression, hyperpyrexia, seizures, ventricular dysrhythmias, pinpoint pupils, stupor leading to coma. (1) associated with some increased risk of complications, but not as severe as IV use (2) CORRECT - IV drug use is associated with increased risk of developing HIV, septicemia, hepatitis, and respiratory failure (3) not directly linked to the development of disease processes (4) not linked to disease processes.

The nurse does discharge teaching with the parent of the child diagnosed with epilepsy. The child is going to be discharged on phenytoin. Which of the following statements, if made by the parent, indicates understanding of the potential side effects of phenytoin? 1. "My child's teeth may become discolored." 2. "My child may develop strange food cravings." 3. "My child may be more sensitive to x-rays." 4. "My child's urine may turn pink, red, or brown."

Question: What is a side effect of phenytoin? Strategy: Eliminate what you know to be wrong. Needed Info: 1) Epilepsy: uncontrolled abnormal discharge of electrical activity in the brain. 2) Sequence of seizures: prodromal stage: vague change in emotions, aura: brief sensation, epileptic cry, convulsion, postictal: change in consciousness. 3) Phenytoin (Dilantin): anticonvulsant and antiarrhythmic. Side effects: thrombocytopenia, leukopenia, ventricular fibrillation, nystagmus, diplopia, gingival hyperplasia, toxic hepatitis. (1) inaccurate (2) inaccurate (3) inaccurate, is more sensitive to ultraviolet rayS (4) CORRECT: harmless side effect

The nurse cares for the client after a below-the-knee amputation. The nurse assesses for infection and is MOST concerned if which of the following is observed? 1. The client appears restless. 2. The client reports a throbbing headache. 3. The client reports persistent pain at the operative site. 4. The skin feels cool proximal to the operative site.

Question: What is a symptom of infection at the operative site? Strategy: Determine what causes each symptom. Does it indicate infection? Needed Info: Major complications after amputation are hemorrhage, infection, and skin breakdown; monitor for bleeding; keep tourniquet placed in site at bedside; skin hygiene important to prevent skin breakdown; wash and dry residual limb twice per day. (1) restlessness indicates hypoxia (2) does not indicate infection (3) CORRECT - infection is frequent complication of amputation; assess for change in color, odor, and consistency of drainage, increased pain, elevated temperature; contact physician immediately (4) does not indicate infection.

The nurse cares for clients in the psychiatric unit. When administering antipsychotic medication, the nurse observes for tardive dyskinesia. The nurse recognizes that which of the following is characteristic of tardive dyskinesia? 1. Masklike face and shuffling gait. 2. Involuntary grimacing and protrusion of the tongue. 3. Motor restlessness and pacing. 4. Severe muscle contractions of the face.

Question: What is a symptom of tardive dyskinesia? Strategy: Think about each answer. Needed Info: Antipsychotics are major tranquilizers used to treat psychotic symptoms; examples are chlorpromazine (Thorazine), thioridazine (Mellaril), fluphenazine (Prolixin), haloperidol (Haldol), clozapine (Clozaril), and risperidone (Risperdal). (1) side effect that describes parkinsonism; notify physician; administer trihexyphenidyl (Artane) or benztropine (Cogentin) as prescribed (2) CORRECT - describes tardive dyskinesia; important to prevent this side effect by maintaining client on lowest possible dose of medication (3) describes akathisia; notify physician; requires reduction of the dose of antipsychotic or may receive benztropine or trihexyphenidyl (4) describes dystonic reaction; can compromise airway; administer diphenhydramine (Benadryl) or benztropine; provide quiet, nonstimulating environment; reassure client that symptoms will resolve.

The patient arrives at the health clinic with reports of dark urine, fever, and flank pain. The initial nursing assessment of the patient reveals which of the following EARLY symptoms of glomerulonephritis? 1. Polyuria. 2. Oliguria. 3. Polydipsia. 4. Enuresis.

Question: What is an EARLY symptom of glomerulonephritis? Strategy: Be careful! The question asks about early symptoms. Needed Info: Acute glomerulonephritis: group of kidney diseases resulting in inflammatory changes from immunological responses. S/S: edema, abdominal pain, hypertension, fever. Nursing responsibilities: restrict sodium and water, daily weight, I + O, bed rest, high-calorie, low-protein diet. (1) excessive output seen with diabetes mellitus, acute renal failure; normal output 1,200 - 1,500 mL/day (2) CORRECT - reduced urinary output (100 - 400/day); also hematuria (blood in urine), proteinuria (protein in urine) (3) excessive thirst seen with diabetes mellitus due to osmotic diuresis; normal intake 1,500 - 2,000 mL/day (4) bedwetting after age 5; not seen with glomerulonephritis.

The nurse assesses the 8 lb 4 oz newborn infant. Which of the following observations, if made by the nurse, requires an intervention? 1. The infant's respirations are 36, shallow and irregular in rate, rhythm, and depth. 2. Rapid pulsations are visible in the fifth intercostal space, left midclavicular line. 3. The infant's axillary temperature is 96.2°F (35.6°C). 4. There is asynchronous spontaneous movement of the infant's extremities.

Question: What is an abnormal finding for a newborn? Strategy: Think about cause of each answer. "Requires an intervention" indicates a complication. Needed Info: Important to assist newborn with heat regulation: wrap newborn to protect from cold, dry infant after birth, place fabric insulated cap on head. Cold stress: infant unable to increase activity and lacks a shivering response to cold; causes metabolic acidosis, hypoxia, and hypoglycemia. (1) normal due to immaturity of respiratory system (2) normal, site of apical pulse, normal rate 120 - 140 bpm (3) CORRECT - subnormal indicates prematurity, infection, low environment temperature, inadequate clothing, dehydration (4) normal, legs move in bicycle fashion, should have equal extension of all extremities.

The patient receives morphine sulfate postoperatively for complaints of pain. Since the patient is receiving morphine, which of the following medications is MOST important for the nurse to have available? 1. Naloxone. 2. Disulfiram. 3. Methadone. 4. Epinephrine.

Question: What is an antidote to morphine? Strategy: Think about each answer choice and how it relates to morphine. Needed Info: Remember that an overdose of morphine causes respiratory depression; need availability of the antidote in case of overdose. (1) CORRECT - narcotic antagonist; rapid onset, duration 3 - 5 hours; reverses respiratory depression (2) treatment for alcohol abuse; reacts with alcohol to cause flushing, HA, vomiting, palpitations, hypotension (3) treatment for drug abuse; narcotic analgesic used to detoxify addicts (4) nervous system stimulant; used in asthmatic attacks, anaphylactic reactions, cardiac arrest.

The nurse cares for the depressed client who frequently verbalizes a negative self-image. The nurse recognizes that which of the following nursing interventions is MOST appropriate for this patient? 1. Help the client identify areas of weakness. 2. Help the client identify unrealistic expectations. 3. Ask the client to identify goals for the next 2 years. 4. Tell the client to stop having negative thoughts.

Question: What is an appropriate nursing action for a client who is depressed? Strategy: Determine the outcome of each answer. Needed Info: Depression may be a response to a real or imagined loss; it may result from anger and aggression toward self that results from feeling of guilt about negative or ambivalent feelings; nursing considerations include being alert for signs of self-destructive behavior, promote eating and rest, support self-esteem. (1) action too negative; reinforces what client already feels (2) CORRECT - unrealistic expectations that client fails to meet reinforces feelings of low self-esteem; the first step is to help client identify what is unreasonable (3) time frame is too long (4) not most appropriate; altering negative thoughts can be helpful in improving self-esteem, but it is not simply a matter of telling a client to stop them; that has a punitive tone and is not realistic.

The nurse observes the student nurse caring for a patient with a tracheostomy tube. The nurse intervenes if which of the following is observed? 1. The student nurse uses clean gloves to remove the tracheostomy dressing. 2. The student nurse cleans the inner cannula by soaking it in hydrogen peroxide. 3. The student nurse removes the soiled trach ties and then reattaches clean ties. 4. The student nurse replaces the dressing with a folded gauze 4 x 4.

Question: What is an incorrect action when caring for a patient with a tracheostomy tube? Strategy: "Need for an intervention" indicates an incorrect action. Needed Info: Perform tracheostomy care every 8 hours and as needed; hyperoxygenate patient prior to suctioning trach tube; sterile procedure. (1) appropriate procedure; dispose of soiled dressing and gloves appropriately (2) appropriate procedure; clean with small brush or pipe cleaner; rinse well in normal saline (3) CORRECT - apply new ties prior to removing old ties to prevent dislodgment of the trach (4) use folded 4 x 4 or commercially prepared 4 x 4; in order to prevent aspiration, do not cut dressing.

The home care nurse assesses the client diagnosed with gout. The nurse is MOST concerned if the client makes which of the following statements? 1. "I drink at least 2 quarts of liquid each day." 2. "I am losing 3 pounds per week." 3. "I limit my protein intake to 3 to 4 ounces per meal." 4. "I have quit drinking beer."

Question: What is an incorrect statement about gout? Strategy: "MOST concerned" indicates an incorrect statement. Needed Info: Gout is overproduction or underexcretion of uric acid; causes joint pain, swelling, limitation of movement, nodules over bony prominences; treatment includes colchicine, analgesics, and anti-inflammatory drugs. (1) increase fluid intake to 2 - 3 quarts per day to promote uric acid excretion and dilute the urine (2) CORRECT - excessive weight loss may precipitate an attack of gout; if overweight, lose 0.5 - 1 pounds per week (3) limit foods high in purines--anchovies, dried peas and beans, and organ meats; cheese, eggs, milk, and vegetables are lower in purines (4) excessive alcohol reduces uric acid excretion.

The nurse instructs the client receiving levothyroxine 100 mcg daily. The nurse identifies that further teaching is necessary if the client states which of the following? 1. "If I have chest pain, I will call my doctor." 2. "If my hands shake, I will call my doctor." 3. "I will take my medication before I go to sleep." 4. "I will inform my other health care providers about this medication."

Question: What is an incorrect statement about levothyroxine? Strategy: Think about the action of the medication. Needed Info: Levothyroxine (Synthroid) increases metabolic rate of body and is used as a thyroid replacement; side effects include nervousness, tremors, insomnia, tachycardia, palpitations, dysrhythmias, and angina; instruct client to report chest pain, palpitations, sweating, nervousness, or shortness of breath. (1) appropriate action: indicates a potentially serious side effect; instruct client to report unusual cardiovascular symptoms (2) appropriate action: may indicate overdose (3) CORRECT - take thyroid replacement at the same time each day; morning is preferred to prevent insomnia (4) appropriate action.

The community health nurse conducts a prevention program at the high school and discusses high-risk groups for suicide. The nurse recognizes that further teaching is necessary if a student states which of the following? 1. "Adolescents are at high risk." 2. "Depressed people are at high risk." 3. "History of previous suicide attempts put people at high risk." 4. "Those who are grieving in response to a loss for 9 months are at high risk."

Question: What is an incorrect statement about suicide? Strategy: Think about each answer. Needed Info: Be alert for signs of self-destructive behavior; behavioral clues of impending suicide include any sudden change in behavior; client becomes energetic after period of severe depression, finalizes business or personal affairs, withdraws from social activities and plans, presence of weapon, razors, or pills, has a death plan. (1) suicide is the third leading cause of death among people 15 to 24 years old (2) symptoms of depression include social withdrawal, feelings of hopelessness, irritability, and difficulty sleeping (3) previous suicide attempts and easy access to lethal methods are risk factors; of those who commit suicide, 80% made previous attempts. (4) CORRECT - grieving is a normal human response that occurs in response to a loss and the entire process may take more than 1 year.

The nurse counsels the young parent about how the parent will know when the child is ready for toilet training. The nurse recognizes that further teaching is necessary if the parent makes which of the following statements? 1. "I can consider toilet training when my child's diaper is dry after naps." 2. "I can begin toilet training when my child begins to walk." 3. "My child must be able to sit for five to ten minutes before I can start toilet training." 4. "It is important that I have the time to spend in toilet training."

Question: What is an incorrect statement about toilet training? Strategy: "Further teaching is necessary" indicates an incorrect statement. Needed Info: Child must be able to control anal and urethral sphincters, recognize the urge to void and defecate, and be able to communicate the need to the parents; readiness occurs around 18 - 24 months; practice sessions should be limited to 5 - 10 minutes, and a parent should stay with the child. (1) staying dry for 2 hours or waking up dry after a nap indicates toilet training readiness (2) CORRECT - voluntary control of anal and urethral sphincters is required; usually occurs at 18 - 24 months; toddlers usually walk at 12 - 13 months (3) must be able to sit without getting up or fussing (4) parent must also recognize child's readiness; if major changes occurring in the family (divorce, moving, vacation) do not begin toilet training.

The nurse assesses the child diagnosed with attention deficit hyperactivity disorder (ADHD). The nurse expects to observe which of the following? 1. Feeding difficulties. 2. Head banging. 3. Easy distractibility. 4. Rigid arms and legs.

Question: What is an indication of ADHD? Strategy: Think about each answer. Needed Info: Characteristics of ADHD include distractibility, immaturity relative to chronologic age, impulsivity, and learning disabilities; nursing considerations include reducing frustration, providing safety and security, and administering medication. (1) exhibited by children diagnosed with failure to thrive (2) observed in children diagnosed with pervasive developmental disorders and mental retardation (3) CORRECT - major clinical manifestation is distractibility; child has difficulty attending to unwanted tasks such as chores or homework (4) side effect of antipsychotic medication.

The nurse manager observes the staff nurse assist the physician with a lumbar puncture. The manager determines that the care is appropriate if the staff nurse does which of the following? 1. The staff nurse instructs the patient to hyperventilate. 2. The staff nurse instructs the patient to maintain a full bladder. 3. The staff nurse explains to the patient that the procedure is always painless. 4. The staff nurse assists the patient into a fetal position.

Question: What is appropriate position for lumbar puncture? Strategy: Determine outcome of each answer. Needed Info: Lumbar puncture is the insertion of needle into subarachnoid space to obtain specimen, relieve pressure, inject dye or medication. Preparation for procedure: explain procedure, confirm that consent has been signed, position in lateral recumbent fetal position. Post-test nursing care: position flat for 4 - 12 hours; encourage PO fluids to 3,000 mL, neurological assessment every 15 - 30 min until stable. Oral analgesics for headache. (1) patient should breathe normally; hyperventilation may lower an elevated pressure (2) patient should empty bladder for comfort (3) patient will feel a needle prick or may feel pain in the leg. (4) CORRECT - the patient uses own arms to hold knees in place, head bent forward; increases space between vertebrae

The nurse cares for clients in the outpatient clinic. The client diagnosed with glaucoma experiences severe restrictions of peripheral vision and asks the nurse if the vision will improve. Which of the following statements by the nurse is BEST? 1. "If you continue to take your medication, the pressure in your eyes will decrease. Your vision will improve." 2. "The physician will perform surgery to remove the lens in your eyes. This will increase your vision." 3. "The current damage to your vision is permanent. Continued use of the eye drops will prevent further damage." 4. "After the eye pressure is stabilized, the physician will reevaluate your vision. Your vision can be corrected with glasses."

Question: What is correct information about glaucoma? Strategy: Think about each answer. Needed Info: Glaucoma is abnormal increase in intraocular pressure leading to visual disability and blindness; signs and symptoms include cloudy, blurry vision, or loss of vision; artificial lights appear to have rainbows or halos around them; decreased peripheral vision; pain, headache, nausea, and vomiting; treatment is miotics. (1) vision will not improve (2) lens is removed during cataract surgery (3) CORRECT - true statement about glaucoma (4) damage is permanent.

The nurse cares for the client who was admitted to the cardiac unit reporting retrosternal chest pain and severe anxiety. The client was diagnosed with arteriosclerotic heart disease (ASHD) and angina. The client is ready for discharge and nitroglycerin is prescribed. Which of the following statements does the nurse include in the discharge teaching? 1. "Store the nitroglycerin tablets in a special clear plastic pillbox with a bright lid." 2. "Take a nitroglycerin tablet before engaging in any activity that may produce chest pain." 3. "Swallow the nitroglycerin tablets, but do not take with water or any other liquids." 4. "You will not have to renew your prescription for nitroglycerin for the next 12 months."

Question: What is correct information about nitroglycerin? Strategy: Determine the outcome of each answer. Needed Info: Nitroglycerin is an antianginal that relaxes vascular smooth muscle; side effects include flushing, hypotension, headache, tachycardia, dizziness, and blurred vision. (1) nitroglycerin is very unstable and should be kept in a dark glass bottle which is securely capped; do not store in metal or plastic (2) CORRECT - prophylactic dose of nitroglycerin increases client's tolerance for stress and exercise (3) nitroglycerin is not swallowed; place under tongue, wet tablet with saliva; do not swallow saliva until tablet dissolves (4) nitroglycerin is unstable, volatile, and inactivated by moisture, air, light, heat, and time; renew supply every 3 months.

The nurse cares for the client who will be taking phenelzine sulfate following discharge. Which of the following is important information for the nurse to include in the teaching plan regarding this medication? 1. The client will see the effects of the medication immediately. 2. The client does not need to use sunblock during outside activities. 3. Drinking coffee or carbonated beverages will decrease the effectiveness of the medication. 4. Combining the medication with certain foods significantly increases blood pressure.

Question: What is correct information about phenelzine? Strategy: Think about the action of the drug. Needed Info: phenelzine sulfate (Nardil) is an MAO inhibitor; interacts with foods containing tyramine or drugs containing sympathomimetic substances to cause a hypertensive crisis. (1) takes 3 - 4 weeks for drug to begin working (2) sunblock is required (3) may precipitate hypertensive crisis (4) CORRECT - instruct client to avoid pickled herring, liver, dry sausage, sauerkraut, aged cheese, yogurt, yeast, and meat extracts, and other pickled, fermented, or smoked foods to prevent a hypertensive crisis

The nurse cares for the client in the outpatient clinic who has received a prescription for verapamil 80 mg TID. The nurse includes which of the following instructions when teaching the client about this medication? 1. "Drink lots of fluids during the day to prevent liver and kidney damage." 2. "When you awaken in the morning, sit on the side of the bed for a few minutes before standing." 3. "If you are feeling stress and develop symptoms, take an extra dose of verapamil." 4. "Take the medication before meals on an empty stomach."

Question: What is correct information about verapamil? Strategy: Determine the outcome of each answer. Needed Info: Verapamil (Calan): calcium-channel blocker; side effects include transient hypotension, dizziness, headache, constipation, elevated liver enzymes; instruct client to take medication with food, monitor vital signs, and instruct not to chew or divide sustained-release medication. (1) encourage client to increase fluid and fiber intake to counteract constipation side effect (2) CORRECT - medication causes transient hypotension; monitor blood pressure when first taking the medication and when dosage is adjusted (3) take medication as prescribed; given as an antihypertensive and antianginal (4) take with food; increase intake of fiber and fluids.

The nurse plans postoperative care for the patient scheduled for a stapedectomy. When the patient is returned to the room after surgery, the nurse expects to observe which of the following? 1. Patient's hearing is completely restored. 2. Patient is still drowsy from the general anesthesia. 3. Patient experiences vertigo, nausea, and vomiting. 4. Patient has drains in both ears.

Question: What is expected after a stapedectomy? Strategy: Think about each answer. Needed Info: Excision of stapes with or without prosthesis to correct hearing loss; during first 24 hours post-op position patient flat in bed with minimal head movement; instruct patient to not blow nose or sneeze; assess for facial nerve damage or muscle weakness or changes in taste. (1) hearing initially worse; improves after 6 weeks (2) local anesthetic used (3) CORRECT - close to inner ear; meclizine (Antivert) (anti-vertigo) and prochlorperazine (Compazine) (antiemetic) used; assist with ambulation, side rails up, change positions slowly (4) drains not used; gel foam (absorbent sponge) packing used to decrease bleeding

The patient with a history of heart failure (HF) is admitted to the hospital with flulike symptoms. When taking the history, the nurse learns that the patient has been taking digoxin 0.125 mg PO daily for 3 years. Last month the physician changed the prescription for digoxin to 0.25 mg PO daily and ordered furosemide 40 mg daily. The nurse expects the physician to order which of the following laboratory tests? 1. Serum electrolytes and digoxin level. 2. White blood cell count and hemoglobin and hematocrit. 3. Cardiac enzymes and an arterial blood gas. 4. Blood cultures and urinalysis.

Question: What is going on with the patient, and which tests will help identify the problem? Strategy: Think about what each test measures. Needed Info: Digoxin (Lanoxin): cardiac glycoside works by strengthening myocardial contraction and slowing conduction through the AV node. Furosemide (Lasix): acts at loop of Henle to inhibit reabsorption of sodium, chloride; side effects: agranulocytosis (decreased WBC), hypokalemia. Heart failure (HF): failure of heart to adequately pump blood. S/S: dyspnea; weight gain, edema, crackles. Treatment: cardiac glycosides (digoxin), diuretics, restricted sodium diet. Nursing responsibilities: promote rest, give oxygen, teach about meds. (1) CORRECT - check potassium; hypokalemia may precipitate dig toxicity: N + V, bradycardia, AV block, visual disturbances, PVCs (2) WBC indicates infection, inflammation; hemoglobin and hematocrit measure functioning of red blood cells, low values indicate anemia (3) CPK + LDH isoenzymes indicate cardiac damage; ABG indicates acid/base balance (4) blood cultures indicate infection; UA indicates urinary problems.

The nurse cares for the recently retired salesman who is brought to the psychiatric hospital by his spouse. The spouse states that since retirement, the patient has been listless and roams around the house complaining of nothing to do. The patient states, "Without a job I have no purpose in life." The spouse adds that the patient recently lost 10 pounds and sleeps for only 2 to 3 hours each night. In order to prioritize the patient's nursing care, the nurse assesses which of the following areas FIRST? 1. Suicidal ideation. 2. Level of insight into his problem. 3. Nutritional deficiencies. 4. Motivation to solve own personal problems.

Question: What is most important for a patient with depression? Strategy: Set priorities according to Maslow's hierarchy of needs. Needed Info: Symptoms of depression: regressive behavior, obsessive thoughts, unkempt appearance, insomnia, withdrawn behavior. Nursing responsibilities: check for possible suicide, report behavioral changes, meet physical needs, structure simple routines, use touch judiciously, encourage expression of feelings. Treatment: antidepressants; group, individual, and family therapy. (1) CORRECT - safety needs highest (2) safety needs higher priority (3) safety needs more important (4) later issue.

The nurse teaches the group of parents of toddlers how to prevent accidental poisoning. Which of the following suggestions does the nurse give regarding medications? 1. Lock all medications in a cabinet. 2. Childproof all the caps to medication bottles. 3. Store medications on the highest shelf in a cupboard. 4. Place medications in different containers.

Question: What is the BEST way to prevent accidental poisoning in children, especially toddlers? Strategy: Picture toddlers at play. Needed Info: Remember that no bottle's cap can be made totally childproof; only a locked cabinet can provide protection. Even the highest shelf is no barrier for some climbing toddlers. Changing the containers will only make life difficult for the parents; it will not prevent accidental poisoning. (1) CORRECT - improper storage most common cause of poisoning; highest incidence in 2-year- olds (2) children can open (3) toddlers climb (4) keep in original containers.

The nurse cares for the patient who is being treated for heart failure (HF) and atrial fibrillation. The physician orders digoxin 0.25 mg PO daily. Prior to administering the medication, the nurse assesses that the patient's heart rate is 98 and irregular. Which of the following actions should the nurse take FIRST? 1. Administer the digoxin and chart the rhythm. 2. Hold the digoxin until the patient's pulse slows down. 3. Hold the digoxin until the patient's pulse increases. 4. Call the physician for clarification of the medication order.

Question: What is the FIRST thing you should do? Strategy: Determine the outcome of each answer choice. Needed Info: Atrial fibrillation: rapid, irregular depolarization of atria. Results in irregular and rapid pulse. Treatment: digoxin (Lanoxin; strengthens the myocardial contraction and slows the rate of conduction), calcium channel blockers nifedipine (Procardia), quinidine, procainamide (Pronestyl), anticoagulants (heparin), cardioversion. (1) CORRECT - drug of choice for atrial fib and flutter; report to physician any increase, decrease, irregularity and/or change in regularity of pulse rate (2) needs digoxin to do (3) already is rapid; would increase myocardial demands (4) not necessary

The nurse cares for the patient receiving neomycin sulfate. The nurse recalls that this medication is given for which of the following reasons? 1. To increase digestive functioning by supporting intestinal bacteria. 2. To decrease postoperative wound infection by suppressing intestinal bacteria. 3. To serve as an adjunct to systemic antibiotic therapy. 4. To prevent the occurrence of ulcerative colitis.

Question: What is the action of neomycin? Strategy: Think about the action of the medication. Needed Info: Neomycin sulfate (Neo-fradin) is an aminoglycoside used to treat infections caused by Pseudomonas and E. coli, used to suppress intestinal bacteria, and as adjunct treatment for hepatic coma; side effects include ototoxicity and nephrotoxicity; nursing considerations include check hearing and renal function, encourage fluids, and offer small frequent meals. (1) no improved digestion (2) CORRECT - neomycin sulfate is an aminoglycoside used to suppress intestinal bacteria; acts as a bowel sterilizer; used to prevent wound and abdominal infections (3) used as adjunctive treatment of hepatic coma (4) sulfasalazine (Azulfidine), not neomycin, is helpful in preventing the recurrence of ulcerative colitis.

The nurse plans care for the patient diagnosed with Graves' disease. The nurse includes which of the following in the patient's plan of care? 1. Provide frequent rest periods. 2. Provide 2 meals per day. 3. Provide extra clothing for warmth. 4. Provide caffeinated beverages.

Question: What is the appropriate action? Strategy: Determine the outcome of each answer. Needed Info: Graves' disease is hyperthyroidism; assessment includes hyperactivity, sensitivity to heat, rest and sleep disturbance, increased perception of stimuli, weight loss, and tachycardia. (1) CORRECT - due to increased metabolic rate, provide for frequent rest periods and provide an environment that is free of stress (2) requires 6 meals per day that are high in calories due to high metabolic rate (3) suffers from heat intolerance; requires cool environment (4) has increased metabolic rate and caffeine would further increase it.

The nurse cares for the infant being evaluated for pyloric stenosis. The nurse recognizes that it is MOST important to offer which of the following feedings? 1. Clear fluids. 2. Continuous nasogastric feedings. 3. Intermittent nasogastric feedings. 4. Small, frequent feedings.

Question: What is the appropriate feeding for an infant with pyloric stenosis? Strategy: Determine the outcome of each answer. Needed Info: Pyloric stenosis is obstruction caused by hypertrophy and hyperplasia of pylorus, the muscular sphincter at the gastroduodenal juncture; projectile vomiting occurs 2 - 4 weeks after birth; postoperative care includes provide parenteral fluids as ordered, check incision site, monitor warmth, offer clear liquids with glucose or electrolyte solution first; if tolerated, infant begins formula or breast feeding. (1) clear fluids offered only in special situations, such as initial feeding after surgery or in preparation for surgery or diagnostic tests of the bowel (2) IV fluids will be given rather than NG tube feedings; exception--NG tube may be inserted prior to surgery for gastric decompression and may be continued immediately after surgery (3) if unable to tolerate oral feedings, infant is NPO and given IV fluids, not NG tube feedings, containing glucose and electrolytes (4) CORRECT - normal amounts of feeding may not be tolerated; may cause aspiration; position upright on right side after feedings

The nurse cares for clients in the outpatient clinic. Which of the following is the MOST important immediate nursing goal for the client just diagnosed with glaucoma? 1. Prepare for required surgery. 2. Prevent further deterioration of the vision. 3. Assist the client to deal with the inevitable effects of blindness. 4. Decrease the ocular pressure and improve vision.

Question: What is the appropriate goal for a client diagnosed with glaucoma? Strategy: Think about each answer. Needed Info: Glaucoma is an abnormal increase in intraocular pressure, leading to visual disability and blindness; signs and symptoms include cloudy, blurry vision or loss of vision; artificial lights appear to have rainbows or halos around them; decreased peripheral vision; pain, headache, nausea, and vomiting; treatment is miotics. (1) surgery is not always indicated (2) CORRECT - damage to vision cannot be corrected; further damage can be prevented with medication; client can complete a trial of medication before contemplating surgery or assuming blindness as an outcome (3) most important immediate goal is to prevent further loss of vision, possible with medication, so that client will not develop blindness, which is NOT inevitable (4) ocular pressure can be decreased, but vision cannot be improved.

The nurse cares for the patient admitted to the recovery room following a total left hip replacement. The nurse positions the patient in which of the following positions? 1. On the right side with the head of the bed slightly elevated and the left hip adducted. 2. On the left side with the head of the bed slightly elevated and the hips flexed 120 degrees. 3. Supine with the knee gatch elevated to 30 degrees and the left hip extended. 4. Supine with the head of the bed slightly elevated and a pillow between the legs.

Question: What is the appropriate position to place a client in after a total left hip replacement? Strategy: Determine the outcome of each answer. Needed Info: Nursing care includes abduction of affected extremity using splints, wedge pillow, or 2 - 3 pillows between legs, turn patient as ordered, ice to operative site, do not sleep on operative side, do not flex hip more than 45 - 60 degrees. (1) adduction should be avoided to prevent dislocation (2) do not flex more than 45 - 60 degrees to prevent dislocation (3) head of bed should be slightly elevated, knee gatch is never used (4) CORRECT - keeps legs abducted and prevents hip flexion; maintains alignment of the prosthesis and prevents dislocation

The nurse is performing a home care visit on the 3-year-old with a cast on the left arm due to a fracture of the radius. The nurse is MOST concerned when which of the following is observed? 1. The mother wraps the cast with plastic wrap prior to bathing the child. 2. The child elevates the arm on a pillow while watching television. 3. The child sits at the table playing with small toy figurines. 4. The mother encourages the child to wiggle the fingers on the left hand.

Question: What is the child doing wrong? Strategy: "MOST concerned" indicates an actual or potential complication. Needed Info: Immediate cast care includes avoid covering cast until dry, handle with palms, not fingertips, watch for danger signs such as blueness or paleness, pain, numbness or tingling sensations on affected area; intermediate cast care includes mobilize client, encourage isometric exercises, do not put anything inside cast, keep small items that might be placed inside the cast away from small children. (1) waterproofs cast against splashes; cast should be kept out of water (2) prevents swelling; can support with a sling or on pillow (3) CORRECT - possibility of child sticking small items down cast (4) indicates good circulation; also check for swelling, discoloration, or decreased sensation.

A neighbor calls the nurse stating that a piece of glass is embedded in the neighbor's child's eye. Which of the following instructions by the nurse is MOST important? 1. Irrigate the injured eye with warm normal saline and apply a dressing. 2. Place a pressure dressing on the injured eye and take the child to the emergency room. 3. Remove the piece of glass from the child's eye. 4. Put an eye patch over both eyes and immediately take the child to the emergency room.

Question: What is the correct action for an eye injury? Strategy: Determine the outcome of each answer. Needed Info: If nonpenetrating abrasion, patch eye for 24 hours; if nonpenetrating contusion, apply cold compresses and take analgesics; if penetrating injury, cover with patch and refer to surgeon. (1) appropriate for nonpenetrating foreign body that is causing an irritation (2) would lead to further eye injury (3) would lead to further eye injury; removal should be done only by a surgeon (4) CORRECT - minimize eye movement in order to prevent further injury

The nurse manager reviews infection prevention practices with the staff caring for clients with central venous catheters (CVCs). Which of the following statements by a staff member indicates the BEST understanding of the precaution required to prevent infections for these clients? 1. "If the dressing is wet or soiled, I will change it immediately." 2. "I will apply the antibiotic ointment as ordered by the physician." 3. "I will assess for swelling in the shoulder, neck, chest, and arm at least twice per shift." 4. "I will flush the catheter at regular intervals."

Question: What is the correct care to prevent infection? Strategy: Determine the outcome of each answer. Needed Info: Central venous catheter used to deliver parenteral nutrition; complications of insertion of central venous catheter include pneumothorax and infection. (1) CORRECT - prevents growth of microorganisms; use aseptic technique when changing dressing (2) relates to treatment, not prevention; if infection occurs, physician may order local antibiotic ointment and /or systemic antibiotic or antifungal (3) swelling indicates possible pneumothorax, which is not an infection (4) important to maintain patency of central venous catheter, but not expected to play a role in infection prevention.

The nurse evaluates care for the client with a diagnosis of vaginal cancer being treated with an internal radium implant. The nurse determines that the nursing care of the client is appropriate if which of the following is observed? 1. The nurse wears a dosimeter film badge when in the client's room. 2. The client uses the bedside commode. 3. The client's 10-year-old grandchild visits for 20 minutes. 4. The nurse stands at the foot of the bed to talk with the client.

Question: What is the correct procedure when caring for clients with an internal radium implant? Strategy: Determine the outcome of each answer. Needed Info: Internal radiation is a sealed source placed in a body cavity or tumor. Place client in private room; save all dressing, bed linens until source is removed; then discard dressing and linens as usual; rotate staff caring for client. (1) CORRECT - measures the amount of radiation that nurse is exposed to; each nurse should have individual badge (2) is on bed rest to prevent dislodgment of the implant (3) children under age of 16 and pregnant women not allowed to visit; limit time in room (4) do not stand in the direct line of the radiation source; limit time in room.

The nurse cares for the postoperative client receiving cephalexin monohydrate 500 mg PO QID. The nurse schedules the administration of this medication at which of the following times? 1. Prior to meals. 2. 9AM, 3PM, 9PM, 3AM. 3. 9AM, 1PM, 5PM, 9PM. 4. After administration of an antacid.

Question: What is the correct schedule to administer cephalexin monohydrate? Strategy: Determine the outcome of each answer. Needed Info: Cephalexin (Keflex) is a first-generation cephalosporin antibiotic; side effects include diarrhea, nausea, dizziness, abdominal pain, superinfection, allergic reactions; take with food, avoid alcohol while taking medication; assess for penicillin allergy (up to 20% have cross-allergy). (1) take with food or milk to avoid GI upset (2) CORRECT - blood level must be achieved and maintained for an antibiotic to be effective; medication should be given around-the-clock, every 6 hours (3) schedule medication around-the-clock, every 6 hours (4) antacids will reduce the effectiveness of the medication.

The nurse cares for clients in the orthopedic clinic. The nurse is MOST concerned if which of the following is observed? 1. The teenager who is 6'4" tall places the crutches about 6" to the side of the feet when ambulating with them. 2. The school-aged child who is 4'8" tall flexes the elbows about 20 degrees when ambulating with crutches. 3. The middle-aged adult who is 5'10" tall advances the crutches first when walking down the stairs. 4. The older adult who is 5'6" tall uses a 4-point gait when ambulating with crutches.

Question: What is the correct technique for crutch-walking? Strategy: "MOST concerned" indicates an incorrect action. Needed Info: To determine crutch height, measure 2 fingers below axilla; support weight on hand pieces, not on axilla; crutches should be kept 8 - 10 inches out to side. (1) CORRECT - taller person requires a broader base of support, at least 8 inches (2) elbows flexed 20 - 30 degrees enables correct hand placement on grips (3) appropriate technique, follow crutches with weak leg, then strong leg (4) provides maximum support, partial weight bearing, both feet required.

The nurse instructs the client about how to perform breast self-examination. The nurse should include which of the following instructions about examining the breasts in a mirror? 1. "Stand with your arms at your sides. Bend from the waist to the left side. Bend from the waist to the right side." 2. "Stand with both arms above your head. Lower the right arm and keep the left arm raised. Lower the left arm and raise the right arm." 3. "Stand with your hands on your hips. Clasp your hands behind your back." 4. "Stand with your arms at your sides. Clasp your hands behind your head and press your hands forward. Place your hands on your hips and bow slightly toward the mirror."

Question: What is the correct way to perform self-breast examination using a mirror? Strategy: Determine the outcome of each answer. Needed Info: Perform breast self-examination monthly beginning at age 20; after inspecting breasts in the mirror, client should palpate the breasts when standing and lying down. (1) should bend forward with hands on hips (2) raise arm to palpate breast to detect unusual growths (3) clasp hands behind head first and then place hands on hips. (4) CORRECT - stand before mirror to inspect breast for discharge from nipples, puckering, dimpling, or scaling of skin; placing hands behind head and on hips will show changes in shape and contour of the breast

The client in the clinic asks the nurse, "What is the difference between rheumatoid arthritis and osteoarthritis?" Which of the following responses by the nurse is BEST? 1. "Rheumatoid arthritis is quickly progressive, and osteoarthritis has periods of remission." 2. "Rheumatoid arthritis is a systemic disease, and osteoarthritis is deterioration of the synovial joints." 3. "Rheumatoid arthritis is often treated surgically, and osteoarthritis is treated by medication." 4. "There is very little clinical difference between rheumatoid arthritis and osteoarthritis."

Question: What is the difference between rheumatoid arthritis and osteoarthritis? Strategy: Think about each answer. Needed Info: Rheumatoid arthritis is a chronic systemic disease that causes inflammatory changes in joints; osteoarthritis is nonsystemic and degenerative; symptoms include joint pain, swelling, and limitation of movement; nursing care includes pain management, rest, activity, and exercise. (1) rheumatoid arthritis is progressive and has periods of remission and exacerbations; osteoarthritis is degenerative and there are no remissions (2) CORRECT - rheumatoid arthritis is a chronic systemic disease that causes inflammatory changes in joints; osteoarthritis is nonsystemic and degenerative affecting the synovial joints (3) both diseases can be treated with medication; both diseases may require joint replacement (4) untrue statement.

The nurse cares for the client who experiences severe panic attacks when planning to go grocery shopping. The nurse expects to administer which of the following oral medications? 1. Chlorpromazine. 2. Carbamazepine. 3. Flurazepam. 4. Imipramine.

Question: What is the drug of choice for clients experiencing panic-level anxiety? Strategy: Think about the action of each medication. Needed Info: Anxiety is feeling of dread or fear in the absence of external threat or disproportionate to the nature of the threat; in panic level anxiety, the client is unable to see, hear or function; assess level of anxiety, decrease environmental stimuli, use unhurried approach and stay with the client. (1) chlorpromazine (Thorazine): antipsychotic medication; not used to treat panic attacks (2) carbamazepine (Tegretol): anticonvulsant used to treat seizures and nightmares (3) flurazepam (Dalmane) : sedative-hypnotic, used to produce sleep. (4) CORRECT - imipramine (Tofranil): tricyclic antidepressant used to treat panic attacks

The patient with a history of alcoholism is brought to the emergency room in an agitated state. The patient is vomiting and diaphoretic. The patient had the last drink 5 hours ago. The nurse expects to administer which of the following medications? 1. Disulfiram. 2. Methadone hydrochloride. 3. Naloxone hydrochloride. 4. Chlordiazepoxide hydrochloride.

Question: What is the drug used to treat acute alcohol withdrawal? Strategy: Think about the action of each drug. Needed Info: Symptoms of acute alcohol withdrawal include tremors, being easily startled, insomnia, anxiety, anorexia, and alcoholic hallucinations. Nursing care includes administering sedation as needed, monitoring pulse, blood pressure, and temperature, seizure precautions, orienting frequently, and not leaving hallucinating, confused client alone. (1) disulfiram (Antabuse): used as a deterrent to impulsive drinking; contraindicated if patient drank alcohol in previous 12 hours (2) methadone hydrochloride (Dolophine): opioid analgesic, used to treat narcotic withdrawal syndrome; S/E seizures, respiratory depression (3) naloxone hydrochloride (Narcan): narcotic antagonist used to reverse narcotic-induced respiratory depression; S/E ventricular fibrillation, seizures, pulmonary edema. (4) CORRECT - chlordiazepoxide hydrochloride (Librium): antianxiety; used to treat symptoms of acute alcohol withdrawal; S/E lethargy, hangover, agranulocytosis

The nurse manager notes that one of the staff members is frequently absent, and this has adversely affected the quality of care given to patients on the unit. When INITIALLY counseling the staff member, which of the following approaches by the nurse manager is BEST? 1. Inform the staff member that the next missed day will be grounds for termination. 2. Talk with the staff member and remind the staff member of the standards of the agency. 3. Give the staff member a written reminder of the standards of the agency. 4. Document the staff member's absenteeism.

Question: What is the first action the nurse manager should take? Strategy: Determine the outcome of each answer. Needed Info: If the staff member does not clearly understand what is expected, the staff member may feel role strain, which might cause withdrawal from the work situation. (1) important to clarify the staff member's role; give oral and written reminders before terminating employee (2) CORRECT - first action is to give employee an oral reminder of the agency's standards; do not threaten discipline; purpose is to clarify role expectation (3) if absenteeism continues to be a problem after the verbal reminder, the staff member is given the same reminder in writing (4) should be documented, but the nurse's first action should be to give the staff member a verbal reminder.

At midnight, 2 days following a hemicolectomy, the patient awakens frightened and agitated. The patient climbs out of bed, removes the indwelling urinary drainage catheter, and runs down the hall screaming. Which of the following actions is most appropriate for the nurse to take INITIALLY? 1. Call the physician and request a sedative for the patient. 2. Return the patient to bed and restrain the patient immediately. 3. Replace the patient's indwelling urinary drainage catheter. 4. Return the patient to bed and assess the patient's condition.

Question: What is the first thing you should do in this situation? Strategy: Establish priorities. Remember the steps of the nursing process. Needed Info: Hemicolectomy: removal of half or less of the colon in order to remove tumors. (1) assessment needed first (2) last resort; needs assessment and reorientation (3) not first priority. (4) CORRECT - assessment first step

The woman in her second trimester of pregnancy tells the clinic nurse that her 5-year-old child has been asking questions "about sex." The client asks the nurse what she should tell her child. Which of the following statements, if made by the nurse, is BEST? 1. "Buy a book about sex designed for young children and read it with your son." 2. "Have your child touch your abdomen and tell him about your pregnancy." 3. "Tell your child that this subject is complicated, and you will discuss it as the child gets older." 4. "Answer your child's questions in a matter-of-fact manner, in words that the child will understand."

Question: What is the most appropriate response? Strategy: "BEST" indicates discrimination is required. Needed Info: Important to determine what the child knows and thinks and to offer honest explanations. (1) not best action, may provide more information than child is seeking (2) does not answer particular questions child has; assumes that questions are about pregnancy (3) questions should be answered as they are asked, not postponed or ignored. (4) CORRECT - helps child understand his concerns, allows for answering exact question that is being asked

The nurse cares for the patient with a diagnosis of chronic obstructive pulmonary disease (COPD) and bronchitis. The patient constantly rings the call bell and rattles the bed rails. Which of the following actions by the nurse is MOST appropriate? 1. Check the patient's pulse oximetry. 2. Send a nursing assistant to sit with the patient. 3. Sit the client in a chair next to the nurses' station. 4. Request that the patient's family order the TV for the patient.

Question: What is the most important action for this patient? Strategy: Remember to assess before implementing. Needed Info: COPD is a group of conditions associated with obstruction of air flow entering or leaving the lungs; indications include change in skin color, weakness and weight loss, dyspnea, use of accessory muscles to breathe, cough, abnormal ABGs. (1) CORRECT - decreased oxygenation will cause confusion; assess before implementing (2) assess before implementing (3) obtain pulse oximetry reading before determining appropriate intervention (4) assess before implementing.

The client is brought to the community mental health center by the spouse. One year ago the client's youngest child was killed in a car accident. The graduation of the child's high school class triggered feelings of sadness and guilt. As a result, the client has been having severe headaches, insomnia, and poor appetite. In planning care for this client, the nurse recognizes that the symptoms are MOST likely an example of which of the following? 1. Turning aggression inward. 2. Receiving inadequate support from her family. 3. Displacement of anger. 4. Delayed grief reaction.

Question: What is the most likely cause of these symptoms in this patient? Strategy: Think about each action. Needed Info: At the one-year time point, a client would often be moving toward the stage of grief called acceptance. However, the stages are not sequential and do not have a guaranteed timeframe. The client has faced a major anniversary, reminding client of the loss. (1) partial possible explanation (2) assumption (3) not accurate; no evidence of displacement (4) CORRECT - anniversaries of loss can trigger symptoms of grief

The patient diagnosed with multiple myeloma is admitted to the unit after developing pneumonia. When the nurse enters the patient's room wearing a mask, the patient says in an irritated tone of voice, "Why are you wearing that mask?" Which of the following responses by the nurse is BEST? 1. "The chest x-ray taken this morning indicates you have pneumonia." 2. "What have you been told about the x-rays that were taken this morning?" 3. "You have been placed on contact precautions due to your infection." 4. "I am trying to protect you from the germs in the hospital."

Question: What is the most therapeutic response? Strategy: Remember to assess before implementing. Needed Info: Multiple myeloma is a neoplastic disease that infiltrates bone and bone marrow, causes anemia, renal lesions, and high globulin levels in blood. Pneumonia is inflammatory process resulting in edema of lung tissue and extravasation of fluid into alveoli, causing hypoxia. (1) does not assess what client knows; physician responsible for telling patient the medical diagnosis (2) CORRECT - assessment; determines what client knows before responding; allows client to verbalize (3) pneumonia requires droplet precautions (4) pneumonia requires droplet precautions.

The nurse has just given a client a subcutaneous injection. What immediate follow-up action does the nurse take? 1. The nurse removes and discards gloves in the designated receptacle. 2. The nurse performs hand hygiene, to protect both the nurse and the client. 3. The nurse discards the uncapped needle with the syringe in the designated receptacle. 4. The nurse caps the needle before discarding the syringe in the designated receptacle.

Question: What is the next step after giving an injection? Strategy: Think about each answer choice. Needed Info: Capping a needle can lead to a needlestick injury. Gloves kept on until syringe disposal, as a safety precaution. Hand hygiene after all other steps. (2) hand hygiene after syringe and gloves disposal (3) CORRECT - the Centers for Disease Control and Prevention (CDC) recommends not capping the needle before disposal (1) discard gloves after syringe disposal (4) violates CDC recommendation

The school nurse observes the group of school-aged children playing on the playground. A child begins to cry and reports being stung by a bee. Which of the following actions should the nurse take FIRST? 1. Inject IM epinephrine. 2. Remove the stinger. 3. Apply a warm compress. 4. Wash with soap and water.

Question: What is the nurse's priority for a bee sting? Strategy: All answers are implementation; determine the outcome of each answer. Needed Info: Hymenopteran stings (bees, wasps, hornets, yellow jackets, fire ants) inject venom through a stinging apparatus; local reaction includes small red area, wheal, itching, and heat. Assess for systemic reaction and instruct client about how to avoid contact. (1) appropriate for hypersensitive individuals or if the client demonstrates a severe life-threatening response (2) CORRECT - remove stinger by scraping skin until stinger is removed; remove the stinger as quickly as possible to avoid injection of venom (3) apply cool compress after removing stinger and clean the bee sting (4) cleanse wound after removing stinger; apply paste made with baking soda or meat tenderizer.

The nurse cares for the 19-year-old client admitted to the emergency department after an auto accident. Even though the client denies drinking alcohol, the nurse notes that the client's breath smells of alcohol, speech is slurred, reflexes are diminished, and the client has difficulty recalling the events of the evening. The physician orders an MRI. Which of the following actions should the nurse take FIRST? 1. Inform the client that since he is of the age of consent, he can sign the consent form for the MRI. 2. Instruct the client to remove his watch. 3. Contact the client's parents to give consent for the MRI. 4. Restrict food and fluids for 4 hours.

Question: What is the nurse's priority in this situation? Strategy: Determine the outcome of each answer. Needed Info: In most states, young adults (18 years and older) can legally give consent; a client cannot give informed consent if s/he has been drinking or is premedicated. (1) cannot give consent since client has apparently been drinking and has altered mental status (2) appropriate action; however, health care provider must obtain consent prior to an MRI (3) CORRECT - MRI provides detailed pictures of body structures; procedure requires consent, and client unable to give informed consent due to probable drinking and altered mental status (4) no food or fluid restrictions for adults.

The nurse auscultates the abdomen of the pregnant woman at 38 weeks gestation to determine fetal heart rate. If the fetal heartbeat is located in the right lower quadrant, which of the following is MOST likely the presenting part? 1. Shoulder. 2. Head. 3. Feet. 4. Buttocks.

Question: What is the position of the fetus? Strategy: Map out the abdomen and picture the position of the fetus. Needed Info: Lower quadrant heartbeat indicates vertex or head/cephalic presentation; fetal heartbeat is best heard over the fetus's back. (1) only 1% of births; uncommon (2) CORRECT - right lower quadrant heartbeat indicates occiput of fetal head is on the right side of the mother's body and facing the front (anterior) of the mother's body; (ROA) (3) breech; would hear FHT in upper quadrant (4) breech; would hear FHT in upper quadrant.

The home care nurse cares for the child diagnosed with hemophilia A recovering from the acute phase of spontaneous bleeding into the joints. It is MOST important for the nurse to give the parents which of the following instructions? 1. Administer ibuprofen for pain. 2. Apply ice to the joint. 3. Decrease the risk of injury. 4. Encourage active range-of-motion exercises.

Question: What is the priority nursing action after the acute phase of spontaneous bleeding? Strategy: "MOST important" indicates a priority. Determine the outcome of each answer. Needed Info: Hemophilia is a bleeding disorder caused by deficiency of factor VIII (most common) or factor IX; symptoms include easy bruising, joint pain with bleeding, prolonged internal or external bleeding. (1) use ibuprofen (Advil) with caution because it inhibits platelet aggregation; offer acetaminophen (Tylenol) at home to control pain (2) appropriate action during bleeding episode; rest, ice, compression, and elevation to prevent excessive blood loss; administer factor VIII concentrate (3) important to prevent bleeding episodes; encourage age-appropriate exercises that strengthen muscles and joints (4) CORRECT - active range-of-motion encouraged after bleeding episode to prevent crippling effects of bleeding; active range-of-motion allows the child to control the amount of exercise according to the pain level; do not perform passive range-of-motion

The nurse cares for the client after an above-the-knee amputation. The client has a closed rigid cast dressing in place. Several days after surgery, the nurse enters the client's room and finds that the cast has come off. Which of the following actions does the nurse take FIRST? 1. Wrap the residual limb with an elastic compression bandage. 2. Observe the residual limb for swelling. 3. Contact the physician. 4. Ask the client how the cast came off.

Question: What is the priority nursing action if the rigid cast dressing comes off an above-the-knee amputation? Strategy: Determine the outcome of each answer. Needed Info: Observe for signs of oozing; elevate residual limb for 24 hours; turn client prone to prevent contractures; client ambulates early with rigid cast dressing because it functions as a socket for fitting of a prosthetic immediately post-op. (1) CORRECT - will prevent edema from developing; edema will delay the rehabilitation process (2) important to prevent edema; if residual limb not wrapped immediately, significant swelling will occur (3) notify surgeon so that a new cast dressing can be applied; wrap residual limb before notifying the physician (4) more important to prevent edema from developing.

The nurse cares for clients in the labor and delivery unit. The nurse notes that a client's membranes have ruptured and the amniotic fluid is meconium-stained. The nurse determines that there is no prolapsed cord. Which of the following actions does the nurse take NEXT? 1. Contact the health care provider. 2. Assess fetal heart tones. 3. Start an intravenous line. 4. Obtain the client's pulse and blood pressure.

Question: What is the priority nursing action when a client passes meconium-stained amniotic fluid? Strategy: Determine whether it is appropriate to assess or implement. Needed Info: Amniotic fluid is straw-colored and pale; meconium-stained fluid (greenish-brown) indicates fetus has probably experienced recent hypoxic episode; meconium-stained fluid may be normal finding in breech presentation. (1) assess for nonreassuring fetal heart tone patterns before contacting health care provider (2) CORRECT - meconium-stained amniotic fluid may be an ominous sign; assess for the nonreassuring fetal heart tone patterns of fetal bradycardia, fetal tachycardia, irregular FHR, late, severe, variable, and prolonged deceleration patterns; if fetal distress, turn client to left side, give supplemental oxygen, start IV (3) assess fetus first (4) no reason to assess mother; meconium-stained fluid might indicate fetal distress.

The nurse cares for the 6-year-old child placed in Russell's traction due to a fracture of the left tibia. After repositioning the child, it is MOST important for the nurse to take which of the following actions? 1. Administer pain medication. 2. Offer the child a book. 3. Check the position of the left hip. 4. Assess the pin site for infection.

Question: What is the priority nursing action when caring for a child in Russell's traction? Strategy: "MOST important" indicates priority. Answers are a mix of assessment and implementation. Needed Info: Skin traction is used on the lower leg and a padded sling is placed under the knee; "pulls" contracted muscles; elevate foot of bed with shock blocks to provide countertraction; check popliteal pulse; do not turn from waist down; lift patient, not leg, to provide assistance. (1) analgesics and muscle relaxants are administered due to discomfort caused by traction pull; offer at regular intervals (2) important to offer children an opportunity for play; explain to child what is happening (3) CORRECT - hip is flexed at a prescribed angle to prevent fracture malalignment; after moving child, assess that the prescribed amount of hip flexion is maintained (4) Russell's traction is a form of skin traction; no pins are used.

The staff nurse observes the newly licensed LPN/LVN prepare to administer iron dextran IM to a patient with iron deficiency anemia. It is MOST important for the staff nurse to give which of the following instructions to the LPN/LVN? 1. "Massage the injection site for one minute after the injection of the medicine." 2. "Tap out the air bubble prior to administering the medication." 3. "Release the skin prior to withdrawing the needle." 4. "Change the needle after drawing up the medication."

Question: What is the priority when administering iron dextran? Strategy: Determine the outcome of each answer. Needed Info: Iron dextran (DexFerrum) is a hematinic used to treat iron deficiency anemia; administer using Z-track method; select large, deep muscle; pull skin and subcutaneous tissue 1.5 inches to the side; release skin after withdrawing needle. (1) causes medication to leak into subcutaneous tissue, staining skin (2) draw up 0.2 mL of air to create an airlock (3) release skin after withdrawing the needle. (4) CORRECT - ensures that no solution remains on outside of needle

The home care nurse visits the client diagnosed with non-Hodgkin's lymphoma who is receiving chemotherapy. After the second round of chemotherapy, the client reports a sore mouth and loss of taste. Which of the following actions does the nurse take FIRST? 1. Examine the client's mouth. 2. Instruct the client to use a saline rinse. 3. Obtain a diet history from the client. 4. Instruct the client to avoid spicy foods.

Question: What is the priority when client is complaining of a sore mouth? Strategy: Assess before implementing. Needed Info: Chemotherapy causes stomatitis; assess frequently; good oral hygiene; use soft-bristled toothbrush; avoid dental floss, water pressure gum cleaners, and mouthwashes containing alcohol or glycerin. (1) CORRECT - assess every 4 hours; instruct client about good oral hygiene (2) implementation; important to rinse mouth with water or saline rinse every 12 hours (3) assessment; encourage client to avoid spicy foods or hard foods (4) implementation; correct action, but assess first.

The home care nurse visits the client who has been receiving lithium carbonate for 3 weeks. The client reports to the nurse experiences of blurred vision and intense dizziness. Which of the following actions does the nurse take FIRST? 1. Encourage the client to increase fluid intake. 2. Notify the physician. 3. Instruct the client to breathe into a paper bag. 4. Teach the client about relaxation techniques.

Question: What is the priority when the client is complaining of adverse effects of lithium? Strategy: Determine the outcome of each answer. Needed Info: Lithium used to treat bipolar disorder; has a narrow therapeutic range (1 - 1.5 mEq/L); side effects include dizziness, headache, impaired vision, fine hand tremors, and reversible leukocytosis; nursing considerations include monitor blood levels frequently, encourage 2,500 - 3,000 mL fluids daily. (1) intake should be 2,500 - 3,000 mL daily; more important to contact physician (2) CORRECT - important to confirm lithium level; levels over 2.0 mEq/L may cause lithium intoxication, agitation, ataxia, blurred vision, confusion, tinnitus, vertigo, hyperreflexia, and myoclonic twitching (3) appropriate action if client hyperventilating (4) not priority; symptoms indicate adverse side effects; need physician management.

The home care nurse is visiting the 82-year-old client living with the client's adult child. The client appears malnourished and has multiple bruises on the body. Which of the following actions, if taken by the nurse, is MOST appropriate? 1. Place a home health aide with the patient to document incidents of abuse. 2. Discuss the nurse's observation with the client's children. 3. Report the situation to the nursing supervisor. 4. Request that another nurse visit the patient to assess the situation.

Question: What is the priority when the nurse suspects elder abuse? Strategy: Determine the outcome of each answer. Needed Info: Indications of elder abuse include battering, fractures, bruises, overmedicated or undermedicated, poor nutritional status, dehydration; nursing care includes providing for the client's safety, providing for physical needs, and reporting to appropriate agency. (1) must report suspected cases (2) report to the nursing supervisor (3) CORRECT - required by state law (4) no reason to involve another nurse.

The male patient is admitted to the hospital for evaluation of hematuria. An intravenous pyelogram (IVP) is ordered. The patient asks the nurse to explain what will happen to him during the IVP. Which of the following explanations, if made by the nurse, is MOST accurate? 1. "An intravenous line will be inserted, dye injected into it, and then x-rays will be taken of your kidneys, ureters, and bladder." 2. "A scope will be inserted into your penis so that the inside of your bladder can be visualized." 3. "A catheter will be inserted into your penis, dye injected into it, and then x-rays will be taken of your bladder and ureters." 4. "A small incision is made in the kidney, dye injected into it, and then x-rays will be taken of your kidneys, ureters, and bladder."

Question: What is the procedure for an IVP? Strategy: Form a mental image of the procedure. Needed Info: Hematuria: blood in the urine. IVP: radiographic exam of the kidney, ureter, bladder. Prep: check for sensitivity to contrast medium, iodine, shellfish; prep bowel (laxatives, enemas); may be NPO or allowed fluids. Post-test: force fluids. (1) CORRECT - describes procedure (2) cystoscopy; prep: NPO, bowel prep (laxatives, enemas), general or local anesthesia used; post-test: check for bleeding and infection (3) cystourethrogram; prep: none; post-test: check for infection (4) inaccurate.

The nurse cares for the 1-year-old patient who is admitted to the hospital with a fractured femur and is placed in Bryant's traction. The nurse recognizes that the child should be maintained in which of the following positions? 1. Buttocks slightly elevated off the bed. 2. Buttocks flat on the bed. 3. Knees slightly flexed. 4. Hips extended.

Question: What is the proper position for a child in Bryant's traction? Strategy: Picture the traction apparatus. Remember the concepts for effective traction. Needed Info: Bryant's traction: type of running traction used to reduce a fractured femur in a child; adhesive strips are applied to both legs and secured with elastic bandages wrapped from foot to groin; both legs are suspended by weights and pulleys. (1) CORRECT - child's weight provides countertraction (2) no countertraction (3) not possible; must be extended (4) must be flexed at 90 degree angle.

The nurse conducts the admission physical examination for the new clinic patient. Which of the following sequences represents the order in which the nurse performs the assessment of the patient's abdomen? 1. Observe, auscultate, percuss, palpate. 2. Auscultate, observe, percuss, palpate. 3. Palpate, percuss, auscultate, observe. 4. Percuss, palpate, auscultate, observe.

Question: What is the proper sequence of steps to take to assess a patient's abdomen? Strategy: Remember to look and listen before you feel. Needed Info: Percussion and palpation alter the mobility of the bowel and heighten bowel sounds. Use the diaphragm of a stethoscope because sounds are high pitched. Percuss, checking for tympany (hollow sound) and dullness (high-pitched sound). To palpate, depress abdominal wall 1 cm using the pads of your fingers. (1) CORRECT (2) inaccurate; look first (3) inaccurate; look first (4) inaccurate; look first.

The nurse cares for the postoperative client who had an abdominal resection for colon cancer, including the insertion of a Jackson-Pratt drain. The nurse recognizes that which of the following is the PRIMARY purpose of the drain? 1. To irrigate the incision with a saline solution. 2. To prevent bacterial infection of the incision. 3. To prevent accumulation of drainage in the wound. 4. To measure the amount of fluid lost after surgery.

Question: What is the purpose of a Jackson-Pratt drain? Strategy: Think about each answer. Is it the primary purpose of a drain? Needed Info: Jackson-Pratt drain: tissue drain used postoperatively to prevent accumulation of fluid in wound. (1) not accurate (2) not best answer; prophylactic antibiotics and sterile technique used (3) CORRECT - portable wound suction; speeds wound healing; document color, odor, amount, consistency of drainage (4) not primary purpose.

The nurse cares for the patient who returns to the nursing unit in stable condition after having a myelogram using a water-soluble dye. An intravenous infusion is in progress. The nurse recognizes that which of the following is the PRIMARY purpose of the intravenous fluid? 1. To replace blood lost during the procedure. 2. To enhance excretion of the dye. 3. To restore cerebrospinal fluid levels. 4. To increase blood flow to the brain.

Question: What is the purpose of administering IV fluids after a myelogram using a water-soluble dye? Strategy: Think about each answer choice. Does it make sense? Needed Info: In a myelogram, contrast dye is injected into the spinal column. This causes the tissue under study to be visible. The spinal cord, subarachnoid space, and other surrounding structures can be visualized more clearly than in standard x-rays. After the procedure is completed, an intravenous infusion enhances renal excretion of the dye. (1) no blood loss (2) CORRECT - dilutes dye and enhances excretion by kidneys (3) none lost (4) not purpose.

The unconscious patient is admitted to the hospital for treatment of an injury sustained in an automobile accident. The patient has a cuffed tracheostomy tube and mechanical ventilation in progress. The nurse recalls that the purpose of the cuff on a tracheostomy tube is to accomplish which of the following? 1. Prevent displacement of the tracheostomy tube. 2. Maintain the alignment of the trachea with the lungs. 3. Separate the upper and lower airways. 4. Maintain the patency of the trachea.

Question: What is the purpose of the cuff on a trach tube? Strategy: Form a mental image of a tracheostomy tube with a cuff inserted into a trachea. Needed Info: Cuff: plastic balloon that encircles the tracheal tube to form a seal between the outer cannula and the trachea. (1) action of twill tapes tied at side of neck; need 2 people to change, or leave in place till new ties on; allow for 2 finger spaces between tie and neck (2) does not change position of trachea (3) CORRECT - seals off lumen; prevents aspiration (4) purpose of trach tube, not cuff.

The patient is admitted to the hospital after sustaining a severe head injury in an automobile accident. When the patient dies, the nurse observes the patient's spouse comforting other family members. Which of the following interpretations of the spouse's behavior is MOST justifiable? 1. The spouse has already moved through the stages of the grieving process. 2. The spouse is repressing anger related to the patient's death. 3. The spouse is experiencing shock and disbelief related to the patient's death. 4. The spouse is demonstrating resolution of the patient's death.

Question: What is the reason for the spouse's behavior? Strategy: "MOST justifiable" indicates that there may be more than one correct response. Think about each answer choice. Is it true? Needed Info: Stages of grief, popularly known as DABDA: 1) denial, 2) anger, 3) bargaining, 4) depression, 5) acceptance. The stages can be experienced in any order; they are not sequential. Acute period: 4 - 8 weeks, usual minimum time for resolution: 1 year. (1) usually takes a minimum of 1 year (2) anger is a possible stage, but the spouse's behavior does not support this interpretation (3) CORRECT - denial is the inability to comprehend reality of situation (4) too soon.

The nurse conducts the family therapy session with the patient being treated for depression. During the therapy session, the patient verbally expresses love toward the mother, but has an angry facial expression and pounds the table with a fist. The nurse understands that the discrepancy between the patient's body language and spoken language is BEST characterized as which of the following? 1. Ambivalence. 2. Scapegoating. 3. Double-bind communication. 4. Loose associations.

Question: What is this behavior called? Strategy: "Best categorized" indicates that there may be more than one correct response. Needed Info: Double-bind communication is characterized by simultaneous communication of two mutually conflicting verbal and nonverbal messages. (1) mixed feelings; confusing emotional experience (2) others blamed for problems (3) CORRECT - emotions communicated verbally are opposite of emotions communicated physically (4) disordered thought processes.

The patient admitted for treatment of bronchitis reports an allergy to sulfa drugs and to penicillin to the nurse. The nurse recognizes that which of the following medications is MOST appropriate for this patient? 1. Co-trimoxazole. 2. Sulfisoxazole. 3. Cephalexin. 4. Ciprofloxacin.

Question: What medication can be given to a patient with a sulfa and penicillin allergy? Strategy: Think about each answer choice. Needed Info: Patients with a sensitivity to penicillin should take cephalosporin medications cautiously due to a cross-allergy. Patients with an allergy to sulfa drugs should not take any sulfa-containing medications. (1) co-trimoxazole (Septra): sulfa medication; used for treatment of traveler's diarrhea, Pneumocystis jiroveci; used prophylactically for women with recurrent UTI; side effects: agranulocytosis, anemia, N + V, diarrhea, toxic nephrosis, rash, photosensitivity (2) sulfisoxazole (Gantrisin): sulfa medication; side effects: aplastic anemia, toxic nephrosis; force fluids (3,000 - 4,000/day) to prevent crystalluria; keep urine alkaline (3) cephalexin (Keflex): cephalosporin medication; side effects: diarrhea, rash, urticaria; take with food/milk (4) CORRECT - ciprofloxacin (Cipro): quinolone antibiotic; side effects: dizziness, seizures, HA, abdominal pain, rash, photosensitivity; give PO 2 hours after meals or 2 hours before or after antacids or medications continuing iron; avoid caffeine; force fluids 3,000 mL/24 hours

The nurse cares for the client with schizophrenia. The nurse recognizes that the patient has developed parkinsonian side effects of chlorpromazine. The nurse expects which of the following medications will be prescribed for the patient? 1. Diazepam. 2. Haloperidol. 3. Amitriptyline. 4. Benztropine.

Question: What medication is given to treat the parkinsonian side effects of chlorpromazine? Strategy: Think about the action of each drug. Needed Info: benztropine: anticholinergic, antiparkinsonian agent; side effects include drowsiness, blurred vision, nausea, constipation, urinary retention, dry mouth, agitation; nursing considerations include monitor intake and output, monitor for muscle weakness or inability to move certain muscle groups, monitor for central nervous system depression or stimulation, provide sugarless gum or lozenges for dry mouth. (1) diazepam: antianxiety medication; side effects: drowsiness, ataxia, cardiovascular collapse (2) haloperidol: antipsychotic medication; would exacerbate symptoms; side effects: extrapyramidal reactions, blurred vision, dry mouth, tardive dyskinesia (3) amitriptyline: antidepressant medication; side effects: drowsiness, dizziness, orthostatic hypotension, blurred vision, dry mouth, urinary retention, photosensitivity (4) CORRECT- benztropine: antiparkinsonian medication; manages extrapyramidal symptoms; side effects: sedation, dry mouth, urinary retention

The nurse cares for the client admitted to the hospital reporting fatigue and weight loss. Physical examination reveals pallor and multiple bruises on the arms and legs. The results of the client's test reveal acute lymphocytic leukemia and thrombocytopenia. Which of the following nursing diagnoses MOST accurately reflects the client's condition? 1. Potential for injury. 2. Self-care deficit. 3. Potential for self-harm. 4. Alteration in comfort.

Question: What nursing diagnosis is seen with acute lymphocytic leukemia and thrombocytopenia? Strategy: Think about each answer choice. Needed Info: Thrombocytopenia: decreased platelet count increases the client's risk for injury, normal count: 150,000 - 350,000 per mm3. Leukemia: group of malignant disorders involving overproduction of immature leukocytes in bone marrow; this shuts down normal bone marrow production of erythrocytes, platelets, normal leukocytes; causes anemia, leukopenia, and thrombocytopenia leading to infection and hemorrhage. Symptoms: pallor of nail beds and conjunctiva, petechiae (small hemorrhagic spots on skin), tachycardia, dyspnea, weight loss, fatigue. Treatment: chemotherapy, antibiotics, blood transfusions, bone marrow transplantation. Nursing responsibilities: private room, no raw fruits or vegetables, small frequent meals, oxygen, good skin care. (1) CORRECT - low platelet count increases risk of bleeding from even minor injuries; safety measures: shave with an electric razor, use soft toothbrush, avoid subcutaneous or IM medications and invasive procedures (urinary drainage catheter or a nasogastric tube), side-rails up, remove sharp objects, frequently assess for signs of bleeding, bruising, hemorrhage (2) may feel weak; does not most address condition (3) implies risk for purposeful self-injury; not given any information, assumption (4) client is not comfortable, and comfort measures would address problem; does not most address condition.

The nurse cares for the patient in the labor unit. During the transitional phase of labor, the umbilical cord becomes prolapsed. The nurse places the patient in which of the following positions? 1. Lithotomy. 2. Side-lying. 3. Semi-Fowler's. 4. Trendelenburg.

Question: What position should you place a patient in if there is a prolapse of the umbilical cord? Strategy: Picture the situation as described. Needed Info: Prolapsed cord: obstetrical emergency in which the umbilical cord is below the presenting part of the fetus. Compression of the cord causes fetal hypoxia resulting in CNS damage. (1) lying on back with thighs flexed on abdomen, legs separated and knees bent with feet in stirrups; used for examination of vagina or rectum (2) used to remove weight from vena cava to prevent maternal hypotension; does not help with prolapsed cord (3) aggravates prolapsed cord pressure. (4) CORRECT - supine on incline with head lower than hips and legs; or put finger against presenting part and shift weight off cord

The nurse cares for the patient who is being discharged after having a transurethral prostatectomy (TURP) for benign prostatic hypertrophy (BPH). The nurse's discharge teaching plan reinforces adherence to which of the following measures? 1. Avoiding vigorous exercise for 3 weeks. 2. Avoiding cold foods for 1 week. 3. Avoiding hot baths for 1 month. 4. Avoiding high-residue foods for 2 weeks.

Question: What should a patient be told to do after discharge for a TURP? Strategy: Think about the outcome of each answer choice. Needed Info: Hemorrhage most common complication after a TURP. During first 3 weeks post-op avoid: lifting more than 8 lbs, mowing lawn, riding in car more than 25 min, climbing stairs quickly, sexual intercourse, engaging in sports. (1) CORRECT - due to danger of bleeding (2) no dietary restrictions (3) no need to avoid; comforting (4) no dietary restrictions.

The nurse develops a care plan for the patient with dementia. The nurse recognizes that it is MOST important to include which of the following measures in the plan? 1. Leave the television on in patient's room all day. 2. Frequently orient patient to surroundings. 3. Provide patient with newspapers and magazines. 4. Assign a staff member to stay with patient while patient is awake.

Question: What should be done for a patient with dementia? Remember to establish priorities. It is necessary to meet physical needs and safety needs before addressing psychosocial needs. Strategy: Determine the outcome of each answer choice. Needed Info: Dementia: progressive loss of cognitive function. Decline in memory, learning, attention, judgment, orientation and language skills. Most common type is Alzheimer's disease. Affects 5 million people in US. Usually lasts between 7 and 15 years, before death. (1) provides sensory stimuli but no orientation (2) CORRECT - provides for safety needs (3) does not address safety needs or orientation (4) provides company but not orientation.

The patient is admitted to the hospital complaining of right-sided weakness and difficulty speaking. The patient reports a fall while at home. It is MOST important for the nurse's initial assessment of the patient to include evaluation for which of the following? 1. Nutritional deficiencies. 2. Ambulation problems. 3. Hearing difficulties. 4. Head injury.

Question: What should the nurse assess for in this situation? Strategy: Establish priorities. Remember your ABCs. Needed Info: Cerebrovascular accident (stroke): caused by thrombosis, embolism, ischemia, or hemorrhage. S/S: loss of movement, thought, memory, speech, or sensation. Aphasia: inability to use or comprehend language, due to damage in cerebral hemisphere. Dysarthria: problem with rate, rhythm, or articulation of speech due to loss of motor function of muscles for speech. Expressive aphasia: difficulty speaking. Nursing responsibilities: repeat directions, break down tasks into components, face patient, and speak clearly and slowly. Give patient time to respond. Assist with facial muscle exercises. (1) not highest priority (2) not highest priority; on bed rest during first 48 - 72 hours (3) not highest priority (4) CORRECT - safety most important

The home care nurse performs an assessment of the elderly client diagnosed with type 2 diabetes and hypertension. The client is following an 1,800-calorie ADA diet and takes furosemide 40 mg PO daily. The client's adult child tells the nurse that the parent has been complaining of dizziness. Which of the following actions does the nurse take FIRST? 1. Instruct the client to change positions slowly. 2. Advise the client to drink more fluids. 3. Obtain the client's blood pressure when lying or sitting, and then when standing. 4. Check the client's blood sugar.

Question: What should the nurse do if the client complains of dizziness? Strategy: "FIRST" indicates priority. Assess before implementing. Needed Info: Hypertension is persistent elevation of systolic blood pressure of 140 or higher mm Hg and diastolic blood pressure of 90 or higher mm Hg; furosemide (Lasix) is a loop diuretic that causes hypotension, hypokalemia, hyperglycemia, GI upset, and weakness. (1) appropriate action if client has postural hypotension due to diuretic therapy; assess before implementing (2) appropriate action if client has fluid volume deficit; assess before implementing (3) CORRECT - dizziness may indicate hypotension; elderly may be more sensitive to fluid loss due to diuretic therapy; obtain blood pressure in lying/sitting and standing positions to determine if client has postural hypotension (4) dizziness is symptom of hypoglycemia; after assessing for postural hypotension, a check of whether the dizziness is related to low blood sugar readings is warranted.

The nurse cares for the patient admitted to the intensive care unit (ICU) with a diagnosis of adult respiratory distress syndrome (ARDS) after a drug overdose. Positive end-expiratory pressure (PEEP) is initiated. Because the patient fights the ventilator, the physician orders vecuronium bromide. After administering the medication, it is MOST important for the nurse to take which of the following actions? 1. Administer analgesia as ordered. 2. Explain all procedures to the patient. 3. Maintain airborne precautions. 4. Administer complete eye care.

Question: What should the nurse do to prevent complications of vecuronium bromide? Strategy: Think Maslow. Needed Info: Adult respiratory distress syndrome (ARDS) is characterized by dyspnea and tachypnea followed by progressive hypoxemia despite oxygen therapy; positive end-expiratory pressure (PEEP): positive pressure is exerted during the expiratory phase of ventilation; vecuronium bromide (Norcuron) is a neuromuscular blocking agent used to provide skeletal relaxation during mechanical ventilation. (1) psychosocial; important because patient unable to communicate pain or discomfort; caring for eyes more important (2) psychosocial; client will be anxious due to mechanical ventilation; even though there is temporary paralysis due to drug, client is still able to hear; explain all care to client (3) physical; universal precautions used; no need for airborne (4) CORRECT - physical; client unable to blink due to vecuronium; eye care will prevent corneal abrasion

The patient is scheduled for a myelogram. The patient asks the nurse if there will be any discomfort during the test. Which of the following responses, if made by the nurse, is MOST accurate? 1. "No, this procedure will not hurt at all." 2. "Yes, this is one of the most painful procedures that you can have." 3. "This is an uncomfortable procedure, but you will receive general anesthesia so you will not be aware of the pain." 4. "This is an uncomfortable procedure, but you will be given medication before the test to lessen the discomfort."

Question: What should the nurse say about pain during a myelogram? Strategy: "MOST accurate" indicates that there may be more than one correct response. Needed Info: Myelogram: insertion of contrast medium into the subarachnoid space of spine via a lumbar puncture in order to visualize the vertebral column. Pretest: encourage fluids, check allergies. Antipsychotics, antidepressants, and anticoagulants may be withheld for several days. Diazepam (Valium) can be given during pre-op. Post-test: position the patient in a supine position with the head slightly elevated for several hours. (1) some discomfort involved (2) inaccurate and nontherapeutic (3) local anesthesia given to decrease discomfort. (4) CORRECT - usually given sedative

The nurse cares for the patient being treated for a myocardial infarction. The patient is receiving heparin 5,000 units subcutaneously every 12 hours. The nurse should assess the patient for which of the following? 1. Pallor or cyanosis. 2. Areas of ecchymosis and petechiae. 3. Varicose veins. 4. Edema and weight gain.

Question: What should you assess in a patient receiving heparin? Strategy: Think about the cause of each symptom and how it relates to heparin. Needed Info: Heparin: anticoagulant that inactivates thrombin and prevents the conversion of fibrinogen to fibrin. Side effects: hemorrhage, thrombocytopenia, hypersensitivity. Nursing responsibilities: check partial thromboplastin time (PTT) to monitor effect: 1.5 - 2 times control. Give subcutaneous into abdomen. Leave needle in place for 10 sec. Do not massage. Rotate sites. Never "piggyback" with other meds. Protamine sulfate is antagonist. Terminology: ecchymosis = bruise; petechiae = pinpoint hemorrhages; melena = black, tarry stool; epistaxis = nosebleed; hematuria = blood in urine. (1) unoxygenated blood in circulation (2) CORRECT - ecchymosis and petechiae can indicate hemorrhage; contact provider (3) incompetent valves appear as tortuous skin veins (4) not seen.

The nurse cares for the patient being treated for injuries sustained in a hunting accident. The patient has a tracheostomy tube in place. The nurse enters the patient's room and discovers that the tracheostomy tube has become dislodged and assesses that the patient is having difficulty breathing through the stoma. Which of the following actions does the nurse take FIRST? 1. Performs mouth to stoma breathing. 2. Hyperextends the patient's neck. 3. Places the patient in high-Fowler's position. 4. Administers oxygen.

Question: What should you do FIRST? Strategy: Remember your ABCs: airway, breathing, circulation. Needed Info: Tube extubation may occur during change of ties or coughing. (1) needs airway first (2) CORRECT - provides patent airway; call for help; place in semi-Fowler's (30 - 45 degrees), then check breath sounds; use hemostat to open airway (3) high-Fowler's position (90 degrees) is too upright; needs airway first (4) needs airway first.

The child is brought to the emergency department by the parents, who state the child fell off a bicycle. Upon examination, the nurse notes several bruises, lacerations, and burns in various stages of healing on the child's body, and the child is hypervigilant to touch. The nurse suspects child abuse. Which of the following statements MOST accurately reflects the nurse's responsibility in cases of suspected child abuse? 1. The nurse should not report child abuse without actual proof. 2. The nurse should report a case of suspected child abuse to proper authorities. 3. The nurse should not report suspected child abuse without discussing it with the child's parents first. 4. The nurse should confirm suspicions of child abuse with at least two other staff members before reporting it.

Question: What should you do if you suspect child abuse? Strategy: "Most accurate" indicates that there may be more than one correct response. Think about the outcome of each answer choice. Needed Info: Each state has laws that specify the individuals who are "mandated reporters." Nurses are in this category. The laws also direct the nurse to the relevant place to make a report. Ongoing education by hospitals makes this responsibility clear. (1) must report suspected cases (2) CORRECT - state law (3) inaccurate (4) inaccurate.

In the dining room of the mental health center, the nurse observes the formerly homeless and malnourished patient diagnosed with chronic schizophrenia putting food into a plastic bag. Which of the following actions, if taken by the nurse, is MOST appropriate? 1. Reprimand the patient immediately. 2. Ask the patient why the food is being put into a plastic bag. 3. Inform the patient that snacks will be available later. 4. Distract the patient and redirect to another activity.

Question: What should you do in this situation? Strategy: "MOST appropriate" indicates that there may be more than one correct response. Determine the outcome of each answer choice. Needed Info: The behavior of the patient is consistent with a distorted view of reality. Concern about food availability leads to storing a personal cache of food. Way of coping with fear. (1) judgmental (2) nontherapeutic; "why" questions make patient defensive, feel threatened (3) CORRECT - reality orientation; talk with patient in nonthreatening way about the patient's needs (4) misses opportunity to reality test and reorient.

The nurse cares for the patient with moderate hearing loss. The nurse teaches the patient's family to use which of the following approaches when speaking to the patient? 1. Raise your voice until the patient is able to hear you. 2. Face the patient and speak quickly using a high voice. 3. Face the patient and speak slowly using a slightly lowered voice. 4. Use facial expressions and speak as you would normally.

Question: What should you do to communicate with a person with a moderate hearing loss? Strategy: Think about the outcome of each answer choice. Needed Info: Presbycusis: age-related hearing loss due to inner ear changes; decreased ability to hear high sounds. (1) would result in high tones patient unable to hear (2) speech should be done slowly, not quickly, and usually unable to hear high tones (3) CORRECT - also decrease background noise; speak at a slow pace, use nonverbal cues (4) nonverbal cues help, but need low tones.

The nurse cares for the child newly diagnosed with epilepsy. Which of the following items does the nurse have available at the bedside? 1. Suction machine and oxygen setup. 2. Catheterization set. 3. Intermittent positive pressure breathing machine (IPPB). 4. Restraints

Question: What should you have at the bedside for a child with a history of seizures? Strategy: Remember your ABCs. Needed Info: Epilepsy: seizure disorder characterized by abnormal, recurring, excessive, and self-terminating electrical disturbances; Dx made after 2 or more seizures; strong genetic component. (1) CORRECT - remove secretions, provide patent airway, provide oxygenation (2) can be incontinent; not done (3) inflates lungs through positive pressure; not necessary (4) muscle contractions could cause fracture; not done.

The patient is admitted to the hospital for evaluation of a gangrenous right foot. A right below-the-knee amputation is scheduled. The patient asks, "Why can't they just amputate my foot instead of my leg?" Which of the following statements, if made by the nurse, is MOST accurate? 1. "It is necessary to have good circulation in your leg for healing to occur." 2. "It will be easier to fit you with a prosthesis." 3. "This is the best method to control the infection." 4. "This will prevent further circulatory problems in your leg."

Question: What should you say to the patient? Strategy: "MOST accurate" indicates that there may be more than one correct response. Needed Info: Most amputations of lower extremities are a result of peripheral vascular disease resulting from diabetes or cardiac disease. (1) CORRECT - amputation done at most distal point that will heal; the most critical factor is circulation in remainder of extremity (2) not accurate (3) not accurate (4) underlying disease will continue.

The nurse cares for the patient receiving morphine sulfate by use of a patient-controlled analgesia (PCA) pump. When making evening rounds, the nurse finds the patient sleeping and the spouse at the bedside. The nurse observes that each time the patient grimaces, the spouse presses the button on the PCA machine. Which of the following actions, if taken by the nurse, is MOST appropriate? 1. Encourage the spouse to continue this practice. 2. Explain to the spouse that this should be done only once every hour while the patient is sleeping. 3. Explain the purpose of the patient-controlled analgesia to the spouse. 4. Instruct spouse to awaken the patient when patient grimaces and ask if patient is having pain.

Question: What should you say to the spouse? Strategy: "Most appropriate" indicates that there may be more than one correct response. Think about why the nurse would make each statement. Needed Info: PCA allows patients to control own administration of IV analgesics. (1) patient should push button (2) inappropriate; patient should push button (3) CORRECT - include family in teaching; spouse's behavior could result in morphine overdose (4) inappropriate.

The patient is being treated for heart failure (HF) and is placed on a 2 gram sodium diet. The nurse performs dietary teaching. Which of the following statements, if made by the patient, indicates that further teaching is necessary? 1. "Some medications and seasonings such as MSG contain sodium." 2. "Milk, fish, and celery are foods naturally high in sodium." 3. "I need to avoid soups and seasoned rice when eating at a restaurant." 4. "I can eat any food I like as long as I don't add additional salt to my food."

Question: What statement is WRONG about a low-salt diet? Strategy: Be careful! This is a negative question. Three statements are correct, one is wrong. Needed Info: Foods to avoid on a low-sodium diet: cured or smoked meat or fish, Kosher meats, peanut butter, processed cheese, salted crackers, seasoning mixes, tomato juice, canned foods. (1) true statement; read labels (2) true statement; intake restricted (3) true statement; plain foods better; soups and seasoned rice may contain significant amount of sodium (4) CORRECT - incorrect info; sodium can be found in many foods; it is not just in table salt itself; 2 g mild sodium restriction

The patient suffers a cerebrovascular accident (stroke) in the left temporal lobe and is admitted to the hospital. When performing an assessment of the patient, the nurse expects some patient impairment in which of the following? 1. Control of the left arm. 2. Glucose metabolism. 3. Corneal reflex in both eyes. 4. Speech.

Question: What symptom would you expect in patient with a CVA? What does the temporal lobe control? Strategy: Think about what functions the temporal lobe controls. Needed Info: CVA or stroke: disruption in blood supply to brain. Causes: thrombus, embolus, or hemorrhage. Risk factors: hypertension, diabetes, heart disease, smoking, substance abuse, obesity, stress, lack of exercise, high cholesterol levels. Usually seen after age 65. S/S: aphasia (impairment in ability to communicate through speech), alexia (difficulty reading), agraphia (impairment in ability to write), HA, syncope, motor or sensory disturbances (paresthesia, paralysis). (1) unaffected; nerve fibers cross in spinal canal, result in disabilities on opposite (contralateral) side (2) symptom of diabetes (3) function of fifth cranial nerve (trigeminal); caused by brain stem disorders (4) CORRECT - left hemisphere controls speech, math skills, analytical thinking

The nurse plans discharge teaching for the client with coronary artery disease (CAD). The client will continue taking warfarin sodium at home. Which of the following instructions does the nurse include in the teaching? 1. Have complete blood count every 1 - 4 weeks. 2. Test stools daily for blood. 3. Wear a MedicAlert bracelet. 4. Stop taking the warfarin sodium before going to the dentist.

Question: What teaching should be done with a patient who is going to be discharged on warfarin? Strategy: Think about the outcome of each answer choice. Needed Info: Coronary artery disease (CAD): narrowing of coronary arteries due to atherosclerosis. Risk factors: hereditary, smoking, age, gender (men higher risk), race (white higher risk), hypertension, elevated serum cholesterol, diabetes mellitus. warfarin sodium (Coumadin): smoking increases required dose; flu vaccine enhances effect for 1 month; fever, prolonged hot weather enhances effect; high-fat diet decreases effect of medication. (1) should have PT check, not complete blood count, every 1 - 4 weeks; (2) no need to check this often; should observe for blood (3) CORRECT - provides for safety; also teach to use soft toothbrush, electric razor (4) should continue to take, but tell dentist.

The nurse cares for the patient admitted to the hospital reporting severe abdominal cramping and diarrhea. The nurse evaluates the effectiveness of the patient's intravenous therapy. The nurse recalls that which of the following laboratory tests BEST reflects hydration status? 1. Erythrocyte sedimentation rate (ESR). 2. White blood cell count (WBC). 3. Hematocrit (HCT). 4. Serum glucose.

Question: What test gives you the BEST indication of hydration status? Strategy: Think about what each value measures. How does it relate to hydration? (1) ESR: rate at which RBCs settle out of unclotted blood in 1 hour; indicates inflammation/necrosis; normal: men 0 - 15 mm/h, women 0 - 20 mm/h (2) WBC: indicates infection (normal 4,500 - 11,000/mm3); reduced: leukopenia, elevated: leukocytosis (3) CORRECT - percentage of red blood cells (RBCs) in the plasma; increased with dehydration, reduced with fluid volume excess; normal: men 40 - 54%, women 36 - 46%; other tests that indicate hydration: BP, urine specific gravity (normal: 1.005 - 1.030) (4) indicates insulin production (normal 60 - 110 mg/dL).

The nurse manager is evaluating care given by the staff of a medical/surgical unit. The nurse manager should intervene if which of the following is observed? 1. A nursing assistant disposes of a patient's used tissue in the bedside container before opening the roommate's milk carton. 2. A student nurse washes hands for 15 seconds after removing gloves following inserting an indwelling urinary catheter. 3. A nurse puts on a gown, gloves, mask, and goggles prior to inserting a nasogastric tube. 4. An LPN/LVN visits with a client diagnosed with methicillin-resistant staphylococcus aureus (MRSA) wound infection while the client eats lunch.

Question: What will cause the spread of infection? Strategy: "Should intervene" indicates an incorrect action. Needed Info: Standard precautions are used to prevent nosocomial infections; wash hands as soon as gloves are removed, between patient contacts, between procedures or tasks with same patient, when touching blood, body fluids, or contaminated surfaces; masks, goggles, and gown if in danger of splashes. (1) CORRECT - contaminated hands cause cross-infections; instruct family about when hand washing is necessary and the correct procedure (2) wash hands for at least 10 seconds after removing gloves after a procedure (3) appropriate technique; splashes may occur (4) requires contact precautions; client in isolation may develop sense of loneliness; visiting with client during meals increases sensory stimulation

The nurse cares for the patient who is receiving warfarin sodium. It is MOST important for the nurse to have which of the following medications available? 1. Ferrous sulfate. 2. Protamine sulfate. 3. Vitamin E. 4. Vitamin K.

Question: What will counteract the actions of warfarin sodium? Strategy: Think about the action of each drug. Needed Info: Should check for hematuria (blood in urine), tarry stools, ecchymosis, petechiae, epistaxis (nosebleed). (1) used for iron deficiency; side effects: nausea, constipation; dilute liquid preparations in water or juice, not milk or antacids; absorption impaired by yogurt, cheese, milk, cereals, coffee, tea, whole grain breads (2) heparin antagonist; 1 mg neutralizes 90 - 115 units; give slowly IV over 1 - 3 min (3) fat-soluble vitamin; used in premature infants and patients with impaired fat absorption (4) CORRECT - Phytonadione (vitamin K) (Mephyton): promotes hepatic formation of prothrombin; controls abnormal bleeding; antidote for warfarin sodium (Coumadin) overdosage; side effects: transient hypotension, bronchospasm, anaphylaxis; used in newborns to prevent and treat hemorrhagic disease of newborns

The nurse does preoperative teaching with the client scheduled to have a transurethral prostatectomy (TURP) under spinal anesthesia. Which of the following statements about the result of the spinal anesthesia does the nurse include in the teaching? 1. "You will be unable to move your arms or legs immediately after surgery." 2. "You will require analgesics to relieve pain in your back." 3. "You will be unable to move your legs immediately after surgery." 4. "You will require a special machine to help you breathe immediately after surgery."

Question: What will result from the spinal anesthesia? Strategy: Think about each answer. Is it expected with spinal anesthesia? Needed Info: Spinal anesthesia: injection into the subarachnoid space. Complications: N + V, HA, resp paralysis, muscular weakness in legs. TURP: removal of enlarged portion of the prostate by the use of a resectoscope inserted through the urethra. (1) arms not affected below T-4 (2) not common problem (3) CORRECT - impulses temporarily blocked; will return (4) awake during procedure; no airway problem.

The nurse cares for the patient postoperatively after a transurethral prostatectomy (TURP) for treatment of benign prostatic hypertrophy (BPH). The patient has a continuous bladder irrigation (CBI) through a three-way urinary catheter with a 30 mL balloon tip. When changing the patient's bed, the nurse notices that the sheets are wet. Which of the following BEST explains this finding? 1. The patient is experiencing acute urinary retention. 2. The patient is experiencing autonomic dysreflexia. 3. The patient has a urinary tract infection. 4. The patient is having bladder spasms.

Question: What would cause leaking of the catheter for this patient? Strategy: Think about how each answer relates to a catheter. Needed Info: BPH: enlargement of the prostate gland that obstructs the urethra. TURP: removal of enlarged portion of the prostate by the use of a resectoscope inserted through the urethra. No incision is made. During the first 24 - 48 hours post-op, a continuous bladder irrigation (CBI) using isotonic fluids (normal saline) is used to keep catheter patent and remove clots and sediment. It should be regulated to provide for clear or pink urine. Traction may be applied to the catheter (by the physician) and the cath tubing taped to the thigh or abdomen. This prevents hemorrhage by applying pressure to the blood vessels. The Foley is usually removed after 2 - 3 days. Teaching: may initially have burning on urination, frequency, dribbling. Force fluids to 3,000 mL/day. Avoid alcohol, spicy foods, strenuous activities for 2 - 3 weeks. (1) urine would not be passed (2) seen after spinal cord injury; caused by distended bladder or colon; causes SNS discharge; results in hypertension, bradycardia, HA (3) would have foul smell, increased temp (4) CORRECT - passing urine around cath; normal to feel urge to void

The nurse is working with the family of the client with Parkinson's disease. The nurse knows that the client has autonomic system dysfunction. Which of the following signs and symptoms are cause by that problem? 1. Diarrhea. 2. Postural hypotension. 3. Depression. 4. Rigidity.

Question: When the client's autonomic system does not work right, what is the effect in the client's body? Strategy: Think about each answer choice. Needed Info: Parkinson's disease (PD) is a progressive, degenerative neurological disorder characterized by tremor, muscle rigidity, bradykinesia, and postural instability. Three kinds of dysfunctions: motor, autonomic system, and cognitive/psychologic. (1) autonomic system problems cause constipation, not diarrhea (2) CORRECT - autonomic system problems cause postural hypotension (3) psychologic system problems cause depression (4) motor problems cause rigidity.

The patient is admitted to the hospital with a fractured right femur. The patient is placed in balanced suspension traction with a Thomas splint and Pearson attachment. The patient's nurse is teaching a student nurse about traction. The student nurse asks, "Where is the pulling force of the traction applied?" Which of the following responses by the nurse is MOST accurate? 1. "It is applied to the quadriceps muscle." 2. "It is applied to the bone distal to the fracture site." 3. "It is applied to the bone proximal to the fracture site." 4. "It is applied to the knee."

Question: Where is the force of the pull for a patient in balanced suspension traction for a fractured femur? Strategy: Review the concepts of traction and picture the traction equipment. Needed Info: Traction: pulling force on part of the body. Used to reduce, align, and immobilize fractures, and to relieve muscle spasms. Balanced suspension traction: exerts pull on affected part and supports extremity in hammock or splint; the splint is held in place by balanced weights attached to overhead bar. Traction provided by system of ropes, pulleys, and weights. Countertraction provided by patient's body weight. Pull of traction on extremity remains constant, despite changes in position. (1) applied to the bone, never muscle in skeletal traction (2) CORRECT - ensures proper alignment of bone fragments (3) inaccurate; would not provide proper alignment (4) inaccurate.

*The picture wouldn't upload to this side of the card, so it's on the answer side :( * The nurse performs a physical assessment on the adult with a history of a mitral murmur. Identify the area where the nurse places the stethoscope to auscultate the mitral murmur. 1. A 2. B 3. C 4. D

Question: Where should the nurse assess for a mitral valve murmur? Strategy: Examine the diagram carefully. Select the area to assess for a mitral valve murmur. Needed Info: The following anatomical landmarks are used to evaluate heart sounds. The Angle of Louis is located at the manubrial sternal junction at the second rib. The aortic and pulmonic areas are found at the second intercostal space. Erb's point is found at the third intercostal space. The mitral area is found at the fifth intercostal space at the left midclavicular line. The point of maximal impulse (PMI), or the impulse of the left ventricle, is felt most strongly on an adult at the left fifth intercostal space in the midclavicular line. (1) INCORRECT (2) INCORRECT (3) mitral valve murmur is assessed at the PMI (4) INCORRECT

The nurse cares for the unresponsive patient admitted to the intensive care unit with a suspected brain injury from a motorcycle accident. It is MOST important for the nurse to intervene if a staff member performs which of the following assessments of pupillary activity? 1. Doll's eye oculocephalic reflex. 2. Direct light response. 3. Conjugate gaze. 4. Corneal reflex.

Question: Which assessment is contraindicated for this client? Strategy: "Nurse to intervene" indicates an incorrect action. Needed Info: Brain injuries include concussion, contusion, laceration, and hematoma; evaluate level of consciousness, perform neurological assessment, elevate head of bed 30 degrees to decrease intracranial pressure, careful intake and output. (1) CORRECT - observe patient's eye movement as head is turned quickly from side-to-side; if eyes move in opposite direction from side to which head is turned, reflex is intact; contraindicated because it requires head to be turned from side-to-side; patient with suspected brain injury may also have a cervical spine injury (2) darken room; eyelid is held open with other eye covered; penlight is swung from patient's ear toward midline of face and shown directly into eye; pupil should constrict immediately; no reason to intervene (3) nurse holds one finger up and asks patent to follow it with eyes alone; nurse moves finger up, down, lateral, and oblique to evaluate if the patient's eyes track together to follow the finger; unable to perform assessment if client unconscious (4) cotton ball touched to cornea; an immediate blink reflex is normal: no contraindication.

The nurse is presented with a group of patients in the emergency department (ED). Which of the following clients does the nurse see FIRST? 1. The client who reports being raped 30 minutes ago and is exhibiting self-blame, anxiety and feelings of worthlessness. 2. The client who reports a miscarriage last evening and has spotting of blood on her underwear. 3. The client who told the family of intent to commit suicide and has easy access to a gun. 4. The client who witnessed a child stabbed to death and is experiencing anxiety and difficulty coping.

Question: Which client should the nurse see first? Strategy: Remember Maslow. Determine most unstable client. Needed Info: Clients with physical needs take priority over clients with psychosocial needs. (1) follow emergency room protocol, may include clothing, hair samples, NPO; focus on here and now; be alert for potential internal injuries, e.g., hemorrhage (2) after spontaneous abortion, scant, dark discharge may persist for 1 - 2 weeks; instruct client to report any heavy, profuse, or bright red bleeding (3) CORRECT - client has expressed intent, and the risk of danger is great due to the lethal weapon; nurse should see this patient first (4) client experiencing a situational crisis; important to focus on the here and now; help client to become aware of feelings and validate them.

The client with a history of cholelithiasis and recurrent urinary tract infections is admitted to the medical unit with reports of fatigue. A small lump is discovered in the client's neck and the physician orders diagnostic testing. The nurse recognizes that which of the following tests should be performed FIRST? 1. Cholecystogram. 2. Intravenous pyelogram (IVP). 3. Myelogram. 4. Thyroid scan.

Question: Which diagnostic test should be performed first? Strategy: Think about how each test is performed. Needed Info: Cholelithiasis is presence of stones in the gallbladder; symptoms include intolerance to fatty foods, indigestion, nausea, vomiting, flatulence, eructation, and severe pain in the upper right quadrant of the abdomen. (1) iodide-containing contrast medium used to visualize the gallbladder (2) radiopaque iodine contrast medium used to visualize kidneys, ureters, and bladder (3) contrast dye is introduced into spinal subarachnoid space, so spinal cord and nerve roots are outlined and dura mater distortions are also visible (4) CORRECT - must be performed before radiographic exams, which use contrast substances; these would interfere with the interpretation of the thyroid scan which measures uptake of radioactive iodine by the thyroid

The adolescent is brought to the emergency room with a compound fracture of the left femur. Vital signs are BP 80/60, pulse 120, respirations 26, temperature 99.0°F (37.2°C). The nurse expects the physician to initially order which of the following fluids? 1. D10 in water. 2. D5 in 0.45% NaCl. 3. Lactated Ringer's solution. 4. 0.45% NaCl.

Question: Which fluids are best? Strategy: Think about the action of each fluid. Needed Info: Hypovolemic shock occurs because bone is vascular; can rapidly develop; nursing considerations: immobilize joint below and above fracture; assess for S/S shock--tachycardia, hypotension, cool, clammy skin, cyanosis, restlessness, decreased alertness; administer large amounts of isotonic fluids. (1) need isotonic fluid, not hypertonic fluid (2) hypertonic fluid; need isotonic fluid (3) CORRECT - need isotonic fluid to restore circulating blood volume; may also use 0.9% NaCl (normal saline solution) (4) hypotonic solution; isotonic fluid is needed at this time.

The nurse changes the dressing of the woman who had a mastectomy two days ago. After the nurse removes the old dressing, the woman turns her head away. Which of the following nursing diagnoses is MOST appropriate? 1. Powerlessness. 2. Knowledge deficit. 3. Dysfunctional grieving. 4. Body image disturbance.

Question: Which nursing diagnosis is most appropriate? Strategy: "MOST appropriate" indicates that discrimination is required to answer the question. Needed Info: Stages of body image readjustment include psychological shock (denial and anger), withdrawal (passive and dependent), acknowledgment (beginning of grief process), and integration (integrate body changes into new image). (1) may feel powerless, but client is most likely reacting to change in body (2) no indication of knowledge deficit (3) no indication of dysfunctional grieving. (4) CORRECT - mastectomy may cause client to question her femininity, attractiveness, and self-esteem; encourage client to verbalize; encourage client to participate in planning of care

The nurse knows that it is important to identify patients at significant risk of developing a deep vein thrombosis (DVT). Which patient would the nurse assess as having the lowest risk profile? 1. A 67-year-old carpenter undergoing a left total knee replacement. 2. A 22-year-old woman who weighs 230 lbs and is 2 months pregnant with her second child. 3. A 44-year-old woman with ovarian cancer experiencing vomiting from chemotherapy. 4. A 50-year-old executive following removal of cataracts.

Question: Which patient is least likely to develop a deep vein thrombosis? Strategy: Think about risk factors. Needed Info: Deep vein thrombosis can cause pulmonary embolism; occurs in patients undergoing joint replacement, pregnancy, ulcerative colitis, heart failure, the immobilized patient, and patients with severe infections. To prevent, early ambulation, antithrombosis stockings, and anticoagulants are used. (1) age, surgery (up to 70% experience DVT as a complication), and immobility puts him at significant risk (2) obesity and pregnancy puts her at significant risk (3) cancer and dehydration put her at significant risk. (4) CORRECT - the patient with cataracts is not immobile after surgery, usually done in an outpatient setting

The nurse cares for patients on the surgical unit. When planning care, the nurse anticipates that which patient will have the MOST difficulty adjusting psychologically? 1. The 13-year-old girl who has a wart removed from her nose. 2. The 26-year-old man who has palliative surgery for stage 4 cancer of the pancreas. 3. The 42-year-old woman who has an elective hysterectomy. 4. The 60-year-old man who has a colostomy for severe diverticular disease.

Question: Which patient will feel most threatened by surgery? Strategy: "MOST difficulty" indicates that discrimination is required to answer the question. Needed Info: Stages of body image readjustment include psychological shock, withdrawal, acknowledgment, and integration. (1) adolescents fear being different from their peers; body image is a concern, which the surgery enhanced (2) CORRECT - average survival rate after diagnosis of cancer of the pancreas is 4.1 months; of all the patients, he is the only one facing a near-term death; the other patients derive some long-term benefit from their operations (3) loss of uterus will affect body image, but the client with the terminal illness has greater challenges (4) body image disturbance is common after colostomy, but the client will not have the debilitating and painful symptoms of his disease.

The nurse approaches the paranoid schizophrenic client on the psychiatric unit to perform an ordered venipuncture to obtain a blood specimen. The client becomes agitated and says to the nurse, "You pretend to take blood, but I know you really want to inject me with a poison that will kill me." Which of the following responses by the nurse is BEST? 1. "No, I do not want to kill you. Why do you think that drawing blood is going to kill you?" 2. "Calm down. I drew your blood last week and nothing bad happened to you, did it?" 3. "You sound frightened. The physician wants to ensure that your medications are working properly." 4. "Look, the tube is empty. I can't inject you with anything if the tube is empty."

Question: Which response is most therapeutic? Strategy: "BEST" indicates discrimination is required to answer the question. Needed Info: Delusions are persistent false beliefs; allow client to verbalize delusion, do not argue or try to convince client that delusion is not real. Point out feeling tone of delusion and provide activities to divert attention from delusion. (1) encourages discussion of delusion; directly counters client's perception; does not acknowledge underlying feelings (2) confrontational and challenging (3) CORRECT - acknowledges client's feelings; gives a clear matter-of-fact response directly related to the reason for the blood draw (4) responds to content of the delusion; assumes that client can engage in logical thinking.

The client scheduled for a cardiac catheterization says to the nurse, "I know you were in here when the doctor had me sign the consent form for the test. I thought I understood everything, but now I'm not so sure." Which of the following responses by the nurse is BEST? 1. "Why didn't you listen more closely?" 2. "You sound as if you would like to ask more questions." 3. "I'll get you a pamphlet about cardiac catheterization." 4. "That often happens when this procedure is explained to clients."

Question: Which response is most therapeutic? Strategy: "BEST" indicates that discrimination is required to answer the question. Needed Info: Informed consent is obtained by the individual who will perform the test; explanation of the test and expected results, anticipated risks, discomforts, potential benefits, and possible alternatives are discussed; consent can be withdrawn at any time. (1) "why" questions are nontherapeutic; does not respond to client's feelings or concerns; judgmental (2) CORRECT - directly responds to client's statement by paraphrasing; implies encouragement of expression of client's concern (3) may be helpful, but first the nurse needs to clarify the client's concerns by discussion (4) does convey acceptance and lets the client know that client's response is not abnormal, but response is closed and does not allow client to express feelings or concerns.

The nurse cares for the client diagnosed with genital herpes. After the client is informed of the diagnosis, the client begins crying. Which of the following responses by the nurse is BEST? 1. "We have support groups that may help you talk about some of your feelings." 2. "I see that you are upset. Share with me your thoughts." 3. "While herpes is a difficult disease, at least you don't have AIDS." 4. "I think the physician should explain more to you about genital herpes."

Question: Which response is most therapeutic? Strategy: "BEST" indicates that discrimination is required. Needed Info: Genital herpes: caused by herpes simplex virus; symptoms: painful, vesicular genital lesions and difficulty voiding; nursing care: offer emotional support, sitz baths, monitor Pap smears on a regular basis; treatment: Acyclovir. (1) passing the buck; nurse should acknowledge the client's feelings and allow the client to verbalize; support group may be suggested later (2) CORRECT - reflects the client's feelings and allows the client to verbalize concerns (3) minimizes the client's diagnosis and feelings (4) passing the buck; the nurse should explain the disease.

The nurse cares for the client receiving vincristine sulfate. The nurse recognizes it is MOST important to assess for which of the following? 1. Fatigue and nausea. 2. Polyphagia and polydipsia. 3. Paresthesia and difficulty with gait. 4. Diarrhea and alopecia.

Question: Which side effect poses a safety issue with the client? Strategy: "MOST important" indicates a priority question. Needed Info: Vincristine sulfate (Oncovin) is an antineoplastic agent; side effects include peripheral neuritis, loss of reflexes, bone marrow depression, alopecia, and GI symptoms; avoid IV infiltration and extravasation; check reflexes, motor and sensory function; allopurinol (Zyloprim) given to increase excretion and decrease buildup of uric acid. (1) does not cause fatigue; does cause nausea and vomiting; not the priority (2) symptoms of diabetes; not caused by vincristine (3) CORRECT - indicate a peripheral neuropathy; to ensure client safety, support client when ambulating (4) diarrhea can be symptomatically treated; alopecia does occur; reassure client that it is usually reversible.

The nurse teaches the parent of the young child who has recently been diagnosed as having epilepsy about the condition. Which of the following statements, if made by the parent, indicates that further teaching is necessary? 1. "Epilepsy does not affect my child's mental capacities." 2. "Grand mal seizures do not cause brain damage." 3. "Epilepsy is a form of mental illness." 4. "Epilepsy can be controlled with medication."

Question: Which statement is NOT TRUE about epilepsy? Strategy: Be careful! This is a NEGATIVE question. Needed Info: Epilepsy: seizure disorder, characterized by abnormal, recurring, excessive and self-terminating electrical disturbances; Dx made after 2 or more seizures; strong genetic component (1) true statement (2) true statement (3) CORRECT - electrical disturbance in brain; can be controlled by medication; not a form of mental illness (4) true statement.

The patient is admitted to the hospital with a diagnosis of carcinoma of the colon and undergoes a colon resection. Two days postoperatively, the patient becomes confused and agitated. It is determined that the patient is delirious. The nurse recalls that delirium is BEST described by which of the following statements? 1. Delirium is characterized by acute onset with symptoms lasting for hours or weeks. 2. Delirium is characterized by gradual onset with symptoms lasting for months or years. 3. Delirium is characterized by either acute or gradual onset with symptoms lasting from several months to several years. 4. Delirium is characterized by either acute or gradual onset with symptoms lasting for several days.

Question: Which statement is TRUE for delirium? Strategy: Think about each answer choice. Needed Info: Delirium: onset rapid, often at night; manifestations fluctuate over 24-hour period: awareness, orientation, recent memory, sleep/wake cycle disturbed; associated with illness or meds. Dementia: onset insidious, develops over years, not associated with physical illness. Alertness not impaired. Nursing responsibilities: ensure safety, meet patient's physical needs. (1) CORRECT; delirium has acute onset with symptoms lasting for hours or weeks (2) inaccurate; delirium has rapid onset (3) inaccurate; delirium has rapid onset and shorter duration of symptoms (4) inaccurate; delirium has rapid onset.

The client comes to the emergency department reporting chest pain that occurs nightly while the client is at rest. The client is diagnosed with resting angina. The nurse instructs the client about how to decrease the anginal attacks. Which of the following statements, if made by the client to the nurse, indicates that teaching is successful? 1. "I am going to sign up for meditation classes at the community center." 2. "I am going to take a brisk walk after dinner every night." 3. "I am going to take a part-time job at a day care center." 4. "I am going to take over-the-counter diet pills to lose weight."

Question: Which statement is correct? Strategy: Think about what the client's words mean. Needed Info: Angina is chest pain due to ischemia that does not cause permanent damage; symptoms include pain that may radiate down left arm; arm pain associated with stress, exertions, or anxiety; administer antianginal exercise program to reduce blood pressure and pulse rate; percutaneous transluminal coronary angioplasty (PCTA), coronary artery bypass graft surgery (CABG) may be performed. (1) CORRECT - will decrease and manage stress; focus on breathing will have calming effect on client as well as assist with oxygenation; will not produce chest pain, shortness of breath, or undue fatigue (2) should not engage in physical exercise for 2 hours after meals (3) may involve too much physical activity and stress (4) obesity is a risk factor of angina, but over-the-counter diet pills contain sympathomimetic substances that can increase heart rate.

The nurse reviews the charts of four antepartal women. The nurse recognizes that which woman is at MOST risk for having a child with a cleft lip and palate? 1. A 22-year-old Asian woman who is having a girl. 2. A 35-year-old African American woman who is having a boy. 3. A 25-year-old Native American woman who is having a boy. 4. A 40-year-old Caucasian who is having a girl.

Question: Which woman is at greatest risk for having a child with a cleft lip and cleft palate? Strategy: Think about each answer. Needed Info: Cleft lip: small or large fissure in facial process of upper lip or up to nasal septum, including anterior maxilla; cleft palate: midline, bilateral, or unilateral fissures in hard and soft palate. (1) individuals of Asian background are more likely than African Americans or Caucasians to have a child with cleft lip and palate; less likely to be found in a girl (2) African Americans are least likely to have a child with cleft lip and palate (3) CORRECT - Native Americans have the highest incidence of cleft lip and palate; males are more likely than females to have both (4) Caucasians are less likely to have a child with cleft lip and cleft palate; more common in males.

The woman delivers a 6 lb 10 oz infant. The mother observes the nurse in the delivery room place drops in the infant's eyes. The mother asks the nurse why this was done. Which of the following responses by the nurse is BEST? 1. "The drops will constrict your infant's pupils to prevent injury." 2. "The drops will remove mucus from your infant's eyes." 3. "The drops will prevent infections that might cause blindness." 4. "The drops will prevent neonatal conjunctivitis."

Question: Why are eye drops placed in a newborn's eyes? Strategy: "BEST" indicates that discrimination may be required to answer the question. Needed Info: Prophylactic care of newborn includes administration of antibiotic eye drops containing erythromycin and tetracycline. Eye irritation may occur, but it is not common and is self-limiting. (1) erythromycin or tetracycline do not cause miosis (2) does not remove mucus from baby's eyes (3) CORRECT - precaution against ophthalmia neonatorum (inflammation of the eyes due to gonorrheal or chlamydia infection) (4) conjunctivitis is inflammation of the conjunctiva.

The nurse at the community mental health center cares for the new client with a diagnosis of depression. The physician prescribes amitriptyline. One week after starting amitriptyline, the client reports to the nurse that there has been no improvement. Which explanation, if made by the nurse, is MOST accurate? 1. "It takes two to four weeks for the medication to work." 2. "You may need more medication." 3. "Your depression is probably deepening." 4. "This medication probably is not the right one for you."

Question: Why are the patient's symptoms not relieved? Strategy: "MOST accurate" indicates that there may be more than one correct response. Think about each statement. Is it true about amitriptyline? Needed Info: Amitriptyline (Elavil): tricyclic antidepressant; take full dose at bedtime; delay of 2 - 4 weeks before effects seen; side effects: drowsiness, dizziness, orthostatic hypotension, blurred vision, dry mouth, urinary retention, constipation, sweating; nursing responsibilities: monitor for risk of suicide. (1) CORRECT - broken down by liver; drowsiness precedes antidepressant effect by several weeks, so med may improve sleep patterns before other symptoms (2) too early to know (3) inaccurate (4) not nursing decision.

The patient is postoperative orthopedic surgery. The physician orders morphine sulfate to be administered using a patient-controlled analgesia (PCA) pump. The nurse checks the PCA pump to determine how many times the patient has triggered the system. Which of the following explanations BEST explains why the patent triggered the system 11 times but received only 6 injections? 1. The patient is developing an addiction to morphine. 2. The patient does not understand how to use the pump. 3. The patent is developing a tolerance to morphine. 4. The amount of narcotic prescribed is not controlling the patient's pain.

Question: Why did the patient receive only 6 injections of medication? Strategy: "Best" indicates that there may be more than one correct response. Think about the outcome of each answer choice. Needed Info: PCA allows patients to control administration of IV analgesics. Preloaded pump system administers preset amount of medication when button is pushed by patient. Predetermined lock-out time interval. Amount of medication is displayed on front of machine. Reduces pulmonary complications, and patient is more alert. (1) develops over long period (2) more likely is pressing button to get pain relief (3) develops over long period. (4) CORRECT - patient is pressing button before lock-out time has expired because is in pain; system keeps track of number of requests for medication

The patient with a fractured right femur has traction applied through the use of a Steinmann pin through the femur. Balanced suspension traction is used with a Thomas splint and a Pearson attachment. The nurse explains to the patient that the purpose of the pin is which of the following? 1. To maintain alignment of the fracture. 2. To hold the Thomas splint in place. 3. To hold the Pearson attachment in place. 4. To immobilize the femur.

Question: Why is a pin used for a fractured femur? Strategy: Visualize the type of traction described. Needed Info: Pin: inserted directly through skin into the bone. Nursing responsibilities: check skin for redness, odor, and drainage. Change dressing and clean with hydrogen peroxide and/or saline if ordered. (1) CORRECT - provides pull directly to bone; results in realignment of bone (2) splint elevated at 45 degree angle to bed; supports thigh (3) fastened to Thomas splint at knee joint; knee is flexed 45 degrees; lower leg lies in Pearson attachment parallel to bed (4) purpose of traction itself.

The nurse plans teaching for the client scheduled for an amniocentesis. It is MOST important for the nurse to include which of the following statements? 1. "The test assesses gestational age using the biparietal circumference." 2. "The test determines the gender of the baby." 3. "The test is used to detect possible birth defects." 4. "The test should not be completed if you have a history of previous miscarriages."

Question: Why is an amniocentesis done? Strategy: Think about each answer. Needed Info: Amniotic fluid is aspirated by needle inserted through the abdominal and uterine walls and is done after 14 weeks gestation to diagnose genetic disorders or neural tube defects; instruct client to empty bladder. (1) age determined by ordering a sonogram, not an amniocentesis; after 30 weeks, can assess the lecithin/sphingomyelin ratio to determine fetal lung maturity (2) can be done but that is not the primary reason (3) CORRECT - completed to determine genetic disorders or neural tube defects; takes 2 - 4 weeks to obtain results; complications include premature labor, infection, Rh isoimmunization (4) not a contraindication.

The nurse plans to administer furosemide 20 mg IV to the patient diagnosed with chronic renal failure. The nurse recalls that the PRIMARY purpose of this medication is which of the following? 1. To increase the blood flow to the renal cortex. 2. To decrease the circulatory blood volume. 3. To increase excretion of sodium and water. 4. To decrease the workload on the heart.

Question: Why is furosemide given to a patient diagnosed with chronic renal disease? Strategy: Think about the action of furosemide. Needed Info: Chronic renal failure is progressive, irreversible kidney injury caused by hypertension, diabetes mellitus, lupus erythematosus, and chronic glomerulonephritis; symptoms include anemia, acidosis, azotemia, fluid retention, and urinary output alterations; nursing care includes monitoring potassium levels, daily weight, intake and output, and diet teaching about regulating protein intake, fluid intake to balance fluid losses, and some restrictions of sodium and potassium. Furosemide (Lasix) is a potassium-wasting diuretic, which increases renal potassium excretion. Monitor blood pressure, serum electrolytes, weight, I + O. Do not give at bedtime. (1) Furosemide is a loop diuretic that inhibits sodium and chloride reabsorption at the proximal and distal tubules and the ascending loop of Henle (2) Furosemide used to treat fluid overload due to chronic renal failure (3) CORRECT - Furosemide is administered in order to augment the kidney's excretory functioning (4) correcting the fluid overload will decrease the workload on the heart, but the primary reason Furosemide is given to patient diagnosed with chronic renal failure is to augment the kidneys' excretory functioning.

The 65-year-old patient is recovering from a right below-the-knee amputation. The patient observes the "figure eight" bandage on the residual limb. The patient asks the nurse why the bandage is applied in this manner. Which of the following explanations, if made by the nurse, is the MOST important reason? 1. "It decreases the possibility of infection." 2. "It helps to minimize postoperative pain." 3. "It reduces the possibility of clot formation." 4. "It reduces postoperative swelling."

Question: Why is the bandage after a BKA applied in a figure eight? Strategy: Picture a BKA with the bandage. Needed Info: Pressure to an operative site reduces postoperative swelling. To promote a return to better circulation, the pressure bandage is changed at regular intervals. As secondary effects, this activity reduces pain caused by swelling and the possibility of clot formation in the residual limb. (1) antibiotics first 48 - 72 hours do this (2) not primary purpose; pain medications play this role (3) reducing the possibility of clot formation happens secondary to the primary purpose of reducing postoperative swelling. (4) CORRECT - hastens venous return, controls edema; must be worn at all times except bathing; must be removed and reapplied several times a day

One afternoon in the hospital day room, the nurse overhears the newly admitted patient with chronic schizophrenia say to another patient, "I hate you. Get away from me or I'll kill you." Which of the following interpretations of this behavior, by the nurse, is MOST justifiable? 1. The patient does not like the other patient. 2. The patient is angry. 3. The patient feels threatened. 4. The patient feels powerful.

Question: Why is the patient acting like this? Strategy: "MOST justifiable" indicates that there may be more than one correct response. Think about each answer choice and how it relates to schizophrenia. Needed Info: Chronic schizophrenia distorts the way a patient thinks, acts, expresses emotions, perceives reality, and relates to others. The patient can misinterpret what the other patient's behavior meant. (1) assumption; not most justifiable (2) assumption; not most justifiable (3) CORRECT - patient not in usual environment; patients with schizophrenia hear voices and have trouble interpreting reality (4) assumption and unlikely.

The client is admitted to the hospital reporting persistent lower back pain. The nurse puts the client in bed in semi-Fowler's position with the hips and knees moderately flexed. The nurse recognizes the PRIMARY rationale for this position is to accomplish which of the following? 1. Relieve tension at the lumbo-sacral region. 2. Maintain proper alignment of the vertebral joints. 3. Improve breathing for better oxygen supply to the sacral musculature. 4. Increase blood flow to the spinal cord.

Question: Why is this position used for this patient? Strategy: Picture the patient as described. Determine the outcome of each answer choice. Needed Info: Causes of low back pain: herniated nucleus pulposus, muscle sprain. S/S: knifelike pain, sensory changes. Diagnosis: CT scan or MRI. Treatment: muscle relaxants, NSAIDS, analgesics, heat, traction (separates vertebrae to relieve pressure on nerve), transcutaneous electrical nerve simulation (TENS), surgery. (1) CORRECT - knees flexed relieves pressure on sciatic nerve or disk (2) done in any position; does not relieve pain (3) done to increase comfort (4) no change.

Caring for an 18-month old toddler who has a blood lead level of 3mcb/dL. Which of the following actions should the nurse take

Recommend re-screening in 1 year

After being examined by a forensic nurse in the emergency department, a rape victim is prepared for discharge. Because of the nature of the attack, this client is at risk for post-traumatic stress disorder (PTSD). Which symptoms are associated with PTSD? Select all that apply.

Recurrent, intrusive recollections or nightmares Sleep disturbances Difficulty concentrating Clients with PTSD typically experience recurrent, intrusive recollections or nightmares, sleep disturbances, difficulty concentrating, chronic anxiety or panic attacks, memory impairment, and feelings of detachment or estrangement that destroy interpersonal relationships. Gingival and dental problems are associated with bulimia. Flight of ideas and unusual talkativeness are characteristic of the acute manic phase of bipolar disorder.

Sanguineous:

Red

The nurse is evaluating a postoperative client for infection. Which sign or symptom would be most indicative of infection?

Red, warm, tender incision

Which of the following nursing interventions would most likely facilitate effective communication with a hearing-impaired patient?

Reduce environmental noise and distractions before communicating.

A patient with a sudden onset of hearing loss tells the nurse that he would like to begin using hearing aids. The nurse understands that the health professional dispensing hearing aids would have what responsibility?

Refer the patient to his primary care physician.

A nurse is caring for a person who is delusional. What is important for the nurse to do while communicating with the client?

Reinforce reality for delusional statements.

A nurse is preparing a client's room to accommodate a visit from a spiritual counselor. Which of the following is a recommended practice?

Remove any unnecessary equipment.

A nurse is caring for a client with myasthenia gravis. The client is having difficulty forming words and his tone is nasal. Which of the following is an effective communication strategy for this client?

Repeat what the client has said to verify the meaning.

Assessing a client after administering epinephrine for anaphylactic reactions. Identify adverse effect of this med

Report of Chest pain

The nurse is giving instructions to a client who's going home with a cast on his leg. Which point is most critical?

Reporting signs of impaired circulation

During a health assessment, a woman age 49 years tells the nurse that she is "just so tired and has been having mood swings and hot flashes." Based on this information, the nurse would conduct a more thorough history and assessment of what body system?

Reproductive

A nurse is caring for a hospitalized client. What intervention can the nurse use to help the client continue normal spiritual practices?

Request dietary consultation for the client's dietary restrictions.

When the South Asian client arrives 25 minutes late to her appointment at the clinic, the nurse recognizes this as a sign of which of the following?

Respect

What are the primary nursing considerations when assisting with, or conducting, a physical assessment of the genitalia?

Respecting the client's privacy and modesty

Reviewing ABG values of a client. Client has a pH of 7.20, PaCO2 of 60 mmHg, and HCO 3 of 25 mEq/l

Respiratory acidosis

After assessing a client, a nurse documents the state of awareness as confused. What part of the brain controls awareness?

Reticular activating system

The nurse is caring for a client who suffered a stroke three days ago, and is assessing the client's state of arousal. The nurse knows that the part of the body responsible for a person being alert or aroused is which of the following?

Reticular activating system

A two-year-old child has tested positive for tuberculosis (TB), and has been started on rifampin. The child's parents ask the nurse if there is any important information they should know about this medication. What important adverse effect should the nurse inform these parents about?

Rifampin and its metabolites will turn urine, feces, sputum, tears, and sweat an orange color. This is not a serious adverse effect. Rifampin may also cause GI upset, headache, drowsiness, dizziness, visual disturbances, and fever. Liver enzyme and bilirubin levels increase because of hepatic metabolism of the drug. Parents should be taught the signs and symptoms of hepatitis and hyperbilirubinemia such as jaundice of the sclera or skin.

A client verbalizes to a mental health counselor that his life is meaningless since his wife divorced him and that he no longer wants to live. What nursing diagnosis, resulting from his spiritual distress, would be appropriate?

Risk for Self-Directed Violence

A woman aged 88 years who lives alone has deficits in vision and hearing, although these deficits are corrected by glasses and hearing aids. Her blood pressure medicine is making her dizzy. What response to these health problems would the home health nurse identify?

Risk for accidental injury

The nurse is planning the care of a patient with a diagnosis of vertigo. What nursing diagnosis risk should the nurse prioritize in this patient's care?

Risk for falls

Which of the following signs and symptoms is most indicative of Ménière disease?

Rotary vertigo and tinnitus

What can the nurse do to prevent lipodystrophy when administering insulin to a diabetic client?

Rotate the injection sites.

While reviewing the health history of an older adult experiencing hearing loss the nurse notes the patient has had no trauma or loss of balance. What aspect of this patient's health history is most likely to be linked to the patient's hearing deficit?

Routine use of quinine for management of leg cramps

A home care client has both visual and hearing deficits. Although all of the following are important, what would be a high priority concern when planning and implementing care?

Safety

A client has just returned to his room after undergoing exploratory abdominal surgery. The nurse notes watery red drainage on his dressing. The nurse will describe the drainage as:

Sanguineous

The nurse is providing information to a client who is taking chlorpromazine. What is the most important information for the nurse to provide?

Schedule routine medication checks. It is important to continually assess for adverse reactions and continued therapeutic effectiveness. The dosage should be changed if ordered by the primary care provider. While chlorpromazine can exacerbate serious sunburns, medication should not be discontinued without an order from the provider. Adverse reactions should be immediately reported to the provider.

Developing a client education program about osteoporosis for older adult clients

Sedentary lifestyle

Manager planning to use a democratic leadership style. Which of the actions demonstrates democratic leadership style?

Seeks input from the other nurses

An adolescent male tells the nurse that he is afraid his penis will be damaged because he masturbates every day. The nurse's response is based on what knowledge?

Self-stimulation is a normal activity.

A nurse is using inspection as an assessment technique. What does the nurse use during inspection?

Senses of vision, hearing, smell

A female patient with rheumatoid arthritis has taken high doses of aspirin for several years, a practice that has resulted in damage to her hearing due to the drug's ototoxic effects. What type of hearing loss has this patient experienced?

Sensorineural hearing loss

A patient has been diagnosed with hearing loss related to damage of the end organ for hearing or cranial nerve VIII. What term is used to describe this condition?

Sensorineural hearing loss

A client tells his nurse that he has difficulty hearing related to working in a loud factory setting for 15 years. What is the term for this condition?

Sensory deficit

A client with hearing loss gets very frustrated trying to carry on conversations with friends. Which type of stressor is the client experiencing?

Sensory deficits

The nurse is working in a clinic and sees a resident of a long-term-care facility, age 82 years, who has come in to be checked by her physician. The caregiver, who accompanies her, reports that the client has been displaying the following: drowsiness, excessive sleeping, decreased attention span, irritability, and signs of depression. The client's daughter and family, who usually visit her, moved away from the area six weeks ago due to the husband's job. You suspect which of the following problems?

Sensory deprivation

A client admitted to the hospital for major surgery is at risk for what type of sensory alteration?

Sensory overload

Caring for a school age child taking valporic acid. Diagnostic tests

Serum liver enzyme levels

Caring for a child who has hypotonic dehydration and is receiving an oral re-hydration solution. Lab results indicates treatment effective

Serum sodium 136 mEq/L

A nurse is responding to sexual harassment from a client at work. Which of the following is a recommended guideline for dealing with this behavior?

Set and enforce limits to the behavior and maintain boundaries.

The nurse is caring for a female age 45 years who discloses during the admission nursing history that she is no longer able to enjoy sex with her husband because it causes too much pain in her vagina. The nurse includes which of the following nursing diagnoses in the client's care plan related to this information?

Sexual Dysfunction: Dyspareunia

Providing discharge teaching for preschool age child with a new prescription for Amoxicillian/clavulanate suspension.

Shake the medication bottle well before each dose is given. Report diarrhea to the provider immediately. Store medication in refrigerator

A nurse meets his/her neighbor and new baby at the local market. The neighbor states that she received outstanding nursing care from one of the nurse's colleagues during her labor and childbirth. What is the best way for the nurse to recognize her nursing colleague's professional efforts?

Share the feedback with the nursing colleague directly.

The nurse is preparing to assess a client's cranial nerves. Which of the following techniques should you use to assess cranial nerve III?

Shine a bright light in the client's eye and observe for bilateral pupillary response.

The client in a rehabilitation unit is having a difficult time adjusting to the scheduled activities on the unit, as well as being dependent on others for meals and medications. Which word best describes what the patient is experiencing?

Shock

Teaching a client undergoing ECT about adverse effects

Short term memory loss

A patient with a diagnosis of retinal detachment has undergone a vitreoretinal procedure on an outpatient basis. What subject should the nurse prioritize during discharge education?

Signs and symptoms of increased intraocular pressure

A patient with otosclerosis has significant hearing loss. What should the nurse do to best facilitate communication with the patient?

Sit or stand in front of the patient when speaking.

The nurse is providing sexual education to a group of teenagers. One of the class members reports feeling sexually aroused when her boyfriend strokes her arms. The nurse points out that the human body has many erogenous zones, the largest being which of the following?

Skin

While conducting a health assessment with an older adult, the nurse notices it takes the person longer to answer questions than is usual with younger clients. What should the nurse do?

Slow the pace and allow extra time for answers.

A nurse is performing a preoperative assessment. Which client statement should alert her to the presence of risk factors for postoperative complications?

Smoking one pack of cigarettes per day reduces the activity of the cilia lining the respiratory tract, increasing the client's risk of ineffective airway clearance after surgery.

A nurse working in a clinic is planning to conduct vision screenings for a group of low-income women. What equipment would be needed to test vision?

Snellen chart

An older adult, newly widowed, has been unable to adjust to her change in roles or form new relationships. What is this experience called?

Social isolation

The mother of an infant with iron deficiency anemia asks the nurse what she could have done to prevent the anemia. The nurse should teach the mother that it is helpful to introduce solid foods into the infant's diet at age:

Solids should be introduced at 6 months. Full-term infants use up their prenatal iron stores within 4 to 6 months after birth. Milk contains insufficient iron.

A home health nurse is working with a patient who quit his job after injury- life has no meaning and lonely- everyone has left him- what is this a sign of:

Spiritual Distress

A client states that his life has meaning and purpose, he feels loved, and has experienced forgiveness in his life. What is the term that describes this state of spirituality?

Spiritual health

A client who is 10 weeks pregnant develops spotting; however, the cervix remains closed. The nurse should suspect which of the following?

Spotting in the first trimester may indicate that the pregnancy is in jeopardy. Bed rest and avoidance of physical and emotional stress are recommended. Abortion is usually inevitable if the bleeding is accompanied by pain with dilation and effacement of the cervix. An inevitable abortion is associated with cervical dilation. An ectopic pregnancy is in the fallopian tubes, and a false positive pregnancy could reflect a missed abortion.

A nurse observes an area on the client's skin and documents: 9 x 12mm shallow abrasion at 2 o'clock position of coccyx; scant amount of clear drainage; mild erythema; no ecchymosis or odor. Which action should the nurse take? A. Leave wound open to air B. Begin negative pressure wound therapy C. Apply moist saline dressing to wound D. Place a dry sterile gauze dressing over the wound

Stage 2 ulcer C. Apply moist saline dressing to wound

A patient has undergone diagnostic testing and has been diagnosed with otosclerosis? What ear structure is primarily affected by this diagnosis?

Stapes

Nurse preceptor is evaluating the performance of newly licensed nurse. which action should the preceptor intervene on

Starts a task by determining what supplies are needed. Should gather prior to beginning task for effective time management.

The registered nurse taking shift report learns that an assigned patient is blind. How should the nurse best communicate with this patient?

State her name and role immediately after entering the patient's room.

A client has been receiving oxytocin to augment her labor. The nurse notes that contractions are lasting 100 seconds. Which immediate action should the nurse take?

Stop the oxytocin infusion

A charge nurse enters a client's room and observes a physician instructing another nurse on how to insert a central line into the client's neck. The nurse is holding the cannula and inserting the line. What would be the appropriate response by the charge nurse?

Stop the procedure and inform the nurse that he/she is practicing outside of a nurse's legal scope of practice.

When taking a dietary history from a newly admitted client, the nurse should remember that which of the following foods is a common allergen?

Strawberries

The daughter of an older adult calls the nurse practitioner to report that her mother is becoming very confused after dark. What is this type of confusion named?

Sundowning syndrome

Planning care for a client with left sided weakness following a stroke

Support the client's left arm on a pillow while sitting

A patient is exploring treatment options after being diagnosed with age-related cataracts that affect her vision. What treatment is most likely to be used in this patient's care?

Surgical intervention

School nurse is notified of an emergency which several children were injured following the collapse of playground equipment.

Survey the scene for potential hazards to staff and children.

The nurse is providing discharge education for a patient with a new diagnosis of Ménière's disease. What food should the patient be instructed to limit or avoid?

Sweet pickles

A nurse is monitoring a client for adverse reactions to atropine eyedrops. Systemic absorption of atropine sulfate through the conjunctiva can cause which adverse reaction?

Systemic absorption of atropine can cause tachycardia, palpitations, flushing, dry skin, ataxia, and confusion. To minimize systemic absorption, the client should apply digital pressure over the punctum at the inner canthus for 2 to 3 minutes after instilling the drops. The drug also may cause dry mouth. It isn't known to cause hypotension or apnea.

A nurse asks the mother of a hospitalized client age 2 years to bring in his favorite blanket or stuffed animal. What sense is the nurse trying to stimulate with this intervention?

Tactile

A client with rheumatoid arthritis reports GI irritation after taking piroxicam (Feldene). To prevent GI upset, the nurse should provide which instruction?

Take piroxicam with food or an antacid.

A client's left leg is in skeletal traction with a Thomas leg splint and Pearson attachment. Which intervention should the nurse include in this client's care plan?

Teach the client how to prevent problems caused by immobility.

What do most nursing interventions pertaining to sexuality involve?

Teaching to promote sexual health

A nurse is caring for a client after internal fixation of a compound fracture in the tibia. The nurse finds that the client has not had his dinner, seems restless, and is tossing on the bed. Keeping in mind that the client is Latino, what is the most appropriate response by the nurse?

Tell me what you are feeling.

A client has been hospitalized with a diagnosis of myasthenia gravis. A friend is visiting the client during lunch. The nurse enters the room after the client recovered from choking on lunch. What should the nurse do next?

Tell the client to swallow when her chin is tipped down on her chest.

Caring for a client who is receiving positive end expiratory pressure via mechanical ventilation. Adverse effects of PEEP

Tension pneumothorax

A client is admitted to the hospital for diagnostic testing to rule out colorectal cancer. Which intervention should the nurse include on the plan of care?

Test all stools for occult blood.

A hospitalized patient with impaired vision must get a picture in his or her mind of the hospital room and its contents in order to mobilize independently and safely. What must the nurse monitor in the patient's room?

That all furniture remains in the same position

Rn observing a LPN and a AP move a client up in bed. When should the nurse intervene

The LPN and the AP grasp the client under his arms to life him up in bed.

A registered nurse (RN) is supervising an unlicensed assistive personnel (UAP). Which principle would the nurse follow when delegating tasks?

The RN delegates a task based on the UAP's skill set

The nurse is providing teaching about accidental poisoning to the family of a 3-year-old. The nurse understands that a child of this age is at increased risk of accidental ingestion due to which sensory alteration? a. A lack of fully developed hearing. b. A less discriminating sense of touch. c. Visual acuity that has not fully developed. d. A less discriminating sense of taste.

The answer is D. Less discriminating sense of taste. The young preschooler may have a less discriminating sense of taste than the older child, making him or her at increased risk for accidental ingestion. A less discriminating sense of touch and developing visual acuity would not increase the risk. Hearing is intact at birth and it does not increase the child's risk for accidental ingestion.

Assessing a preschooler with facial laceration. Identify potential indication of sexual abuse

The child exhibits discomfort while sitting.

Inpatient mental health unit is monitoring a visit between client with aggressive behavior and the client's partner

The client is pacing around the chair in which his partner is sitting.

A nurse is counseling a female victim of sexual assault. Which of the following statements accurately describes the increased risks for this client?

The client is three times more likely to suffer from depression.

Assessing client who is 11 wks gestation and reports drinking ginger tea. signs of it's effectiveness

The client reports a decrease in episodes of nausea

Caring for a client who is 2 days postpartum. Which behaviors should indicate the client is bonding with newborn

The client tells visitors how much the newborn looks like her sister.

A client who's paralyzed on the left side has been receiving physical therapy and attending teaching sessions about safety. Which behavior indicates that the client accurately understands safety measures related to paralysis?

The client uses a mirror to inspect the skin.

A client is being admitted with a nursing home-acquired pneumonia. The unit has four empty beds in semiprivate rooms. The room that would be most suitable for this client is the one with a:

The client with a possible transient ischemic attack is the only client who has not had surgery and is not immunocompromised.

The mother of a 3-year-old is concerned about her child's speech. She describes her preschooler as hesitating at the beginning of sentences and repeating consonant sounds. What is the nurse's best response? a. Hesitancy and dysfluency are normal during this period of development. b. Reading to the child will help model appropriate speech. c. Expressive language concerns warrant a developmental evaluation. d. The mother should ask her child's physician for a speech therapy evaluation.

The correct response is A. Preschoolers often have a period of dysfluency and hesitancy in their language development, but it usually resolves by about age 4 years.

. What is the best advice about nutrition for the toddler? a. Encourage cup drinking and give water between meals and snacks. b. Encourage unlimited milk intake, because toddlers need the protein for growth. c. Avoid sugar-sweetened fruit drinks and allow as much natural fruit juice as desired. d. Allow the toddler unlimited access to the sippy cup to ensure adequate hydration.

The correct response is A. The toddler should wean to the cup by age 12 to 15 months. Limit real fruit juice to 4 to 6 oz per day and milk to 16 to 24 oz per day, and give it with meals and snacks. Offer water between meals and snacks.

A 9-month-old infant's mother is questioning why cow's milk is not recommended in the first year of life as it is much cheaper than formula. What rationale does the nurse include in her response? a. It is permissible to substitute cow's milk for formula at this age as he is so close to 1 year old. b. Cow's milk is poor in iron and does not provide the proper balance of nutrients for the infant. c. As long as the mother provides whole milk, rather than skim, she can start cow's milk in infancy. d. If the mother cannot afford the infant formula, she should dilute it to make it last longer.

The correct response is B. Although whole milk contains a sufficient quantity of fat, cow's milk is inappropriate for use in infancy as it does not provide the appropriate balance of nutrients (especially iron) and may overload the infant's renal system with inappropriate amounts of sodium, protein, and minerals. Cow's milk use should be delayed until 1 year of age. Infant formula should always be reconstituted according to the manufacturer's recommendations.

To gain cooperation from a toddler, what is the best approach by the nurse? a. Immediately pick the toddler up from the mother's lap. b. Kneel in front of the toddler while he or she is on the mother's lap. c. Do the nursing tasks quickly so the toddler can play. d. Ask the toddler if it is okay if you begin the needed task.

The correct response is B. Being at the toddler's level and allowing the toddler to stay with his mother allow him to feel more secure. If the toddler perceives the nurse to be nonthreatening, the nurse is more likely to gain cooperation. Toddlers ordinarily answer "no," so asking the toddler's permission is not helpful. Simply jumping in and starting the task without allowing the toddler to warm up will threaten the child.

The father of a 2-month-old girl is expressing concern that his infant may be getting spoiled. The nurse's best response is: a. "She just needs love and attention. Don't worry; she's too young to spoil." b. "Consistently meeting the infant's needs helps promote a sense of trust." c. "Infants need to be fed and cleaned; if you're sure those needs are met, just let her cry." d. "Consistency in meeting needs is important, but you're right, holding her too much will spoil her."

The correct response is B. Infants need to have both their physical needs (e.g., feeding, changing, clothing) and their emotional needs (e.g., attention, holding) met consistently so that they can develop a sense of trust, which is the basis for the later development of self-esteem.

The nurse is caring for a hospitalized 30-month-old who is resistant to care, is angry, and yells "no" all the time. The nurse identifies this toddler's behavior as a. problematic, as it interferes with needed nursing care. b. normal for this stage of growth and development. c. normal because the child is hospitalized and out of his routine.

The correct response is B. Negativism is characteristic of the stage, no matter what the situation (hospital or home).

The successful resolution of developmental tasks for the school-age child, according to Erikson, would be identified by: a. Learning from repeating tasks b. Developing a sense of worth and competence c. Using fantasy and magical thinking to cope with problems d. Developing a sense of trust

The correct response is B. School-age children develop a sense of worth and competence. Toddlers learn from repeating tasks. Preschoolers use fantasy and magical thinking. It is in infancy that the child develops a sense of trust.

The mother of a 4-year-old asks for advice on using time-out for discipline with her child. What advice should the nurse give the mother? a. If spanking is not working, then time-out is not likely to be helpful either. b. Place the child in time-out for 4 minutes. c. Use time-out only if removing privileges is unsuccessful. d. The child should stay in time-out until crying ceases.

The correct response is B. The generally accepted guidelines recommend keeping the child in time-out for 1 minute per year of age.

What has the most influence in deterring an adolescent from beginning to drink alcohol? a. Drinking habits of parents b. Drinking habits of peers c. Drinking philosophy of adolescent's culture d. Drinking philosophy of adolescent's religion

The correct response is B. The teenager's peer group has the greatest influence on his or her behavior. If peers are drinking, the teenager will be at greater risk of drinking.

The mother of two sons, ages 6 and 9, states they want to play on the same baseball team. As the school nurse, what advice would you give their mother? a. Having the boys on the same team will make it more convenient for the mother. b. Levels of coordination and concentration differ, so the boys need to be on different teams. c. Put the boys on the same team because they are both school-age children. d. It is best to avoid putting the boys on the same team to prevent sibling rivalry.

The correct response is B. With age, concentration and coordination increase, so the 9-year-old would be operating at a higher level of maturity.

Parents of an 8-month-old girl express concern that she cries when left with the babysitter. How does the nurse best explain this behavior? a. Crying when left with the sitter may indicate difficulty with building trust. b. Stranger anxiety should not occur until toddlerhood; this concern should be investigated. c. Separation anxiety is normal at this age; the infant recognizes parents as separate beings. d. Perhaps the sitter doesn't meet the infant's needs; choose a different sitter.

The correct response is C. As the infant realizes she is separate from her parents, it may distress her when the parents leave, as she understands they are no longer with her.

. The nurse is caring for a hospitalized 4-year-old who insists on having the nurse perform every assessment and intervention on her imaginary friend first. She then agrees to have the assessment or intervention done to herself. The nurse identifies this preschooler's behavior as: a. Problematic; the child is old enough to begin to have a basis in reality. b. Normal, because the child is hospitalized and out of her routine. c. Normal for this stage of growth and development. d. Problematic, as it interferes with needed nursing care.

The correct response is C. Imaginary friends help the preschooler cope with stress in his or her life.

The nurse is providing anticipatory guidance to the mother of a 6-month-old infant. What is the best instruction by the nurse in relation to the infant's oral health? a. "Start brushing her teeth after all the baby teeth come in." b. "Use a washcloth with toothpaste to clean her mouth." c. "Clean your baby's gums, then new teeth, with a washcloth." d. "Rinse your baby's mouth with water after every feeding."

The correct response is C. The infant's mouth should be cleansed with a damp washcloth as should the baby's new teeth. It is important to clean the mouth and the teeth in order to prevent dental caries. Toothpaste is unnecessary in infancy. Rinsing the infant's mouth would present a safety hazard.

The mother of a 15-month-old is concerned about a speech delay. She describes her toddler as being able to understand what she says, sometimes following commands, but using only one or two words with any consistency. What is the nurse's best response to this information? a. The toddler should have a developmental evaluation as soon as possible. b. If the mother would read to the child, then speech would develop faster. c. Receptive language normally develops earlier than expressive language. d. The mother should ask her child's physician for a speech therapy evaluation.

The correct response is C. Young toddlers understand far more language than they can actually express themselves.

The mother of a 3-month-old boy asks the nurse about starting solid foods. What is the most appropriate response by the nurse? a. "It's okay to start puréed solids at this age if fed via the bottle." b. "Infants don't require solid food until 12 months of age." c. "Solid foods should be delayed until age 6 months, when the infant can handle a spoon on his own." d. "The tongue extrusion reflex disappears at age 4 to 6 months, making it a good time to start solid foods."

The correct response is D. As the tongue extrusion reflex disappears, the infant is better able to accept the spoon and learn to take solid foods.

A 2-year-old is having a temper tantrum. What advice should the nurse give the mother? a. For safety reasons, the toddler should be restrained during the tantrum. b. Punishment should be initiated, as tantrums should be controlled. c. The mother should promise the toddler a reward if the tantrum stops. d. The tantrum should be ignored as long as the toddler is safe.

The correct response is D. Ignoring tantrums is the best method for discouraging them. Any additional attention received because of the outburst may only contribute to another occurrence in the future.

As the school nurse conducting screening for vision in a 6-year-old child, you would refer the child to a specialist if the visual acuity in both eyes is: a. 20/20 b. 20/25 c. 20/30 d. 20/50

The correct response is D. Visual acuity of 20/20 is not expected until 7 years of age.

When giving parents guidance for the adolescent years, the nurse would advise the parents to: (Choose all that apply.) a. Accept the adolescent as a unique individual b. Provide strict, inflexible rules c. Listen and try to be open to the adolescent's views d. Screen all of his or her friends e. Respect the adolescent's privacy f. Provide unconditional love

The correct responses are A, C, E, and F. Adolescents need to be accepted as unique individuals. Parents should provide unconditional love, respect their privacy, and listen to them. Screening all of their friends and providing strict, inflexible rules would only lead to poor interactions between the parents and the adolescent.

Which of the following are reasons that stealing occurs in school-age children? (Choose all that apply.) a. To escape punishment b. High self-esteem c. Low expectations of family/peers d. Lack of sense of property e. Strong desire to own something

The correct responses are A, D, and E. Stealing in the school-age years occurs for multiple reasons: to escape punishment, because of a lack of sense of propriety or ownership, and because of a strong desire to own something they do not have because of lack of money or refusal by parents. Stealing also occurs when a school-age child has low self-esteem and high expectations from his family or peers that the child cannot meet.

Which is associated with early adolescence? (Choose all that apply.) a. Uses scientific reasoning to solve problems b. Still at times wants to be dependent upon parents c. Incorporates own set of morals and values d. Is influenced by peers and values memberships in cliques

The correct responses are B and D. During early adolescence (11 to 14 years of age), adolescents are in conflict over becoming independent from their parents. They still at times want the role of the parents to be as it was during the school-age years. They are influenced by peers and value membership in cliques. Adolescents develop scientific reasoning and incorporate their own set of morals and values in middle and late adolescence.

In developing a weight-loss plan for an adolescent, which would the nurse include? (Choose all that apply.) a. Have parents make all of the meal plans. b. Eat slowly and place the fork down between each bite. c. Have the family exercise together. d. Refer to an adolescent weight-loss program. e. Keep a food and exercise diary.

The correct responses are B, C, D, and E. These are steps that promote weight loss in adolescents. Adolescents want to be involved in the process, so having parents make all of the meal plans would not promote acceptance by the adolescent.

Which activities will promote weight loss in an obese school-age child? (Choose all that apply.) a. Unlimited computer and TV time b. Role modeling by family c. Becoming active in sports d. Eating unstructured meals e. Involving child in meal planning and grocery shopping f. Drinking three glasses of water per day

The correct responses are B, C, and E. Inactivity while watching TV and playing computer games leads to weight gain. Sports facilitate the expenditure of calories while the child is participating in age-appropriate activities; unstructured mealtimes lead to increased consumption of fast foods and decreased role modeling by parents. The family needs to eat regular meals. Involving the child in meal preparation and grocery shopping facilitates interest in the weight-loss project. Intake of water increases metabolism and creates a feeling of fullness.

A new client has come to the clinic wanting a method of birth control. The client asks about a diaphragm. What would the nurse teach this client about a diaphragm?

The diaphragm must be used during each episode of sexual activity.

An older adult woman of Chinese ancestry refuses to eat at the nursing home, stating, "I'm just not hungry." What factors should the staff assess for this problem?

The food served may not be culturally appropriate.

A patient diagnosed with arthritis has been taking aspirin and now reports experiencing tinnitus and hearing loss. What should the nurse teach this patient?

The hearing loss will likely resolve with time after the drug is discontinued.

A home health nurse is visiting a client 60 years of age. During the initial visit, the client's husband answers all of the questions. What would the nurse assess based on this behavior?

The husband is the dominant member of the family.

Which of the following occurs in the male during the resolution phase of the sexual response cycle?

The male experiences a period during which he is incapable of sexual response.

A 4-year-old child is admitted for a cardiac catheterization. Which is most important to include as the nurse teaches this child about the cardiac catheterization?

The most important aspect of teaching a preschooler is to have the family members there for support. Preschoolers are able to understand information that is individualized to their level.

The nurse is caring for a child whose mother is deaf and untrusting of staff. She frequently cries at the bedside, but refuses intervention from social work or the chaplain. Which issue is most important for the nurse to address with the mother to promote a trusting relationship?

The mother's fear that the staff do not respect her

The home health nurse is making an initial home visit to a male widow age 76 years. During the assessment the nurse finds that the client is taking multiple medications. The client states that he has also been taking some herbal remedies. What should the nurse be sure to include in the client education?

The need to inform his physician and pharmacist about the herbal remedies

The nurse is teaching a patient to care for her new ocular prosthesis. What should the nurse emphasize during the patient's health education?

The need to perform thorough hand hygiene before handling the prosthesis

A client who fell through ice and was submerged for longer than 1 minute is admitted to the emergency department with hypothermia and near-drowning. At which point will the nurse best be able to determine the client's outcome status?

The neural or hemodynamic status of the client cannot be determined until the client is warmed.

Assessing a 2 wk old newborn with a birth weight of 3.64 kg being breastfed. Indicators of effective breastfeeding

The newborn has 6 to 8 wet diapers per day

When providing care on an Indian reservation, the nurse has prioritized assessments for diabetes and fetal alcohol syndrome when working with residents of the reservation. How should this nurse's practice be best understood?

The nurse is correct in assessing for health problems that have a higher incidence and prevalence among this population.

A nurse is sitting near a client while conducting a health history. The client keeps edging away from the nurse. What might this mean in terms of personal space?

The nurse is in the client's personal space.

A nurse in a large metropolitan city enjoys working in a health clinic that primarily serves Hispanic clients. What does this statement imply about the nurse?

The nurse respects and values providing culturally competent care.

The nurse has entered a client's room after receiving a morning report. The nurse rapidly assessed the client's airway, breathing, and circulation and greeted the client by saying "Good morning." The client has made no reciprocal response to the nurse. How should the nurse best respond to the client's silence?

The nurse should ask appropriate questions to understand the reasons for the client's silence.

Checking for Chvostek's sign

The nurse should assess for a Chvostek's sign by tapping the client's facial nerve about an inch in front of the tragus of the ear. Facial twitching is a positive finding that indicates hypocalcemia.

Which instruction about levothyroxine administration should a nurse teach a client?

The nurse should instruct the client to take levothyroxine on an empty stomach (to promote regular absorption) in the morning (to help prevent insomnia and to mimic normal hormone release).

A client scheduled for a total laryngectomy and radical neck dissection begins talking rapidly, commenting, "I'm really nervous and scared about the operation." What is the most therapeutic action by the nurse?

The nurse should listen attentively and provide realistic verbal reassurance.

An infant's mother thinks there may be something wrong with him because "he spits up so much." What further information should the nurse obtain?

The nurse would need to determine the frequency, consistency, characteristics, and volume of emesis; the volume and frequency of feeding; the adequacy of burping; and growth parameters.

Which desired outcome demonstrates effective parent teaching about disciplining a toddler?

The parents will call immediate attention to undesirable behavior. Explanation: Calling immediate attention to undesirable behavior reflects effective teaching. This approach helps the child learn socially acceptable behavior and maintain self-esteem and a positive self-concept while learning to adapt to the rules of the larger group and society. Rules should be established clearly and enforced consistently. To reinforce desirable behavior, parents should voice requests for behavior in positive terms and use a normal speaking voice and tone when talking to or reprimanding the child. Screaming and shouting should be minimized.

A patient with low vision has called the clinic and asked the nurse for help with acquiring some low-vision aids. What else can the nurse offer to help this patient manage his low vision?

The patient has diabetes.

A nurse is planning preoperative teaching for a patient with hearing loss due to otosclerosis. The patient is scheduled for a stapedectomy with insertion of a prosthesis. What information is most crucial to include in the patient's preoperative teaching?

The patient is likely to experience resolution of conductive hearing loss after the procedure.

A patient has been diagnosed with serous otitis media for the third time in the past year. How should the nurse best interpret this patient's health status?

The patient needs to be assessed for nasopharyngeal cancer.

A patient with glaucoma has presented for a scheduled clinic visit and tells the nurse that she has begun taking an herbal remedy for her condition that was recommended by a work colleague. What instruction should the nurse provide to the patient?

The patient should discuss this new remedy with her ophthalmologist promptly.

An advanced practice nurse has performed a Rinne test on a new patient. During the test, the patient reports that air-conducted sound is louder than bone-conducted sound. How should the nurse best interpret this assessment finding?

The patient's hearing is likely normal.

A child who is of preschool age is diagnosed as having severe autism. The most effective therapy involves which intervention?

The preschool-aged child with severe autism will benefit from one-on-one play therapy. The therapist can develop a rapport with this child with nonverbal play. Antipsychotic medications are not indicated for the autism client. The child has difficulty with interpersonal relationships; therefore, group psychotherapy and social skills groups would not be effective.

The nurse is discussing the results of a patient's diagnostic testing with the nurse practitioner. What Weber test result would indicate the presence of a sensorineural loss?

The sound is heard better in the ear in which hearing is better.

Preparing to transfer a client from ICU to the medical floor. Client was recently weaned from mechanical ventilation follow a pneumonectomy. Information to include in change of shift report

The time of the client's last dose of pain medication. So the receiving nurse can anticipate what time to give the next dose.

When assessing a toddler's growth and development, the nurse understands that a child in this age-group displays behavior that fosters which developmental task?

The toddler's developmental task is to achieve autonomy while overcoming shame and doubt. Developing initiative is the preschooler's task whereas developing trust is the infant's task. Developing industry is the task of the school-age child.

The nurse is providing care for a client who has a sacral pressure ulcer with a wet-to-dry dressing. Which guideline is appropriate for a wet-to-dry dressing?

The wound should remain moist from the dressing.

The focal point of nursing is the nurse-client interaction. What must nurses consider when conducting the necessary assessment of their clients and significant others?

Their own cultural orientation.

While caring for an older adult male, the nurse observes that his skin is dry and wrinkled, his hair is gray, and he needs glasses to read. Based on these observations, what would the nurse conclude?

These are normal physiologic changes of aging.

A client scheduled for complex heart surgery has been reading the Bible for hours each day, cries often, and is not sleeping well. What might these observations cue the nurse about the client?

These behaviors are signs of spiritual distress.

A patient is postoperative day 6 following tympanoplasty and mastoidectomy. The patient has phoned the surgical unit and states that she is experiencing occasional sharp, shooting pains in her affected ear. How should the nurse best interpret this patient's complaint?

These pains are an expected finding during the first few weeks of recovery.

A patient comes to the ophthalmology clinic for an eye examination. The patient tells the nurse that he often sees floaters in his vision. How should the nurse best interpret this subjective assessment finding?

This is a normal aging process of the eye.

A 40-year-old nurse is taking a health history from a Hispanic man aged 20 years. The nurse notes that he looks down at the floor when he answers questions. What should the nurse understand about this behavior?

This is culturally appropriate behavior.

Nurse administers incorrect dose of medication. Which facts related to incident report should the nurse document in the client's medical record?

Time the medication was given

Caring for a client who asks about taking ginseng to improve her appetite. Identify that ginseng can decrease the effectiveness of which medication.

Timolol

Which use of restraints in a school-age child should the nurse question?

To substitute for observation

A 60-year-old patient's long history of poorly controlled hypertension has culminated in a diagnosis of retinal detachment. What type of retinal detachment is this patient most likely to have experienced?

Traction retinal detachment

Caring for a client who has hyperthyroidism

Tremors

A nurse in the hospital is caring for a Native American male. What person is most important to include in the care of the client?

Tribal medicine man

Can you use essential oils for Asthma: True or False

True

Series of "small successes is positive way to help clients True or False

True

A middle adult client requests visits by the hospital chaplain and reads the Bible each day while hospitalized for treatment of heart problems. What is the individual illustrating?

Trust in spiritual strength

A client with schizophrenia believes his room is bugged by the Central Intelligence Agency (Canadian Security Intelligence Service) and that his roommate is a foreign spy. The client has never had a romantic relationship, has no contact with family members, and has not been employed in the past 14 years. Based on Erikson's theories, the nurse should recognize that this client is in which stage of psychosocial development?

Trust versus mistrust. This client's paranoid ideation indicates difficulty trusting others. The stage of autonomy versus shame and doubt deals with separation, cooperation, and self-control. Generativity versus stagnation is the normal stage for this client's chronologic age. Integrity versus despair is the stage for accepting the positive and negative aspects of one's life, which would be difficult or impossible for this client.

A school nurse is preparing to test the auditory function of grade school students. What equipment will be needed for this examination?

Tuning fork

Providing teaching for teaching for a client who has a fracture of the right fibula with a short leg cast in place. Client is non weight bearing for 6 weeks.

Use a three-point gait

What intervention is recommended to reduce sensory stimulation for infants in the neonatal ICU?

Use limited light

Providing teaching to a client who speaks a different language than the nurse about an upcoming diagnostic procedure.

Use pictures to illustrate the procedure to the client

A nurse is preparing to administer an IM injection to a client who is obese

Use the ventrogluteal site because it has a thick area of muscle and contains no large nerves or blood vessels.

The nurse is caring for a client who speaks Chinese, and the nurse does not speak Chinese. An appropriate approach for communication with this client includes what?

Using a caring voice and repeating messages frequently

During a class for 5th- and 6th-grade girls about menstruation, one student comments that she has heard that girls smell bad during their menses. Other students chime in saying they have heard the same thing and ask how to prevent odors. The nurse correctly answers with which of the following solutions?

Utilize good hygiene and regular bathing

A heterosexual couple enjoys both anal and vaginal intercourse. What may result from these sexual activities?

Vaginal infections

When providing nursing care to an African American individual, which of the following cultural factors should the nurse consider?

Values and beliefs are often present oriented.

A nurse is weaning a client from mechanical ventilation. Which assessment finding indicates the weaning process should be stopped?

Ventricular tachycardia indicates that the client isn't tolerating the weaning process. The weaning process should be stopped before lethal ventricular arrhythmias occur. A respiratory rate of 16 breaths/minute and an oxygen saturation of 93% are normal findings. Although the client's blood pressure has increased, it hasn't increased more than 20% over baseline, which would indicate that the client isn't tolerating the weaning process.

Preparing to administer a blood transfusion to a client . Ensure proper client identification

Verify the client and blood product information with another licensed nurse

Hospice nurse consulting about receiving home services

We can expect the hospice nurse to provide support for us after our mother's death

Discharge instructions to client to received home oxygen therapy

Wear clothing made with cotton fabrics while oxygen is in use.

A community health nurse is conducting a seminar on vision self-care. What might be one topic included in the education plan?

Wear sunglasses when working outside.

A child goes to the school nurse and complains of not being able to hear the teacher. What test could the school nurse perform that would preliminarily indicate hearing loss?

Whisper test

During a preparation for parenting class, one of the participants asks the nurse, "How will I know if I am really in labor?" What should the nurse tell the participant about true labor contractions?

With true labor, the contractions are felt first in the lower back and then the abdomen. They gradually increase in frequency and duration and do not disappear with ambulation, rest, or sleep. In true labor, the cervix dilates and effaces. Walking tends to increase true contractions. False labor contractions disappear with ambulation, rest, or sleep. False labor contractions commonly remain the same in duration and frequency. Clients who are experiencing false labor may have pain, even though the contractions are not very effective.

A nurse is assessing a client who is receiving clozapine. The nurse reviews the medical record. What should the nurse do next?

Withhold the clozapine, and notify the health care provider (HCP). Because clozapine can cause tachycardia, the nurse should withhold the medication if the pulse rate is greater than 140 bpm and notify the HCP. Giving the drug or telling the client to exercise could be detrimental to the client.

People in crisis - can they work through crisis if someone works with them

Yes

Is it important for a nurse to consider a client's culture when providing physical care?

Yes, culture dictates the amount of sensory stimulation considered normal.

Caring for a client who recently signed informed consent to donate a kidney to her sibling in ERF. States "I don't want my brother to die, but what if I need this kidney one day?"

You afraid that you other kidney will fail at some point after the organ donation

Teaching for client about right to confidentiality

You can provide a list of family members who can receive information about the diagnosis.

Provide teaching to client 24 wks gestation scheduled for glucose tolerance test

You will need to fast the night before the test.

Client teaching about the basal body temp method for birth control

Your body temperature might decrease slightly just prior to ovulation

The wife of a 67-year-old client who has been taking imipramine (Tofranil) for 3 days asks the nurse why her husband isn't better. The nurse should tell the wife: a) "It takes 2 to 4 weeks before the full therapeutic effects are experienced." b) "Your husband may need an increase in dosage." c) "A different antidepressant may be necessary." d) "It can take 6 weeks to see if the medication will help your husband."

a - Imipramine, a tricyclic antidepressant, typically requires 2 to 4 weeks of therapy before the full therapeutic effects are experienced. Because the client has been taking the drug for only 3 days, it is too soon to determine if the current dosage of imipramine is effective. It is also too soon to consider taking another antidepressant.

A client with a bleeding ulcer is vomiting bright red blood. The nurse should assess the client for which of the following indicators of early shock? a) Tachycardia. b) Dry, flushed skin. c) Increased urine output. d) Loss of consciousness.

a - In early shock, the body attempts to meet its perfusion needs through tachycardia, vasoconstriction, and fluid conservation. The skin becomes cool and clammy. Urine output in early shock may be normal or slightly decreased. The client may experience increased restlessness and anxiety from hypoxia, but loss of consciousness is a late sign of shock.

Total parenteral nutrition (TPN) is prescribed for a client who has recently had a significant small and large bowel resection and is currently not taking anything by mouth. The nurse should: a. administer TPN through a nasogastric or gastrostomy tube b. handle TPN using strict aseptic technique c. auscultate for bowel sounds prior to administering TPN d. designate a peripheral intravenous (IV) site for TPN administration

a - TPN is hypertonic, high-calorie, high-protein, intravenous (IV) fluid that should be provided to clients without functional gastrointestinal tract motility, to better meet their metabolic needs and to support optimal nutrition and healing. TPN is ordered once daily, based on the client's current electrolyte and fluid balance, and must be handled with strict aseptic technique (because of its high glucose content, it is a perfect medium for bacterial growth). Also, because of the high tonicity, TPN must be administered through a central venous access, not a peripheral IV line. There is no specific need to auscultate for bowel sounds to determine whether TPN can safely be administered

A nurse is caring for a client with poorly managed diabetes mellitus who has a serious foot ulcer. When she informs him that the physician has ordered a wound care nurse to examine his foot, the client asks why he should see anyone other than this nurse. He states, "It's no big deal. I'll keep it covered and put antibiotic ointment on it." What is the nurse's best response? a) "We're very concerned about your foot and we want to provide the best possible care for you." b) "This is a big deal and you need to recognize how serious it is." c) "This is the physician's recommendation. The wound care nurse will see you today." d) "You could lose your foot if you don't see the wound care nurse."

a - The client's response indicates that he's in denial and needs further insight and education about his condition. Letting the client know that the nurse has his best interests in mind helps him accept the wound-care nurse. Although telling the client that his condition is serious and that the wound care nurse will see him that day are true statements, they're much too direct and may increase client resistance. Telling the client he could lose his foot is inappropriate and isn't therapeutic communication.

A client is scheduled for surgery under general anesthesia. The night before surgery, the client tells the nurse, "I can't wait to have breakfast tomorrow." Based on this statement, which nursing diagnosis should be the nurse's priority? a) Deficient knowledge related to food restrictions associated with anesthesia b) Fear related to surgery c) Risk for impaired skin integrity related to upcoming surgery d) Ineffective coping related to the stress of surgery

a - The client's statement reveals a Deficient knowledge related to food restrictions associated with general anesthesia. Fear related to surgery, Risk for impaired skin integrity related to upcoming surgery, and Ineffective coping related to the stress of surgery may be applicable nursing diagnoses but they aren't related to the client's statement.

A child, age 3, is brought to the emergency department in respiratory distress caused by acute epiglottiditis. Which clinical manifestations should the nurse expect to assess? a. severe sore throat, drooling, and inspiratory strider b. low grade fever, stridor, and a barking cough c. pulmonary congestion, a productive cough, and a fever d. sore throat, a fever, and general malaise

a - a child with acute epiglottiditis appears acutely ill and clinical manifestations may include drooping (because of difficulty swallowing), severe sore throat, hoarseness, a high temperature, and severe inspiratory stridor. A low grade fever, stridor, and barking cough that worsens at night are suggestive of croup. Pulmonary congestion, productive cough, and fever along with nasal flaring, retractions, chest pain, dyspnea, decreased breath sounds, and crackles indicate pneumococcal pneumonia. A sore throat, fever, and general malaise point to viral pharyngitis.

A nurse is caring for a client with lower back pain who is scheduled for myelography using metrizamide (a water-soluble contrast dye). After the test, the nurse should place the client in which position? a. head of the bed elevated 45 degrees b. prone c. supine with feet raised d. supine with the head lower than the trunk

a - after a myelogram, positioning depends on the dye injected. When a water-soluble dye such as metrizamide in injected, the head of the bed is elevated to a 45-degree angle to slow the upward dispersion of the dye. The prone and supine positions are contraindicated when a water-soluble contrast dye is used. The client should be positioned supine with the head lower than the trunk after an air-contrast study.

when caring for a client after a closed renal biopsy, the nurse should: a. maintain the client on strict bed rest in a supine position for 6 hours b. insert an indwelling catheter to monitor urine output c. apply a sandbag to the biopsy site to prevent bleeding d. administer IV opioid medications to promote comfort

a - after a renal biopsy, the client is maintained on strict bed rest in a supine position for at least 6 hours to prevent bleeding. If no bleeding occurs, the client typically resumes general activity after 24 hours. Urine output is monitored, but an indwelling catheter is not typically inserted. A pressure dressing is applied over the site, but a sandbag is not necessary. Opioids to control pain would not be anticipated; local discomfort at a biopsy site can be controlled with analgesics.

A nurse is documenting a variance that has occurred during the shift, and this report will be used for quality improvement to identify high-risk patterns and potentially initiate in-services programs. This is an example of which type of report? a. incident report b. nurse's shift report c. transfer report d. telemedicine report.

a - an incident report, also termed a variance report or occurrence report, is a tool healthcare agencies use to document anything out of the ordinary that results in or has the potential to result in harm to a client, employee, or visitor. These reports are used for quality improvement and not for disciplinary action. They are a means of identifying risks and high-risk patterns and initiating in-service programs to prevent further problems. A nurse's shift report is given by a primary nurse to the nurse replacing him or her by the charge nurse to the nurse who assumes responsibility for continuing client care. A transfer report is a summary of a client's condition and care when transferring clients from one unit or institution to another. A telemedicine report can link healthcare professional immediately and enable nurses to receive and give critical information about clients in a timely fashion.

the comatose victim of the car accident is to have a gastric lavage. Which of the following positions would be most appropriate for the client during this procedure? a. lateral b. supine c. trendelenburg's d. lithotomy

a - an unconscious client is best positioned in a lateral or semiprone position because these positions allow the jaw and tongue to fall forward, facilitate drainage of secretions, and prevent aspiration. Positioning the client supine carries a major risk of airway obstruction from the tongue, vomit, or nasopharyngeal secretions. Trendeleburg's position, with the head lower than the heart, decreases effective lung volume and increases the risk of cerebral edema. The lithotomy position has no purpose in this situation.

The nurse is assessing a client at her postpartum checkup 6 weeks after a vaginal delivery. The mother is bottle feeding her baby. Which client finding indicates a problem at this time? a. firm fundus at the symphysis b. white, thick vaginal discharge c. striae that are silver in color d. soft breasts without milk

a - by 4 to 6 weeks postpartum, the fundus should be deep in the pelvis and the size of a non-pregnant uterus. Subinvolution, caused by infection or retained placental fragments, is a problem associated with a uterus that is larger than expected at this time. Normal expectations include a white, thick vaginal discharge, striae that are beginning to fade to silver, and breasts that are soft without evidence of milk production (in a bottle feeding mother).

A client with type 1 diabetes must undergo bowel resection in the morning. How should the nurse proceed while caring for him on the morning of surgery? a. administer half of the client's typical morning insulin dose as ordered b. administer an oral antidiabeteic agent as ordered c. administer an I.V. insulin infusion as ordered d. administer the client's normal daily dose of insulin as ordered

a - if the nurse administers the client's normal daily dose of insulin while he's on nothing-by-mouth status before surgery, he'll experience hypoglycemia. Therefore, the nurse should administer half the daily insulin dose as ordered. Oral antidiabetic agents aren't effective for type 1 diabetes I.V. insulin infusions aren't necessary to manage blood glucose levels in clients undergoing routine surgery.

The best indicator that the client has learned how to give an insulin self-injection correctly is when the client can: a. perform the procedure safely and correctly b. critique the nurse's performance of the procedure c. explain all of the steps of the procedure correctly d. correctly answer a post-test about the procedure

a - the nurse should judge that learning has occurred from evidence of a change I the client's behavior. A client who performs a procedure safely and correctly demonstrates that he has acquired a skill. Evaluation of this skill acquisition requires performance of that skill by the client with observation by the nurse. The client must also demonstrate cognitive understanding, as shown by the ability to critique the nurse's performance. Explaining the steps demonstrates of knowledge at the cognitive level only. A post-test does not indicate the degree to which the client has learned a psychomotor skill.

Glucose GH challenge

a 2 hr oral glucose tolerance test (OGTT) with 75-100 gm glucose - a sample is taken before the test begins and again 30-60 minutes after drinking the glucose solution (test takes up to 3 hrs) - High levels may be r/t Cushing's Syndrome or diabetes (normal GH <2 ng/ml)

A client has been taking furosemide for 2 days. The nurse should assess the client for:

a decreased potassium level.

myxedema

a severe form of hypothyroid disease where the brain is unable to control crucial processes like breathing - in some cases, myxedema is fatal

A college student was referred to the campus health service because of difficulty staying awake in class. What should be included in the nurse's assessment? Select all that apply: a) Amount of sleep he usually obtains during the week and on weekends b) How much alcohol he usually consumes c) Onset and duration of symptoms d) Whether or not his classes are boring e) What medications including herbal remedies, he is taking

a) Amount of sleep he usually obtains during the week and on weekends c) Onset and duration of symptoms e) What medications, including herbal remedies, he is taking

The nurse is performing a dressing change for a client and notices that there is a new area of skin breakdown near the site of the dressing. On closer examination, it appears to be caused from the tape used to secure the dressing. This would be an example of which phase of the nursing process: a) Assessment b) Diagnoses c) Implementation d) Evaluation

a) Assessment

Which of the following client statements indicates that the client with hepatitis B understands discharge teaching? a) "I will not drink alcohol for at least 1 year." b) "I must avoid sexual intercourse." c) "I should be able to resume normal activity in a week or two. d) "Because hepatitis B is a chronic disease, I know I will always be jaundiced."

a) CORRECT ANSWER "I will not drink alcohol for at least 1 year." Reason: It is important that the client understand that alcohol should be avoided for at least 1 year after an episode of hepatitis. Sexual intercourse does not need to be avoided, but the client should be instructed to use condoms until the hepatitis B surface antigen measurement is negative. The client will need to restrict activity until liver function test results are normal; this will not occur within 1 to 2 weeks. Jaundice will subside as the client recovers; it is not a permanent condition.

A nurse preceptor is working with a student nurse who is administering medications. Which statement by the student indicates an understanding of the action of an antacid? a) "The action occurs in the stomach by increasing the pH of the stomach contents and decreasing pepsin activity." b) "The action occurs in the small intestine, where the drug coats the lining and prevents further ulceration." c) "The action occurs in the esophagus by increasing peristalsis and improving movement of food into the stomach." d) "The action occurs in the large intestine by increasing electrolyte absorption into the system that decreases pepsin absorption."

a) CORRECT ANSWER "The action occurs in the stomach by increasing the pH of the stomach contents and decreasing pepsin activity." Reason: The action of an antacid occurs in the stomach. The anions of an antacid combine with the acidic hydrogen cations secreted by the stomach to form water, thereby increasing the pH of the stomach contents. Increasing the pH and decreasing the pepsin activity provide symptomatic relief from peptic ulcer disease. Antacids don't work in the large or small intestine or in the esophagus.

A client is scheduled for an excretory urography at 10 a.m. An order directs the nurse to insert a saline lock I.V. device at 9:30 a.m.. The client requests a local anesthetic for the I.V. procedure and the physician orders lidocaine-prilocaine cream (EMLA cream). The nurse should apply the cream at: a) 7:30 a.m. b) 8:30 a.m. c) 9 a.m. d) 9:30 a.m.

a) CORRECT ANSWER 7:30 a.m. Reason: It takes up to 2 hours for lidocaine-prilocaine cream (EMLA cream) to anesthetize an insertion site. Therefore, if the insertion is scheduled for 9:30 a.m., EMLA cream should be applied at 7:30 a.m. The local anesthetic wouldn't be effective if the nurse administered it at the later times.

Which of the following statements would provide the best guide for activity during the rehabilitation period for a client who has been treated for retinal detachment? a) Activity is resumed gradually, and the client can resume her usual activities in 5 to 6 weeks. b) Activity level is determined by the client's tolerance; she can be as active as she wishes. c) Activity level will be restricted for several months, so she should plan on being sedentary. d) Activity level can return to normal and may include regular aerobic exercises.

a) CORRECT ANSWER Activity is resumed gradually, and the client can resume her usual activities in 5 to 6 weeks. Reason: The scarring of the retinal tear needs time to heal completely. Therefore, resumption of activity should be gradual; the client may resume her usual activities in 5 to 6 weeks. Successful healing should allow the client to return to her previous level of functioning.

A nurse is caring for a client who has a tracheostomy and temperature of 103° F (39.4° C). Which intervention will most likely lower the client's arterial blood oxygen saturation? a) Endotracheal suctioning b) Encouragement of coughing c) Use of a cooling blanket d) Incentive spirometry

a) CORRECT ANSWER Endotracheal suctioning Reason: Endotracheal suctioning removes secretions as well as gases from the airway and lowers the arterial oxygen saturation (SaO2) level. Coughing and using an incentive spirometer improve oxygenation and should raise or maintain oxygen saturation. Because of superficial vasoconstriction, using a cooling blanket can lower peripheral oxygen saturation readings, but SaO2 levels wouldn't be affected.

The nurse is serving on the hospital ethics committee which is considering the ethics of a proposal for the nursing staff to search the room of a client diagnosed with substance abuse while he is off the unit and without his knowledge. Which of the following should be considered concerning the relationship of ethical and legal standards of behavior? a) Ethical standards are generally higher than those required by law. b) Ethical standards are equal to those required by law. c) Ethical standards bear no relationship to legal standards for behavior. d) Ethical standards are irrelevant when the health of a client is at risk.

a) CORRECT ANSWER Ethical standards are generally higher than those required by law. Reason: Some behavior that is legally allowed might not be considered ethically appropriate. Legal and ethical standards are often linked, such as in the commandment "Thou shalt not kill." Ethical standards are never irrelevant, though a client's safety or the safety of others may pose an ethical dilemma for health care personnel. Searching a client's room when they are not there is a violation of their privacy. Room searches can be done with a primary health care provider's order and generally are done with the client present.

Which nursing action is required before a client in labor receives epidural anesthesia? a) Give a fluid bolus of 500 ml. b) Check for maternal pupil dilation. c) Assess maternal reflexes. d) Assess maternal gait.

a) CORRECT ANSWER Give a fluid bolus of 500 ml. Reason: One of the major adverse effects of epidural administration is hypotension. Therefore, a 500-ml fluid bolus is usually administered to prevent hypotension in the client who wishes to receive an epidural for pain relief. Assessing maternal reflexes, pupil response, and gait isn't necessary.

A client is prescribed metaproterenol (Alupent) via a metered-dose inhaler, two puffs every 4 hours. The nurse instructs the client to report adverse effects. Which of the following are potential adverse effects of metaproterenol? a) Irregular heartbeat. b) Constipation. c) Pedal edema. d) Decreased pulse rate.

a) CORRECT ANSWER Irregular heartbeat. Reason: Irregular heartbeats should be reported promptly to the care provider. Metaproterenol (Alupent) may cause irregular heartbeat, tachycardia, or anginal pain because of its adrenergic effect on beta-adrenergic receptors in the heart. It is not recommended for use in clients with known cardiac disorders. Metaproterenol does not cause constipation, pedal edema, or bradycardia.

Which of the following should the nurse use to determine achievement of the expected outcome for an infant with severe diarrhea and a nursing diagnosis of Deficient fluid volume related to passage of profuse amounts of watery diarrhea? a) Moist mucous membranes. b) Passage of a soft, formed stool. c) Absence of diarrhea for a 4-hour period. d) Ability to tolerate intravenous fluids well.

a) CORRECT ANSWER Moist mucous membranes. Reason: The outcome of moist mucous membranes indicates adequate hydration and fluid balance, showing that the problem of fluid volume deficit has been corrected. Although a normal bowel movement, ability to tolerate intravenous fluids, and an increasing time interval between bowel movements are all positive signs, they do not specifically address the problem of deficient fluid volume.

Prochlorperazine (Compazine) is prescribed postoperatively. The nurse should evaluate the drug's therapeutic effect when the client expresses relief from which of the following? a) Nausea. b) Dizziness. c) Abdominal spasms. d) Abdominal distention.

a) CORRECT ANSWER Nausea. Reason: Prochlorperazine is administered postoperatively to control nausea and vomiting. Prochlorperazine is also used in psychotherapy because of its effects on mood and behavior. It is not used to treat dizziness, abdominal spasms, or abdominal distention.

A nurse takes informed consent from a client scheduled for abdominal surgery. Which of the following is the most appropriate principle behind informed consent? a) Protects the client's right to self-determination in health care decision making. b) Helps the client refuse treatment that he or she does not wish to undergo. c) Helps the client to make a living will regarding future health care required. d) Provides the client with in-depth knowledge about the treatment options available.

a) CORRECT ANSWER Protects the client's right to self-determination in health care decision making. Reason: Informed consent protects the client's right to self-determination in health care decision making. Informed consent helps the client to refuse a treatment that the client does not wish to undergo and helps the client to gain in-depth knowledge about the treatment options available, but the most important function is to encourage shared decision making. Informed consent does not help the client to make a living will.

A client has the following arterial blood gas values: pH, 7.30; PaO2, 89 mm Hg; PaCO2, 50 mm Hg; and HCO3-, 26 mEq/L. Based on these values, the nurse should suspect which condition? a) Respiratory acidosis b) Respiratory alkalosis c) Metabolic acidosis d) Metabolic alkalosis

a) CORRECT ANSWER Respiratory acidosis Reason: This client has a below-normal (acidic) blood pH value and an above-normal partial pressure of arterial carbon dioxide (PaCO2) value, indicating respiratory acidosis. In respiratory alkalosis, the pH value is above normal and the PaCO2 value is below normal. In metabolic acidosis, the pH and bicarbonate (HCO3-) values are below normal. In metabolic alkalosis, the pH and HCO3- values are above normal.

A 10-year-old child diagnosed with acute glomerulonephritis is admitted to the pediatric unit. The nurse should ensure that which action is a part of the child's care? a) Taking vital signs every 4 hours and obtaining daily weight b) Obtaining a blood sample for electrolyte analysis every morning c) Checking every urine specimen for protein and specific gravity d) Ensuring that the child has accurate intake and output and eats a high-protein diet

a) CORRECT ANSWER Taking vital signs every 4 hours and obtaining daily weight Reason: Because major complications — such as hypertensive encephalopathy, acute renal failure, and cardiac decompensation — can occur, monitoring vital signs (including blood pressure) is an important measure for a child with acute glomerulonephritis. Obtaining daily weight and monitoring intake and output also provide evidence of the child's fluid balance status. Sodium and water restrictions may be ordered depending on the severity of the edema and the extent of impaired renal function. Typically, protein intake remains normal for the child's age and is only increased if the child is losing large amounts of protein in the urine. Checking urine specimens for protein and specific gravity and daily monitoring of serum electrolyte levels may be done, but their frequency is determined by the child's status. These actions are less important nursing measures in this situation.

A nurse has received change-of-shift-report and is briefly reviewing the documentation about a client in the client's medical record. A recent entry reads, "Client was upset throughout the morning." How could the charting entry be best improved? a) The entry should include clearer descriptions of the client's mood and behavior. b) The entry should avoid mentioning cognitive or psychosocial issues. c) The entry should list the specific reasons that the client was upset. d) The entry should specify the subsequent interventions that were performed.

a) CORRECT ANSWER The entry should include clearer descriptions of the client's mood and behavior. Reason: Entries in the medical record should be precise, descriptive, and objective. An adjective such as "upset" is unclear and open to many interpretations. As such, the nurse should elaborate on this description so a reader has a clearer understanding of the client's state of mind. Stating the apparent reasons that the client was "upset" does not resolve the ambiguity of this descriptor. Cognitive and psychosocial issues are valid components of the medical record. Responses and interventions should normally follow assessment data but the data themselves must first be recorded accurately.

A nurse is facilitating mandated group therapy for clients who have sexually abused children. Children who are victims of sexual abuse are typically: a) from any segment of the population. b) of low socioeconomic background. c) strangers to the abuser. d) willing to engage in sexual acts with adults.

a) CORRECT ANSWER from any segment of the population. Reason: Victims of childhood sexual abuse come from all segments of the population and from all socioeconomic backgrounds. Most victims know their abuser. Children rarely willingly engage in sexual acts with adults because they don't have full decision-making capacities.

A client with chronic heart failure is receiving digoxin (Lanoxin), 0.25 mg by mouth daily, and furosemide (Lasix), 20 mg by mouth twice daily. The nurse instructs the client to notify the physician if nausea, vomiting, diarrhea, or abdominal cramps occur because these signs and symptoms may signal digoxin toxicity. Digoxin toxicity may also cause: a) visual disturbances. b) taste and smell alterations. c) dry mouth and urine retention. d) nocturia and sleep disturbances.

a) CORRECT ANSWER visual disturbances. Reason: Digoxin toxicity may cause visual disturbances (such as, flickering flashes of light, colored or halo vision, photophobia, blurring, diplopia, and scotomata), central nervous system abnormalities (such as headache, fatigue, lethargy, depression, irritability and, if profound, seizures, delusions, hallucinations, and memory loss), and cardiovascular abnormalities (abnormal heart rate and arrhythmias). Digoxin toxicity doesn't cause taste and smell alterations. Dry mouth and urine retention typically occur with anticholinergic agents, not inotropic agents such as digoxin. Nocturia and sleep disturbances are adverse effects of furosemide — especially if the client takes the second daily dose in the evening, which may cause diuresis at night.

Coping with stress differs from adaption to stress in that: a) coping is a more immediate, short term response to stress b) Coping is a later response to stress

a) Coping is a more immediate, short term response to stress

hyperpituitarism

a. Hypersecretion of growth hormone by the anterior pituitary gland in an adult; caused primarily by pituitary tumors b. Leads to conditions such as acromegaly and Cushing's disease

POST OP HYPOPHYSECTOMY

a. Initiate postoperative care similar to craniotomy care. b. Monitor vital signs, neurological status, and level of consciousness. c. Elevate the head of the bed. d. Monitor for increased intracranial pressure. e. Monitor for bleeding. f. Instruct the client to avoid sneezing, coughing, and blowing the nose. g. Monitor for signs of temporary diabetes insipidus or syndrome of inappropriate antidiuretic hormone resulting from ADH disturbances. h. Monitor intake and output, and avoid water intoxication. i. Administer glucocorticoids and other hormone replacements as prescribed. j. Administer antibiotics, analgesics, and antipyretics as prescribed. k. Instruct the client in the administration of prescribed medications.

assessment findings for hyperpituitarism

a. Large hands and feet b. Thickening and protrusion of the jaw c. Arthritic changes and joint pain d. Visual disturbances e. Diaphoresis f. Oily, rough skin g. Organomegaly h. Hypertension i. Dysphagia j. Deepening of the voice

assessment findings for anterior pituitary hypofunction

a. Mild to moderate obesity (GH, TSH) b. Reduced cardiac output (GH, ADH) c. Infertility, sexual dysfunction (gonadotropins, ACTH) d. Fatigue, low blood pressure (TSH, ADH, ACTH, GH) e. Tumors of the pituitary also may cause headaches and visual defects (pituitary is located near the optic nerve).

INTERVENTIONS FOR DI

a. Monitor vital signs and neurological and cardiovascular status. b. Provide a safe environment, particularly for the client with postural hypotension. c. Monitor electrolyte values and for signs of dehydration. d. Maintain client intake of adequate fluids. e. Monitor intake and output, weight, serum osmolality, and specific gravity of urine. f. Instruct the client to avoid foods or liquids that produce diuresis. g. Vasopressin tannate (Pitressin) or desmopressin acetate (DDAVP, Stimate) may be prescribed; these are used when the ADH deficiency is severe or chronic. h. Instruct the client in the administration of medications as prescribed; DDAVP may be administered by injection, intranasally, or orally. i. Instruct the client to wear a Medic-Alert bracelet.

ASSESSMENT FINDINGS FOR DI

a. Polyuria of 4 to 24 L/ day b. Polydipsia c. Dehydration (decreased skin turgor and dry mucous membranes) d. Inability to concentrate urine e. Low urinary specific gravity, 1.006 or lower f. Fatigue g. Muscle pain and weakness h. Headache i. Postural hypotension that may progress to vascular collapse without rehydration j. Tachycardia

nursing interventions for pituitary hypofunction

a. Provide emotional support to the client and family. b. Encourage the client and family to express feelings related to disturbed body image or sexual dysfunction. c. Client may need hormone replacement for the specific deficient hormones. d. Client education is needed regarding the signs and symptoms of hypofunction and hyperfunction related to insufficient or excess hormone replacement

Caring for a client receiving a continuous heparin infusion. Lab test to review prior to adjusting the client's heparin?

aPTT

Assessing older adult client with pneumonia

acute confusion

*hypothyroid patient has numbness and tingling - what is the nurse's priority action?

admin calcium gluconate

*post thyroidectomy - patient develops stridor and right hand cramp - nurse expects to...

admin calcium gluconate

*presurgical medication teaching for a patient with acromegaly?

admin sandostatin IM

*addisons crisis patient admitted

administer IV NS

When prioritizing a client's care plan based on Maslow's hierarchy of needs, the nurse's first priority would be:

administering pain medication.

small pox transmission

airborne through direct deposit of infective droplets, bodily fluids, contaminated objects

A client experiencing a severe asthma attack has the following arterial blood gas results: pH 7.33; PCO2 48 (6.4 kPa); PO2 58 (7.7 kPa); HCO3 26 (26 mmol/L). Which prescriptions should the nurse implement first?

albuterol nebulizer

A client is confused and continuously attempts to get out of bed. The physician prescribes a vest restraint. When applying a vest restraint, the nurse should:

allow room for the client to turn.

*what should nurse do when assess CVAD?

always document site

A nurse is assessing a 15-year-old girl who has lost 30 lb (13.6 kg) over 3 months. What other finding is the nurse likely to assess?

amenorrhea.

A client has three children and his mother lives with them. This is called:

an extended family.

Although both vertigo and dizziness can result from peripheral or central vestibular disorders, vertigo is distinctly different because it causes:

an illusion of motion.

*patient yells and throws a fit at the nurse - what stage of dying?

anger

Signs/symptoms of adrenal insufficiency

anorexia, nausea, vomiting, abdominal pain, fatigue, lethargy, fever, confusion or coma if left uncorrected

*most likely available form of weaponized biological agent

anthrax

To prepare the community for the possible threat of anthrax, a nurse must teach that:

anthrax can infect the integumentary, GI, and respiratory systems.

The nurse is caring for a client with a right ankle sprain. When applying cold to the client's injury, the nurse should:

apply it immediately after the injury occurs.

A client undergoes a surgical procedure that requires the use of general anesthesia. Following general anesthesia, the client is most at risk for:

atelectasis.

The nurse is administering eardrops to an adult client. To straighten the ear canal in an adult client before instilling the drops, the nurse should gently pull the:

auricle up and back.

The nurse is preparing to remove a previously applied topical medication from a client. The rationale for removing previously applied topical medications before applying new medications is to:

avoid administering more than the prescribed dose.

Teaching a client 26 wks gestation about a glucose tolerance test.

avoid caffeine

small pox vaccine site care

avoid contact with site thorough hand washing after contact with site loosely cover site with porous bandage change dressing daily dispose of contaminated bandages and site scab properly immunity = major reaction at site - scab separates - scar

To prevent back injury, the nurse should instruct the client to:

avoid prolonged sitting and standing.

After staying several hours with her 9-year-old daughter who is admitted to the hospital with an asthma attack, the mother leaves to attend to her other children. The child exhibits continued signs and symptoms of respiratory distress. Which of the following findings should lead the nurse to believe the child is experiencing anxiety? a) Not able to get comfortable. b) Frequent requests for someone to stay in the room. c) Inability to remember her exact address. d) Verbalization of a feeling of tightness in her chest.

b - A 9-year-old child should be able to tolerate being alone. Frequently asking for someone to be in the room indicates a degree of psychological distress that, at this age, suggests anxiety. The inability to get comfortable is more characteristic of a child in pain. Inability to answer questions correctly may reflect a state of anoxia or a lack of knowledge. Tightness in the chest occurs as a result of bronchial spasms.

After surgery for an ileal conduit, the nurse should closely assess the client for the occurrence of which of the following complications related to pelvic surgery? a) Peritonitis. b) Thrombophlebitis. c) Ascites. d) Inguinal hernia.

b - After pelvic surgery, there is an increased chance of thrombophlebitis owing to the pelvic manipulation that can interfere with circulation and promote venous stasis. Peritonitis is a potential complication of any abdominal surgery, not just pelvic surgery. Ascites is most frequently an indication of liver disease. Inguinal hernia may be caused by an increase in intra-abdominal pressure or a congenital weakness of the abdominal wall; ventral hernia occurs at the site of a previous abdominal incision.

The nurse has administered aminophylline to a client with emphysema. The medication is effective when there is: a) Relief from spasms of the diaphragm. b) Relaxation of smooth muscles in the bronchioles. c) Efficient pulmonary circulation. d) Stimulation of the medullary respiratory center.

b - Aminophylline, a bronchodilator that relaxes smooth muscles in the bronchioles, is used in the treatment of emphysema to improve ventilation by dilating the bronchioles. Aminophylline does not have an effect on the diaphragm or the medullary respiratory center and does not promote pulmonary circulation.

A pregnant client in her third trimester is started on chlorpromazine (Thorazine) 25 mg four times daily. Which of the following instructions is most important for the nurse to include in the client's teaching plan? a) "Don't drive because there's a possibility of seizures occurring." b) "Avoid going out in the sun without a sunscreen with a sun protection factor of 25." c) "Stop the medication immediately if constipation occurs." d) "Tell your doctor if you experience an increase in blood pressure."

b - Chlorpromazine is a low-potency antipsychotic that is likely to cause sun-sensitive skin. Therefore the client needs instructions about using sunscreen with a sun protection factor of 25 or higher. Typically, chlorpromazine is not associated with an increased risk of seizures. Although constipation is a common adverse effect of this drug, it can be managed with diet, fluids, and exercise. The drug does not need to be discontinued. Chlorpromazine is associated with postural hypotension, not hypertension. Additionally, if postural hypotension occurs, safety measures, such as changing positions slowly and dangling the feet before arising, not stopping the drug, are instituted.

A 7 year old with a history of tonic-clonic seizures has been actively seizing for 10 minutes. The child weighs 22 kg and currently has an intravenous (IV) line of D5 1/2 NS + 20 meq KCL/L running at 60 ml/hr. Vital signs are a temperature of 38 degrees C, heart rate of 120, respiratory rate of 28, and oxygen saturation of 92%. Using the SBAR (Situation-Background-Assessment-Recommendation) technique for communication, the nurse calls the primary healthcare provider with a recommendation for: a) Rectal diazepam (Diastat). b) IV lorazepam (Ativan). c) Rectal acetaminophen (Tylenol). d) IV fosphenytoin.

b - IV ativan is the benzodiazepine of choice for treating prolonged seizure activity. IV benzodiazepines potentiate the action of the gamma-aminobutyric acid (GABA) neurotransmitter, stopping seizure activity. If an IV line is not available, rectal Diastat is the benzodiazepine of choice. The child does have a low-grade fever; however, this is likely caused by the excessive motor activity. The primary goal for the child is to stop the seizure in order to reduce neurologic damage. Benzodiazepines are used for the initial treatment of prolonged seizures. Once the seizure has ended, a loading dose of fosphenytoin or phenobarbital is given.

A 10-month-old child with recurrent otitis media is brought to the clinic for evaluation. To help determine the cause of the child's condition, the nurse should ask the parents: a) "Does water ever get into the baby's ears during shampooing?" b) "Do you give the baby a bottle to take to bed?" c) "Have you noticed a lot of wax in the baby's ears?" d) "Can the baby combine two words when speaking?"

b - In a young child, the eustachian tube is relatively short, wide, and horizontal, promoting drainage of secretions from the nasopharynx into the middle ear. Therefore, asking if the child takes a bottle to bed is appropriate because drinking while lying down may cause fluids to pool in the pharyngeal cavity, increasing the risk of otitis media. Asking if the parent noticed earwax, or cerumen, in the external ear canal is incorrect because wax doesn't promote the development of otitis media. During shampooing, water may become trapped in the external ear canal by large amounts of cerumen, possibly causing otitis external (external ear inflammation) as opposed to internal ear inflammation. Asking if the infant can combine two words is incorrect because a 10-month-old child isn't expected to do so.

The mother of a client with chronic undifferentiated schizophrenia calls the visiting nurse in the outpatient clinic to report that her daughter has not answered the phone in 10 days. "She was doing so well for months. I don't know what's wrong. I'm worried." Which of the following responses by the nurse is most appropriate? a) "Maybe she's just mad at you. Did you have an argument?" b) "She may have stopped taking her medications. I'll check on her." c) "Don't worry about this. It happens sometimes." d) "Go over to her apartment and see what's going on."

b - Noncompliance with medications is common in the client with chronic undifferentiated schizophrenia. The nurse has the responsibility to assess this situation. Asking the mother if they've argued or if the client is mad at the mother or telling the mother to go over to the apartment and see what's going on places the blame and responsibility on the mother and therefore is inappropriate. Telling the mother not to worry ignores the seriousness of the client's symptoms.

A client with bipolar disorder, manic phase, just sat down to watch television in the lounge. As the nurse approaches the lounge area, the client states, "The sun is shining. Where is my son? I love Lucy. Let's play ball." The client is displaying: a) Concreteness. b) Flight of ideas. c) Depersonalization. d) Use of neologisms.

b - The client is demonstrating flight of ideas, or the rapid, unconnected, and often illogical progression from one topic to another. Concreteness involves interpreting another person's words literally. Depersonalization refers to feelings of strangeness concerning the environment or the self. A neologism is a word made up by a client.

The nurse is caring for several mother-baby couplets. In planning the care for each of the couplets, which mother would the nurse expect to have the most severe afterbirth pains? a) G 4, P 1 client who is breastfeeding her infant. b) G 3, P 3 client who is breastfeeding her infant. c) G 2, P 2 cesarean client who is bottle-feeding her infant. d) G 3, P 3 client who is bottle-feeding her infant.

b - The major reasons for afterbirth pains are breast-feeding, high parity, overdistended uterus during pregnancy, and a uterus filled with blood clots. Physiologically, afterbirth pains are caused by intermittent contraction and relaxation of the uterus. These contractions are stronger in multigravidas in order to maintain a contracted uterus. The release of oxytocin when breast-feeding also stimulates uterine contractions. There is no data to suggest any of these clients has had an overdistended uterus or currently has clots within the uterus. The G 3, P 3 client who is breast-feeding has the highest parity of the clients listed, which—in addition to breast-feeding—places her most at risk for afterbirth pains. The G 2, P 2 postcesarean client may have cramping but it should be less than the G 3, P 3 client. The G 3, P 3 client who is bottle-feeding would be at risk for afterbirth pains because she has delivered several children, but her choice to bottle-feed reduces her risk of pain.

A nurse is monitoring a client receiving tranylcypromine sulfate (Parnate). Which serious adverse reaction can occur with high dosages of this monoamine oxidase (MAO) inhibitor? a) Hypotensive episodes b) Hypertensive crisis c) Muscle flaccidity d) Hypoglycemia

b - The most serious adverse reaction associated with high doses of MAO inhibitors is hypertensive crisis, which can lead to death. Although not a crisis, orthostatic hypotension is also common and may lead to syncope with high doses. Muscle spasticity (not flaccidity) is associated with MAO inhibitor therapy. Hypoglycemia isn't an adverse reaction of MAO inhibitors.

Which scenario complies with Health Insurance Portability and Accountability Act of 1996 (HIPAA) regulations? a) Two nurses in the cafeteria are discussing a client's condition. b) The health care team is discussing a client's care during a formal care conference. c) A nurse checks the computer for the laboratory results of a neighbor who has been admitted to another floor. d) A nurse talks with her spouse about a client's condition.

b - To provide interdisciplinary continuity of care, nurses must share relevant information during client care conferences. Nurses discussing information in the cafeteria may be overheard; this indiscretion violates HIPAA regulations. Looking up laboratory results for a neighbor is a HIPAA violation, as is discussing a client's condition with one's spouse.

A nurse is instructing a client with bipolar disorder on proper use of lithium carbonate (Eskalith), the drug's adverse effects, and symptoms of lithium toxicity. Which client statement indicates that additional teaching is required? a. "I can still eat my favorite salty foods." b. "when my moods fluctuate, I'll increase my dose of lithium." c. "a good blood level of the drug means the drug concentration has stabilized." d. "eating too much watermelon will affect my lithium level."

b - a client who states that he'll increase his dose of lithium if his mood fluctuates requires additional teaching because increasing the dose of lithium without evaluating the client's laboratory values can cause serious health problems, such as lithium toxicity, overdose, and renal failure. Clients taking lithium don't need to limit their sodium intake. A low sodium diet causes lithium retention. A therapeutic lithium blood level indicates that the drug concentration has stabilized. The client demonstrates effective teaching by stating his lithium levels will be affected by foods that have a diuretic effect, such as watermelon, cantaloupe, grapefruit juice and cranberry juice.

A 10-month-old child has cold symptoms. The mother asks how she can clear infant's nose. Which of the following would be the nurse's best recommendation? a. use a cool air vaporizer with plain water b. use saline nose drops and then a bulb syringe c. blow into the child's mouth to clear the nose d. administer a nonprescription vasoconstrictive nose spray.

b - although a cool air vaporizer may be recommended to humidify the environment, using saline nose drops and then a bulb syringe before meals and at nap and bed times will allow the child to breathe more easily. Saline helps to loosen secretions and keep the mucous membranes moist. The bulb syringe then gently aids in removing the loosened secretions. Blowing into the child's mouth to clear the nose introduces more organisms to the child. A nonprescription vasoconstrictive nasal spray is not recommended for infants because if the spray in used for longer than 3 days a rebound effect with increased inflammation occurs.

A client with Rh isoimmunization gives birth to a neonate with an enlarged heart and severe, generalized edema. The neonate is immediately transferred to the neonatal intensive care unit. Which nursing diagnosis is most appropriate for the client? a. ineffective parenting related to the neonate's transfer to the intensive care unit b. impaired parenting related to the neonate's transfer to the intensive care unit c. deficient fluid volume related to severe edema d. fear related to removal and loss of the neonate by statute

b - because the neonate is severely ill and needs to be placed in the neonatal intensive care unit, the client may have a nursing diagnosis of 'impaired parenting related to the neonate's transfer to the neonatal intensive care unit.' (another pertinent nursing diagnosis may be 'compromised family coping related to lack the opportunity for bonding.) Rh isoimmunization isn't a socially unacceptable infection. This condition causes an excess fluid volume (not deficient) related to cardiac problems. Rh isoimmunization doesn't lead to loss of the neonate by statute.

The major goal of therapy in crisis intervention is to: a. withdraw from the stress b. resolve the immediate problem c. decrease anxiety d. provide documentation of events

b - during a period of crisis, the major goal is to resolve the immediate problem, with hopes of getting the individual to the level of functioning that existed before the crisis or to a higher level of functioning. Withdrawing from stress doesn't address the immediate problem and isn't therapeutic. The client's anxiety will decrease after the immediate problem is resolved. Providing support and safety are necessary interventions while working toward accomplishing the goal. Documentation is necessary for maintaining accurate records of treatment, but isn't a major goal.

A client has a herniated disk in the region of the third and fourth lumbar vertebrae. Which nursing assessment finding most supports this diagnosis? a. hypoactive bowel sounds b. severe lower back pain c. sensory deficits in one arm d. weakness and atrophy of the arm muscles

b - the most common finding in a client with a herniated lumbar disk is severe lower back pain, which radiates to the buttocks, legs, and feet - usually unilaterally. A herniated disk also may cause sensory and motor loss (such as foot drop) in the area innervated by the compressed spinal nerve root. During later stages, it may cause weakness and atrophy of leg muscles. The condition doesn't affect bowel sounds or the arms.

Before discharge, which instruction should a nurse give to a client receiving digoxin (Lanoxin)? a. "take an extra dose of digoxin if you miss one dose." b. "call the physician if your heart rate is above 90 beats/minute c. "call the physician if your pulse drops below 80 beats/minute." d. "take digoxin with meals."

b - the nurse should instruct the client to notify the physician if his heart rate is greater than 90 beats/ minute because cardiac arrhythmias may occur with digoxin toxicity. To prevent toxicity, the nurse should instruct the client never to take an extra dose of digoxin if he misses a dose. The nurse should show the client how to take his pulse and tell him to call the physician if his pulse rate drops below 60 beats/minute - not 80 beats/minute, which is a normal pulse rate and doesn't warrant action. The client shouldn't take digoxin with meals; doing so slows the absorption rate.

The nurse observes that the right eye of an unconscious client does not close completely. Which nursing intervention is most appropriate a. have the client wear eyeglasses at all time b. lightly tape the eyelid shut c. instill artificial tears once every shift d. clean the eyelid with a washcloth every shift

b - when the blink reflex is absent or the eyes do not close completely, the cornea may become dry and irritated. Corneal abrasion can occur. Taping the eye closed will prevent injury. Having the client wears eyeglasses or cleaning the eyelid will not protect the cornea from dryness or irritation. Artificial tears instilled once per shift are not frequent enough for preventing dryness.

The nurse has discussed sexuality issues during the prenatal period with a primigravida who is at 32 weeks' gestation. She has had one episode of preterm labor. The nurse determines that the client understands the instructions when she says: a) "I can resume sexual intercourse when the bleeding stops." b) "I should not get sexually aroused or have any nipple stimulation." c) "I can resume sexual intercourse in 1 to 2 weeks." d) "I should not have sexual intercourse until my next prenatal visit."

b) CORRECT ANSWER "I should not get sexually aroused or have any nipple stimulation." Reason: This client has already had one episode of preterm labor at 32 weeks' gestation. Sexual intercourse, arousal, and nipple stimulation may result in the release of oxytocin which can contribute to continued preterm labor and early delivery. The client should be advised to refrain from these activities until closer to term, which is 6 to 8 weeks later. Telling the client that intercourse is acceptable after the bleeding stops is incorrect and may lead to early delivery of a preterm neonate. The client should not have intercourse for at least 6 weeks because of the danger of inducing labor. There is no indication when the client's next prenatal visit is scheduled.

When planning care for a client with a head injury, which position should the nurse include in the care plan to enhance client outcomes? a) Trendelenburg's b) 30-degree head elevation c) Flat d) Side-lying

b) CORRECT ANSWER 30-degree head elevation Reason: For clients with increased intracranial pressure (ICP), the head of the bed should be elevated to 30 degrees to promote venous outflow. Trendelenburg's position is contraindicated because it can raise ICP. Flat or neutral positioning is indicated when elevating the head of the bed would increase the risk of neck injury or airway obstruction. A side-lying position isn't specifically a therapeutic treatment for increased ICP.

A client with an incomplete small-bowel obstruction is to be treated with a Cantor tube. Which of the following measures would most likely be included in the client's care once the Cantor tube has passed into the duodenum? a) Maintain bed rest with bathroom privileges. b) Advance the tube 2 to 4 inches at specified times. c) Avoid frequent mouth care. d) Provide ice chips for the client to suck.

b) CORRECT ANSWER Advance the tube 2 to 4 inches at specified times. Reason: Once the intestinal tube has passed into the duodenum, it is usually advanced as ordered 2 to 4 inches every 30 to 60 minutes. This, along with gravity and peristalsis, enables passage of the tube forward. The client is encouraged to walk, which also facilitates tube progression. A client with an intestinal tube needs frequent mouth care to stimulate saliva secretion, to maintain a healthy oral cavity, and to promote comfort regardless of where the tube is placed in the intestine. Ice chips are contraindicated because hypotonic fluid will draw extra fluid into an already distended bowel.

A woman who has recently immigrated from Africa who delivered a term neonate a short time ago requests that a "special bracelet" be placed on the baby's wrist. The nurse should: a) Tell the mother that the bracelet is not recommended for cleanliness reasons. b) Apply the bracelet on the neonate's wrist as the mother requests. c) Place the bracelet on the neonate, limiting its use to when the neonate is with the mother. d) Recommend that the mother wait until she is discharged to apply the bracelet.

b) CORRECT ANSWER Apply the bracelet on the neonate's wrist as the mother requests. Reason: The nurse should abide by the mother's request and place the bracelet on the neonate. In some cultures, amulets and other special objects are viewed as good luck symbols. By allowing the bracelet, the nurse demonstrates culturally sensitive care, promoting trust. The neonate can wear the bracelet while with the mother or in the nursery. The bracelet can be used while the neonate is being bathed, or if necessary and acceptable to the client removed and replaced afterward.

When measuring the fundal height of a primigravid client at 20 weeks' gestation, the nurse will locate the fundal height at which of the following points? a) Halfway between the client's symphysis pubis and umbilicus. b) At about the level of the client's umbilicus. c) Between the client's umbilicus and xiphoid process. d) Near the client's xiphoid process and compressing the diaphragm.

b) CORRECT ANSWER At about the level of the client's umbilicus. Reason: Measurement of the client's fundal height is a gross estimate of fetal gestational age. At 20 weeks' gestation, the fundal height should be at about the level of the client's umbilicus. The fundus typically is over the symphysis pubis at 12 weeks. A fundal height measurement between these two areas would suggest a fetus with a gestational age between 12 and 20 weeks. The fundal height increases approximately 1 cm/week after 20 weeks' gestation. The fundus typically reaches the xiphoid process at approximately 36 weeks' gestation. A fundal height between the umbilicus and the xiphoid process would suggest a fetus with a gestational age between 20 and 36 weeks. The fundus then commonly returns to about 4 cm below the xiphoid owing to lightening at 40 weeks. Additionally, pressure on the diaphragm occurs late in pregnancy. Therefore, a fundal height measurement near the xiphoid process with diaphragmatic compression suggests a fetus near the gestational age of 36 weeks or older.

A nurse is assessing the legs of a client who's 36 weeks pregnant. Which finding should the nurse expect? a) Absent pedal pulses b) Bilateral dependent edema c) Sluggish capillary refill d) Unilateral calf enlargement

b) CORRECT ANSWER Bilateral dependent edema Reason: As the uterus grows heavier during pregnancy, femoral venous pressure rises, leading to bilateral dependent edema. Factors interfering with venous return, such as sitting or standing for long periods, contribute to edema. Absence of pedal pulses and sluggish capillary refill signal inadequate circulation to the legs — an unexpected finding during pregnancy. Unilateral calf enlargement, also an abnormal finding, may indicate thrombosis.

A parent brings a 5-year-old child to a vaccination clinic to prepare for school entry. The nurse notes that the child has not had any vaccinations since 4 months of age. To determine the current evidence for best practices for scheduling missed vaccinations the nurse should: a) Ask the primary care provider. b) Check the website at the Center for Disease Control and Prevention (CDC). c) Read the vaccine manufacturer's insert. d) Contact the pharmacist.

b) CORRECT ANSWER Check the website at the Center for Disease Control and Prevention (CDC). Reason: The CDC is the federal body that is ultimately responsible for vaccination recommendations for adults and children. A division of the CDC, the Advisory Committee on Immunization Practices, reviews vaccination evidence and updates recommendation on a yearly basis. The CDC publishes current vaccination catch-up schedules that are readily available on their website. The lack of vaccinations is a strong indicator that the child probably does not have a primary care provider. If consulted, the pharmacist would most likely have to review the CDC guidelines that are equally available to the nurse. Reading the manufacturer's inserts for multiple vaccines would be time consuming and synthesis of the information could possibly lead to errors.

A 10-year-old with glomerulonephritis reports a headache and blurred vision. The nurse should immediately: a) Put the client to bed. b) Obtain the child's blood pressure. c) Notify the physician. d) Administer acetaminophen (Tylenol).

b) CORRECT ANSWER Obtain the child's blood pressure. Reason: Hypertension occurs with acute glomerulonephritis. The symptoms of headache and blurred vision may indicate an elevated blood pressure. Hypertension in acute glomerulonephritis occurs due to the inability of the kidneys to remove fluid and sodium; the fluid is reabsorbed, causing fluid volume excess. The nurse must verify that these symptoms are due to hypertension. Calling the physician before confirming the cause of the symptoms would not assist the physician in his treatment. Putting the client to bed may help treat an elevated blood pressure, but first the nurse must establish that high blood pressure is the cause of the symptoms. Administering Tylenol for high blood pressure is not recommended.

A client who took an overdose of Tylenol in a suicide attempt is transferred overnight to the psychiatric inpatient unit from the intensive care unit. The night shift nurse called the primary health care provider on call to obtain initial prescriptions. The primary health care provider prescribes the typical routine medications for clients on this unit: Milk of Magnesia, Maalox and Tylenol as needed. Prior to implementing the prescriptions, the nurse should? a) Ask the primary health care provider about holding all the client's PM prescriptions. b) Question the primary health care provider about the Tylenol prescription. c) Request a prescription for a medication to relieve agitation. d) Suggest the primary health care provider write a prescription for intravenous fluids.

b) CORRECT ANSWER Question the primary health care provider about the Tylenol prescription. Reason: The nurse should question the Tylenol order because the client overdosed on Tylenol, and that analgesic would be contraindicated as putting further stress on the liver. There is no need to hold the PM Milk of Magnesia or Maalox. There is no indication that the client is agitated or needs medication for agitation. There is little likelihood that the client needs an IV after being transferred out of an intensive care unit, as the client will be able to take oral fluids.

A nurse assesses a client's respiratory status. Which observation indicates that the client is having difficulty breathing? a) Diaphragmatic breathing b) Use of accessory muscles c) Pursed-lip breathing d) Controlled breathing

b) CORRECT ANSWER Use of accessory muscles Reason: The use of accessory muscles for respiration indicates the client is having difficulty breathing. Diaphragmatic and pursed-lip breathing are two controlled breathing techniques that help the client conserve energy.

A primigravid client gives birth to a full-term girl. When teaching the client and her partner how to change their neonate's diaper, the nurse should instruct them to: a) fold a cloth diaper so that a double thickness covers the front. b) clean and dry the neonate's perineal area from front to back. c) place a disposable diaper over a cloth diaper to provide extra protection. d) position the neonate so that urine will fall to the back of the diaper.

b) CORRECT ANSWER clean and dry the neonate's perineal area from front to back. Reason: When changing a female neonate's diaper, the caregiver should clean the perineal area from front to back to prevent infection and then dry the area thoroughly to minimize skin breakdown. For a male, the caregiver should clean and dry under and around the scrotum. Because of anatomic factors, a female's diaper should have the double thickness toward the back. The diaper, not the neonate, should be positioned properly. Placing a disposable diaper over a cloth diaper isn't necessary. The direction of urine flow can't be ensured.

A male client should be taught about testicular examinations:

before age 20.

The nurse is caring for a client who's showing signs of hypoglycemia. This client will most likely have a blood glucose level:

below 70 mg/dl.

In which maternal locations would the nurse place the ultrasound transducer of the external electronic fetal heart rate monitor if a fetus at 34 weeks' gestation is in the left occipitoanterior (LOA) position?

below the umbilicus on the left side. As the uterus contracts, the abdominal wall rises and, when external monitoring is used, presses against the transducer. This movement is transmitted into an electrical current, which is then recorded. With the fetus in the LOA position, the cardiotransducer should be placed below the umbilicus on the side where the fetal back is located and uterine displacement during contractions is greatest. If the fetal back is near the symphysis pubis, the fetus is presenting as a transverse lie. If the fetus is in a breech position, the fetal back may be at or above the umbilicus.

When instilling erythromycin ointment into the eyes of a neonate 1 hour old, the nurse would explain to the parents that the medication is used to prevent which problem?

blindness secondary to gonorrhea

A client is prescribed transcutaneous electrical nerve stimulation (TENS) for pain relief. The rationale for using TENS is to:

block painful stimuli traveling over small nerve fibers.

When inserting a urinary catheter, the nurse can facilitate the insertion by asking the client to:

breathe deeply.

When teaching a preschool-age child how to perform coughing and deep-breathing exercises before corrective surgery for tetralogy of Fallot, which teaching and learning principle should the nurse address first?

building the teaching on the child's current level of knowledge

A nurse is helping a physician insert a subclavian central line. After the physician has gained access to the subclavian vein, he connects a 10-ml syringe to the catheter and withdraws a sample of blood. He then disconnects the syringe from the port. Suddenly, the client becomes confused, disoriented, and pale. The nurse suspects an air embolus. She should: a) place the client in a supine position and prepare to perform cardiopulmonary resuscitation. b) place the client in high-Fowler's position and administer supplemental oxygen. c) turn the client on his left side and place the bed in Trendelenburg's position. d) position the client in the shock position with his legs elevated.

c - A nurse who suspects an air embolism should place the client on his left side and in Trendelenburg's position. Doing so allows the air to collect in the right atrium rather than enter the pulmonary system. The supine position, high-Fowler's position, and the shock position are therapeutic for other situations but not for air embolism.

When obtaining the vital signs of a client with multiple traumatic injuries, a nurse detects bradycardia, bradypnea, and systolic hypertension. The nurse must notify the physician immediately because these findings may reflect which complication? a) Shock b) Encephalitis c) Increased intracranial pressure (ICP) d) Status epilepticus

c - When ICP increases, Cushing's triad may develop, which involves decreased heart and respiratory rates and increased systolic blood pressure. Shock typically causes tachycardia, tachypnea, and hypotension. In encephalitis, the temperature rises and the heart and respiratory rates may increase from the effects of fever on the metabolic rate. (If the client doesn't maintain adequate hydration, hypotension may occur.) Status epilepticus causes unceasing seizures, not changes in vital signs.

A nurse, a licensed practical nurse (LPN), and a nursing assistant are caring for a group of clients. The nurse asks the nursing assistant to check the pulse oximetry level of a client who underwent laminectomy. The nursing assistant reports that the pulse oximetry reading is 89%. The client Kardex contains an order for oxygen application at 2 L/min should the pulse oximetry level fall below 92%. The nurse is currently assessing a postoperative client who just returned from the postanesthesia unit. How should the nurse proceed? a. immediately go the client's room and assess vital signs, administer oxygen at 2 L/minute, and notify the physician. b. ask the nursing assistant to notify the physician of the low pulse oximetry level c. ask the LPN to obtain vital signs and administer oxygen at 2 L/min to the client who underwent laminectomy d. complete the assessment of the new client before attending to the client who underwent laminectomy

c - because it's important to get more information about the client with a decreased pulse oximetry level, the nurse should ask the LPN to obtain vital signs and administer oxygen as ordered. The nurse must attend to the newly admitted client without delaying treatment to the client who is already in her care. The nurse can effectively do this by delegating tasks to an appropriate health team member such as an LPN. The nurse doesn't need to immediately attend to the client with a decreased pulse oximetry level; she may wait until she completes the assessment of the newly admitted client. The physician doesn't need to be notified at this time because an order for oxygen administration is already on record.

A client has an episiotomy to widen her birth canal. Birth extends the incision into the anal sphincter. This complication is called: a. a first-degree laceration b. a second-degree laceration c. a third-degree laceration d. a fourth-degree laceration

c - birth may extend an episiotomy incision to the anal sphincter (a third degree laceration) or the anal canal (a fourth degree laceration). A first degree laceration involves the fourchette, perineal skin, and vaginal mucous membranes. A second degree laceration extends to the fasciae and muscle of the perineal body.

The nurse should instruct the family of a child with newly diagnosed hyperthyroidism to: a. keep their home warmer than usual b. encourage plenty of outdoor activities c. promote interactions with one friend instead of groups d. limit bathing to prevent skin irritation

c - children with hyperthyroidism experience emotional labiality that may strain interpersonal relationships. Focusing on one friend's is easier than adapting to group dynamics until the child's condition improves. Because of their high metabolic rate, children with hyperthyroidism complain of being too warm. Bright sunshine may be irritating because of disease-related ophthalmopathy. Sweating is common and bathing should be encouraged.

A client with pneumonia has a temperature of 102.6F (39.2C), is diaphoretic, and has a productive cough. The nurse should include which of the following measures in the plan of care? a. position changes every 4 hours b. nasotracheal suctioning to clear secretions c. frequent linen changes d. frequent offering of a bedpan

c - frequent linen changes are appropriate for this client because of the diaphoresis. Diaphoresis produces general discomfort. The client should be kept dry to promote comfort. Position changes need to be done every 2 hours. Nasotracheal suctioning is not indicated with the client's productive cough. Frequent offering of a bedpan is not indicated by the data provided in this scenario.

The neonoate of a client with type 1 diabetes is at high risk for hypoglycemia. An initial sign the nurse should recognize as indicating hypoglycemia in a neonate is: a. peripheral acrocyanosis b. bradycardia c. lethargy d. jaundice

c - lethargy in the neonate may be seen with hypoglycemia because of a glucose in the nerve cells. Peripheral acrocyanosis is normal in the neonate because of immature capillary function. Tachycardia - not bradycardia - is seen with hypoglycemia. Jaundice isn't a sign of hypoglycemia.

The physician ordered IV naloxone (Narcan) to reverse the respiratory depression from morphine administration. After administration of the naloxone the nurse should: a. check respirations in 5 minutes because naxolone is immediately effective in relieving respiratory depression b. check respirations in 30 minutes because the effects of morphine will have worn off by then c monitor respirations frequently for 4 to 6 hours because the client may need repeated doses of naloxone d. monitor respirations each time the client receives morphine sulfate 10 mg IM

c - the nurse should monitor the client's respirations closely for 4 to 6 hours because naloxone has a shorter duration of action than opioids. The client may need repeated doses of naloxone to prevent or treat a recurrence of the respiratory depression. Naloxone is usually effective in a few minutes; however, its effects last only 1 to 2 hours and ongoing monitoring of the client's respiratory rate will be necessary. The client's dosage of morphine will be decreased or a new drug will be ordered to prevent another instance of respiratory depression.

After discussing asthma as a chronic condition, which of the following statements by the father of a child with asthma best reflects the family's positive adjustment to this aspect of the child's disease? a) "We try to keep him happy at all costs; otherwise, he has an asthma attack." b) "We keep our child away from other children to help cut down on infections." c) "Although our child's disease is serious, we try not to let it be the focus of our family." d) "I'm afraid that when my child gets older, he won't be able to care for himself like I do."

c) CORRECT ANSWER "Although our child's disease is serious, we try not to let it be the focus of our family." Reason: Positive adjustment to a chronic condition requires placing the child's illness in its proper perspective. Children with asthma need to be treated as normally as possible within the scope of the limitations imposed by the illness. They also need to learn how to manage exacerbations and then resume as normal a life as possible. Trying to keep the child happy at all costs is inappropriate and can lead to the child's never learning how to accept responsibility for behavior and get along with others. Although minimizing the child's risk for exposure to infections is important, the child needs to be with his or her peers to ensure appropriate growth and development. Children with a chronic illness need to be involved in their care so that they can learn to manage it. Some parents tend to overprotect their child with a chronic illness. This overprotectiveness may cause a child to have an exaggerated feeling of importance or later, as an adolescent, to rebel against the overprotectiveness and the parents.

The nurse meets with the client and his wife to discuss depression and the client's medication. Which of the following comments by the wife would indicate that the nurse's teaching about disease process and medications has been effective? a) "His depression is almost cured." b) "He's intelligent and won't need to depend on a pill much longer." c) "It's important for him to take his medication so that the depression will not return or get worse." d) "It's important to watch for physical dependency on Zoloft."

c) CORRECT ANSWER "It's important for him to take his medication so that the depression will not return or get worse." Reason: Improved balance of neurotransmitters is achieved with medication. Clients with endogenous depression must take antidepressants to prevent a return or worsening of depressive symptoms. Depression is a chronic disease characterized by periods of remission; however, it is not cured. Depression is not dependent on the client's intelligence to will the illness away. Zoloft is not physically addictive.

A worried mother confides in the nurse that she wants to change physicians because her infant is not getting better. The best response by the nurse is which of the following? a) "This doctor has been on our staff for 20 years." b) "I know you are worried, but the doctor has an excellent reputation." c) "You always have an option to change. Tell me about your concerns." d) "I take my own children to this doctor."

c) CORRECT ANSWER "You always have an option to change. Tell me about your concerns." Reason: Asking the mother to talk about her concerns acknowledges the mother's rights and encourages open discussion. The other responses negate the parent's concerns.

A client in the triage area who is at 19 weeks' gestation states that she has not felt her baby move in the past week and no fetal heart tones are found. While evaluating this client, the nurse identifies her as being at the highest risk for developing which problem? a) Abruptio placentae. b) Placenta previa. c) Disseminated intravascular coagulation. d) Threatened abortion.

c) CORRECT ANSWER Disseminated intravascular coagulation. Reason: A fetus that has died and is retained in utero places the mother at risk for disseminated intravascular coagulation (DIC) because the clotting factors within the maternal system are consumed when the nonviable fetus is retained. The longer the fetus is retained in utero, the greater the risk of DIC. This client has no risk factors, history, or signs and symptoms that put her at risk for either abruptio placentae or placenta previa, such as sharp pain and "woody," firm consistency of the abdomen (abruption) or painless bright red vaginal bleeding (previa). There is no evidence that she is threatening to abort as she has no complaints of cramping or vaginal bleeding.

Which of the following laboratory findings are expected when a client has diverticulitis? a) Elevated red blood cell count. b) Decreased platelet count. c) Elevated white blood cell count. d) Elevated serum blood urea nitrogen concentration.

c) CORRECT ANSWER Elevated white blood cell count. Reason: Because of the inflammatory nature of diverticulitis, the nurse would anticipate an elevated white blood cell count. The remaining laboratory findings are not associated with diverticulitis. Elevated red blood cell counts occur in clients with polycythemia vera or fluid volume deficit. Decreased platelet counts can occur as a result of aplastic anemias or malignant blood disorders, as an adverse effect of some drugs, and as a result of some heritable conditions. Elevated serum blood urea nitrogen concentration is usually associated with renal conditions.

A nurse is developing a nursing diagnosis for a client. Which information should she include? a) Actions to achieve goals b) Expected outcomes c) Factors influencing the client's problem d) Nursing history

c) CORRECT ANSWER Factors influencing the client's problem Reason: A nursing diagnosis is a written statement describing a client's actual or potential health problem. It includes a specified diagnostic label, factors that influence the client's problem, and any signs or symptoms that help define the diagnostic label. Actions to achieve goals are nursing interventions. Expected outcomes are measurable behavioral goals that the nurse develops during the evaluation step of the nursing process. The nurse obtains a nursing history during the assessment step of the nursing process.

A client who has been diagnosed with gastroesophageal reflux disease (GERD) complains of heartburn. To decrease the heartburn, the nurse should instruct the client to eliminate which of the following items from the diet? a) Lean beef. b) Air-popped popcorn. c) Hot chocolate. d) Raw vegetables.

c) CORRECT ANSWER Hot chocolate. Reason: With GERD, eating substances that decrease lower esophageal sphincter pressure causes heartburn. A decrease in the lower esophageal sphincter pressure allows gastric contents to reflux into the lower end of the esophagus. Foods that can cause a decrease in esophageal sphincter pressure include fatty foods, chocolate, caffeinated beverages, peppermint, and alcohol. A diet high in protein and low in fat is recommended for clients with GERD. Lean beef, popcorn, and raw vegetables would be acceptable.

A client who has a history of Crohn's disease is admitted to the hospital with fever, diarrhea, cramping, abdominal pain, and weight loss. The nurse should monitor the client for: a) Hyperalbuminemia. b) Thrombocytopenia. c) Hypokalemia. d) Hypercalcemia.

c) CORRECT ANSWER Hypokalemia. Reason: Hypokalemia is the most expected laboratory finding owing to the diarrhea. Hypoalbuminemia can also occur in Crohn's disease; however, the client's potassium level is of greater importance at this time because a low potassium level can cause cardiac arrest. Anemia is an expected development, but thrombocytopenia is not. Calcium levels are not affected.

The nurse is caring for a client with asthma. The nurse should conduct a focused assessment to detect which of the following? a) Increased forced expiratory volume. b) Normal breath sounds. c) Inspiratory and expiratory wheezing. d) Morning headaches.

c) CORRECT ANSWER Inspiratory and expiratory wheezing. Reason: The hallmark signs of asthma are chest tightness, audible wheezing, and coughing. Inspiratory and expiratory wheezing is the result of bronchoconstriction. Even between exacerbations, there may be some soft wheezing, so a finding of normal breath sounds would be expected in the absence of asthma. The expected finding is decreased forced expiratory volume [forced expiratory flow (FEF) is the flow (or speed) of air coming out of the lung during the middle portion of a forced expiration] due to bronchial constriction. Morning headaches are found with more advanced cases of COPD and signal nocturnal hypercapnia or hypoxemia.

Which of the following is a priority during the first 24 hours of hospitalization for a comatose client with suspected drug overdose? a) Educate regarding drug abuse. b) Minimize pain. c) Maintain intact skin. d) Increase caloric intake.

c) CORRECT ANSWER Maintain intact skin. Reason: Maintaining intact skin is a priority for the unconscious client. Unconscious clients need to be turned every hour to prevent complications of immobility, which include pressure ulcers and stasis pneumonia. The unconscious client cannot be educated at this time. Pain is not a concern. During the first 24 hours, the unconscious client will mostly likely be on nothing-by-mouth status.

A nurse is performing a psychosocial assessment on a 14-year-old adolescent. Which emotional response is typical during early adolescence? a) Frequent anger b) Cooperativeness c) Moodiness d) Combativeness

c) CORRECT ANSWER Moodiness Reason: Moodiness may occur often during early adolescence. Frequent anger and combativeness are more typical of middle adolescence. Cooperativeness typically occurs during late adolescence.

An anxious young adult is brought to the interviewing room of a crisis shelter, sobbing and saying that she thinks she is pregnant but does not know what to do. Which of the following nursing interventions is most appropriate at this time? a) Ask the client about the type of things that she had thought of doing. b) Give the client some ideas about what to expect to happen next. c) Recommend a pregnancy test after acknowledging the client's distress. d) Question the client about her feelings and possible parental reactions.

c) CORRECT ANSWER Recommend a pregnancy test after acknowledging the client's distress. Reason: Before any interventions can occur, knowing whether the client is pregnant is crucial in formulating a plan of care. Asking the client about what things she had thought about doing, giving the client some ideas about what to expect next, and questioning the client about her feelings and possible parental reactions would be appropriate after it is determined that the client is pregnant.

A man of Chinese descent is admitted to the hospital with multiple injuries after a motor vehicle accident. His pain is not under control. The client states, "If I could be with my people, I could receive acupuncture for this pain." The nurse should understand that acupuncture in the Asian culture is based on the theory that it: a) Purges evil spirits. b) Promotes tranquility. c) Restores the balance of energy. d) Blocks nerve pathways to the brain.

c) CORRECT ANSWER Restores the balance of energy. Reason: Acupuncture, like acumassage and acupressure, is performed in certain Asian cultures to restore the energy balance within the body. Pressure, massage, and fine needles are applied to energy pathways to help restore the body's balance. Acupuncture is not based on a belief in purging evil spirits. Although pain relief through acupuncture can promote tranquility, acupuncture is performed to restore energy balance. In the Western world, many researchers think that the gate-control theory of pain may explain the success of acupuncture, acumassage, and acupressure.

The nurse walks into a client's room to administer the 9:00 a.m. medications and notices that the client is in an awkward position in bed. What is the nurse's first action? a) Ask the client his name. b) Check the client's name band. c) Straighten the client's pillow behind his back. d) Give the client his medications.

c) CORRECT ANSWER Straighten the client's pillow behind his back. Reason: The nurse should first help the client into a position of comfort even though the primary purpose for entering the room was to administer medication. After attending to the client's basic care needs, the nurse can proceed with the proper identification of the client, such as asking the client his name and checking his armband, so that the medication can be administered.

A client was hospitalized for 1 week with major depression with suicidal ideation. He is taking venlafaxine (Effexor), 75 mg three times a day, and is planning to return to work. The nurse asks the client if he is experiencing thoughts of self-harm. The client responds, "I hardly think about it anymore and wouldn't do anything to hurt myself." The nurse should make which judgment about the client? a) The client is decompensating and in need of being readmitted to the hospital. b) The client needs an adjustment or increase in his dose of antidepressant. c) The depression is improving and the suicidal ideation is lessening. d) The presence of suicidal ideation warrants a telephone call to the client's primary care provider.

c) CORRECT ANSWER The depression is improving and the suicidal ideation is lessening. Reason: The client's statements about being in control of his behavior and his or her plans to return to work indicate an improvement in depression and that suicidal ideation, although present, is decreasing. Nothing in his comments or behavior indicate he is decompensating. There is no evidence to support an increase or adjustment in the dose of Effexor or a call to the primary care provider. Typically, the cognitive components of depression are the last symptoms eliminated. For the client to be experiencing some suicidal ideation in the second week of psychopharmacologic treatment is not unusual.

Before an incisional cholecystectomy is performed, the nurse instructs the client in the correct use of an incentive spirometer. Why is incentive spirometry essential after surgery in the upper abdominal area? a) The client will be maintained on bed rest for several days. b) Ambulation is restricted by the presence of drainage tubes. c) The operative incision is near the diaphragm. d) The presence of a nasogastric tube inhibits deep breathing.

c) CORRECT ANSWER The operative incision is near the diaphragm. Reason: The incisions made for upper abdominal surgeries, such as cholecystectomies, are near the diaphragm and make deep breathing painful. Incentive spirometry, which encourages deep breathing, is essential to prevent atelectasis after surgery. The client is not maintained on bed rest for several days. The client is encouraged to ambulate by the first postoperative day, even with drainage tubes in place. Nasogastric tubes do not inhibit deep breathing and coughing.

A child with a poor nutritional status and weight loss is at risk for a negative nitrogen balance. To help diagnose this problem, the nurse anticipates that the physician will order which laboratory test? a) Total iron-binding capacity b) Hemoglobin (Hb) c) Total protein d) Sweat test

c) CORRECT ANSWER Total protein Reason: The nurse anticipates the physician will order a total protein test because negative nitrogen balance may result from inadequate protein intake. Measuring total iron-binding capacity and Hb levels would help detect iron deficiency anemia, not a negative nitrogen balance. The sweat test helps diagnose cystic fibrosis, not a negative nitrogen balance.

A client received chemotherapy 24 hours ago. Which precautions are necessary when caring for the client? a) Wear sterile gloves. b) Place incontinence pads in the regular trash container. c) Wear personal protective equipment when handling blood, body fluids, and feces. d) Provide a urinal or bedpan to decrease the likelihood of soiling linens.

c) CORRECT ANSWER Wear personal protective equipment when handling blood, body fluids, and feces. Reason: Chemotherapy drugs are present in the waste and body fluids of clients for 48 hours after administration. The nurse should wear personal protective equipment when handling blood, body fluids, or feces. Gloves offer minimal protection against exposure. The nurse should wear a face shield, gown, and gloves when exposure to blood or body fluid is likely. Placing incontinence pads in the regular trash container and providing a urinal or bedpan don't protect the nurse caring for the client.

A client has just been diagnosed with early glaucoma. During a teaching session, the nurse should: a) provide instructions on eye patching. b) assess the client's visual acuity. c) demonstrate eyedrop instillation. d) teach about intraocular lens cleaning.

c) CORRECT ANSWER demonstrate eyedrop instillation. Reason: Eyedrop instillation is a critical component of self-care for a client with glaucoma. After demonstrating eyedrop instillation to the client and family, the nurse should verify their ability to perform this measure properly. An eye patch isn't necessary unless the client has undergone surgery. Visual acuity assessment isn't necessary before discharge. Intraocular lenses aren't implanted in clients with glaucoma.

When developing a care plan for a client with a do-not-resuscitate (DNR) order, a nurse should: a) withhold food and fluids. b) discontinue pain medications. c) ensure access to spiritual care providers upon the client's request. d) always make the DNR client the last in prioritization of clients.

c) CORRECT ANSWER ensure access to spiritual care providers upon the client's request. Reason: Ensuring access to spiritual care, if requested by the client, is an appropriate nursing action. A nurse should continue to administer appropriate doses of pain medication as needed to promote the client's comfort. A health care provider may not withhold food and fluids unless the client has a living will that specifies this action. A DNR order does not mean that the client does not require nursing care.

A client with diabetes who needs to learn to inject his own insulin states, "Ive had a good night's sleep, so let's tackle that syringe." The client if showing: a) Feedback c) Readiness

c) Readiness

thyroid hormones

calcitonin, T3, T4

*what diet should you provide a patient with hypoparathyroidism?

calcium and vitamin d supplements (not milk and milk products because they are high in phosphorous)

When a new mother asks the nurse whether her newborn infant can see her, the best response by the nurse is to tell the mother that her infant ...

can see light and dark patterns

cardiac s&s hyperthyroid

cardiac effects may include a heart rate ranging from 90 to 160 bpm, atrial fibrillation may occur, cardiac decompensation e.g. heart failure is common especially in the elderly

The nurse is teaching a group of women to perform breast self-examination. The nurse should explain that the purpose of performing the examination is to discover:

changes from previous self-examinations.

The nurse is about to administer a medication to a client. To verify the client's identity, the nurse should:

check the client's identification bracelet.

A client who underwent a lobectomy and has a water-seal chest drainage system is breathing with a little more effort and at a faster rate than 1 hour ago. The client's pulse rate is also increased. The nurse should:

check the tubing to ensure that the client is not lying on it or kinking it. In this case, there may be some obstruction to the flow of air and fluid out of the pleural space, causing air and fluid to collect and build up pressure. This prevents the remaining lung from reexpanding and can cause a mediastinal shift to the opposite side. The nurse's first response is to assess the tubing for kinks or obstruction. Increasing the suction is not done without a health care provider's prescription. The normal position of the drainage bottles is 2 to 3 feet (61 to 91.4 cm) below chest level. Clamping the tubes obstructs the flow of air and fluid out of the pleural space and should not be done.

inhalation anthrax diagnostics

chest x-ray (pleural effusion, widened mediastinum) blood culture sputum culture ELISA for IgG spinal tap for CSF

A client is recovering from an infected abdominal wound. Which foods should the nurse encourage the client to eat to support wound healing and recovery from the infection?

chicken and orange slices

When planning care for a client who has ingested phencyclidine (PCP), the nurse's highest priority should be meeting the:

client's safety needs. The highest priority for a client who has ingested PCP is meeting his safety needs as well as those of the staff. Drug effects are unpredictable and prolonged, and the client may easily become aggressive and physically violent. After safety needs have been met, the client's physical, psychosocial, and medical needs may be addressed.

In community-based nursing, primary responsibility for decisions related to health care belongs to the:

client.

The nurse is administering eyedrops to a client with glaucoma. To achieve maximum absorption, the nurse should instill the eyedrop into the:

conjunctival sac.

norepinephrine

constricts all blood vessels to increase BP greatly

A triple-lumen indwelling urinary catheter is inserted for continuous bladder irrigation following a transurethral resection of the prostate. In addition to balloon inflation, the functions of the three lumens include:

continuous inflow and outflow of irrigation solution.

The nurse is collecting data on a client admitted with a diagnosis of small bowel obstruction. When assessing the client's pulse rate, the nurse should:

count the apical or radial pulse for 60 seconds.

GI anthrax diagnostics

culture and gram stain of peritoneal fluid or ulcers for gram positive rods - stool culture

cutaneous anthrax diagnostics

culture of the skin lesion punch bx

The client is taking risperidone (Risperdal) to treat the positive and negative symptoms of schizophrenia. Improvement of which of the following negative symptoms indicate the drug is effective? a) Abnormal thought form. b) Hallucinations and delusions. c) Bizarre behavior. d) Asocial behavior and anergia.

d - Asocial behavior, anergia, alogia, and affective flattening are some of the negative symptoms of schizophrenia that may improve with risperidone therapy. Abnormal thought form is a positive symptom of schizophrenia. Hallucinations and delusions are positive symptoms of schizophrenia. Bizarre behavior is a positive symptom of schizophrenia.

A client was talking with her husband by telephone, and then she began swearing at him. The nurse interrupts the call and offers to talk with the client. She says, "I can't talk about that bastard right now. I just need to destroy something." Which of the following should the nurse do next? a) Tell her to write her feelings in her journal. b) Urge her to talk with the nurse now. c) Ask her to calm down or she will be restrained. d) Offer her a phone book to "destroy" while staying with her.

d - At this level of aggression, the client needs an appropriate physical outlet for the anger. She is beyond writing in a journal. Urging the client to talk to the nurse now or making threats, such as telling her that she will be restrained, is inappropriate and could lead to an escalation of her anger.

A 56-year-old client is receiving chemotherapy that has the potential to cause pulmonary toxicity. Which of the following symptoms indicates a toxic response to the chemotherapy? a) Decrease in appetite. b) Drowsiness. c) Spasms of the diaphragm. d) Cough and shortness of breath.

d - Cough and shortness of breath are significant symptoms because they may indicate decreasing pulmonary function secondary to drug toxicity. Decrease in appetite, difficulty in thinking clearly, and spasms of the diaphragm may occur as a result of chemotherapy; however, they are not indicative of pulmonary toxicity.

Which of the following is an early symptom of glaucoma? a) Hazy vision. b) Loss of central vision. c) Blurred or "sooty" vision. d) Impaired peripheral vision.

d - In glaucoma, peripheral vision is impaired long before central vision is impaired. Hazy, blurred, or distorted vision is consistent with a diagnosis of cataracts. Loss of central vision is consistent with senile macular degeneration but it occurs late in glaucoma. Blurred or "sooty" vision is consistent with a diagnosis of detached retina.

While assessing a male neonate whose mother desires him to be circumcised, the nurse observes that the neonate's urinary meatus appears to be located on the ventral surface of the penis. The primary health care provider is notified because the nurse suspects which of the following? a. phimosis b. hydrocele c. epispadias d. hydrospadias

d - The condition in which the urinary meatus is located on the ventral surface of the penis, termed hypospadias, occurs in 1 of every 500 male infants. Circumcision is delayed until the condition is corrected surgically, usually between 6 and 12 months of age. Phimosis is an inability to retract the prepuce at an age when it should be retractable or by age 3 years. Phimosis may necessitate circumcision or surgical intervention. Hydrocele is a painless swelling of the scrotum that is common in neonates. It is not a contraindication for circumcision. Epispadias occurs when the urinary meatus is located on the dorsal surface of the penis. It is extremely rare and is commonly associated with bladder extrophy.

A client with burns on his groin has developed blisters. As the client is bathing, a few blisters break. The best action for the nurse to take is to: a) remove the raised skin because the blister has already broken. b) wash the area with soap and water to disinfect it. c) apply a weakened alcohol solution to clean the area. d) clean the area with normal saline solution and cover it with a protective dressing.

d - The nurse should clean the area with a mild solution such as normal saline, and then cover it with a protective dressing. Soap and water and alcohol are too harsh. The body's first line of defense broke when the blisters opened; removing the skin exposes a larger area to the risk of infection.

A client diagnosed with pain disorder is talking with the nurse about fishing when he suddenly reverts to talking about the pain in his arm. Which of the following should the nurse do next? a) Allow the client to talk about his pain. b) Ask the client if he needs more pain medication. c) Get up and leave the client. d) Redirect the interaction back to fishing.

d - The nurse should redirect the interaction back to fishing or another focus whenever the client begins to ruminate about physical symptoms or impairment. Doing so helps the client talk about topics that are more therapeutic and beneficial to recovery. Allowing the client to talk about his pain or asking if he needs additional pain medication is not therapeutic because it reinforces the client's need for the symptom. Getting up and leaving the client is not appropriate unless the nurse has set limits previously by saying, "I will get up and leave if you continue to talk about your pain."

Nurses teach infant care and safety classes to assist parents in appropriately preparing to take their neonates home. Which statement about automobile restraints for infants is correct? a) An infant should ride in a front-facing car seat until he weighs 20 lb (9.1 kg) and is 1 year old. b) An infant should ride in a rear-facing car seat until he weighs 25 lb (11.3 kg) or is 1 year old. c) An infant should ride in a front-facing car seat until he weighs 30 lb (13.6 kg) or is 2 years old. d) An infant should ride in a rear-facing car seat until he weighs 20 lb and is 1 year old.

d - Until the infant weighs 20 lb and is 1 year old, he should ride in a rear-facing car seat.

A client has refused to take a shower since being admitted 4 days earlier. He tells a nurse, "there are poison crystals hidden in the showerhead. They'll kill me if I take a shower." Which nursing action is most appropriate? a. dismantling the showerhead and showing the client that there is nothing in it b. explaining that other clients are complaining about the client's body odor c. asking a security officer to assist in giving the client a shower d. accepting these fears and allowing the client to take a sponge bath

d - by acknowledging the client's fears, the nurse can arrange to meet the client's hygiene needs in another way. Because these fears are real to the client, providing a demonstration of reality by dismantling the shower head wouldn't be effective at this time.

a nurse is caring for a client who required chest tube insertion for a pneumothorax. To assess for pneumothorax resolution, the nurse can anticipate that the client will require: a. monitoring of arterial oxygen saturation (SaO2) b. arterial blood gas (ABG) studies c. chest auscultation d. chest x ray

d - chest x ray confirms diagnosis by revealing air or fluid in the pleural space. SaO2 values may initially decrease with a pneumothorax but typically return to normal within 24 hours. ABG studies may show hypoxemia, possibly with respiratory acidosis and hypercapnia but these are not necessarily related to a pneumothorax. Chest auscultation will determine overall lung status, but it's difficult to determine if the best has re-expanded sufficiently.

The client with a hearing aid does not seem to be able to hear the nurse. The nurse should do which of the following? a. contact the client's audiologist b. cleanse the hearing aid ear mold in normal saline c. irrigate the ear canal d. check the hearing aid's placement

d - inadequate amplification can occur when a hearing aid is not place properly. The certified audiologist is licensed to dispense hearing aids. The ear mold is the only part of the hearing aid that may be wash frequently; it should be washed daily with soap and water. Irrigation of the ear canal is done to remove impacted cerumen or a foreign body

To prevent development of peripheral neuropathies associated with isoniazid administration, the nurse should teach the client to: a. avoid excessive sun exposure b. follow a low-cholesterol diet c. obtain extra rest d. supplement the diet with pyridoxine (vitamin B6)

d - isoniazid competes for the available vitamin B6 in the body and leaves the client at risk for developing neuropathies related to vitamin deficiency. Supplemental vitamin B6 is routinely prescribed to address this issue. Avoiding sun exposure is a preventative measure to lower the risk of skin cancer. Following a low-cholesterol diet lowers the individual's risk of developing atherosclerotic plaque. Rest is important in maintaining homeostasis but has no real impact on neuropathies.

The nurse receives change-of-shift report about the following four patients. Which patient will the nurse assess first: a) A patient who has malnutrition associated with 4+generalized pitting edema b) A patient whose potential nutrition has 10 mL of solution left in the infusion bag d) A patient who is receiving continuous internal feedings and has new onset crackles throughout the lungs

d) A patient who is receiving continuous internal feedings and has new onset crackles throughout the lungs

A 16-year-old academically gifted boy is about to graduate from high school early, because he has completed all courses needed to earn a diploma. Within the last 3 months, he has experienced panic attacks that have forced him to leave classes early and occasionally miss a day of school. He is concerned that these attacks may hinder his ability to pursue a college degree. What would be the best response by the school nurse who has been helping him deal with his panic attacks? a) "It is natural to be worried about going into a new environment. I am sure with your abilities you will do well once you get settled." b) "You are putting too much pressure on yourself. You just need to relax more and things will be alright." c) "It might be best for you to postpone going to college. You need to get these panic attacks controlled first." d) "It sounds like you have a real concern about transitioning to college. I can refer you to a health care provider for assessment and treatment."

d) CORRECT ANSWER "It sounds like you have a real concern about transitioning to college. I can refer you to a health care provider for assessment and treatment." Reason: The client's concerns are real and serious enough to warrant assessment by a physician rather than being dismissed as trivial. Though he is very intelligent, his intelligence cannot overcome his anxiety. In fact, his anxiety is likely to interfere with his ability to perform in college if no assessment and treatment are received. Just postponing college is likely to increase rather than lower the client's anxiety, because it does not address the panic he is experiencing.

A 16-year-old primigravida at 36 weeks' gestation who has had no prenatal care experienced a seizure at work and is being transported to the hospital by ambulance. Which of the following should the nurse do upon the client's arrival? a) Position the client in a supine position. b) Auscultate breath sounds every 4 hours. c) Monitor the vital signs every 4 hours. d) Admit the client to a quiet, darkened room.

d) CORRECT ANSWER Admit the client to a quiet, darkened room. Reason: Because of her age and report of a seizure, the client is probably experiencing eclampsia, a condition in which convulsions occur in the absence of any underlying cause. Although the actual cause is unknown, adolescents and women older than 35 years are at higher risk. The client's environment should be kept as free of stimuli as possible. Thus, the nurse should admit the client to a quiet, darkened room. Clients experiencing eclampsia should be kept on the left side to promote placental perfusion. In some cases, edema of the lungs develops after seizures and is a sign of cardiovascular failure. Because the client is at risk for pulmonary edema, breath sounds should be monitored every 2 hours. Vital signs should be monitored frequently, at least every hour.

During the first feeding, the nurse observes that the neonate becomes cyanotic after gagging on mucus. Which of the following should the nurse do first? a) Start mouth-to-mouth resuscitation. b) Contact the neonatal resuscitation team. c) Raise the neonate's head and pat the back gently. d) Clear the neonate's airway with suction or gravity.

d) CORRECT ANSWER Clear the neonate's airway with suction or gravity. Reason: If a neonate gags on mucus and becomes cyanotic during the first feeding, the airway is most likely closed. The nurse should clear the airway by gravity (by lowering the infant's head) or suction. Starting mouth-to-mouth resuscitation is not indicated unless the neonate remains cyanotic and lowering his head or suctioning doesn't clear his airway. Contacting the neonatal resuscitation team is not warranted unless the infant remains cyanotic even after measures to clear the airway. Raising the neonate's head and patting the back are not appropriate actions for removing mucus. Doing so allows the mucus to remain lodged causing further breathing difficulties.

A nurse is performing a baseline assessment of a client's skin integrity. What is the priority assessment parameter? a) Family history of pressure ulcers b) Presence of pressure ulcers on the client c) Potential areas of pressure ulcer development d) Overall risk of developing pressure ulcers

d) CORRECT ANSWER Overall risk of developing pressure ulcers Reason: When assessing skin integrity, the overall risk potential of developing pressure ulcers takes priority. Overall risk encompasses existing pressure ulcers as well as potential areas for development of pressure ulcers. Family history isn't important when assessing skin integrity.

A nurse is caring for a client with a diagnosis of Impaired gas exchange. Based upon this nursing diagnosis, which outcome is most appropriate? a) The client maintains a reduced cough effort to lessen fatigue. b) The client restricts fluid intake to prevent overhydration. c) The client reduces daily activities to a minimum. d) The client has normal breath sounds in all lung fields.

d) CORRECT ANSWER The client has normal breath sounds in all lung fields. Reason: If the interventions are effective, the client's breath sounds should return to normal. The client should be able to cough effectively and should be encouraged to increase activity, as tolerated. Fluids should help thin secretions, so fluid intake should be encouraged.

Based on a client's history of violence toward others and her inability to cope with anger, which of the following should the nurse use as the most important indicator of goal achievement before discharge? a) Acknowledgment of her angry feelings. b) Ability to describe situations that provoke angry feelings. c) Development of a list of how she has handled her anger in the past. d) Verbalization of her feelings in an appropriate manner.

d) CORRECT ANSWER Verbalization of her feelings in an appropriate manner. Reason: Verbalizing feelings, especially feelings of anger, in an appropriate manner is an adaptive method of coping that reduces the chance that the client will act out these feelings toward others. The client's ability to verbalize her feelings indicates a change in behavior, a crucial indicator of goal achievement. Although acknowledging feelings of anger and describing situations that precipitate angry feelings are important in helping the client reach her goal, they are not appropriate indicators that she has changed her behavior. Asking the client to list how she has handled anger in the past is helpful if the nurse discusses coping methods with the client. However, based on this client's history, this would not be helpful because the nurse and client are already aware of the client's aggression toward others.

On the second postpartum day a gravida 6, para 5 complains of intermittent abdominal cramping. The nurse should assess for: a) endometritis. b) postpartum hemorrhage. c) subinvolution. d) afterpains.

d) CORRECT ANSWER afterpains. Reason: In a multiparous client, decreased uterine muscle tone causes alternating relaxation and contraction during uterine involution, which leads to afterpains. The client's symptoms don't suggest endometritis, hemorrhage, or subinvolution.

A nurse is providing care for a pregnant client in her second trimester. Glucose tolerance test results show a blood glucose level of 160 mg/dl. The nurse should anticipate that the client will need to: a) start using insulin. b) start taking an oral antidiabetic drug. c) monitor her urine for glucose. d) be taught about diet.

d) CORRECT ANSWER be taught about diet. Reason: The client will need to watch her overall diet intake to control her blood glucose level. The client's blood glucose level should be controlled initially by diet and exercise, rather than insulin. Oral antidiabetic drugs aren't used in pregnant clients. Urine glucose levels aren't an accurate indication of blood glucose levels.

When assessing an elderly client, the nurse expects to find various aging-related physiologic changes. These changes include: a) increased coronary artery blood flow. b) decreased posterior thoracic curve. c) decreased peripheral resistance. d) delayed gastric emptying.

d) CORRECT ANSWER delayed gastric emptying. Reason: Aging-related physiologic changes include delayed gastric emptying, decreased coronary artery blood flow, an increased posterior thoracic curve, and increased peripheral resistance.

A client with chronic undifferentiated schizophrenia is admitted to the psychiatric unit of a local hospital. During the next several days, the client is seen laughing, yelling, and talking to himself. This behavior is characteristic of: a) delusion. b) looseness of association. c) illusion. d) hallucination.

d) CORRECT ANSWER hallucination. Reason: Auditory hallucination, in which one hears voices when no external stimuli exist, is common in schizophrenic clients. Such behaviors as laughing, yelling, and talking to oneself suggest such a hallucination. Delusions, also common in schizophrenia, are false beliefs or ideas that arise without external stimuli. Clients with schizophrenia may exhibit looseness of association, a pattern of thinking and communicating in which ideas aren't clearly linked to one another. Illusion is a less severe perceptual disturbance in which the client misinterprets actual external stimuli. Illusions are rarely associated with schizophrenia.

During which stage of NREM sleep would you expect a client to be most difficult to arouse: c) Stage III d) Stage IV

d) Stage IV

To help alleviate spiritual distress effectively, the nurse must: b) offer to pray with the client d) find out what the client perceives his/her spiritual needs to be

d) find out what the client perceives his/her spiritual needs to be

calcitonin

decrease serum calcium which inhibits bone reabsorption

GI s&s hypothyroid

decreases and slows down the movement in the GI tract - this often results in constipation - Other GI problems associated are irritable bowel syndrome (IBS)

In the stages of death and dying as defined by Elizabeth Kubler-Ross, feelings of loss, grief, and intense sadness are symptoms of:

depression.

The nurse is caring for a client with a colostomy. The client tells the nurse that he makes small pin holes in the drainage bag to help relieve gas. The nurse should teach him that this action:

destroys the odor-proof seal.

The nurse is collecting data on an elderly client. When collecting data, the nurse should consider that one normal aging change is:

diminished reflexes.

A client who recently had a stroke requires a cane to ambulate. When teaching about cane use, the rationale for holding a cane on the uninvolved side is to:

distribute weight away from the involved side.

While examining a 12-month-old child, the nurse notes that the child can stand independently but cannot walk without support. The nurse should:

do nothing because this is a normal finding in a child this age.

*important assessment question to ask a hypothyroid patient

do you feel fatigued even if you sleep a lot?

*question to ask patient with hypoparathyroidism?

do you have bone pain?

meds to supress prolactin

dostinex, paroldel

The mother of an infant with a cleft lip asks when the repair will be scheduled. What is the nurse's best response?

during the first 6 months of life

When developing a therapeutic relationship with a client, the nurse should begin preparing the client for termination of the relationship:

during the first meeting.

small pox diagnostics

dx based on hx and px findings - throat swab and/or pustule swab

Client receiving IV fluids of 150 mL/h which indicates fluid overload

dyspnea

respiratory s&s hyperthyroid

dyspnea due to oxygen demands/hypoxia

When caring for a client with a nursing diagnosis of Impaired swallowing related to neuromuscular impairment, the nurse should:

elevate the head of the bed 90 degrees during meals.

After undergoing a left thoracotomy, a client has a chest tube in place. When caring for this client, the nurse must:

encourage coughing and deep breathing. When caring for a client who's recovering from a thoracotomy, the nurse should encourage coughing and deep breathing to prevent pneumonia. Fluctuations in the water-seal chamber are normal. Clamping the chest tube could cause a tension pneumothorax. Chest tube milking is controversial and should be done only to remove blood clots that obstruct the flow of drainage.

Goiter

enlargement of the thyroid - Occurs in both hyperthyroidism and hypothyroidism - Diffuse - enlarging the whole thyroid

A client is transferred from the emergency department to the locked psychiatric unit after attempting suicide by taking 200 acetaminophen tablets. The client is now awake and alert but refuses to speak with the nurse. In this situation, the nurse's first priority is to:

ensure safety by initiating suicide precautions.

The nurse is preparing to begin one-person cardiopulmonary resuscitation. The nurse should first:

establish unresponsiveness.

A 25-year-old client asks the nurse how often and when she should perform breast self-examinations. The nurse should tell her:

every month, 7 to 10 days after menses starts.

In comparison to children with acute otitis media (AOM), those with otitis media with effusion (OME) have:

excess middle ear fluid.

One of the causes of conductive hearing loss is:

excess middle ear fluid.

Mineralocorticoids

exert their main effects on electrolyte metabolism (the "salt" hormones) aldosterone - essential to life - The release of aldosterone is also increased by hyperkalemia - Aldosterone is the long-term regulation of sodium balance - Increases retention of sodium and water by the kidneys - Stimulated by changes in serum sodium, sodium and the renin-angiotensin system - maintains blood volume and BP - Increases excretion of potassium

*woman on menopause complains of painful sex

experiencing vaginal dryness

The nurse is teaching a client diagnosed with basal cell epithelioma. The most common cause of basal cell epithelioma is:

exposure to the sun.

The nurse is collecting data on a 47-year-old client who has come to the physician's office for his annual physical. One of the first physical signs of aging is:

failing eyesight, especially close vision.

A client expresses confidence in his traditional healer to relieve symptoms of an illness. What is the client demonstrating?

faith

*signs to look for in person with thyroid dysfunction

fatigue after sleeping long periods

A person's psychosocial needs during the dying process of a relative may include:

flexible visitation, participation in client care, and rest breaks.

The nurse is instructing a client about the use of antiembolism stockings. Antiembolism stockings help prevent deep vein thrombosis (DVT) by:

forcing blood into the deep venous system.

*how would you give instructions to patient diagnosed with hyperthyroidism

give written instructions

*patient asks why she must take steroid every other day - feels better the days she takes it

gives change for adrenal gland to function normally

The nurse is speaking to a group of women about early detection of breast cancer. The average age of the women in the group is 47. Following the American Cancer Society guidelines, the nurse should recommend that the women:

have a mammogram annually.

*priority patient with hypothyroidism

heart rate 48

A nurse received an accidental needle stick while giving an I.M. injection. The greatest threat for the nurse is:

hepatitis B (HBV).

bioterrorism - category a

high priority agents including organisms that pose a risk to large number of people at once - intentional attack - criminal act - results in high mortality rate - easily transmitted - ie. anthrax, small pox

When determining the parents' compliance with treatment for their infant who has otitis media, the nurse should ask the parents if they are:

holding the child upright when feeding with a bottle.

*what to expect to see in a patient with cushings...

hypertension, peripheral edema, and petichiae

Before advising a 24-year-old client desiring oral contraceptives for family planning, the nurse would assess the client for which signs and symptoms?

hypertension. Before advising a client about oral contraceptives, the nurse needs to assess the client for signs and symptoms of hypertension. Clients who have hypertension, thrombophlebitis, obesity, or a family history of cerebral or cardiovascular accident are poor candidates for oral contraceptives. In addition, women who smoke, are older than 40 years of age, or have a history of pulmonary disease should be advised to use a different method.

hematologic s&s hyperthyroid

hyperthyroidism emerged to have an increased risk of thrombotic events - A number of case reports have documented acute venous thrombosis complications in patients with overt hyperthyroidism, especially at cerebral sites - A small fall in hemoglobin is therefore usual in hyperthyroidism - may sometimes be sufficient to cause a mild degree of anemia

The nurse is collecting data on a client who has developed a paralytic ileus. The client's bowel sounds will be:

hypoactive

A client who has a history of Crohn's disease is admitted to the hospital with fever, diarrhea, cramping, abdominal pain, and weight loss. The nurse should monitor the client for:

hypokalemia.

*RN would not administer morphine to patient w/...

hypothyroidism

The nurse is instructing a premenopausal woman about breast self-examination. The nurse should tell the client to do her self-examination:

immediately after her menses.

Acute Adrenal Insufficiency/Adrenal Crisis

inadequate secretion of ACTH from the pituitary gland d/t decreased stimulation of the adrenal cortex or stress from surgery

The nurse is teaching a client with a family history of atherosclerosis. To decrease the risk of atherosclerosis, the nurse should encourage the client to:

increase his activity level.

GI s&s hyperthyroid

increased appetite and dietary intake, abdominal pain, changes in bowel function - diarrhea

During accommodation, pupillary dilation partially compensates for the reduced size of the retinal image by:

increasing light entering the pupil.

The nurse must assess skin turgor of an elderly client. When evaluating skin turgor, the nurse should remember that:

inelastic skin turgor is a normal part of aging.

A client is to have a below-the-knee amputation. Prior to surgery, the circulating nurse in the operating room should:

initiate a time-out.

The nurse is caring for a client admitted to the hospital with a bowel obstruction. The nurse should wear sterile gloves when:

inserting an indwelling urinary catheter.

Diabetic and hypertensive retinopathy are both characterized by the appearance of:

intraretinal hemorrhages.

The nurse is caring for a client who underwent a subtotal gastrectomy 24 hours earlier. The client has a nasogastric (NG) tube. The nurse should:

irrigate the NG tube gently with normal saline solution as prescribed.

neurological s&s hyperthyroid

it also stimulated the person becomes highly irritable, anxious and nervous

cutaneous anthrax s&s

itchy sore lesion similar to insect bite - develops 1-12 days after exposure - painful lymph nodes, fever, malaise, headache

The nurse is caring for a client who recently underwent a tracheostomy. The first priority when caring for a client with a tracheostomy is:

keeping his airway patent.

The nurse is caring for a client with a history of falls. The first priority when caring for a client at risk for falls is:

keeping the bed in the lowest possible position.

The nurse is working on a surgical floor. The nurse must logroll a client following a:

laminectomy.

When preparing a client for an enema, the nurse should help him into the:

left-lateral Sims' position.

oropharyngeal anthrax s&s

lesions at base of tongue or tonsils sore throat, dysphagia, fever, lymphadenopathy

The nurse is administering I.M. injections to an older client. The nurse should remember that an older client has:

less subcutaneous tissue and muscle mass than a younger client.

The nurse is caring for a client who recently underwent a total hip replacement. The nurse should:

limit client hip flexion while sitting.

The nurse should assess the client for digoxin toxicity if serum levels indicate that the client has a:

low potassium level.

PTH intact

measures the level of parathyroid hormone in the blood - This test is used to help identify hyperparathyroidism or to find the cause of abnormal calcium levels - 10-65 pg/ml

*patient with cushings - teach

moderate exercise

A client with pneumonia is experiencing pleuritic chest pain. The nurse should assess the client for:

moderate pain that worsens on inspiration.

IGF (Insulin-like Growth Factor)

more reliable measure of the GH levels (normal 135-250 ng/ml)

*small pox vaccine administration method

multiple punctures

GI (intestinal/abdominal) anthrax s&s

nausea, vomiting, fever, anorexia, abdominal pain, hematemesis, bloody diarrhea, septicemia, toxemia, cyanosis, shock, death

ACTH Deficit Replacement

need to replace cortisol

TSH Deficit Replacement

need to replace the thyroid hormone

Hypercalcemia causes...

neuromuscular irritation

small pox treatment

no cure - supportive care while body fights - alleviating sx - antipyretics, IV fluids, pain relief, skin care, antibiotics, vaccine within 4 days after exposure

*patient with cushings

no strenuous exercise

*post partial thyroidectomy - nurse knows patient has airway clearance b/c

no tracheal stridor, speaks clearly, denies numbness & tingling

*patient with graves disease is receiving RAI therapy - I131 - should be informed?

not to expect relief from symptoms immediately

A primigravid client at 38 weeks' gestation comes to the labor room because "my water broke." The health care provider (HCP) asks the nurse to verify spontaneous rupture of membranes using nitrazine paper. The nurse observes that the nitrazine paper turns bright blue. The nurse's next action should be to:

notify the HCP that the membranes are ruptured. Nitrazine paper responds to alkaline fluids by changing blue; amniotic fluid is alkaline so the color verifies that the membranes are ruptured. The nurse notifies the provider that membranes are ruptured so that a plan of action can be developed. Rupture of membranes in the absence of labor increases the risk of infection. Vaginal examinations are limited until labor is initiated. Wearing a sanitary pad increases potential for infection. Documentation of the Nitrazine test is completed after notifying the provider.

In order to maintain fixation on stable objects during head rotation, compensatory ______ movements occur.

nystagmus

The nurse is providing teaching to a client who's at risk for coronary artery disease (CAD). The nurse tells the client that CAD has many risk factors. Risk factors that can be controlled or modified include:

obesity, inactivity, diet, and smoking.

The nurse is caring for a client with a fractured hip. The client is combative, confused, and trying to get out of bed. The nurse should:

obtain a physician's order to restrain the client when less restrictive interventions fail.

A person calls the neonatal intensive care unit stating that his son is receiving care there. He tells the nurse that he and the mother "aren't together," and requests information about his son's condition. The nurse should:

obtain more data before giving the caller any confidential information.

anthrax transmission

occurs in humans when they come in contact with infected sick animals, contaminated products, or when directly exposed to spores - not transmitted from person to person

treatment for hyperthyroid

often consists of a combination of therapies including antithyroid agents, radioactive iodine (the most common treatment for Graves' disease) and surgery

The nurse should instruct a woman taking folic acid supplements for folic acid-deficiency anemia that:

oral contraceptive use, pregnancy, and lactation increase daily requirements. Oral contraceptive use, pregnancy, and lactation are situations that increase demand for folic acid. With supplementation, a response should cause the reticulocyte count to increase within 2 to 3 days after therapy has begun. It is not necessary to take folic acid on an empty stomach. A client may safely take both iron and folic acid supplementation.

Adrenal Cortex

outer layer of the gland and is necessary for life - Adrenocortical secretions make it possible for the body to adapt to stress of all kinds - ACTH maintains cortex function - Produces 3 types of steroid hormones which are glucocorticoids, mineralocorticoids, and androgens.

*patient that is diagnosed with an incurable disease is recommended what kind of care?

palliative

*presurgery for GH excess

patient will take sandostatin to reduce levels

*what patient should you assign the LPN?

patient with DM and hypothyroidism (LPNs can not teach and they cannot assess)

A client has undergone a colon resection. While turning him, wound dehiscence with evisceration occurs. The nurse's first response is to:

place saline-soaked sterile dressings on the wound.

A client with seizure disorder is having a grand mal seizure. During the active seizure phase, the nurse should:

place the client on his side, remove dangerous objects, and protect his head.

skeletal s&s hyperthyroid

premature osteoporosis and fractures

The nurse is caring for a client who has suffered a severe stroke. During data collection, the nurse notices Cheyne-Stokes respirations. Cheyne-Stokes respirations are:

progressively deeper breaths followed by shallower breaths with apneic periods.

An acute episode of angle-closure glaucoma is preceded by:

prolonged pupil dilation.

*benefit of progesterone in HRT

protect against uterine cancer

The nurse is providing home care instructions to a client who has recently had a skin graft. It's most important that the client remember to:

protect the graft from direct sunlight.

s&s of congenital hypothyroidism

puffy face, coarse facial features, dull look, thick protruding tongue, poor feeding, choking episodes, constipation or reduced stooling, jaundice prolonged, short stature, swollen, protuberant belly button, decreased activity, sleeps a lot, rarely cries or hoarse cry, dry brittle hair; low hairline, poor muscle tone, cool and pale skin, goiter, (enlarged thyroid), birth defects (eg, heart valve abnormality), poor weight gain due to poor appetite, poor growth, difficult breathing, slow pulse, swollen hands, feet and genitals

*patient with primary aldosteronism

quiet comfortable room - hypertension - hypokalemia

*what important vital sign should be reported immediately in a patient with hypothyroidism?

rapid irregular heart rate

*patient with graves given beta blockers because

reduces s&s

*what can you assign the NA to do?

remind patient to turn q2h

*hyperthyroid patient

report tachycardia

*what is covered by medicare and medicaid - inpatient stay with 5 days - rest for family member caring for patient

respite care

renal s&s hyperthyroid

results in increased GFR as well as increased renin - angiotensin -aldosterone activation

*bilateral adrenalectomy

risk for infection r/t open wound

T4

slow and long acting

small pox vs chicken pox death

small pox: 1/10 chicken pox: uncommon

small pox vs chicken pox fever

small pox: 2-4 days before rash chicken pox: at time of rash

small pox vs chicken pox distribution

small pox: more on arms and legs - on palms and soles chicken pox: more on body - absent on palms and soles

small pox vs chicken pox stages

small pox: pocks in same stage chicken pox: pocks in several stages

small pox vs chicken pox development

small pox: pocks slow - starts in mouth and spreads to face, forearms, then hands, lower limbs, then trunk chicken pox: rapid - from arms to legs, to trunk

Reviewing lab report for client in end stage kidney disease who received hemodialysis 24 hr ago. Findings to report to provider

sodium 148 mEq/

inhalation anthrax s&s

stage 1 - hours-days flu like sx stage 2 - severe dyspnea, shock, mediastinitis, neurological sx, hemorrhagic meningitis (cause of death)

A client receiving total parental nutrition is prescribed a 24-hour urine test. When initiating a 24-hour urine specimen, the collection time should:

start after a known voiding.

*assess client following thyroidectomy

state name b/c checking for laryngeal nerve damage

As a nurse is talking to a client, the client begins choking on his lunch. He's coughing forcefully. The nurse should:

stay with him but not intervene at this time.

When a labor and delivery nurse tells a coworker that an Asian client probably did not want any pain medication because "Asian women typically are stoic," the nurse is expressing a belief known as what?

stereotype

*M/M - instructions for 24 hour urine collection for catecholamines?

store in cold place and no stress 3 days before and during test

During labor, a low-risk multigravid client in active labor has begun pushing, and the fetal head is beginning to crown. To prevent perineal lacerations during the birth, the nurse should:

stretch the perineal tissues with sterile gloved fingers. Sterile gloves should always be worn by birth attendants to prevent infection to the laboring client and the fetus. Stretching the perineal muscles can decrease the incidence of tearing or lacerations.

When planning home care for a 3-year-old child with eczema, what should the nurse teach the mother to remove from the child's environment at home?

stuffed animals

A 12-year-old with asthma wants to exercise. Which activity should the nurse suggest to improve breathing?

swimming

A charge nurse observes two nurses using inappropriate technique when starting an I.V. on a child. The charge nurse should first:

talk with the nurses about proper technique and the risk of infection resulting from improper technique. A nurse has the responsibility to do no harm. If a nurse observes other health care professionals implementing inappropriate practices, the charge nurse should address the problem. The charge nurse's first action should be to counsel the nurses on correct I.V. techniques. If the behaviors continue, the nurse manager should be notified. The situation should not be ignored. Interrupting the nurses during the procedure may unduly cause the parents to lose trust in the nurses' care. The charge nurse should not talk with the child's parents regarding the incident unless a situation develops that requires the parents to be informed.

For the client who has difficulty falling asleep at night because of withdrawal symptoms from alcohol, which are abating, which nursing intervention is likely to be most effective?

teaching the client relaxation exercises to use before bedtime. The best action by the nurse to help a client who has difficulty falling asleep would be to teach the client relaxation exercises to use before bedtime to reduce anxiety and promote relaxation.

The red eyes of bacterial conjunctivitis are accompanied by ________, which differentiates it from the red eyes of acute glaucoma.

tearing, itching and burning

Client with cancer deciding between two treatment plans

tell me more about your understanding of the options.

*patient with cushings - what finding should be reported immediately?

temp of 101.6

Hypocalcemia causes...

tetany

The nurse is teaching a client with genital herpes. Education for this client should include an explanation of:

the importance of informing his partner of the disease.

The nurse is assessing a client who may be in the early stages of dehydration. Early manifestations of dehydration include:

thirst or confusion.

*patient receiving thyroid scan is afraid of being radioactive - I131 - appropriate response by nurse

this is a tracer dose - it wont harm you or others

*What do you want to monitor in a patient with addison's?

thready pulse

The nurse is administering sublingual nitroglycerin to a client. Immediately afterward, the client may experience:

throbbing headache or dizziness.

child with pituitary dwarfism - tx goal is...

to see linear growth

The nurse is to check a client's gag reflex. The most effective technique for testing the gag reflex is to:

touch the back of the client's throat with a tongue blade.

treatment for congenital hypothyroidism

treated with hormone replacement therapy e.g.,Levothroid, Levoxyl, Synthroid, Levothyroxine

*a patient with thyroid storm - the nurse should expect to admin

tylenol and steroids - never aspirin

The nurse is administering sublingual nitroglycerin to a client with chest pain. The nurse should place the medication:

under the tongue.

*RN knowns declomycin is effective in SIADH when?

urine output increases

A client has an order for 5,000 units of subcutaneous heparin every 12 hours. When injecting heparin subcutaneously, the nurse should:

use a 45- to 90-degree angle to insert.

The nurse is preparing a client for nonemergency surgery. The nurse should:

verify the client understands the informed consent form. The surgeon is responsible for explaining the surgical procedure to be performed and the risks of the procedure, as well as for obtaining the informed consent from the client. A nurse may be responsible for obtaining and witnessing a client's signature on the consent form. The nurse is the client's advocate, verifying that a client (or family member) understands the consent form and its implications, and that consent for the surgery is truly voluntary.

*M/M - what are come recommendation you can give to a menopausal woman to relieve hot flashes and insomnia?

vitamin e, yoga, medication, avoid strenuous activity at night - NOT ask md for hypnotic

The nurse is providing care for a client who underwent mitral valve replacement. The best example of a measurable client outcome goal is to:

walk from his room to the end of the hall and back before discharge.

The nurse is developing a teaching plan for a client with diabetes mellitus. A client with diabetes mellitus should:

wash and inspect feet daily.

The nurse is with a group of patient-care attendants reviewing infection-control measures. The nurse tells the group that the first line of intervention for preventing the spread of infection is:

washing hands.

*what indicates patient with DI or addison's needs further treatment?

weaning off vasopressin

Eighty percent of older adults have one chronic illness, and 60% have at least two. The older adult's ability to adapt determines:

whether they are ill or healthy.

With otosclerosis, early hearing loss is identified by an inability to hear:

whispering

*pheochromocytoma

will have high BP

*patient to delegate to LPN/UAP

woman who had MI and will soon be discharged


Kaugnay na mga set ng pag-aaral

causes and effects of the embargo act of 1807

View Set

Nutrition Chapter 5(definitions) and questions.

View Set

unit 5 packet- political participation

View Set

9.3: ¿Qué? and ¿cuál? and 9.4: Pronouns after Prepositions

View Set

trigonometric functions of special angles

View Set

Ch. 16 (DT w. Beta-Lactam Antibact. Agents)

View Set

Sexually Transmitted Disease Study Guide

View Set

Topic 1 Lesson 3 Reconstruction's Impact

View Set